You are on page 1of 146

CHAPTER 1

Child and Ado le s c e nt


Ps yc hiatry
Que s tions

DIRECTIONS (Questions 1 through 75): For each of d u ring her p reviou s m ajor d ep ressive ep i-
the multiple-choice questions in this section, select sod e, she d enies it at this visit. At this tim e,
the lettered answ er that is the one best response in she also d enies self-inju rious u rges. Which of
each case. the follow ing w ou ld be the m ost ap propriate
p lan for treatm ent?
Questions 1 and 2 (A) Ad m it her to the hosp ital given her
history and potential risk.
A 15-year-old girl w ith a history of m ajor d epressive
(B) Ask you r p sychiatric colleagu e to assess
d isord er (MDD) com es to you r p rim ary care office
her at their next available intake.
for a rou tine visit. When you w alk into the exam -
ining room , you notice that she is w ithd raw n w ith (C) Prescribe antid ep ressant m ed ications
her head bent d ow n. She d isplays p oor eye con- w ith a follow -u p ap p ointm ent in
tact and barely speaks d uring you r interview . She 2 w eeks.
ad m its that she u ses m arijuana ap proxim ately tw ice (D) Refer her to a social w orker for
per w eek and cu ts on her arm w hen stressed . H er p sychotherapy.
m other reveals that she is particu larly w orried as (E) Tell the m other you w ill follow u p w ith
the girl’s paternal u ncle com m itted su icid e 10 years her at you r next rou tine visit.
ago. You su spect that she is having a recu rrence of
d ep ressive sym ptom s and are concerned about her Questions 3 and 4
risk for su icid e.
A m other brings her 7-year-old son to you becau se
1. Which of the follow ing factors m ost increases she is w orried that he sits u p in bed in the m id d le of
this patient’s risk of com m itting suicid e? the night and scream s. She says that at those tim es
he is inconsolable but eventually falls back to sleep.
(A) Cu tting behavior
(B) Gend er 3. Which of the follow ing is the m ost likely
(C) Presence of d ep ression d iagnosis?
(D) Relative w ho com m itted su icid e (A) Interm ittent exp losive d isord er
(E) Su bstance u se (B) N arcolep sy
(C) N ightm are d isord er
2. Fu rther history is obtained , and the p atient
(D) N onrap id eye m ovem ent (N REM) sleep
reveals that she has, in fact, been feeling m ore
arou sal d isord er, sleep terror typ e
d ep ressed recently, w ith d ifficu lty sleep ing,
low ap p etite, fatigu e, and p roblem s con- (E) N REM sleep arou sal d isord er,
centrating. While she had su icid al id eation sleep w alking type

1
2 1: Child a nd Ad ole s c e nt P syc hia try

4. Du ring w hich stage of sleep d o these ep isod es Questions 7 and 8


m ost likely occur?
Parents bring their 6-year-old boy to the clinic at the
(A) Stage 1 requ est of the boy’s teachers. The teachers rep ort
(B) Stage 2 that he is quiet in class. When he d oes talk, he fre-
(C) Stages 3 to 4 quently m akes errors w ith verb tense. H is parents
(D) Rap id eye m ovem ent (REM) stage recall that his sp eech w as d elayed . On exam ination,
the boy is friend ly and cooperative. H is speech is
(E) Any stage
clear, but he u ses sim ple sentences w ith a lim ited
vocabulary. Otherw ise, his physical and laboratory
Questions 5 and 6
exam ination is norm al.
A 17-year-old girl w ith a history of asthm a p res-
ents for a p hysical exam ination p rior to entering 7. Which of the follow ing is the m ost likely
college. You note that she appears angry. Upon d iagnosis?
fu rther qu estioning, you learn that she has felt irri- (A) Child hood -onset flu ency d isord er
table for the past 6 m onths since breaking up w ith
(B) Langu age d isord er
her boyfriend of 2 years. She says she feels tired all
the tim e and com es hom e from school every d ay, (C) Social (p ragm atic) com m u nication
lies on the cou ch, and w atches You Tu be vid eos. H er d isord er
grad es have d rop p ed becau se she cannot concen- (D) Sp ecific learning d isability
trate. Despite her fatigue, she com plains of d ifficu lty (E) Sp eech sou nd d isord er
sleeping. She has lost 12 lb over the last 6 m onths.
She rep orts that she quit the senior celebration com - 8. By w hat age w ou ld failu re to sp eak 200 w ord s
m ittee, no longer “hangs ou t” w ith her friend s, can- be m ost consistent w ith a sp eech d elay in this
not im agine things w ill im p rove, and is consid ering patient?
not going to college. H er p hysical and laboratory
(A) 1 year
exam ination is norm al.
(B) 2 years
5. Which of the follow ing m ed ications w ou ld be (C) 3 years
the m ost app ropriate to treat this p atient? (D) 4 years
(A) Carbam azep ine (Tegretol) (E) 5 years
(B) Im ip ram ine (Tofranil)
Questions 9 and 10
(C) Lithiu m
(D) Olanzap ine (Zyp rexa) A 15-year-old boy w ithou t prior p sychiatric his-
(E) Sertraline (Zoloft) tory is on his school concert trip. H e is brought to a
local em ergency room becau se of the acu te onset of
6. Accord ing to the Diagnostic and Statisti- increased anger, agitation, and p aranoia. On inter-
cal M anual of M ental Disorders, Fifth Edition view , he reports feeling u nsafe because a govern-
(DSM -5), w hich of the follow ing sym p tom s m ent agency is spying on him .
in this p atient d ifferentiate her d isord er from
that in an ad u lt? 9. Which of the follow ing tests w ou ld be the
m ost im portant to ord er first?
(A) Anhed onia
(A) Electroencep halogram (EEG)
(B) Decreased concentration
(B) Glu cose tolerance test
(C) Insom nia
(C) Positron em ission tom ograp hy (PET)
(D) Irritable m ood
scan of his head
(E) Weight loss
(D) Thyroid fu nction tests
(E) Urine toxicology screen
Que s tions : 4–15 3

10. All approp riate laboratory evaluations and She rep orts that he ap pears extrem ely revved up
stu d ies com e back w ithin norm al lim its. Which and hyp eractive.
of the follow ing d iagnoses w ou ld be m ost
consistent w ith this patient’s presentation? 13. Which of the follow ing is the m ost likely
d iagnosis?
(A) Anorexia nervosa
(B) Bip olar d isord er, m anic (A) Attention-d eficit/ hyp eractivity d isord er
(C) Bord erline p ersonality d isord er (ADH D)
(D) Generalized anxiety d isord er (B) Bip olar d isord er, m ost recent ep isod e
m anic
(E) Major d ep ressive d isord er
(C) Major d ep ressive d isord er w ith
Questions 11 and 12 p sychotic featu res
(D) Panic d isord er
An 8-year-old boy is referred to you by a school (E) Schizop hrenia
nu rse because he has been com p laining of stom ach-
aches every m orning in school. On interview ing the 14. Ingestion of w hich of the follow ing su b-
boy’s m other, you learn that he d oes not like to go stances w ou ld m ost likely bring abou t sim ilar
to school, insists on com ing hom e im m ed iately after sym ptom s?
school each d ay, and sleep s in his parents’ bed at
(A) Alcohol
night. The m other d enies other com p laints.
(B) Cannabis
11. Which of the follow ing is the m ost likely (C) Cocaine
d iagnosis? (D) H eroin
(A) Post-trau m atic stress d isord er (PTSD) (E) Phencyclid ine (PCP)
(B) Reactive attachm ent d isord er
15. A 9-year-old boy w alks into you r office
(C) Sep aration anxiety d isord er accom panied w ith his mother. They are argu ing
(D) Social anxiety d isord er (social p hobia) abou t his w anting a new portable vid eo gam ing
(E) Sp ecific p hobia system . H is m other, exasp erated w ith her son’s
behavior, tells you that they w ere late becau se
12. Which of the follow ing w ou ld you m ost likely it took him a long tim e to finally agree to get
find in this p atient’s history? into the car to come to the appointm ent w ith
you. Alone w ith you in you r office, he appears
(A) Low levels of p arental control
irritated and refu ses to answ er your questions
(B) Parent w ith an anxiety d isord er or look up from his lap. After 10 minutes, he
(C) Parents w ith a w arm and accep ting style tells you about the “annoying kid s” in his
(D) Secu re attachm ent class and how they “mad e m e get in trouble.”
(E) Tem p eram ent characterized by Fu rther history from the m other reveals that,
sociability and extroversion d espite the above behavior, he has never been
violent or d estructive, and he has not had any
Questions 13 and 14 legal problems. Which of the follow ing is the
m ost likely d iagnosis?
A 16-year-old boy is brou ght to the p ed iatric em er-
gency d ep artm ent by his grand m other w ho rep orts (A) Agorap hobia
he is not acting like him self. H e rep orts that he (B) Attention-d eficit/ hyp eractivity d isord er
feels “incred ibly great” and d oes not need a d octor (ADH D)
becau se he has p ow ers to heal him self of all sickness. (C) Cond u ct d isord er
H e sp eaks rapid ly and his grand m other rep orts he (D) Generalized anxiety d isord er (GAD)
has not slept m ore than 6 hou rs over the p ast 3 d ays. (E) Op p ositional d efiant d isord er (ODD)
4 1: Child a nd Ad ole s c e nt P syc hia try

Questions 16 through 18 into the street in front of a car on the w ay home from
school. H e is physically unharmed , but refused to
A 15-year-old girl w ho is a com p etitive figu re speak to the emergency room d octor. You are asked
skater presents w ith concerns about her w eight. She to consult as you are on your psychiatry rotation. The
believes that she w ou ld be a better skater if she cou ld emergency room doctor is suspicious that the boy’s
lose w eight and feels very u p set and fru strated that behavior reflected und erlying suicid al impulses.
she has failed in her attem pts. The girl relu ctantly
ad m its that she som etim es eats “a w hole lot” of food 19. Which of the follow ing op ening statem ents
at one tim e su ch as a qu art of ice cream , a large bag w ou ld be the m ost valu able in facilitating the
of potato chips, and a jar of peanu t butter. You also boy’s d iscu ssion of the situ ation?
notice abrasions on the back of her right hand .
(A) “You su re w ere lu cky the car sw erved
16. Which of the follow ing opening statem ents at the last m inu te.”
w ou ld be the m ost ap p rop riate? (B) “You w eren’t trying to actu ally get hit
by the car, w ere you ?”
(A) “Tell m e abou t the scratches on you r
hand .” (C) “What were you thinking w hen you ran
into the street?”
(B) “I’ve noticed the cu ts on you r hand . Are
you trying to hu rt you rself?” (D) “It sou nd s like qu ite a d ay. Can you tell
m e abou t w hat hap pened after school
(C) “H ow d id the scratches hap p en?”
tod ay?”
(D) “I see you have scratches on you r hand .
(E) “Child ren w ho try to hu rt them selves
Do you have a cat?”
are very confu sed . Are you confu sed ?”
(E) “Som etim es I see girls w ho m ake
them selves throw up. H ave you ever 20. Which of the follow ing sym p tom s of MDD
d one that?” w ou ld be m ore likely in this p atient com p ared
to an ad olescent w ith MDD?
17. Which of the follow ing laboratory abnorm ali-
ties w ould you m ost likely find in this patient? (A) Dru g u se
(B) H op elessness
(A) Elevated iron
(C) H yp ersom nia
(B) Elevated p rotein
(D) Psychom otor agitation
(C) H yp erchlorem ia
(E) Weight change
(D) H yp okalem ia
(E) H yp onatrem ia 21. Which of the follow ing w ou ld be this child ’s
m ost likely m ethod of attem p ting suicid e?
18. Which of the follow ing typ es of p sychother-
ap y w ould likely be the m ost effective for this (A) Firearm s
p articular p atient? (B) H anging
(A) Cognitive-behavioral therap y (C) Ju m p ing from a significant height
(B) Fam ily therap y (D) Stabbing
(C) Grou p therap y (E) Su bstance ingestion
(D) Psychoanalysis
22. An 11-year-old boy w ith enu resis presents
(E) Psychod ynam ic p sychotherap y to the clinic for rou tine follow -up. H is bed -
w etting had not resp ond ed to behavioral
Questions 19 through 21 interventions, so you had p reviou sly initiated
An 8-year-old boy w ith a history of major depressive treatm ent w ith intranasal d esm opressin
d isorder (MDD) treated w ith fluoxetine (Prozac) is (Dd avp ) after com p letion of a fu ll p hysical
brought to the emergency department after running and laboratory exam ination. Which of the
Que s tions : 16–27 5

follow ing signs/ sym ptom s w ou ld be the m ost m ake od d , rep etitive m ovem ents w ith her hand s,
likely ad verse effect? and p refers to line her toys u p . She becom es acu tely
d istressed w hen rou tines are changed . The child ’s
(A) H ead ache
physical exam ination is u nrem arkable, bu t you note
(B) H ypotension that she d oes not seem to cry w hen she trip s over
(C) Liver toxicity you r exam ination stool, keep ing her attention on
(D) Sed ation aligning you r colored p encils.
(E) Trem or
25. Which of the follow ing is the m ost likely
Questions 23 and 24 d iagnosis?

A 9-year-old boy w ith a history of p anic d isord er (A) Au tism sp ectru m d isord er (ASD)
treated w ith cognitive-behavioral therapy is brought (B) Intellectu al d isability (ID)
to your office by his m other becau se he has been (C) Rett synd rom e
irritable and d ep ressed . On p hysical exam ination, (D) Selective m u tism
the boy appears d epressed bu t otherw ise norm al. (E) Social (p ragm atic) com m u nication
Laboratory exam ination is norm al. d isord er
23. What w ou ld be the likelihood of this p atient 26. The m other asks you abou t p rognosis for her
having a com orbid m ajor d ep ressive d isord er d au ghter ’s cond ition. Which of the follow ing
(MDD)? is the m ost p ositive p rognostic factor?
(A) 5% (A) Com orbid ep ilep sy
(B) 15% (B) Engagem ent in social skills training
(C) 25% (C) Fu nctional language by 5 years of age
(D) 50% (D) Intellectu al d isability
(E) 75% (E) Presence of catatonic sym ptom s
24. After a thorou gh history and m ental statu s Questions 27 and 28
exam ination, you d iagnose the boy w ith MDD
and d ecid e to initiate treatm ent w ith flu ox- A 12-year-old boy is referred by the cou rt for evalu -
etine. You inform the boy and his m other of ation. H e skip s school, stays ou t late at night, and
possible ad verse effects of flu oxetine. Which verbally abu ses his parents. H e has run aw ay from
of the follow ing w ou ld be the m ost likely sid e hom e on three sep arate occasions, p rom p ting his
effect? p arents to call the p olice. H e has been cau ght shop-
lifting and has been in nu m erou s p hysical fights
(A) H yp otension
w ith his p eers.
(B) Liver toxicity
(C) N au sea 27. Up on fu rther history, w hich of the follow ing
(D) Sed ation w ou ld m ost likely be fou nd in this p atient?
(E) Weight gain (A) Absence of a biological father
(B) Absence of a biological m other
Questions 25 and 26
(C) Mother w ith an anxiety d isord er
A 30-month-old girl is brou ght to the clinic by her (D) Patient being an only child
m other for a rou tine visit. The m other tells you that (E) Parents w ho d o not u se corp oral
the girl seem s to avoid affection, often d oes not look p u nishm ent
her in the eye, has stopp ed speaking in preschool,
and d oes not really engage w ith her p reschool peers
nor w ith her 5-year-old brother. She continu es to
6 1: Child a nd Ad ole s c e nt P syc hia try

28. Which of the follow ing p ersonality d isord ers that at any time, w ithout w arning, the boy w ill make a
is this boy m ost likely to d evelop ? d isruptive sound or shout out in class. They d escribe
him as polite and neat but restless and jumpy.
(A) Antisocial p ersonality d isord er
(B) Avoid ant p ersonality d isord er 31. Which of the follow ing is the m ost likely
(C) Paranoid p ersonality d isord er d iagnosis?
(D) Schizoid p ersonality d isord er
(A) Cond u ct d isord er
(E) Schizotyp al p ersonality d isord er
(B) Op p ositional d efiant d isord er
Questions 29 and 30 (C) Panic d isord er
(D) Sep aration anxiety d isord er
A 10-year-old girl w ho has recently been d iagnosed (E) Tou rette d isord er
w ith d iabetes m ellitu s typ e I is referred to you by
her ped iatrician for an evalu ation. You notice that 32. Which of the follow ing m ed ications w ou ld be
she seem s sad . H er parents are concerned about her the m ost appropriate to prescribe initially?
being d ep ressed . Consid eration is given for d iag-
nosing ad justm ent d isord er w ith d epressed m ood (A) Bu p rop ion (Wellbu trin)
versu s m ajor d epressive d isord er (MDD). (B) Clonid ine (Catap res)
(C) H alop erid ol (H ald ol)
29. Which of the follow ing criteria for the d iagno- (D) Paroxetine (Paxil)
sis of ad ju stm ent d isord er m ost d istinguishes
(E) Venlafaxine (Effexor)
it from MDD?
(A) Sym p tom s cau se m arked d istress or Questions 33 and 34
significant im pairm ent in fu nctioning.
A 7-year-old boy w ith leukem ia is referred to you
(B) Sym p tom s d evelop follow ing an
becau se of concerns abou t his m ood . H is p ar-
id entifiable stressor.
ents rep ort that he flu ctu ates betw een app earing
(C) Sym p tom s d evelop w ithin 3 m onths of d epressed and acting angry. At tim es, he plays qui-
the onset of the stressor. etly in his room , bu t at other tim es he d isp lays anger
(D) Sym p tom s d o not p ersist for m ore than outbu rsts, often hitting his 4-year-old brother. H is
6 m onths follow ing term ination of the m other ad m its that she has d ecreased her expecta-
stressor. tions of him , and feels that since he is ill he should
(E) Sym p tom s d o not rep resent norm al not receive any p u nishm ents.
bereavem ent.
33. Which of the follow ing m ethod s w ould be the
30. Approxim ately w hat p ercentage of child ren m ost effective w ay to engage his m other in a
w ho are d iagnosed w ith d iabetes m ellitu s d iscussion regard ing the role of her actions on
typ e I d evelop ad justm ent d isord er follow ing the boy’s behavior?
their m ed ical d iagnosis?
(A) Acknow led ge her gu ilt and anxiety
(A) 1% abou t her son’s illness and explain the
(B) 5% im p ortance of p rovid ing lim its and
(C) 10% stru cture for his em otional w ell-being.
(D) 33% (B) Em p athize w ith the trau m a of having a
sick child .
(E) 75%
(C) Refer her to a p arent su p p ort grou p .
Questions 31 and 32 (D) Sit qu ietly and m ake no com m ents
abou t her p arenting style.
A 9-year-old boy is referred to you for evaluation (E) Tell her that treating her son like a baby
because of repeated teasing at school related to his is hu rting him em otionally.
inappropriate peer interactions. The teachers report
Que s tions : 28–38 7

34. It is d eterm ined that the boy is su ffering from Questions 37 and 38
m ajor d epressive d isord er. You d iscu ss both
p sychopharm acologic and psychotherapeu tic An 8-year-old boy w ith a family history of tic d isor-
treatm ent options, but she is concerned abou t d ers is referred to you for an evaluation of behav-
his ongoing leukem ia treatm ent. Which of ioral d ifficulties in school. H is teachers report that he
the follow ing approaches w ou ld be the m ost is unable to sit still, constantly fid gets, and is unable
ap propriate regard ing treating both his leu ke- to complete class w ork becau se he is so easily d is-
m ia and m ajor d epression? tracted . When he com pletes his w ork, it is often d one
carelessly and is frequently not tu rned in. The teach-
(A) Treat the d ep ression p rior to the ers also note that he blurts ou t replies and has a hard
leu kem ia. tim e d u ring quiet tim e. The boy’s m other reports that
(B) Treat the d ep ression after the leu kem ia. he has alw ays had a lot of energy. Preparing to leave
(C) Treat the d ep ression concu rrently w ith for school in the m orning is challenging because of
the leu kem ia. her son’s d isorganization and forgetfulness. Other-
(D) Treat the leu kem ia first and the w ise, she has no complaints. She d enies that her son
d epression w ill resolve. prod uces any repetitive m ovem ents or sou nd s.
(E) Treat the leu kem ia as treating the
37. Which of the follow ing is the m ost likely
d epression w ill not be effective in the
d iagnosis?
setting of a m alignancy.
(A) Attention-d eficit/ hyp eractivity d isord er
Questions 35 and 36 (ADH D)
(B) Bip olar d isord er
An 8-year-old boy p resents to you r office for a rou -
tine visit. One m onth earlier, you d iagnosed him (C) Cond u ct d isord er
w ith Tou rette d isord er and p rescribed m ed ication. (D) Op p ositional d efiant d isord er
H e and his p arents rep ort that the m ed ication has (E) Unsp ecified d isru p tive behavior
been help fu l. d isord er

35. Which of the follow ing d isord ers w ou ld this 38. The p atient is su bsequ ently treated for the
p atient be m ost likely to d evelop ? above cond ition. H e retu rns to an app oint-
m ent after several w eeks, now w ith rep etitive
(A) Au tism sp ectru m d isord er
grim acing and blinking m ovem ents, w hich
(B) Bip olar d isord er have resu lted in his getting teased in school.
(C) Langu age d isord er Which of the follow ing classes of m ed ications
(D) Obsessive-com p u lsive d isord er (OCD) is m ost likely to be resp onsible?
(E) Sep aration anxiety d isord er (A) Benzod iazep ines
(B) D 2 antagonists
36. Prior infection w ith w hich of the follow ing
w ou ld be m ost likely fou nd in the history of (C) Monoam ine oxid ase inhibitors (MAOIs)
this patient? (D) Selective serotonin reu p take inhibitors
(SSRIs)
(A) Haemophilus influenzae
(E) Stim u lants
(B) Influ enza viru s
(C) Parainflu enza viru s Questions 39 and 40
(D) Resp iratory syncytial viru s
A 6-year-old boy is referred to you by his school to
(E) Streptococcus pyogenes
evalu ate his d ifficu lty w ith keep ing u p w ith read ing
and m ath d esp ite his above average intelligence. It
is su sp ected that he su ffers from a learning d isord er,
so further testing and evaluation is ind icated .
8 1: Child a nd Ad ole s c e nt P syc hia try

39. Which of the follow ing find ings w ou ld be (C) Methylp henid ate (Ritalin)
required for a d iagnosis of sp ecific learning (D) Paroxetine (Paxil)
d isord er w ith im p airm ent in read ing? (E) Trazod one (Desyrel)
(A) The child has an above average IQ
(intelligence qu otient) bu t below Questions 43 through 45
average read ing achievem ent.
A 10-year-old boy is referred to you becau se his
(B) The child has an average IQ and below ped iatrician su spects that he m ay have attention-
average read ing achievem ent. d eficit/ hyperactivity d isord er (ADH D). After a
(C) The child ’s read ing achievem ent is thorou gh history, physical exam ination, and labora-
substantially below the child ’s IQ. tory investigation you m ake the d iagnosis of ADH D.
(D) The child ’s read ing achievem ent is After d iscu ssing the ad verse effects of m ed ications,
substantially above the child ’s IQ. you p rescribe m ethylp henid ate to be taken in the
(E) The child ’s read ing achievem ent and IQ m orning and at lu nch on school d ays.
are both below average.
43. Which of the follow ing w ou ld be the m ost
40. What w ou ld be the ap p roxim ate risk of this likely sid e effect of m ethylp henid ate?
child having a com orbid psychiatric d isord er? (A) H yp otension
(A) 5% (B) Insom nia
(B) 10% (C) Liver toxicity
(C) 25% (D) Trem or
(D) 50% (E) Weight gain
(E) 75%
44. What is the likelihood that this patient w ill sig-
Questions 41 and 42 nificantly benefit from the m ethylp henid ate?

An 8-year-old child is referred to you for an evalu- (A) 10%


ation of bed -w etting. Several behavioral interven- (B) 25%
tions have been attem pted , inclu d ing eliminating (C) 33%
fluid intake in the evening, sched uled aw akenings at (D) 50%
night to use the bathroom, and a urine alarm (a bell (E) 70%
and pad ). These techniqu es have been u nsu ccessful,
and the child continues to u rinate in bed every night. 45. Despite education and reassurance, the mother
remains opposed to stimulant medications but
41. What is the likelihood of this p atient having a still w ishes her son to receive pharmacologic
com orbid m ental illness? treatment for his ADH D. Which of the follow -
(A) 5% ing med ications w ould be the most appropriate
(B) 10% to prescribe?
(C) 20% (A) Arip ip razole (Abilify)
(D) 50% (B) Atom oxetine (Strattera)
(E) 75% (C) Citalop ram (Celexa)
(D) Mixed am p hetam ine salts (Ad d erall)
42. Which of the follow ing m ed ications w ou ld be (E) Valp roic acid (Dep akote)
the m ost ap propriate to prescribe to treat this
p atient? Questions 46 and 47
(A) Benztrop ine (Cogentin)
Du ring a rou tine office visit, the m other of a
(B) Desm op ressin (Dd avp ) 37-m onth-old girl tells you that she is concerned
Que s tions : 39–51 9

abou t her d au ghter’s behavior. Since the birth of 48. In which of the following areas would you most
her son 4 m onths earlier, the m other states that her likely expect additional difficulties in this patient?
d aughter has been m ore irritable and angry. The
(A) Attention
child has told her m other that she d oes not w ant the
baby anym ore and to take him back. The m other is (B) Fine m otor skills
esp ecially concerned becau se her d au ghter tried to (C) Gross m otor skills
bite the baby the w eek before. (D) Im aginary p lay
(E) Potty training p rogress
46. Which of the follow ing statem ents w ou ld be
the m ost approp riate response to this m other? 49. Which of the follow ing qu alities w ou ld be
(A) “If you sim p ly ignore you r d au ghter’s m ost associated w ith a m ore favorable p rog-
behavior, it w ill p ass.” nosis in this child ?
(B) “It is u nd erstand able that you r d au ghter (A) Easily toilet trained
is angry and exp eriences jealou sy w ith (B) Interested in m echanical toys
the new baby joining the fam ily.” (C) Organized in p lay
(C) “The next tim e she tries to bite him , you (D) Recip rocal conversation
should bite her back so she know s how
(E) Reciting songs and p oem s from m em ory
it feels.”
(D) “Tell you r d au ghter she is being a very Questions 50 and 51
bad girl and you w on’t love her if she
bites the baby.” A 17-year-old girl com es to you r office for a rou tine
(E) “Tell you r d au ghter that she need s to visit. She states that she feels fine and offers no com -
love the baby and be a w ond erfu l big p laints. On p hysical exam ination, you find that her
sister.” w eight is 92 lb and her height is 65 in. One year ear-
lier, her w eight w as 126 lb, and height 65 in. After
47. Which of the follow ing tasks w ou ld this girl fu rther d iscu ssion, you learn that she is terrified of
be able to p erform at her cu rrent age? gaining w eight and believes that she is fat and need s
to lose m ore w eight. She ad d itionally reports that
(A) Acknow led ge her angry and she has not m enstru ated in the past 6 m onths.
com petitive feelings tow ard her sibling.
(B) Be able to state her age and gend er. 50. Which of the follow ing laboratory abnorm a-
(C) Cou nt to 50. lities are you m ost likely to find in this p atient?
(D) Recognize that w ater p ou red from one (A) H yp ercholesterolem ia
glass into another of a d ifferent size is
(B) H yp erkalem ia
the sam e volu m e.
(C) H yp ocarotenem ia
(E) Rid e a bicycle.
(D) Increased thyroid -stim u lating horm one
(TSH )
Questions 48 and 49
(E) Leu kocytosis
A 4-year-old boy is referred to you for evalu ation
becau se he has p oor social related ness. Up on inter- 51. Which of the follow ing com p lications w ould
view , he appears healthy and w ell-kem pt. H e grabs be the m ost likely ind ication for ad m itting this
you r office key off you r d esk. At you r office d oor, p atient to the hosp ital?
he takes the key and locks and u nlocks you r d oor
rep eated ly. Desp ite attem p ts to red irect and d istract (A) Anem ia
him , he rem ains preoccu pied w ith this task. After (B) Arrhythm ia
abou t 10 m inu tes, you attem p t to take the key aw ay (C) Brad ycard ia
from him and he becom es extrem ely u p set, m aking (D) H yp otension
an insistent, p iercing cry. (E) Lanu go
10 1: Child a nd Ad ole s c e nt P syc hia try

Questions 52 and 53 54. Which of the follow ing item s in his history
w ou ld be m ost consistent w ith you r p rovi-
An 8-m onth-old boy is brought to the clinic by his sional d iagnosis?
m other. She com plains that her son has been expe-
riencing scream ing and crying fits w hen she leaves (A) Alw ays social and ou tgoing
him w ith a babysitter. She says that in the p ast he (B) Father w ith schizotyp al p ersonality
d id not object to being left w ith a babysitter and asks d isord er
you w hy he becom es so u p set now , and w hat she (C) Parents getting d ivorced
can d o about it. (D) Recently transferred schools
(E) Recently u sed m ariju ana
52. Which of the follow ing statem ents w ou ld be
the m ost app ropriate response?
55. Which of the follow ing featu res w ou ld ind i-
(A) “This behavior is characteristic of cate a poorer p rognosis in this patient?
au tistic child ren.”
(A) Acu te onset
(B) “It is p ossible that you r son has
(B) Affective sym p tom s
separation anxiety d isord er.”
(C) Good p rem orbid ad ju stm ent
(C) “This behavior su ggests that you ’re not
spend ing enou gh tim e w ith you r son.” (D) Onset before the age of 13 years
(D) “It sou nd s as thou gh you r son is overly (E) Well-d ifferentiated sym p tom s
attached to you .”
Questions 56 through 58
(E) “This behavior is norm al at you r son’s
age and w ill p ass w ith tim e.” A 9-year-old girl w ith a fam ily history of bipolar
d isord er is referred to you by her school because of
53. Which of the follow ing d iagnoses w ou ld be d isruptive behavior in class that has been w orsen-
m ost ap prop riate if the boy in the p reviou s ing over the past 3 m onths. H er teachers report that
qu estion (Qu estion 52) w ere an 8-year-old boy her energy level is high, and she is m arked ly more
w ith sim ilar behavior? d istractible. She is also sleeping more poorly and is
(A) Agorap hobia increasingly m ore intrusive into her siblings’ per-
sonal space. Both attention-d eficit/ hyp eractivity d is-
(B) N orm al behavior
ord er (ADH D) and a m anic episod e are consid ered .
(C) Obsessive-com p u lsive d isord er
(D) Sep aration anxiety d isord er 56. Which of the follow ing sym p tom s w ou ld
(E) Social anxiety (social p hobia) be m ore consistent w ith ADH D rather than
m ania?
Questions 54 and 55
(A) Distractibility
A 10-year-old boy is referred to you d u e to extrem e (B) Im p u lsivity
d ifficulties in school. H e has been held back a grad e (C) Low self-esteem
and is still not p assing his classes. Du ring the cou rse (D) Motoric hyp eractivity
of your evalu ation, you learn that the boy hears
(E) Pressu red sp eech
voices telling him that he is stupid and to leave the
classroom . Afraid to d isobey, he goes to the bath-
57. If this child ’s school is u nable to m anage her
room frequ ently. H e also has d ifficu lty falling asleep
behavior in her classroom d esp ite ou tp atient
at night becau se the voices keep him aw ake. In ad d i-
treatm ent and m ed ication, w hich of the fol-
tion, you learn that the boy believes others can read
low ing long-term op tions w ou ld be the m ost
his thou ghts. Physical and laboratory exam inations
optim al school placem ent?
are norm al. You su sp ect that the boy m ay be su ffer-
ing from schizop hrenia. (A) H om e schooling
(B) Parochial school
Que s tions : 52–63 11

(C) School for child ren w ith learning Questions 61 and 62


d isabilities
A 12-year-old boy w ith Tou rette d isord er com es to
(D) Resid ential treatm ent
you r office for a rou tine visit. Tw o w eeks earlier,
(E) Therap eu tic d ay school you had p rescribed clonid ine for his illness. The
boy rep orts that his tics have su bsid ed slightly since
58. Further history and evaluation over time result starting the clonid ine, bu t he com plains abou t the
in the d iagnosis of bipolar d isord er, most m ed icine.
recent episod e manic. Which of the follow ing
med ications w ou ld be the m ost appropriate 61. Which of the follow ing ad verse effects is this
treatm ent? boy m ost likely exp eriencing?
(A) Bu p rop ion (Wellbu trin) (A) Dry m ou th
(B) Du loxetine (Cym balta) (B) H yp otension
(C) Methylp henid ate (Ritalin) (C) N au sea
(D) Mixed am p hetam ine salts (Ad d erall) (D) Sed ation
(E) Valp roic acid (Dep akote) (E) Trem or

Questions 59 and 60 62. The p arents bring in the boy’s 7-year-old


An 8-year-old boy is brou ght to you r office by his brother for evalu ation. After fu rther history
m other for evaluation of an u pp er resp iratory infec- is obtained , he is d iagnosed w ith attention-
tion. The m other m entions that her son has started deficit/ hyperactivity d isord er (ADHD). Which
w etting the bed again. In ad d ition, she m entions of the follow ing classes of med ications w ould
that the boy’s grand m other d ied recently and w on- be the most appropriate choice for the brother?
d ers if this is affecting him . (A) Antip sychotic
(B) Monoam ine oxid ase inhibitor (MAOI)
59. At w hich of the follow ing ages w ou ld a child
(C) Serotonin-sp ecific reu p take inhibitor
norm ally be able to ap preciate that d eath is
(SSRI)
irreversible?
(D) Stim ulant
(A) 2 years (E) Tricyclic antid epressant (TCA)
(B) 3 years
(C) 5 years 63. A 6-year-old boy is brou ght to the em ergency
(D) 7 years d ep artm ent by his m other, w ho reports that
(E) 12 years he w as p laying on som e step s in front of the
hou se w hen he slip p ed and fell. She tells
60. Which of the follow ing d efense m echanism s you that she is concerned that he m ight have
is the boy m ost likely em p loying w hen he is broken his arm . An x-ray of the boy’s arm
w etting the bed ? show s a fracture of the u lna, as w ell as signs of
several old fractures of varying ages. Which of
(A) Acting ou t the follow ing is the m ost app rop riate cou rse
(B) Denial of action?
(C) Regression (A) Recom m end calcium su pp lem ents and a
(D) Repression m ultivitam in d aily.
(E) Som atization (B) Refer the boy to an orthoped ist for
fu rther evalu ation.
(C) Set the cu rrent broken bone in a cast
and have the boy see his p ed iatrician for
follow -u p care.
12 1: Child a nd Ad ole s c e nt P syc hia try

(D) Tell the boy that you notice that he has (C) N eu rology consu lt
had m u ltip le broken bones and ask him (D) Pu lm onary function tests
how each of these fractures happened . (E) Routine laboratory stu d ies
(E) Tell the m other that you notice that
the boy has had m ultiple broken bones 67. A 5-year-old girl d iagnosed w ith lu pu s is seen
and recom m end that she lim it the boy’s by her fem ale p ed iatrician for a rou tine visit.
sports activities. After retu rning hom e from the clinic, the girl
asks her friend to “p lay d octor.” Which of the
64. Th e m oth er of a 6-year-old boy calls an d follow ing d efense m echanism s best d escribes
asks you for ad vice. She says th at h er son this behavior?
still su cks his th u m b, an d sh e is concerned
abou t th is beh avior. Wh ich of th e follow - (A) Displacem ent
in g su ggestion s for h er to d o is th e m ost (B) Dissociation
ap p rop riate? (C) Id entification
(A) Ask the d entist to construct a m outh (D) Rationalization
ap p liance that w ill d eter su cking. (E) Reaction form ation
(B) Coat her son’s thum b in hot pepper
sauce. Questions 68 and 69
(C) Give him gu m frequ ently. A 10-year-old girl w ith a history of asthm a is
(D) Ignore the behavior. brou ght to the clinic after a recent increase in her
(E) Im p lem ent a behavioral system to asthm a sym p tom s. Du ring the visit, you learn that
rew ard stop p ing. she is being physically beaten by her m other’s boy-
friend on a regu lar basis.
Questions 65 and 66
68. Und er w hich of the follow ing circu m stances
A 14-year-old girl presents to her pediatrician com- d oes the law requ ire m and atory rep orting by
plaining that she has been “freaking out.” The girl a p hysician of su sp ected child abu se?
describes episodes of shaking, gasping for air, and
feeling like she is going to die. The feelings intensify (A) In all cases
for a few minutes and resolve spontaneously. These (B) Only in cases in w hich the child show s
episodes have occurred at various times, in various sit- behavioral m anifestations of abu se.
uations, and the girl is worried that she is going crazy. (C) Only w hen consent of a p arent or
A complete history and physical examination does not gu ard ian is obtained .
reveal any further relevant symptoms or signs. (D) Only w hen the p hysician believes it is in
a child ’s best interest.
65. Which of the follow ing is the m ost app rop riate (E) Only w hen the p hysician has exam ined
p harm acologic treatm ent? all child ren in the fam ily.
(A) Arip ip razole
(B) Carbam azep ine 69. Which of the follow ing m anifestations w ou ld
be the m ost likely ou tcom e of the abu se?
(C) Du loxetine
(D) Sertraline (A) Aggression
(E) Valp roic acid (B) Dissociative d isord er
(C) Generalized anxiety d isord er
66. Prior to prescribing m ed ications, w hich of the (D) Major d ep ressive d isord er
follow ing shou ld the p ed iatrician ord er next? (E) Post-trau m atic stress d isord er
(A) Electrocard iogram (ECG)
(B) Electroencep halogram (EEG)
Que s tions : 64–75 13

70. A 4-year-old boy is referred to you becau se he 73. An 8-year-old boy is brou ght in by his m other
w ill not sp eak in p reschool. Over the cou rse of w ho com p lains that she cannot get her son to
abou t 2 m onths, he grad u ally stop ped talking. listen to her. She is fru strated becau se he fre-
H is m other rep orts that he initially objected quently ignores her requ ests and instru ctions.
to going to preschool, but now no longer Consid eration is given tow ard the d iagnosis of
com plains. She states that at tim es her son is oppositional d efiant d isord er (ODD). Which
qu iet and stays in his room , bu t that she has of the follow ing featu res w ou ld best sup port
not otherw ise noticed a significant change in the d iagnosis in this child ?
his sp eech or behavior. Which of the follow ing
(A) Aggression to p eop le
is the m ost likely d iagnosis?
(B) Dep ressive sym p tom s
(A) Major d ep ressive d isord er (MDD) (C) Disobed ience tow ard teachers
(B) Persistent d ep ressive d isord er (D) Lack of p articip ation in tasks requ iring
(d ysthym ia) attention
(C) Selective m u tism (E) Violation of ru les
(D) Sep aration anxiety d isord er
(E) Social anxiety d isord er (social p hobia) 74. A 13-year-old girl is seen by her p sychiatrist
1 year after an au tom obile accid ent. She d em -
71. A 2-year-old boy is referred to you for evalu - onstrates intact langu age ability and com plex
ation d u e to the su sp icion that the child is m otor skills. She has no id entifiable abnor-
the victim of abu se second ary to factitiou s m alities in the perception of stim uli, bu t she
d isord er im posed by another. Which of the has lost the ability to read since the accid ent.
follow ing fam ily m em bers is the m ost likely Which of the follow ing d eficits is she m ost
p erp etrator fabricating the illness? likely d em onstrating?
(A) Father (A) Agnosia
(B) Brother (B) Alexia
(C) Mother (C) Anom ia
(D) Sister (D) Ap hasia
(E) Uncle (E) Ap raxia

72. A fru strated m other brings her 14-year-old son 75. A 10-year-old girl w ithou t significant m ed ical
to a child p sychiatrist after he is expelled from history is brou ght by her father to the p ed ia-
three high schools in 1 year. She reports the trician for evaluation. Over the past school
boy has tried tw ice to set his school on fire, has year, she has been having increasing d ifficu l-
slashed school bus tires, and has broken into ties going to sleep. Althou gh she has “alw ays
the principal’s office to steal athletic trophies. had bed tim e ritu als,” they have extend ed in
In ad d ition, he has been su sp end ed nu m er- com plexity and length. Most of her tim e in
ou s tim es for getting into fights w ith other the evening is now spent going arou nd the
stu d ents. She shu d d ers and tearfully relates hou se nu m erou s tim es, locking and u nlock-
that she recently caught him singeing one of ing the d oors and w ind ow s. While she know s
the fam ily cats w ith a cigarette butt. Which the chances of a bu rglary are slim , she is
of the follow ing p ersonality d isord ers is this extrem ely anxiou s abou t her safety, and she
boy m ost at risk of d evelop ing in the fu tu re? “can’t stop ” the u rges to perform these behav-
iors. As a resu lt, she only obtains 5 hou rs of
(A) Antisocial
sleep, and she has been falling asleep in class
(B) Bord erline w ith d im inishing grad es. Which of the follow -
(C) H istrionic ing therap eu tic interventions is consid ered the
(D) Obsessive-com p u lsive first-line treatm ent for this d isord er?
(E) Schizotyp al
14 1: Child a nd Ad ole s c e nt P sychia try

(A) Cognitive-behavioral therapy (CBT) interp reting social cu es and u tilizes less com -
(B) Fam ily therapy p lex sp oken langu age, is able to retain basic
(C) Grou p therap y langu age for social com m u nication. H e can
care for his basic personal need s after exten-
(D) Short-term psychod ynam ic therapy
sive teaching, bu t w ou ld eventu ally requ ire
(E) Supp ortive therapy consid erable su pport from co-w orkers and
sup ervisors to m anage responsibilities in the
D IRECTION S (Questions 76 through 93): The fol- fu tu re.
low ing group of numbered items are preceded by
a list of lettered options. For each question, select
the one lettered option that is most closely asso- Questions 80 through 83
ciated w ith it. Each lettered option may be used Match the m ost likely d isord er w ith the appropriate
once, multiple times, or not at all. p atient.

Questions 76 through 79 (A) Anorexia nervosa


(B) Au tism sp ectru m d isord er
Match the severity level of intellectual d isability w ith
(C) Bu lim ia nervosa
the patient’s ad aptive functioning.
(D) Binge eating d isord er
(A) N o intellectu al d isability (E) Illness anxiety d isord er
(B) Mild intellectu al d isability (F) Interm ittent exp losive d isord er
(C) Mod erate intellectu al d isability (G) Obsessive-com p u lsive d isord er
(D) Severe intellectu al d isability (H ) Panic d isord er
(E) Profou nd intellectu al d isability (I) Pica
(J) Pyrom ania
76. A 14-year-old boy w ho has lim ited attainm ent
of conceptual skills, has sp oken language lim - (K) Tou rette d isord er
ited to single w ord s or p hrases, requ ires su p - (L) Trichotillom ania
p ort for all activities of d aily living, requ ires
sup ervision at all tim es, and w ho cannot m ake 80. A 14-year-old girl w ith ep isod es of palpita-
resp onsible d ecisions regard ing the w ell-being tions, chest p ain, shortness of breath, and
of others. d iaphoresis w ho has a norm al physical and
laboratory exam inations.
77. An 8-year-old girl w ho has no obviou s d eficits
in learning, is equ ally m atu re as her p eers, is 81. A 10-year-old overw eight girl feels a loss of
able to perform d aily living tasks w ithout su p - control w hen she qu ickly consum es large
p ort, and has sou nd ju d gm ent. am ou nts of food , d enies heavy exercise, vom -
iting or laxative u se afterw ard s, and feels
significant guilt and sham e regard ing her
78. An 11-year-old boy w ho has d ifficu lties in
behavior.
acad em ic skills of w riting, read ing, and m ath,
is im m atu re in social situ ations, and w ho
has d ifficulties w ith regu lating em otions and 82. An 8-year-old boy w ith erythem atou s,
behavior in age-ap p rop riate fashion. While he chapped hand s, and an otherw ise norm al
is able to p rovid e his ow n p ersonal care, he p hysical and laboratory exam ination.
requ ires su p p ort in m ore com p lex d aily living
tasks, ju d gm ent, and organization. 83. A 13-year-old girl w ith a bald p atch on the
back of her head and an otherw ise norm al
79. A 17-year-old boy w hose concep tu al skills p hysical and laboratory exam ination.
consistently lag behind those of his peers,
and , w hile he has d ifficu lties p erceiving or
Que s tions : 76–94 15

Questions 84 through 87 90. Establishing tru st in the w orld throu gh


resp onsiveness and em p athy of a caregiver.
Match the m ost likely d isord er w ith the appropriate
patient.
91. Preoccup ation w ith su perheroes w ho rep re-
(A) Attention-d eficit/ hyp eractivity d isord er sent id ealized caregivers as a resu lt of con-
(ADH D) flicted feelings tow ard caregivers.
(B) Au tism sp ectru m d isord er
(C) Generalized anxiety d isord er 92. The d evelop m ent of the ability to think abou t
(D) Langu age d isord er and m anipu late id eas abstractly.
(E) Selective m u tism
(F) Social anxiety d isord er 93. The d evelop m ent of the ability to ap p ly rea-
(G) Social (p ragm atic) com m u nication soning so that the child is not lim ited only by
d isord er p ercep tions.

84. A 6-year-old girl whose parents are going DIRECTIONS (Questions 94 through 99): For each of
through a divorce will not speak while at school. the multiple-choice questions in this section, select
the lettered answer that is the one best response in
85. A 9-year-old boy w ho frequ ently blu rts ou t each case.
com m ents in class w ithou t w aiting his turn to
be called on. 94. A 16-year-old girl is brou ght to you by her
m other because of d ropping grad es, apathy,
and p oor m otivation. You learn that she has
86. A 7-year-old boy w ho p erform s w ell in school
recently started sm oking m ariju ana on a
althou gh seem s to talk as if reciting a m ono-
regu lar basis. Which of the follow ing p atterns
logu e rather than interacting in conversation
w ou ld m ost su p p ort the d iagnosis of cannabis
and generally avoid s other child ren, bu t d oes
u se d isord er, severe?
not d isplay rep etitive or restricted interests.
(A) Over 12 m onths, u sing m ore than
87. An 8-year-old boy w ho is having d ifficu lty in intend ed and being u nsu ccessfu l at
school and avoid s interactions w ith his class- stopp ing, w ith a loss of her sum m er job
m ates and others for fear of em barrassm ent. and legal consequ ences from her u se.
(B) Over 12 m onths, having cravings to
u se, spend ing significant tim e obtaining
Questions 88 through 93
m arijuana, and m issing school in ord er
Match the age range w ith the corresp ond ing d evel- to sm oke.
opm ental m ilestone. (C) Over 12 m onths, continu ing to use
d esp ite recu rrent argu m ents, being
(A) Infant (0–18 m onths)
“high” w hile d riving, u sing d espite
(B) Tod d ler (18–36 m onths) know ing it is affecting her m otivation,
(C) Preschool age (3–6 years) need ing increased am ounts to get the
(D) School age (7–12 years) sam e “high,” and having ongoing
(E) Ad olescence (13–17 years) cravings w hen not u sing.
(D) Over 18 m onths, sm oking by herself bu t
88. Focu s on follow ing the ru les. need ing to sm oke m ore to achieve the
sam e effect.
89. Establishing self as au tonom ou s, sep arate (E) Over 18 m onths, sm oking m ost d ays
from caregiver, by p racticing leaving and p er w eek, m ultip le tim es p er d ay, and
retu rning to the caregiver. d eveloping w ithd raw al sym ptom s once
the m ariju ana is stopped .
16 1: Child a nd Ad ole s c e nt P sychia try

95. As the school p sychologist, you are asked to (C) N orm al variant in acad em ic attainm ent
see a fou rth grad er w ho has been consistently (D) Post-trau m atic stress d isord er (PTSD)
acting out in class. H e often lies abou t things (E) Vision d eficit
he has d one in class, such as trying to cheat
on tests, and he physically bu llies you nger 98. A 7-year-old girl is brou ght to the p ed iatri-
child ren. You learn that at hom e he practices cian d u e to w eight loss. She has been grad u -
shooting his BB gu n at squirrels as w ell as ally refu sing food s over the p ast several w eeks
at the fam ily d og. H e exp resses no concern to the point she is now barely even d rinking
for these creatu res nor rem orse at his behav- su pplem ental shakes. This has resu lted in sig-
ior. Which of the follow ing is the m ost likely nificant w eight loss and fatigu e. The girls tells
d iagnosis? you she is aw are she is losing w eight, that she
(A) Au tism sp ectru m d isord er w ou ld like to fit into the ou tfits she has, bu t
(B) Bip olar d isord er that she is afraid of vom iting if she eats the
w rong thing. Physical exam ination reveals a
(C) Child hood onset schizop hrenia
child in the 5th p ercentile of w eight, w hile her
(D) Cond u ct d isord er baseline had p reviou sly been in the 30th per-
(E) Op p ositional d efiant d isord er centile, and basic laboratory stu d ies reveal a
m ild m icrocytic anem ia. Which of the follow -
96. An 8-year-old boy is brou ght to the p ed iatri- ing is the m ost likely d iagnosis in this case?
cian for concerns that he has significant tan-
tru m s. The tantru m s occur at least three tim es (A) Anorexia nervosa
p er w eek (bu t frequ ently m ore) and inclu d e (B) Avoid ant/ restrictive food intake
p hysical aggression tow ard inanim ate objects d isord er (ARFID)
to the extent that he has d estroyed property. (C) Binge eating d isord er
H is parents note that the fam ily w alks on egg- (D) Bu lim ia nervosa
shells to p revent setting him off, and they are (E) Sp ecific p hobia
starting to feel “held hostage” by his chroni-
cally irate m ood . They believe this change in 99. A 7-year-old boy is brou ght to the p ed iatri-
behavior started a year ago, bu t has m arked ly cian becau se, for the p ast 6 m onths, he has
w orsened in the p ast several w eeks. Which of been telling his m other that he is a girl. Ini-
the follow ing is the m ost likely d iagnosis? tially his parents felt this w as “a p hase,” bu t
(A) Bip olar d isord er the patient has becom e increasingly d istressed
(B) Disru p tive m ood d ysregu lation d isord er and m ore insistent that he hop es his breasts
(DMDD) w ill grow d u ring p u berty. H e has been p u t-
ting on his old er sister ’s clothing, and prefers
(C) Generalized anxiety d isord er
p laying w ith her d olls rather than engaging in
(D) Major d ep ressive d isord er (MDD) rou ghhou sing w ith his p eers. H e has rep eat-
(E) Op p ositional d efiant d isord er (ODD) ed ly stated he d islikes his p enis and w ishes he
never had one. Physical exam ination reveals
97. A healthy 16-year-old stu d ent, w ho started norm al m ale genitalia of the app rop riate Tan-
school in the United States after fleeing a ner stage. Based on this inform ation, w hich of
w ar-engulfed region 1 year ago, is struggling the follow ing is the m ost likely d iagnosis?
to keep up in her classes, m ore so w ith Eng-
lish than m ath. Which of the follow ing best (A) Bod y d ysm orp hic d isord er
explains w hy her d ifficu lty is not d u e to a (B) Delu sional d isord er
learning d isability? (C) Gend er d ysp horia in child ren
(A) H earing d eficit (D) N onconform ity to gend er roles
(B) N eu rocognitive d isord er d u e to (E) Transvestic d isord er
traum atic brain injury
CHAPTER 2

Adult Ps yc ho patho lo g y
Que s tions

DIRECTIONS (Questions 1 through 120): For each (D) 50%


of the multiple-choice questions in this section select (E) 100%
the lettered answer that is the one best response in
each case. Questions 3 and 4

A 40-year-old man w ith schizophrenia comes for his


Questions 1 and 2 regular outpatient med ication management appoint-
A 22-year-old single m an is referred to you for a ments. H e reports that over the last w eek his intestines
1-year history of strange behavior characterized and heart have been removed . H e has subsequently
by talking to the television, accu sing local p olice of w ithd raw n and been staying in his basement apart-
bu gging his room , and carrying on conversations ment, avoid ing friend s and family members. When
w ith him self. H is m other d escribes a 3-year history asked about his lack of getting out in the w orld , he
of progressive w ithd raw al from social activities, responds, “What w orld? There is no w orld!”
and reports the p atient d ropp ed ou t of college and
since has been living in his room at hom e. Attem pts 3. Which of the follow ing term s best d escribes
to hold a job as a bu sboy at a local restaurant have this sym p tom ?
abruptly end ed after d isp u tes w ith the em p loyers. (A) Cap gras synd rom e
(B) Cotard synd rom e
1. What is the p revalence of this patient’s likely
(C) Folie à d eu x
illness in the general p op u lation?
(D) Fregoli d elu sion
(A) 0.1% (E) Major d ep ressive d isord er
(B) 1%
(C) 2% 4. Up on retu rning for a follow -u p visit 15 d ays
(D) 3% later, the p atient now claim s that “cyborg
(E) 5% alien robots” that look id entical to his p arents
have recently rep laced his m other and father.
2. The p atient’s m other inform s you that he has Which of the follow ing term s best d escribes
an id entical tw in brother. What is the likely this sym ptom ?
chance of the patient’s tw in also having the (A) Cap gras synd rom e
sam e illness? (B) Cotard synd rom e
(A) 1% (C) Delu sional d isord er
(B) 10% (D) Folie à d eu x
(C) 20% (E) Fregoli d elu sion

33
34 2: Ad ult P sychop a thology

5. A 32-year-old w om an w ith a history of d ep res- her blood alcohol is und etectable. H er urine
sion as a teenager, now 6 d ays p ostp artu m , is toxicology results com e back positive for can-
brou ght into the em ergency room by her hu s- nabis, w hich she confirm s she started sm oking
band . She has not slep t for the p ast several abou t 3 m onths ago. Which of the follow ing
d ays, even w hile the new born is nap p ing. w ou ld be the m ost ap p rop riate p rovisional
She has been irritable, and has been pacing in d iagnosis?
the m id d le of the night and w eeping, w hile
(A) Brief p sychotic d isord er
talking to no one in particular. Yesterd ay she
began to ignore the infant, bu t tod ay she vol- (B) Schizop hreniform d isord er
u nteered that the child “is the Antichrist and (C) Schizop hrenia
m ust be d estroyed .” She ad m its to you that (D) Su bstance-ind u ced m ood d isord er
she w ants to sm other the infant in a hu m ane (E) Su bstance-ind u ced p sychotic d isord er
w ay to p revent the ap ocalyp se. Which of the
follow ing d iagnoses is the m ost likely? 8. She w as started on appropriate treatm ent. One
(A) Bip olar d isord er year later this w om an retu rns to you r office
w ith her m other for follow -u p . H er sym p tom s
(B) Delu sional d isord er
rem itted w ithin a m onth. H ow ever, she has
(C) Major d ep ressive d isord er (MDD) w ith not d one w ell in her freshm an year and for the
p sychotic featu res p ast several m onths has continu ed to exp eri-
(D) Schizoaffective d isord er ence w orsening social isolation and am otiva-
(E) Schizop hrenia tion. While she has not u sed any su bstances
since she last saw you , she relu ctantly ad m its
Questions 6 through 8 to occasionally hearing the d evil com m u nicat-
ing w ith her. She tries to ignore the com m u ni-
An 18-year-old college freshm an w ithou t p rior p sy- cation, and has taken to arranging her books
chiatric history is brought to the em ergency room in a certain m anner to p revent his controlling
after being fou nd on her d orm itory roof d ressed only her thoughts. On her m ental statu s exam ina-
in her u nd erw ear, d esp ite freezing tem p eratu res. tion she m akes poor eye contact and her affect
Cam pu s p olice report she w as flapping her hand s is blu nted . H er m other rep orts that the p atient
and clim bing the banister on the roof, stating to an now rarely calls hom e, thou gh before she’d d o
u nseen other, “I w ill d o as you com m and —soar to so tw ice w eekly. Which of the follow ing is the
m y d eath to fu lfill your p rophecy!” She accu sed the m ost likely d iagnosis?
p olicem en of being “Satan’s horsem en” and cu rsed
as she w as being taken d ow n. H er room m ates con- (A) Brief p sychotic d isord er
firm that for the p ast 5 w eeks the patient has been (B) Schizop hreniform d isord er
acting bizarrely, and her speech has been increas- (C) Schizop hrenia
ingly d isorganized . You p lan to obtain collateral (D) Su bstance-ind u ced m ood d isord er
inform ation from her fam ily.
(E) Su bstance-ind u ced p sychotic d isord er
6. Which of the follow ing features w ou ld you be
9. A 42-year-old w om an p resents to a therap ist
m ost likely to find in her history?
w ith a history of d ram atic m ood sw ings since
(A) H ead trau m a early ad olescence, w here she w ill qu ickly
(B) Low intelligence becom e d eep ly d ep ressed for hou rs to d ays,
(C) N eglectfu l m other u su ally in resp onse to sep aration from a
loved one. She also ad m its to “rage attacks,”
(D) Physical or sexu al abu se
w here she w ill break item s, scream , or scratch
(E) Progressive social w ithd raw al herself su p erficially on her arm s. She inter-
m ittently binge d rinks and has frequ ently
7. She is ad m itted to the psychiatric unit. H er engaged in u np rotected sexu al intercou rse
p hysical exam ination is unrem arkable, and w ith new p artners. Others d escribe her as
Que s tions : 5–13 35

“reactive” and intense. Which of the follow - 11. Which of the follow ing d iagnoses is the m ost
ing d efense m echanism s d oes this p atient likely?
m ost likely em p loy?
(A) Catatonia associated w ith m ajor
(A) Altru ism d ep ressive d isord er
(B) Intellectu alization (B) Catatonia associated w ith schizop hrenia
(C) Sp litting (C) H yp othyroid ism
(D) Sublim ation (D) N eu rolep tic m alignant synd rom e
(E) Und oing (E) Unsp ecified catatonia

10. The fam ily of a 26-year-old patient w ith 12. Recognizing the clinical situ ation in front of
schizophrenia brings him in for follow -u p. you , you ad m it the p atient to the p sychiatric
H e w as initially d iagnosed at age 25 after a w ard for inability to care for self. Which of the
psychotic break that, in retrosp ect, follow ed a follow ing treatm ents w ou ld be best started
protracted cou rse of increasing isolation and im m ed iately?
am otivation. Desp ite d ifficu lties, he w as able
(A) Am itriptyline
to grad u ate from college and hold a fu ll-tim e
job by age 23. H e reports interm ittent hallu ci- (B) Electroconvulsive therapy (ECT)
nations, bu t has been able to m aintain ind e- (C) Lithiu m
pend ent living and p art-tim e em p loym ent. (D) Lorazep am
On exam ination, he is a d isheveled m an w ho (E) Sertraline
articu lates a m u ltitu d e of d elu sional beliefs
w ith a sophisticated vocabu lary. Which of 13. A 49-year-old bank teller w ith no know n psy-
the follow ing characteristics in this patient is chiatric history is referred to your office by
m ost strongly associated w ith a better overall her internist for an evaluation. For the p ast
prognosis? 2 m onths, she has been increasingly convinced
(A) Age at presentation that a w ell-know n m usic star is in love w ith
her and that they have had an ongoing affair.
(B) Gend er of p atient
She is w ell-groom ed , and there is no evid ence
(C) Insid iou s sym p tom onset of thou ght d isord er or hallucinations. She has
(D) Pred om inantly p ositive sym p tom s been fu nctioning w ell at w ork and in other
(E) Prem orbid fu nctioning social relationships. Which of the follow ing is
the m ost likely d iagnosis?
Questions 11 and 12
(A) Brief psychotic d isord er
A 36-year-old patient w ith no previou s psychiatric (B) Delusional d isord er
history is brou ght to the em ergency room by his (C) Paranoid p ersonality d isord er
fam ily. For the p ast m onth he has not been eating (D) Schizop hrenia
regu larly and has isolated him self in the ap artm ent
(E) Schizop hreniform d isord er
w here he lives alone. Tw o m onths ago he stop p ed
fishing, and rep orted feeling like a bu rd en on his
Questions 14 and 15
fiancée. On m ental statu s exam ination, he d isp lays
p sychom otor retard ation and rarely blinks. H e A 46-year-old d ivorced w om an w ith a history of
resp ond s in the negative to any qu estion asked . m ajor d ep ressive d isord er is ad m itted to you r
On physical exam ination, he app ears in no acute inp atient p sychiatric u nit follow ing an intentional
d istress, is afebrile, m arginally hypotensive, and acetam inop hen overd ose. She has had m u ltip le
m ild ly tachycard ic. H e resists you r m otions w ith psychiatric hospitalizations w ith similar presen-
strength proportional to w hat you exert and crud ely tations. The patient reports a 3-w eek history of
m im ics your m ovem ents. H is laboratory stu d ies are insom nia, d ifficulty concentrating, low energy,
u nremarkable. hop elessness, and a d ecreased app etite. She has been
36 2: Ad ult P sychop a thology

unable to w ork recently because of her d epression Questions 16 and 17


and has lost interest in activities she once enjoyed .
She has no history of manic episod es. H er past psy- A 34-year-old w hite m ale is referred by his p rim ary
chiatric history is significant for a prior episod e of care physician for d epression. Upon initial inter-
d epression after the birth of her second child . She view , he com plains of feeling “blue.” H is m ental
has u nd ergone treatment w ith several ad equ ate tri- statu s exam ination reveals a d isheveled ap pearance,
als of m ed ications, inclu d ing au gm entation w ith d ep ressed m ood , p sychom otor retard ation, and su i-
lithium, yet she has continued to have resid ual cid al id eation w ithout plan. H is thought processes
sym ptoms of d epression. She has never been treated are significant for thou ght blocking and som e slow -
w ith psychotherapy. In the past she has experienced ing. Deficits w ith rem ote and short-term m em ory
au d itory hallucinations w hen her d epression w as are noted . Ju d gm ent and insight are also im p aired .
most severe. H er med ical history is significant for You r provisional d iagnosis is m ajor d epressive
hyp othyroid ism , w hich is ad equately m anaged w ith d isord er.
levothyroxine. She also had prior surgery after a leg
fracture from a suicid e attem pt w here she jumped 16. Which type of sleep d istu rbance you w ould
ou t of a w ind ow . m ost exp ect to see in this patient?
(A) Decreased resp onse to sed ative d ru gs
14. Which of the following factors would be most (B) Early m orning aw akening
influ ential in you r recom m end ing electro-
(C) Increased rap id eye m ovem ent (REM)
convu lsive therapy (ECT) for this patient?
stage latency
(A) Concu rrent thyroid d ysfu nction w ith (D) Sleep ing too d eep ly (d ifficu lty being
ad equate treatm ent aw akened )
(B) H istory of associated p sychotic (E) Sleep ing too lightly (aw akened too
sym ptom s w ith prior d epression easily)
(C) H istory of p erip artu m d ep ression
(D) Severe d ep ression that has not 17. Which of the follow ing m etabolic changes
resp ond ed to several m ed ications w ou ld be m ost likely fou nd in this p atient?
(E) Treatm ent-resistant d ep ression w ith (A) Decreased m onoam ine oxid ase (MAO)
recu rrent su icid al id eation activity
(B) Increased catecholam ine activity
15. The ind ications for ECT are d iscu ssed w ith
(C) Increased cortisol secretion
the patient, as w ell as the risks, benefits, and
(D) Increased sex horm ones
sid e effects. She asks appropriate qu estions
and consents to ECT. Fu rther m ed ical history (E) Increased im m u ne fu nctions
is exp lored p rior to beginning the p roced u re.
Which of the follow ing cond itions w ou ld be a Questions 18 and 19
relative contraind ication to p roceed ? A 35-year-old m an is being treated for m ajor d ep res-
(A) Coronary artery d isease, w ith a sive d isord er w ith paroxetine. H e has m issed his last
m yocard ial infarction 2 years ago tw o appointm ents, leaving m essages telling you that
(B) Im p lanted p acem aker for p eriod ic “I’ve been sp ectacular!” H is w ife has since called
arrhythm ia you to rep ort that her hu sband has been sp end ing
m oney on frivolou s item s and overd rew the cou -
(C) Incid entally fou nd frontal m eningiom a
p le’s bank account. She states that her hu sband now
m easu ring 10 cm in d iam eter
quickly becom es agitated and angry. During the
(D) Second -trim ester p regnancy interview in you r office, the p atient qu estions you r
(E) Trau m atic brain inju ry su stained in cred entials and accu ses you of being m ore loyal to
teenage years his w ife than to him . Most of the interview is spent
interru p ting the p atient as you try to d ecip her his
Que s tions : 14–23 37

rap id sp eech. In his rant he threatens to cu t the brake 20. Which of the follow ing d iagnoses is m ost
lines on his m other-in-law ’s car becau se he feels she likely?
has been intru d ing in his m arriage.
(A) Bip olar I d isord er
18. Which of the follow ing is the next m ost (B) Bip olar II d isord er
ap propriate step ? (C) Cyclothym ic d isord er
(D) Dou ble d ep ression
(A) Discharge the p atient hom e as he
refu ses ad m ission, bu t see him (E) Persistent d ep ressive d isord er
tom orrow
21. The patient returns w ithin 4 w eeks, and he now
(B) Escort the p atient (w ith p olice assistance
d escribes w orsening d epression for the past
as need ed ) to the nearest em ergency
3 w eeks, w ith ongoing insomnia, poor appetite,
room
little energy, anhed onia, and poor concentra-
(C) Inform the m other-in-law that she is in tion. H e also ad mits to passive suicidal ideation
d anger w ithout plan. Which of the follow ing w ould be
(D) Inform the p olice that a threat has been the most likely d iagnosis?
m ad e against the m other-in-law
(A) Bip olar I d isord er
(E) Tell the w ife to have the m other-in-law
stay w ith her (B) Bip olar II d isord er
(C) Cyclothym ic d isord er
19. After ad d ressing the above situ ation, the (D) Dou ble d ep ression
p atient is su bsequently started on valp roic (E) Persistent d ep ressive d isord er
acid . Which of the follow ing ad d itional p har-
m acologic interventions w ould be the m ost Questions 22 and 23
ap prop riate?
A 26-year-old fem ale p resents to the psychiatric
(A) Check a seru m p aroxetine level em ergency d ep artm ent in an acu tely d istressed ,
(B) Cross tap er the p aroxetine to nervou s state. She com p lains of terrible anxiety,
nortrip tyline and the em ergency d ep artm ent staff is u nable to
(C) Discontinu e the p aroxetine calm her d ow n or gain an ad equ ate history from
(D) Initiate bu p rop ion the patient. On physical exam ination, she is slightly
(E) Initiate lorazep am d iaphoretic, tachycard ic, and her pup ils are m ild ly
d ilated . She is on no m ed ications.
Questions 20 and 21
22. Which of the follow ing tests w ould be the
A new patient com es to you r office for a p sychiat- m ost useful in d eterm ining the etiology of her
ric evalu ation. H e d escribes m any years w here he sym ptom s?
experiences episod es of 5 to 7 d ays of feeling very
(A) Blood glu cose
d epressed , w ith insom nia, low energy, and poor
concentration. H e d enies any suicid al id eation or (B) Catecholam ine m etabolites
prior attem p ts. Up on further history he reveals (C) Electrocard iogram (ECG)
ad d itional period s w here “I feel the opp osite,” w ith (D) Thyroid fu nction
d ecreased sleep, elevated energy, feeling “on top of (E) Urine toxicology
the w orld ,” and increased sex d rive. These tim es last
1 to 2 w eeks, and he d enies any p aranoia, d elu sions, 23. Which of the follow ing su bstances w ou ld be
or hallu cinations. Desp ite these recu rrent ep isod es, m ost likely to app ear on her urine toxicology
he has alw ays been able to fu nction ad equ ately, resu lts?
although they rem ain d istressing to him . H e d enies
any d ru g or alcohol u se, and he has no significant (A) Caffeine
m ed ical p roblem s. (B) Cannabis
38 2: Ad ult P sychop a thology

(C) Cocaine the staff reports that she has becom e extremely d oc-
(D) Op iates ile and d isplays very little emotion. She has a large
(E) LSD appetite and compu lsively puts both food and non-
food item s in her mouth. She also d isplays sexual
Questions 24 through 26 d isinhibition, often w alking out of her room w ithout
her pants on.
A 46-year-old m an is ad m itted to the hospital for
elective cholecystectom y. On hosp ital d ay 4, he is 27. Which of the follow ing clinical cond itions best
noted to be afebrile, but acutely d iaphoretic, tachy- d escribes her behavior?
card ic, hypertensive, trem ulou s, and agitated . H e
(A) Arnold –Chiari synd rom e
tears ou t his sutures and insists on leaving against
m ed ical ad vice (AMA). H e is ap parently hallu cinat- (B) Klü ver–Bu cy synd rom e
ing, ju d ging from his insistence that he be allow ed (C) Möbiu s synd rom e
to “squ ash those bu gs on the w all” (there are none). (D) Pick d isease
(E) Pu nch-d ru nk synd rom e
24. Which of the follow ing d iagnoses is the m ost
likely? 28. N eu roim aging w ou ld m ost likely show d am -
(A) Brief psychotic d isord er age to w hich tem p oral lobe stru ctu re?
(B) Delirium (A) Am ygd ala
(C) Delu sional d isord er (B) H ip p ocam p u s
(D) Fu nctional neu rological sym p tom (C) Inferior horn of the lateral ventricle
d isord er (D) Insu la
(E) Postoperative sepsis (E) Su p erior tem p oral gyri

25. Which of the follow ing m ed ications w ou ld be Questions 29 and 30


m ost app rop riate to treat his cond ition?
A 22-year-old w oman presents w ith fatigue for 4
(A) Diazep am months. She also reports irritability and poor energy,
(B) Disulfiram w ith dismal sleep and poor concentration. She has
(C) Lorazep am maintained a rigorous exercise routine, as she states
(D) Phenobarbital it makes her feel good to run off her boyfriend’s cook-
(E) Phenytoin ing. Her oral intake has sharply d ropped and she has
lost a significant amount of w eight, but she explains
26. If u ntreated , w hat w ou ld be his m ost likely that she “hasn’t been as hungry” d ue to increased
mortality rate? stress. H er thyroid stud ies are normal, her CBC
reveals anemia, and she is not pregnant, in fact stat-
(A) 5% ing that she hasn’t had a period in several months.
(B) 10%
(C) 30% 29. Up on fu rther qu estioning, w hich of the
(D) 50% follow ing qu alities w ou ld you m ost exp ect to
find in her social history?
(E) 60%
(A) H er p arents likely p raise her su ccesses.
Questions 27 and 28 (B) She has a legal history significant for
A 62-year-old w om an presents to the nu rsing home reckless d riving.
w here you w ork as a consulting psychiatrist. She has (C) She has asp ired to be a m od el.
a history of a bilateral tem poral lobectomy for intrac- (D) She is of low socioeconom ic statu s.
table seizures. After a few w eeks at the new facility, (E) She is a scholastically gifted
in ad d ition to her short-term m em ory d ifficulties, perfectionist.
Que s tions : 24–34 39

30. Which of the follow ing d iagnoses is the m ost (D) Major d ep ressive d isord er
likely? (E) N orm al reaction
(A) Anorexia nervosa
33. Seven m onths later, your patient’s m ood has
(B) Avoid ant/ restrictive food intake
w orsened , and he feels w orthless and hop e-
d isord er (ARFID)
less that he w ill never find another girlfriend .
(C) Bu lim ia nervosa H e has lost a few p ou nd s and has not resu m ed
(D) Exercise-ind u ced am enorrhea d ating, p referring to isolate him self in his
(E) Obsessive-com p u lsive d isord er (OCD) stu d io apartm ent. H e is still feed ing his d og,
bu t he has had to m iss increasing am ou nts of
31. A 23-year-old college stu d ent has been su f- w ork becau se he ju st can’t get him self to get
fering w ith frequ ent ep isod es of feeling read y for class. Which of the follow ing is the
“u tter d oom ” for the p ast 3 m onths. Du ring m ost likely d iagnosis?
these p eriod s, he also experiences trem u lou s-
(A) Acu te stress d isord er
ness, sw eating, d izziness, and tingling in his
extrem ities. H e rep orts having these attacks at (B) Ad ju stm ent d isord er
least once a w eek and is now becom ing fear- (C) Generalized anxiety d isord er
fu l of attend ing classes lest he has an ep isod e. (D) Major d ep ressive d isord er
Which of the follow ing m ed ications w ou ld be (E) N orm al reaction
the m ost appropriate for im m ed iate relief of
his sym ptom s? 34. A 21-year-old college stu d ent is brou ght to
(A) Alprazolam (Xanax) you r office by her p arents after com p leting
her spring sem ester w ith u ncharacteristically
(B) Chlord iazep oxid e (Libriu m )
low grad es. H er p arents rep ort that since
(C) Divalp roex sod iu m (Dep akote) the sp ring session end ed 2 m onths ago their
(D) Fluoxetine (Prozac) d au ghter has been staying in her room , bu t
(E) Phenelzine (N ard il) is irritable w hen they try to engage her in
conversation. She is not interested in fam ily
Questions 32 and 33 barbecu es or being a cou nselor for the p ark
d istrict soccer cam p . After her p arents leave
A 36-year-old grad u ate stu d ent com es to you r
the exam ination room , your patient d iscloses
office because of d ifficulty sleep ing since breaking
she has been feeling d epressed since a m onth
up w ith his fiancée 5 m onths ago. H e is sp end ing
before final exam inations, and her concen-
at least 2 hours in bed thinking abou t his ex-fiancée
tration and sleep have been “horrible.” She
and w hat he cou ld have d one d ifferently. H is con-
had su icid al thoughts after she got her report
centration is w orsening and he’s having d ifficu lties
card back, but d enies them cu rrently. She also
com pleting his coursew ork. While he feels d ow n, he
ad m its to d rinking alcohol to m ake herself feel
is not su icid al, and he is seeking su p p ort of friend s
better, and her intake ranges from 3 to 4 beers
in the p ost-breaku p p eriod . H e is attend ing m ost
or shots of liqu or m ost d ays of the w eek. Asid e
classes. H e d escribes feeling tenser overall, espe-
from likely alcohol u se d isord er, w hich of the
cially w hen he is at a bar. H e is having m ore trouble
follow ing d iagnoses is the m ost ap p rop riate at
relaxing and has noted the new onset of low back
this tim e?
and should er pain.
(A) Acu te stress d isord er
32. Which of the follow ing is the m ost likely (B) Ad ju stm ent d isord er
d iagnosis? (C) Am otivation synd rom e
(A) Acu te stress d isord er (D) Major d ep ressive d isord er (MDD)
(B) Ad ju stm ent d isord er (E) Su bstance-ind u ced m ood d isord er
(C) Generalized anxiety d isord er
40 2: Ad ult P sychop a thology

35. A 37-year-old w om an, w ho w orks the night (A) Bipolar d isord er w ith psychotic features
shift at a local grocery store taking inventory, (B) Major d ep ressive d isord er w ith
reports that her child hood and college years p sychotic featu res
w ere u neventfu l bu t hap p y. She sp end s m ost (C) Schizoaffective d isord er
of her tim e alone w hen she is not at w ork.
(D) Schizop hrenia
She d oes not ventu re out of her house and
her social contacts are lim ited to w ork-related (E) Schizop hreniform d isord er
interactions w ith cow orkers. She is an avid
p lant lover, and she spend s m ost of her free 38. A 42-year-old bu siness execu tive p resents for
tim e taking care of her ind oor nursery. She his first contact w ith a m ental health provid er.
reports that she is quite content w ith her life. H e rep orts that for the last 4 m onths he has
Which of the follow ing is the m ost likely d iag- been feeling d ep ressed . H is low -energy level
nosis for this p atient? and p oor m otivation are affecting his job
p erform ance and the CEO of his com p any
(A) Agorap hobia ad vised him to “take a cou p le of w eeks off.”
(B) Au tism sp ectru m d isord er The p atient rep orts that he started feeling
(C) Avoid ant p ersonality d isord er d ow n w hen his w ife d iscovered that he w as
(D) Schizoid p ersonality d isord er involved in his third extram arital affair. Since
then he has m oved into a sm all apartm ent by
(E) Schizotyp al p ersonality d isord er
him self. H e is sleep ing alm ost 12 hou rs every
night, has a p oor app etite, and is exp erienc-
36. An 18-year-old , p regnant, hu m an im m u no-
ing financial d ifficu lty d u e to ind iscrim inate
d eficiency virus (H IV)–p ositive w om an pres-
p u rchases. H e lam ents the loss of his form er
ents for the treatm ent of op iate u se d isord er.
self. H e reports that he u sed to have several
She reports u sing heroin for the last 8 m onths
p eriod s of tim e p er year, lasting for u p to
w ith su bstantial (bu t u nsu ccessfu l) efforts to
1 w eek, w hen he only need ed 4 to 5 hou rs
qu it for the last 4 m onths. She is now hom eless
of sleep, had large am ou nts of energy, spoke
and has recently been arrested for shop lifting.
qu ickly, and cou ld “p arty all night and w ork
Which of the follow ing pharm acological ther-
all d ay.” Which of the follow ing d iagnoses is
ap ies w ould be the m ost ap prop riate at this
m ost likely in this case?
tim e?
(A) Bipolar I d isord er
(A) Bu prenorphine
(B) Bip olar II d isord er
(B) Clonid ine
(C) Bord erline p ersonality d isord er
(C) H alop erid ol
(D) Major d ep ressive d isord er
(D) Methad one
(E) N arcissistic p ersonality d isord er
(E) N aloxone
39. A 52-year-old w om an w ho has been treated
37. A 47-year-old w om an transfers care to you r
w ith m ed ication for 3 years for a chronic m ood
office. She has a long history of recu rrent
d isord er reports d ry m ou th, trou ble u rinating,
d epression. She also has a history since her
and occasional d izziness w hen she gets ou t
early tw enties that is significant for interm it-
of bed . Which of the follow ing m ed ications is
tent com m and au d itory hallucinations and
she m ost likely being prescribed ?
p ersecu tory d elu sions that are often p res-
ent even w hen her affective sym p tom s are in (A) Divalp roex sod iu m
rem ission. She hold s a p art tim e job and lives (B) Flu oxetine
ind ep end ently. She has been com p liant w ith (C) Im ip ram ine
m ed ications and has not been hosp italized
(D) Lithiu m
since a su icid e attem p t over 7 years ago. Which
of the follow ing is the m ost likely d iagnosis? (E) Phenelzine
Que s tions : 35–45 41

Questions 40 and 41 and lau ghs w hen she cannot recall any of the three
item s you ask her to rep eat. H er attention is intact.
A 72-year-old m an is brou ght in by his w ife to you r She has som e d ifficulties nam ing objects. She states
geriatric p sychiatry clinic. The p atient’s w ife is con- her m ood as “good ” and her affect is euthym ic and
cerned abou t his progressive confu sion over the last fu ll range.
year. She is p articu larly d istressed that he rep eat-
ed ly asks the sam e qu estions throu ghou t the d ay. 42. Which of the follow ing is the m ost ap propriate
H er hu sband has becom e increasingly u nstead y on p rovisional d iagnosis?
his feet and need s to u se a w alker w hen they go ou t.
She w ond ers if these sym ptom s m ay be related to (A) Deliriu m
the m eningitis he su ffered from 3 years ago. (B) Major d ep ressive d isord er
(C) Major neu rocognitive d isord er
40. Which of the follow ing sym p tom s or signs (D) Mild neu rocognitive d isord er
w ou ld m ost likely be fou nd in this p atient?
(E) N orm al p ressu re hyd rocep halu s
(A) Elevated opening p ressu re u p on lu m bar
p unctu re 43. A w orkup is p erform ed . H er laboratory val-
(B) Frontal release signs u es are u nrem arkable, and a CT scan of her
(C) H istory of incontinence brain d em onstrates d iffu se cortical atrop hy
and norm al ventricles. Which of the follow ing
(D) Ocu lom otor d ifficu lties
d iagnoses is m ost likely?
(E) Perseveration
(A) Major neu rocognitive d isord er d u e to
41. Which of the follow ing w ou ld m ost likely Alzheim er d isease
be fou nd on neu roim aging w ith com p u ted (B) Major neu rocognitive d isord er d u e
tom ography (CT) in this p atient? to Lew y bod y d isease
(A) Cerebellar atrop hy (C) Major neu rocognitive d isord er d u e
to trau m atic brain inju ry
(B) Dilated lateral ventricles
(D) Major neu rocognitive d isord er d u e to
(C) Focal su bcortical hyp ointensities
vascular d isease
(D) Frontoparietal atrop hy
(E) Unsp ecified m ajor neu rocognitive
(E) Frontotem p oral atrop hy d isord er

Questions 42 through 44 44. If the p atient’s CT scan revealed a p rep ond er-
A 69-year-old w om an is brou ght to you r geriatric ance of atrophy in the frontal and tem p oral
psychiatry clinic by her hu sband . She d enies any- regions, w hich of the follow ing behavioral
thing is w rong, but he is concerned that for the past m anifestations w ou ld be m ost likely?
year or so she has been less able to recall things she (A) Decline in object nam ing
read s abou t in her m agazines. She u sed to be in
(B) Mem ory p roblem s
charge of grocery shopping, but over the past year
(C) Poor organization
or so she has been leaving things off the list, and
her husband has taken over the job since the p atient (D) Social d isinhibition
can’t seem to organize it anym ore. She is no longer (E) Word find ing d ifficu lty
able to keep track of the cou p le’s finances, and there
have been several occasions w hen her son fou nd 45. A 48-year-old m an has been d rinking u p to
her w and ering ou tsid e, a nu m ber of hou ses aw ay. 6 beers p er night d u ring the w eek and u p to
She is otherw ise healthy, and takes a m ultivitam in 12 beers a night on the w eekend . A year ago,
d aily. H er physical exam ination is unrem arkable. he had his d river ’s license su spend ed for
Mental status exam ination reveals a healthy ap pear- d ru nk d riving. H is m arriage is failing becau se
ing w om an w ho is coop erative w ith you r qu estions, of these d ifficulties. Last m onth, he w as d iag-
nosed w ith a gastric u lcer as a result of alcohol
42 2: Ad ult P sychop a thology

consum ption. H e ad m its to an alcohol p rob- becom es “ju m p y” w henever she hears the
lem and has tried to stop on nu m erou s occa- train going by her hom e. Which of the follow -
sions. H e find s that he experiences insom nia if ing d iagnoses is the m ost ap p rop riate for this
he d oes not d rink for m ore than 2 d ays. Which p atient?
of the follow ing featu res of this case su ggests a
(A) Acu te stress d isord er
severe versus m od erate alcohol u se d isord er?
(B) Ad ju stm ent d isord er
(A) H igh qu antity of alcohol consu m ed on a (C) Generalized anxiety d isord er (GAD)
regu lar basis
(D) Posttrau m atic stress d isord er (PTSD)
(B) H istory of legal problem s
(E) Major d ep ressive d isord er (MDD)
(C) Inability to stop d rinking d esp ite
know ing the harm fu l effects Questions 48 and 49
(D) Marital conflict d u e to d rinking
(E) N u m ber of d ifficu lties he is A 40-year-old w om an w ith a 20-year history of
experiencing schizophrenia p resents to the p sychiatric em ergency
d ep artm ent after a su icid e attem p t by carbon m on-
46. A 26-year-old com pu ter p rogram m er w ithou t oxid e poisoning. She has ongoing com m and hal-
p reviou s p sychiatric history has been m ar- lu cinations to harm herself, and has acted on them
ried for 4 years. H is w ife is exp ecting their at least 10 tim es since her initial d iagnosis. She also
first child . She rep orts that 3 m onths ago the has p ersistent d elu sions that she is resp onsible for
p atient becam e p reoccup ied w ith the id ea w orld d isasters, w hich is w hy she m u st elim inate
that she becam e pregnant by another m an. herself as the sou rce of pain and su ffering in the
Du ring this tim e, he began m issing w ork and w orld . She has been tried on both typ ical and atyp i-
isolated him self in his bed room . H is affect has cal antipsychotics, thou gh none have been effective
p rogressively becom e m ore blu nted . Recently, in fu lly elim inating her p sychotic sym p tom s. H er
he believes that his w ife is carrying a child level of fu nctioning rem ains p oor, and she p resently
conceived by extraterrestrial forces. H e u rged lives in a grou p hom e.
her to have an abortion and she refu sed .
The p atient d enies any history of significant 48. Which of the follow ing is this p atient’s m ost
alcohol or illicit su bstance u se and his recent likely lifetim e risk of su icid e?
m ed ical evalu ation w as w ithin norm al lim its. (A) 1%
Which of the follow ing is the most app ropriate (B) 5%
d iagnosis?
(C) 10%
(A) Brief p sychotic d isord er (D) 30%
(B) Delu sional d isord er (E) 50%
(C) Major d ep ressive d isord er w ith
p sychotic featu res 49. Which of the follow ing m ed ications w ou ld
(D) Schizop hreniform d isord er be the m ost ap p rop riate to p rescribe for this
(E) Schizop hrenia p atient?
(A) Clozap ine
47. A 27-year-old w om an w as involved in a train (B) Flu p henazine
d erailm ent 2 w eeks ago. Since that event, she
(C) H alop erid ol
has felt d ow n, has not slept w ell, has exp e-
(D) Lithiu m
rienced rep eated and intru sive thou ghts of
the accid ent, and has recu rrent nightm ares. (E) Zip rasid one
Lately, she has changed her com m u te to avoid
the train, even thou gh this ad d s 3 hours to her 50. A 27-year-old internal m ed icine resid ent gen-
com m ute d aily. When on the train she has an erally d islikes w orking in an ou tp atient clinic.
acu te increase in her anxiety. She also often Tod ay, how ever, he is looking forw ard to his
Que s tions : 46–55 43

clinical w ork becau se one of his app ointm ents 53. In this p atient, w hich typ e of p sychotherap y
is a follow -u p visit for a single, attractive w ou ld be the m ost efficaciou s?
31-year-old w om an w ho is finishing her anti-
(A) Cognitive-behavioral therap y (CBT)
biotic regim en for treatm ent of p neu m onia.
Which of the follow ing term s bests d escribes (B) Dialectical behavioral therap y (DBT)
this d octor ’s resp onse to his patient? (C) Grou p therap y
(D) Psychoanalysis
(A) Cou ntertransference
(E) Psychod ynam ic therap y
(B) Em pathy
(C) Id entification Questions 54 and 55
(D) Projection
(E) Transference A 34-year-old w om an p resents for the treatm ent of
her severe, m ed ication-refractory, m ajor d epressive
Questions 51 through 53 d isord er. After review ing her p ast p sychiatric his-
tory and interview ing the patient, you conclu d e that
A 31-year-old w om an w as ad m itted to a p sychiatric she w ou ld be app ropriate for ECT.
u nit after attem pting suicid e by overd ose. She had
recently broken u p w ith her boyfriend of 4 m onths. 54. In d iscu ssing the effects of ECT w ith the
She also d escribes episod es of m ood lability, m arked patient, w hich of the follow ing should you tell
by feelings of d ep ression and anger d irected tow ard her is the m ost likely sid e effect?
the psychiatric resid ent w ho com pleted the rota-
(A) Am nesia
tion 5 d ays after her ad m ission. When the resid ent
left, she rep orted that she w as having u rges to cu t (B) Asp iration
her w rists. She has had nu m erou s ad m issions for (C) Card iac arrhythm ias
su icid al gestu res and a lifelong history of tu m u ltu - (D) Convu lsion fractu res
ou s rom antic relationships. As the patient nears her (E) Psychosis
d ischarge d ate, she rep orts that “all the staff hates
m e except for Dr. Johnson.” Dr. Johnson, a m ed ical 55. For the best p ossible ou tcom e, how m any
stu d ent, had a recent d ifference of opinion w ith the treatm ents is this patient likely to requ ire?
nursing staff regard ing the patient’s d ischarge.
(A) 2
51. Which of the follow ing d iagnoses is the m ost (B) 4
likely for this patient? (C) 10
(D) 15
(A) Bord erline personality d isord er
(E) 20
(B) Cyclothym ic d isord er
(C) H istrionic p ersonality d isord er
Questions 56 and 57
(D) Major d ep ressive d isord er (MDD)
(E) Schizoaffective d isord er A 38-year-old w om an p resents to your clinic telling
you that she has had d istu rbing, recu rrent thou ghts
52. Which of the follow ing w ou ld be the m ost abou t harm ing her 7-m onth-old infant. She im ag-
ap p rop riate ou tp atient treatm ent for this ines u sing a knife to stab her child , bu t she has no
p atient? d esire to hu rt her child . As a resu lt of having these
d istressing thou ghts, she has rem oved all sharp
(A) Antid epressants objects from her kitchen. Because of this, she has not
(B) Benzod iazepines been able to p rep are m eals at hom e and has chosen
(C) Grou p p sychotherap y to buy fast food or take ou t for the fam ily m eals. She
(D) Ind ivid u al psychotherap y feels very anxiou s regard ing these, althou gh she has
(E) Mood stabilizers not shared these thou ghts w ith her hu sband .
44 2: Ad ult P sychop a thology

56. Which of the follow ing d iagnoses w ou ld be 59. The p atient is su bsequ ently ad m itted , bu t tries
the m ost likely for this patient? to hit several staff as they tend to her; she also
rep eated ly tries to get ou t of bed and d em and s
(A) Delusional d isord er
to be let go. Which of the follow ing w ou ld be
(B) Obsessive-com p u lsive d isord er the m ost appropriate im m ed iate pharm aco-
(C) Obsessive-com p u lsive p ersonality logic m anagem ent for this p atient?
d isord er
(A) Dip henhyd ram ine
(D) Schizophrenia
(B) H alop erid ol
(E) Schizotyp al p ersonality d isord er
(C) Lorazep am
57. Which of the follow ing w ould be the m ost (D) Phenobarbital
ap propriate first-line p harm acotherap y for (E) Valp roic acid
this cond ition?
60. Which of the follow ing w ou ld be the m ost
(A) Lithium
ap p rop riate long-term treatm ent of this
(B) Lorazep am p atient?
(C) Flu voxam ine
(A) Antibiotic therap y
(D) H alop erid ol
(B) Chest x-ray every 6 m onths
(E) N ortrip tyline
(C) CT head every 6 m onths
Questions 58 through 61 (D) Ind w elling Foley catheter
(E) Intravenou s flu id s
A 72-year-old w om an is brou ght to the em ergency
d epartm ent from a nu rsing hom e for poor oral 61. Which of the follow ing is the m ost likely
intake. She is afebrile, has a pu lse of 95, and a blood 6-m onth m ortality of the d iagnosis in Qu es-
pressu re of 90/ 60. Mental statu s exam ination (MSE) tion 58?
reveals an aw ake and alert, bu t frail, m alnou rished
and d ehyd rated w om an w ho is oriented to p er- (A) 5%
son only. She rep orts that the p resid ent is George (B) 15%
Bu sh. She is easily d istracted and cannot recall any (C) 20%
of three item s after a few m inu tes. She is irritable (D) 35%
and sw ings at the staff w hen they try to insert an IV.
(E) 45%
The team starts IV flu id s as blood and u rine are sent
to the laboratory. A chest x-ray is u nrem arkable, as
Questions 62 and 63
is the head CT. One hou r later, she is calm er and
rep orts the correct d ay, tim e, and p lace; she is less You are a research psychiatrist cond u cting a d ou ble-
d istractible. blind , p lacebo-controlled trial of a new antid ep res-
sant. You have enrolled 200 p atients in the stud y, all
58. Labs are rem arkable for leu kocytosis and d irty of w hom m eet the criteria for u ncom plicated m ajor
u rinalysis. Asid e from cystitis, w hich of the d ep ression. You p lan to rand om ize 100 p atients to a
follow ing is the m ost ap p rop riate d iagnosis p lacebo m ed ication and the other 100 p atients to the
for the p atient? exp erim ental antid ep ressant.
(A) Deliriu m
62. Of the 100 p atients taking the placebo,
(B) Major d ep ressive d isord er
ap p roxim ately how m any p atients w ou ld be
(C) Major neu rocognitive d isord er d u e to exp ected to im p rove after 6 w eeks?
Alzheim er d isease
(D) Substance-ind u ced neu rocognitive (A) 5
d isord er (B) 10
(E) Unsp ecified neu rocognitive d isord er (C) 30
Que s tions : 56–68 45

(D) 50 66. A 32-year-old single su ccessfu l Wall Street


(E) 70 execu tive tells you that on w eekend s he likes
to visit a d om inatrix. H is regular, paid app oint-
63. Of the 100 p atients taking the exp erim ental m ent w ith this p erson is d escribed as hum ili-
antid ep ressant (assu m ing this d ru g is as effi- ating and som ew hat p ainfu l bu t also very
cacious as stand ard antid epressants), ap proxi- sexually arou sing. While he ad m its that this
m ately how m any p atients w ou ld be exp ected behavior “m ay be w eird to som e p eop le,” he
to im p rove after 6 w eeks? enjoys it, can afford it, and it d oesn’t interfere
w ith his w ork or other hobbies. Which of the
(A) 10 follow ing d iagnoses is the m ost ap p rop riate?
(B) 30
(A) Fetishistic d isord er
(C) 50
(B) Frotteu ristic d isord er
(D) 60
(C) N o d iagnosis
(E) 80
(D) Sexu al m asochism d isord er
Questions 64 and 65 (E) Sexu al sad ism d isord er

You r p atient is a 38-year-old d ivorced Catholic Questions 67 and 68


m ale w ith a m onth long history of d ep ressed m ood ,
anhed onia, initial insom nia, low energy, and p oor A 28-year-old w om an com p lains of falling asleep
app etite. H e ad m its to su icid al id eation w ith a d u ring the d ay. This problem has been occu rring for
plan to overd ose for the past several d ays, and he 3 m onths and is now interfering w ith her w ork as a
has gathered p ills this m orning. Up on fu rther his- telephone operator as she falls asleep tw o or three
tory, he ad m its to sim ilar sym ptom s 5 years prior, tim es a d ay w hile speaking w ith cu stom ers. At tim es,
also w ith a p rior su icid e attem p t by overd osing. H e she find s herself falling asleep at her d esk, and she
d rinks 1 beer w eekly and d enies illicit d ru gs. H e is is aw akened w hen her head hits the com p u ter con-
on no other m ed ications except for a m u ltivitam in. sole in front of her. Od d ly enou gh, she reports, this
H e is su bsequ ently ad m itted after being m ed ically can happen w hen she becom es particularly stressed
cleared . ou t, for exam ple, if she is m anaging m any calls.
The p atient also states that this d istu rbance has not
64. What p ercentage of p atients w ith this illness im p roved d esp ite her sleep ing 8 hou rs each night.
eventu ally com m it su icid e?
67. Which of the follow ing is the m ost likely
(A) 1% d iagnosis?
(B) 5%
(A) Circad ian rhythm sleep–w ake d isord er
(C) 10%
(B) H yp ersom nolence d isord er
(D) 20%
(C) Insom nia d isord er
(E) 30%
(D) N arcolep sy
65. Which of the follow ing characteristics of this (E) N ightm are d isord er
patient is the nu m ber one p red ictor of a fu tu re
com p leted su icid e? 68. Which of the follow ing is the m ost app rop ri-
ate p harm acotherap y for this cond ition?
(A) Age
(B) Gend er (A) Bu p rop ion (Wellbu trin)
(C) Previous attem pt (B) Flu oxetine
(D) Relationship statu s (C) Lorazep am
(E) Religion (D) Methylp henid ate (Ritalin)
(E) Phenelzine
46 2: Ad ult P sychop a thology

69. A 28-year-old w om an presents for her annu al 71. Which of the follow ing d ru gs is m ost likely to
gynecology ap p ointm ent. She com p lains cau se this p resentation?
that in the w eek before her period , she often
(A) Alcohol
exp eriences m arked anger and irritability
and argu es m ore w ith her boyfriend . She also (B) Cocaine
reports d im inished energy and concentration, (C) H eroin
and is sleep ing m ore than is u sual for her. (D) Inhalants
These sym p tom s, in ad d ition to breast tend er- (E) Phencyclid ine (PCP)
ness and head aches, alw ays rem it in the w eek
after her m enses is finished . Which of the 72. Which of the follow ing w ould be m ost im por-
follow ing is the m ost likely d iagnosis? tant to ad m inister im m ed iately?
(A) Major d ep ressive d isord er (A) Acetylcysteine
(B) N o d iagnosis (B) Deferoxam ine
(C) Persistent d ep ressive d isord er (C) Methad one
(D) Prem enstru al d ysp horic d isord er (D) Methylene blu e
(PMDD) (E) N aloxone
(E) Prem enstru al synd rom e (PMS)
73. A 30-year-old male veteran from Operation
70. A 37-year-old accou ntant presents to the p ri- Iraqi Freed om / Operation End uring Freed om
m ary care clinic w ith com p laints of insom - presents to the m ental health clinic at the urg-
nia. H e ad m its to feeling “blu e” for 6 w eeks ing of his w ife. While stationed in Iraq on his
since getting p assed over for prom otion. third d eployment, his u nit w as hit by a road sid e
Since that tim e, he has had poor sleep, often bom b. H e saw a fellow Marine killed , w hile he
aw akening early in the m orning. H e also has and several other members of the comm and
had a d ecreased ap p etite w ith a 15-lb w eight sustained nonlethal injuries. Since that time, he
loss, p oor energy, gu ilt over “not being good has had chronic insom nia w ith ongoing night-
enou gh,” and he has been d istracted at w ork. mares of the event as w ell as occasional flash-
H e ad m its to passive su icid al thoughts w ith- backs. H e d escribes alw ays feeling “on ed ge,”
ou t a plan. Which of the follow ing w ou ld be avoid ing crow d s, and becoming easily startled
the likely cou rse of this patient’s illness if not w ith loud noises. H e ad m its to regular alcohol
treated ? use, especially w hen his sym ptoms are w orse.
(A) 3 to 6 m onths H e feels d etached from events w ith his fam-
ily, preferring to isolate himself. Which of the
(B) 3 to 7 m onths
follow ing is the most appropriate treatment to
(C) 6 to 13 m onths begin for this patient?
(D) 9 to 15 m onths
(A) Atyp ical antip sychotic
(E) 12 to 18 m onths
(B) Benzod iazep ine
Questions 71 and 72 (C) Lithiu m
(D) Serotonin-sp ecific reu p take inhibitor
A 36-year-old m an is brou ght to the em ergency (SSRI)
d epartm ent in respiratory arrest. On exam ination,
(E) Valp roic acid
he is brad ycard ic and u nresponsive, w ith constricted
p u pils bilaterally. There are no other obviou s inju -
74. A 40-year-old m arried w om an is referred by
ries on the p atient, bu t no one is im m ed iately avail-
her internist to a p sychologist for fu rther treat-
able to p rovid e collateral history. There is susp icion
m ent. She p resents an 8-m onth history of recu r-
that the patient’s cond ition m ay be the result of an
rent bou ts of “terror,” associated w ith chest
overd ose.
p ain, tachypnea, trem ors, flushing, nausea,
and fears of im p end ing d oom . These ep isod es
Que s tions : 69–79 47

last for ap proxim ately 15 m inu tes and d o not (C) Unconsciou s d esire to assu m e sick role
have a particu lar trigger. As a resu lt, she has (D) Unconsciou s d esire to avoid w ork
had increasing d ifficu lty traveling far from (E) Unconsciou s conflict p rod u cing
her hom e d ue to concerns over having fu rther sym p tom s
attacks in p u blic. Despite ad equate treatm ent
w ith sertraline, she rem ains sym p tom atic and Questions 77 and 78
in significant d istress. Which of the follow ing
psychotherapies would be the most appropriate A 32-year-old m an is brou ght to the psychiatric
for her cond ition? em ergency d epartm ent by the p olice after having
been arrested for p u blic nu d ity. On m ental statu s
(A) Cognitive-behavioral therap y (CBT)
exam ination, the p atient cannot sit d ow n and is
(B) Eye m ovem ent d esensitization and only partly cooperative. H e interrup ts the inter-
rep rocessing (EMDR) view several tim es d em and ing to contact his law -
(C) Insight-oriented therap y yer, “becau se m y rights given to m e by God and
(D) Interpersonal p sychotherap y ord ained by the Jeffersonians have been infringed .”
(E) Su p p ortive p sychotherap y H is sister ind icates that this p atient has been treated
for at least tw o ep isod es of m ajor d ep ression in the
Questions 75 and 76 p ast, one of w hich resu lted in a su icid e attem pt. H e
is cu rrently a com p u ter p rogram m er, bu t has been
You are treating a 48-year-old m arried fem ale on increasingly stressed at w ork. One m onth ago, his
the inpatient m ed ical u nit for pyelonephritis; she girlfriend broke u p w ith him , and since then he has
has resp ond ed w ell to app ropriate antibiotic ther- been increasingly irritable. Over the p ast 2 w eeks he
apy and has been afebrile for the last 24 hou rs. You has m axed ou t his cred it card from online gam bling
inform her of likely d ischarge if she continu es to sites, w hich he has continu ed to visit over the past
im p rove. The next m orning, how ever, she com - fou r consecu tive nights. Laboratory tests show a
p lains of feeling feverish and achy, and having d ys- negative d ru g screen and alcohol levels.
u ria again. The nu rsing staff reports that she has a
su d d en fever of 103°F. You treat the fever w ith acet- 77. Which of the follow ing is the m ost likely
am inophen and p erform a physical exam ination, d iagnosis?
ord er chest x-rays, d raw blood , and ord er a u ri-
nalysis w ith cu ltu re. While you are aw aiting these (A) Ad ju stm ent d isord er
resu lts, the nu rse inform s you that she w itnessed (B) Bip olar d isord er
the patient d ipping her therm om eter into a hot cup (C) Brief p sychotic d isord er
of tea before her tem p erature w as taken. (D) Cyclothym ic d isord er
(E) Major d ep ressive d isord er (MDD)
75. Which of the follow ing d iagnoses m ost likely
accou nts for this w om an’s behavior? 78. Which of the follow ing w ould be the m ost
(A) Factitiou s d isord er ap p rop riate pharm acologic treatm ent for this
(B) Fu nctional neurological sym ptom p atient?
d isord er (A) Carbam azep ine
(C) Illness anxiety d isord er (B) H alop erid ol
(D) Malingering (C) Lam otrigine
(E) Som atic sym p tom d isord er (D) Lithiu m
(E) Sertraline
76. Which of the follow ing is the m ost likely m oti-
vator of this patient’s behavior? 79. A 68-year-old m an w ithou t p rior p sychiatric
(A) Conscious d esire to assu m e sick role history, bu t w ith a history of hypertension,
(B) Consciou s d esire to avoid w ork hyperlip id em ia, and arthritis is ad m itted for a
new left m id d le cerebral artery stroke. Which
48 2: Ad ult P sychop a thology

of the follow ing psychiatric sym ptom s w ou ld 82. A 33-year-old male w ith a history of frequent
be the m ost likely as a resu lt? fighting, aggression, impulsivity, and sui-
cid e attempts is referred to a research facility.
(A) Anxiety
H e receives multiple tests and studies. Upon
(B) Depression cerebrospinal fluid (CSF) analysis, d ecreased
(C) Mania metabolites of w hich of the follow ing neu-
(D) Obsessions rotransmitters w ould most likely be seen as
(E) Panic attacks compared to ind ivid uals w ithout his problems?
(A) Dop am ine
80. A 45-year-old w om an w ith no p reviou s p sy-
(B) Gam m a-am inobu tyric acid (GABA)
chiatric history is ad m itted to neu rology for
the treatm ent of an acu te m ultiple sclerosis (C) Glu tam ate
flare. She d oes not sm oke, d rink alcohol, or (D) N orep inephrine
u se other illicit d ru gs. She is started on ap p ro- (E) Serotonin
p riate therap y and by the third d ay her initial
low er extrem ity w eakness has resolved . H ow - 83. A 38-year-old w om an w ith m u ltip le sclerosis
ever, she also begins to act strangely, and asks of m od erate severity has had sym ptom s of
you to change her room to p revent the “nin- d ep ression and m em ory loss increasing over
jas ou tsid e from creeping in and stealing m y the last year. On m ental statu s exam ination,
sou l.” She also tells you the nurses have been you notice a blu nted affect and d ecreased
p oisoning her m ed icine. Which of the follow - sp eed of m ental processing. A m agnetic reso-
ing d iagnoses w ou ld be the m ost likely? nance im aging (MRI) exam ination is m ost
likely to reveal w hich of the follow ing?
(A) Ad ju stm ent d isord er
(B) Bipolar d isord er (A) Global cerebral atrop hy
(C) Psychotic d isord er d u e to m u ltip le (B) Mu ltip le p laqu es in frontal w hite m atter
sclerosis (C) N orm al brain
(D) Schizophrenia (D) Periventricu lar lacu nar infarcts
(E) Substance-ind uced psychotic d isord er (E) Ventricular enlargem ent

81. A 42-year-old m an p resents w ith a history of 84. An 80-year-old w id ow ed w om an is ad m itted


recu rrent ep isod es of m ajor d ep ression, w ith to the hospital for “confu sion.” On exam ina-
one prior hospitalization. For the past several tion she is som nolent, thou gh earlier in the
m onths he has felt increasingly d ep ressed , m orning she had been alert and aw are. She
w ith insom nia, low ap p etite, little energy, cry- is u ncoop erative, hostile, and hallu cinating.
ing sp ells, and p oor concentration. H e is not H er insight and m em ory are p oor. The p ri-
taking any current m ed ications. H e states that m ary team w ishes to know if she is “d eliriou s
he has been on fluoxetine, p aroxetine, sertra- or d em ented .” Which of the follow ing signs/
line, and venlafaxine in the p ast, and , w hile sym ptom s in this p atient is the m ost sp ecific
they’ve all been efficaciou s, he has stop ped for d eliriu m ?
them d u e to significant erectile d ysfu nction.
(A) Com bativeness
Which of the follow ing m ed ications w ou ld be
the m ost ap propriate to prescribe? (B) Flu ctu ating consciou sness
(C) Poor m em ory
(A) Aripip razole
(D) Psychosis
(B) Bup ropion
(E) Uncoop erativeness
(C) Citalop ram
(D) Lithium 85. A happily m arried grad uate stu d ent com es
(E) Sertraline to you r clinic w ith com p laints of insom nia.
She has been u nable to fall asleep because
Que s tions : 80–88 49

she ru m inates abou t grad es, m oney, her rela- office reporting that his m ood has been d ow n
tionship , and her you ng child ren. She has in the d u m p s, and he feels like he m ay never
alw ays been concerned abou t these, bu t lately feel better. Recently, he has been thinking that
her w orries have gotten ou t of control. She is life w as not w orth living anym ore and has
u nable to relax and feels constant tension in p assive thou ghts of su icid e. H e reports ongo-
her m uscles. While she d enies sym p tom s of ing thou ghts of overd osing on p ills. Which of
p anic attacks, she has noticed an increase in the follow ing w ould be the m ost appropriate
head aches and gastrointestinal d istu rbances next step ?
over the p ast few m onths. She d enies any
(A) Call his p arents and arrange a fam ily
p roblem s w ith m ood . She d enies any recent
m eeting
stressors, changes to her rou tine, or changes to
her hu sband ’s rou tine. Which of the follow ing (B) Discontinu e his SSRI and have him
is the m ost likely d iagnosis? retu rn in 1 w eek
(C) Refer the p atient to the p sychiatric
(A) Ad ju stm ent d isord er em ergency d ep artm ent
(B) Generalized anxiety d isord er (D) Sched u le w eekly ou tp atient visits
(C) Obsessive-com p u lsive d isord er (E) Sw itch the SSRI and have him follow -u p
(D) Panic d isord er w ith you in 1 m onth
(E) Social anxiety d isord er (social p hobia)
88. A 36-year-old m arried w hite fem ale p resents
86. You are a research p sychiatrist w ho is stu d ying to the em ergency room w ith a 2-m onth his-
signs and sym p tom s associated w ith certain tory of d epression, term inal insom nia, fatigu e,
p sychiatric d isord ers, and notice a category d ecreased ap p etite, anhed onia, and exces-
of patients w ho have sensory gating d eficits, sive guilt. She feels hopeless and relu ctantly
short-term m em ory d ifficulties, and abnor- ad m its to su icid al id eation for the p ast w eek,
m alities in sm ooth-p u rsu it eye m ovem ents. w ith thou ghts of “taking all of m y m ed icines.”
Which of the follow ing d iagnoses is m ost After fu rther qu estioning, she states that “I
likely to be fou nd in this p atient p op u lation? w ou ld never d o it” as she is a d evou t Catho-
lic w ho attend s chu rch regu larly. Which of the
(A) Attention-d eficit/ hyp eractivity d isord er
follow ing characteristics m ost increases this
(ADH D)
p articu lar patient’s risk of su icid e?
(B) Major d ep ressive d isord er (MDD)
(C) Obsessive-com p u lsive d isord er (OCD) (A) Age
(D) Posttrau m atic stress d isord er (PTSD) (B) Gend er
(E) Schizop hrenia (C) H op elessness
(D) Marital statu s
87. A 23-year-old single m ed ical stu d ent com es (E) Religion
to you r office com p laining of d ifficu lty sleep-
ing, excessive w orry abou t his stu d ies, his Questions 89 and 90
relationship w ith his p arents, and that his girl-
friend of 2 years is going to break u p w ith him A 29-year-old m arried w hite w om an w ith a p ast
d espite being happ y w ith their relationship. m ed ical history of recu rrent m igraines is brou ght to
H e feels irritable at tim es, and has significant the psychiatric em ergency d epartm ent by her hu s-
m u scle tension. Because of these sym p tom s, band w ho rep orts that, d esp ite feeling d ep ressed
his grad es have su ffered . H e d oes not rep ort 2 m onths ago and being com p liant w ith p rescribed
a d epressed m ood . You d iagnose him w ith treatm ent, she now has been acting bizarre for sev-
generalized anxiety d isord er, and p rescribe a eral d ays. On initial interview , the p atient states, “I
com bination of psychotherapy and a selective- feel su p erbly su p rem e, and you have no id ea w hat
serotonin reuptake inhibitor (SSRI). Three an am azing person I am !” The p atient is talking so
m onths later, the patient com es back to you r rap id ly that you cannot interru p t her. H er hu sband
50 2: Ad ult P sychop a thology

rep orts that the p atient has not slep t in over a w eek, (D) Retrograd e
and d u ring the sam e tim e p eriod , she has p u t a d ow n (E) Selective
p aym ent for a car, has p urchased a d iam ond tennis
bracelet, and has booked an extravagant vacation. Questions 93 and 94

89. Which of the follow ing is the m ost likely A 75-year-old man comes to you r office com plain-
d iagnosis? ing of poor sleep since his w ife’s d eath 1 m onth ago.
Since that time, he has been unable to fall asleep,
(A) Anxiety d isord er and he has felt “d ow n.” H e is slightly more isolative
(B) Bip olar and related d isord er now , as many of the activities he enjoyed d oing w ere
(C) N eu rocognitive d isord er w ith his w ife. H is appetite is d ecreased , but he is still
(D) Psychotic d isord er bathing and cooking. H e sometim es feels guilty that
(E) Som atic sym p tom d isord er she d ied before him , and is angry w ith God that he
is alive w ith no “soul mate” anymore. H e sometim es
90. You suspect a medication is the cause for her hears the voice of his d eceased w ife encouraging
current condition, but neither she nor her hus- him to move on, and he d enies suicid al id eation.
band recall the medication prescribed 2 months
ago. Which of the following medications is the 93. Which of the follow ing is the m ost ap p rop riate
most likely etiology? d iagnosis at this tim e?

(A) Am itriptyline (A) Ad ju stm ent d isord er w ith d ep ressed


m ood
(B) Clonazep am
(B) Bereavem ent
(C) Flu oxetine
(C) Major d ep ressive d isord er
(D) Lithium
(D) Persistent com p lex bereavem ent
(E) Sertraline
d isord er
91. A 62-year-old m ale patient w ith schizop hrenia (E) Schizop hrenia
is brou ght into the em ergency d ep artm ent by
the police for trespassing. Up on interview, he 94. Which of the follow ing w ou ld be the m ost
tells the p hysician that over the past 3 w eeks app rop riate treatm ent for this p atient?
his television has been giving him cod ed (A) H osp italize the p atient im m ed iately
m essages. Which of the follow ing term s best (B) Prescribe an antid ep ressant
d escribes this p henom enon?
(C) Prescribe an antip sychotic
(A) H allu cination (D) Refer for brief su p portive therap y
(B) Id ea of reference (E) Refer for cognitive-behavioral therap y
(C) Illu sion
(D) Thou ght broad casting 95. When asked abou t his level of ed u cation, a
(E) Thou ght insertion 48-year-old m an w ith a history of schizoaf-
fective d isord er, d ep ressed typ e, sp end s the
92. A 45-year-old p atient tells her d octor that after next 5 m inutes d escribing his high school
hearing that her hu sband d ied , she cou ld not bu ild ing, friend s he had at the tim e, clu bs he
rem em ber leaving her office and going hom e. joined , and eventu ally his high school grad u a-
In every other resp ect, her m em ory is intact. tion cerem ony. H e conclud es by saying, “And
Which of the follow ing types of am nesia is that w as the end of m y schooling.” Which
this an exam ple of? of the follow ing term s d oes this response best
d em onstrate?
(A) Continuou s
(A) Circu m stantiality
(B) Generalized
(B) Loosening of associations
(C) Localized
Que s tions : 89–100 51

(C) Perseveration 99. A 21-year-old w om an com p lains of recu r-


(D) Pressured sp eech rent ep isod es w here she has extrem e anxiety,
(E) Tangentiality along w ith p alp itations, d izziness, nau sea,
abd om inal cram p s, d iarrhea, tingling of her
96. In her p sychiatrist’s office, a p atient su d d enly fingers, shortness of breath, and fearing she
low ers herself to the floor, begins flailing m ay d ie. These p eriod s last u nd er 30 m inu tes,
abou t w ild ly, then flings a garbage p ail against and they’ve been increasing in frequency. She
the w all, and runs out of the office. Im m ed i- is u nable to p red ict w hen they w ill occu r, bu t,
ately afterw ard , she returns. She is alert and as a resu lt, she is often w orried abou t hav-
oriented , yet d oes not rem em ber the incid ent. ing another attack, and she has been u nw ill-
Which of the follow ing typ es of seizu res is the ing to leave her ap artm ent for several w eeks.
m ost likely? She d rinks 1 to 2 d rinks of alcohol p er m onth
and d enies illicit d ru g u se. She has no m ed i-
(A) Com p lex p artial seizu re cal problem s, is only on oral contraceptives,
(B) Jacksonian seizure and her physical and laboratory w orku p are
(C) N onep ilep tic seizu re negative. Which of the follow ing w ould be the
(D) Tem p oral lobe seizu re m ost app rop riate treatm ent?
(E) Tonic–clonic seizu re (A) Bu sp irone
(B) Lithiu m
97. A 26-year-old m an new ly d iagnosed w ith (C) Lorazep am
narcolep sy exp lains that he has ep isod es of
(D) Valp roic acid
falling d ow n w ithou t any loss of consciou s-
ness, p recip itated by lau ghter or anger. Which (E) Venlafaxine
of the follow ing term s best d escribes this
sym p tom ? Questions 100 and 101

(A) Catalepsy A 46-year-old m an w ith a history of hyp ertension


(B) Catap lexy and hyperlip id em ia p resents to his p rim ary care d oc-
(C) H yp ersom nia tor w ith com plaints that “I’m d rinking too m u ch.”
(D) H yp nagogic hallu cinations H e gives a long history of binges over 4 to 5 d ays
at a tim e, w here he w ill d rink 1 to 2 pints of gin per
(E) Sleep p aralysis
d ay. H e has had several DUIs and is at risk of losing
his job. In ad d ition, his d rinking has put a strain on
98. A 21-year-old m an w ith new ly d iagnosed
his m arriage. Unfortunately he has been u nable to
schizophrenia has been com p liant w ith his
maintain sobriety for any significant period of tim e.
med ications and is less p sychotic. At his next
H e is “stressed out” because of the consequences of
follow -u p appointm ent, he is noted to be rest-
his d rinking, and he ad mits to chronic, intermittent
less and constantly m oving. H e states that
insom nia. H e has a good appetite, how ever, and his
he feels as if he has to be m oving all the tim e
energy is ad equate u nless he is significantly hun-
and is u ncom fortable if he sits still. Which
gover. H e feels helpless but d enies suicid al id eation.
of the follow ing sid e effects is he m ost likely
exp eriencing?
100. Which of the follow ing m ed ications w ou ld be
(A) Akathisia m ost appropriate to prescribe for this patient?
(B) Akinesia
(A) Acam p rosate
(C) Dystonia
(B) Bu p rop ion
(D) Rabbit synd rom e
(C) Citalop ram
(E) Tard ive d yskinesia
(D) Lithiu m
(E) Olanzap ine
52 2: Ad ult P sychop a thology

101. Tw o w eeks later this sam e patient is seen (A) Atrop hy of brainstem and cerebellu m
in the em ergency room after being p icked (B) Atrop hy of frontal and tem p oral lobes
u p by the police for d istu rbing the peace (C) Enlarged ventricles
and w and ering. On exam ination he app ears
(D) Global atrop hy
confu sed , has nystagm us, and an unstead y
gait. Which of the follow ing shou ld be ad m in- (E) Periventricu lar w hite-m atter lesions
istered im m ed iately?
104. Which of the follow ing w ou ld best d escribe
(A) Glucose his clinical course?
(B) H alop erid ol
(A) Continu ed im p rovem ent
(C) Lorazep am
(B) N o im p rovem ent or w orsening
(D) N altrexone
(C) Rap id d ecline
(E) Thiam ine
(D) Stead y d ecline
102. A 36-year-old man presents to the em ergency (E) Step -w ise d eterioration
d epartm ent after being found w ithout cloth-
ing in the street. H e has m ultiple excoriations 105. An 86-year-old w om an w ith m u ltip le m ed ical
all over his bod y, is paranoid and agitated , and problem s and a recent hip fractu re is ad m itted
states that bugs are craw ling all over him. H is to the intensive care u nit. While in the u nit, she
vitals are as follow s: T-99.1, BP-160/ 93, P-105, aw akens at night and m istakes her intrave-
R-20. Which of the follow ing substances w ould nou s (IV) p ole for a fam ily m em ber com ing for
be m ost likely found in his laboratory results? a visit. She then becom es agitated , attem p ting
to pu ll out her IV and get ou t of the hospital
(A) Alcohol bed . Which of the follow ing ap p roaches is the
(B) Cannabis m ost im p ortant in the u ltim ate m anagem ent
(C) Cocaine of this patient?
(D) Opiates (A) Ad m inister d ip henhyd ram ine
(E) PCP (B) Ad m inister halop erid ol
(C) Ad m inister lorazep am
Questions 103 and 104
(D) App ly soft restraints
A 72-year-old m arried m ale w ith a history of hyper- (E) Determ ine and correct the u nd erlying
tension, bord erline d iabetes, hyperlipid em ia, and cond ition
coronary artery d isease is brou ght in by his w ife.
Over the p ast year he has becom e increasingly for- 106. A 24-year-old grad u ate stu d ent in p hiloso-
getfu l, m isp lacing his keys, getting lost w hile d riv- p hy is referred by his stu d ent health center
ing, and starting to w and er. When confronted w ith for a p sychiatric evalu ation. Althou gh he
these situations he becom es d efensive, m aking up claim s to have had sim ilar bu t attenu ated
excu ses for his behavior. H is sleep is erratic, bu t he is sym p tom s in ch ild hood , since beginn ing
eating w ell and enjoys w atching his favorite m ovies his thesis, he d escribes an acu te w orsening
on the television. H is w ife d escribes extrem e m ood of fears that he w ill contract H IV. While he
lability, w here he w ill be lau ghing and then crying. u nd erstand s the m od es and risks of contrac-
H e d enies suicid al id eation. H e d oesn’t d rink alco- tion and p ractices safe sex, he is u nable to “get
hol and d enies d ru g u se. H e w as a p rior sm oker bu t rid of these thou ghts.” As a resu lt, he feels
qu it 20 years ago. H is p hysical exam ination and labs com p elled to w ash his hand s m any tim es p er
are u nrem arkable. d ay, even to the p oint of their becom ing raw
an d bleed ing. Desp ite his insigh t that his
103. As p art of his w orku p , an MRI of his brain concerns are irrational, he is not able to stop
is ord ered . What w ou ld be the m ost likely the behaviors. A p ositron em ission tom ogra-
find ing? p hy (PET) scan of this p atien t’s brain w ou ld
Que s tions : 101–111 53

m ost likely d em on strate increased activity in easily agitated , striking at nu rsing staff and
w hich of the follow ing stru ctu res? his d au ghter. Which of the follow ing m ed ica-
tions w ou ld be m ost appropriate to prescribe
(A) Am ygd ala
in this case?
(B) Cau d ate nu cleus
(C) Cerebellu m (A) Am itriptyline
(D) H ip p ocam p u s (B) Clonazepam
(E) Parietal lobes (C) Dip henhyd ram ine
(D) Donep ezil
Questions 107 and 108 (E) H aloperid ol

A 70-year-old w id ow ed m an w ithou t significant 109. A 36-year-old w om an presents w ith com -


m ed ical history is brou ght in by his d au ghter d ue to p laints of a d epressed m ood for the p ast
concerns abou t his being able to ad equately care for m onth. She reports p oor sleep, little app etite
him self. H e has been fou nd lost in the d ow ntow n w ith w eight loss, low energy, d ecreased con-
area on several occasions, attem p ting to w alk into centration, and little libid o. She ad m its to feel-
strangers’ hou ses, as w ell. H e is unable to cook for ing hop eless and su icid al, althou gh she d enies
him self bu t is unable to calculate the correct am ou nt a sp ecific p lan or intent. She is su bsequently
for p u rchases. H is d au ghter has noticed that he begu n on p aroxetine 20 m g at bed tim e. Which
is d izzy w hen stand ing u p and has a w id e-based , of the follow ing sym p tom s w ou ld be m ost
slow w alk. Up on interview , he ap pears d isheveled likely to im p rove the earliest?
and smells of u rine. H e is friend ly and coop erative
(A) Decreased libid o
overall, althou gh easily d istracted and confu sed .
While he know s his nam e, he believes the year is (B) Dep ressed m ood
1989. When confronted abou t his m em ory d ifficu l- (C) H op elessness
ties, he states that there are sm all gnom es living in (D) Poor sleep
his furnitu re, w ho play tricks on him by m oving (E) Su icid al id eation
item s arou nd in his cond om iniu m . Accord ing to the
d au ghter, he rarely d rinks alcohol, d oes not sm oke 110. A 35-year-old m an com p lains to his therap ist
tobacco, and has never u sed illicit d ru gs. that his new p artner enjoys sexual activity
only w hen inflicting pain on him . This d is-
107. Which of the follow ing d iagnoses is m ost turbs and fru strates the patient. Which of the
likely? follow ing best d escribes the behavior exhib-
(A) Major frontotem p oral neu rocognitive ited by his p artner?
d isord er (A) Exhibitionistic d isord er
(B) Major neu rocognitive d isord er d u e to (B) Frotteu ristic d isord er
Alzheim er d isease (C) Sexu al m asochism d isord er
(C) Major neu rocognitive d isord er w ith (D) Sexu al sad ism d isord er
Lew y bod ies
(E) Transvestic d isord er
(D) Major vascu lar neu rocognitive d isord er
(E) Su bstance-ind u ced m ajor 111. An anxiou s 23-year-old Asian m ale u niver-
neu rocognitive d isord er sity stu d ent p resents to stu d ent health ser-
vices claim ing that his p enis is shrinking into
108. The above p atient is ap p rop riately d iagnosed his abd om en. Desp ite reassu rances from the
and then placed in an interm ed iate care facil- staff and the p hysician, he rem ains convinced
ity. Eight m onths later he is brou ght by the of this belief. Which of the follow ing syn-
d aughter becau se the nu rsing hom e is having d rom es is this p atient m ost likely su ffering
increasing d ifficu lty caring for him . H e is m ore from ?
confused , resp ond ing to internal stim uli, and
54 2: Ad ult P sychop a thology

(A) Cap gras synd rom e (A) Bip olar d isord er


(B) Koro (B) Delu sional d isord er
(C) Ku ru (C) Major d ep ressive d isord er w ith
(D) Taijin-kyofu sho psychotic featu res
(E) Zar (D) Schizoaffective d isord er
(E) Schizop hrenia
112. A 21-year-old m ale college stu d ent is evalu -
ated by his college stu d ent health center after Questions 114 and 115
being arrested for m astu rbating outsid e of a
sorority w ind ow late at night. H e ad m its to A 63-year-old Vietnam veteran w ith p osttrau m atic
having w atched a particu lar fem ale stu d ent stress d isord er (PTSD) p resents to the ou tpatient
insid e the build ing over a period of several m ental health clinic. H e has been prescribed nu m er-
m onths. Which of the follow ing is the m ost ous m ed ications over the years, and he has recently
likely d iagnosis? been taking citalop ram 40 m g d aily. H e d enies sid e
effects and strongly believes this has been the “m ost
(A) Exhibitionistic d isord er
effective” m ed ication he has taken. While he d enies
(B) Frotteu ristic d isord er intru sive thou ghts or overt flashbacks, he continu es
(C) Gend er d ysp horia to have severe nightm ares several tim es p er w eek
(D) Transvestic d isord er w hich significantly interfere in his sleep . H e rem ains
(E) Voyeuristic d isord er “ju m p y” at tim es, and becau se of this d oesn’t attend
fu nctions w here there are large crow d s. H e has not
113. A 45-year-old sep arated fem ale is brou ght by u sed alcohol or d rugs in 18 years, and he d enies any
her brother into the em ergency room w ith the su icid al id eation.
chief com p laint of “strange behavior.” She
has been living w ith her brother for the p ast 114. Which of the follow ing m ed ications w ou ld
2 years, and she stop ped her m ed ications 12 be the m ost beneficial to p rescribe for
m onths ago. The brother states that over the au gm entation?
past 6 m onths she has becom e increasingly (A) Alp razolam
paranoid and d elu sional, believing that the
(B) Bu p rop ion
neighbors are sp ying on her. She insists that
the Chinese governm ent is using her as an (C) Prazosin
agent to com bat terrorism , and that they com - (D) Risp erid one
m unicate their instru ctions via a w ireless sig- (E) Valp roic acid
nal transm itted d irectly to her brain. She is
often noticed to be talking w hen no one else is 115. In ad d ition to m ed ications, this p atient w ishes
in her room . In ad d ition, over the past m onth to pursu e “talk therapy.” Which of the fol-
she has been staying u p m ost of the night, p ac- low ing form s of p sychotherap y w ou ld be the
ing arou nd the hou se and attem p ting to con- m ost app rop riate to recom m end ?
struct an anti-terrorism m achine, u sing p arts
(A) Behavior therap y
of various household electronics. She states
that it is im portant that she alone com p letes (B) Cognitive p rocessing therap y
her m ission, that she is the m ost senior sp y (C) Cognitive therap y
in the organization w ith a top-secret clear- (D) Dialectical behavioral therapy
ance “that only the Presid ent and I p ossess.” (E) Psychod ynam ic p sychotherap y
H er brother has had a hard tim e calm ing her
d ow n as she sp eaks too qu ickly to follow, goes 116. A 23-year-old grad u ate stu d ent com es to the
from one top ic to another, and “w on’t sit still.” psychiatric em ergency room com p laining of
Which of the follow ing d iagnoses w ou ld be anxiety. She has never been seen by a psychia-
the m ost likely for this patient? trist before and is not taking any m ed ications.
Que s tions : 112–120 55

H er vital signs are notable for a heart rate of 119. A 26-year-old m an w ith schizophrenia is
110 beats/ m in. She is also slightly d iap horetic being d ischarged from his third ad m ission
and has m ild ly d ilated p u pils. Given her p res- to the p sychiatric u nit in 3 years. This m ost
ent state, w hich area of this p atient’s brain recent stay w as p rom p ted by acu te exacerba-
w ou ld m ost likely d em onstrate increased tion of com m and aud itory hallu cinations and
activity? paranoia that resulted in bizarre behaviors
su ch as lying in the m id d le of the road and
(A) Am ygd ala
berating cu stom ers that cost him his restau -
(B) Basal ganglia rant job. H is sym p tom s are im p roved w ith
(C) H ip p ocam p u s pharm acotherap y, w hich he is tolerating, w ith
(D) Locu s ceru leu s m ild sid e-effects of sed ation that are grad u ally
(E) Thalam u s im p roving. Which of the follow ing tim e p eri-
od s m ost accu rately d escribes w hen his risk of
117. You are asked to give a p sychiatric consu lta- su icid e is the highest?
tion on a 28-year-old w om an w ith system ic (A) At his first p sychotic break
lu pu s erythem atosu s w ho w as ad m itted to
(B) Du ring his first hosp italization
the m ed ical service. After you see her, one of
you r m ed ical colleagu es tells you that she w ill (C) In the w eek p reced ing this
no longer sp eak to any of them becau se she hosp italization
“hates all of them ” and now insists on seeing (D) In the w eek w hile he is hosp italized
you becau se you are the “best d octor in the (E) In the w eeks follow ing hosp italization
hospital.” Which of the follow ing term s best
d escribes the patient’s behavior? 120. You are asked to review neu rop sychological
testing for a 19-year-old p atient w ho is fail-
(A) Acting out
ing classes at his local com m u nity college. H is
(B) Externalization resu lts ind icate an intelligence qu otient (IQ)
(C) Regression of 55. H e is has a grou p of p eers bu t is often
(D) Sp litting im m atu re, and his friend s often p revent him
(E) Su blim ation from being taken ad vantage of as he is very
gu llible. H e takes care of his p hysical need s
118. A 42-year-old w oman w ith recurrent episod es bu t p arents ensu re that he has enou gh grocer-
of major d epression is ad mitted to a med i- ies and need to rem ind him to check his bank
cal unit after a car accident that rend ered her accou nt balance. Which of the follow ing is the
unconscious. The patient regains conscious- m ost ap propriate d iagnosis?
ness after 3 days and corroborates that she w as, (A) Mild intellectu al d isability
ind eed , on an antid epressant, but she says she
(B) Mod erate intellectu al d isability
also cannot remember w hich one. She is started
on paroxetine (Paxil) for her d epression. Tw o (C) N o d iagnosis (norm al intellectu al
days after beginning this med ication, she fu nctioning)
develops tachycard ia, d iaphoresis, and myo- (D) Severe intellectu al d isability
clonic jerks. The neurotransmitter most likely (E) Profou nd intellectu al d isability
associated w ith the above reaction is synthe-
sized in w hich of the follow ing central nervous D IRECTION S (Questions 121 through 145): The
system structures? follow ing group of numbered items are preceded
by a list of lettered options. For each question,
(A) Cau d ate nu cleu s
select the one lettered option that is most closely
(B) Locu s ceru leus associated w ith it. Each lettered option may be
(C) N u cleu s accu m bens used once, multiple times, or not at all.
(D) Rap he nu cleu s
(E) Su bstantia nigra
56 2: Ad ult P sychop a thology

Questions 121 through 130: Match each scenario 125. A 27-year-old grad u ate stu d ent w oke u p late
w ith its most likely description. and skip p ed breakfast. She now has a m assive
head ache and is irritable w hen she w alks into
(A) Alcohol intoxication
her first m orning class. She feels like falling
(B) Alcohol w ithd raw al asleep and as if she “has the flu ” by the end of
(C) Am p hetam ine intoxication the m orning.
(D) Am phetam ine w ithd raw al
(E) Caffeine intoxication 126. A veterinarian technician is brou ght to the
(F) Caffeine w ithd raw al em ergency d epartm ent after attacking w hat
(G) Cannabis intoxication he thought w as a cougar (it w as a hou secat).
(H ) Cannabis w ithd raw al In the em ergency room he is febrile, ap p ears
(I) Cocaine intoxication p anicked , has nystagm u s, and d em onstrates
u nexpected strength and rage.
(J) Cocaine w ithd raw al
(K) H allu cinogen intoxication
127. A 35-year-old p ilot is brou ght in for evalu ation
(L) Inhalant intoxication
becau se he has not slep t for d ays, and now he
(M) Opioid intoxication is anxiou s, tachycard ic, trem u lou s, and u nable
(N ) Opioid w ithd raw al to give coherent history. H is pu pils are d ilated
(O) Phencyclid ine intoxication and his blood p ressure is high, d esp ite norm al
valu es 2 w eeks ago on a flight physical.
121. A 32-year-old single, m ale w ith injected con-
ju nctiva can’t concentrate at w ork, lau ghs 128. A 54-year-old w om an w ith arthritis com p lains
read ily at his cow orkers’ d ood les, feels of yaw ning, d iarrhea, abd om inal cram ps, and
“relaxed ,” and sp eaks slow ly w hile seem ingly nausea. H er pup ils are d ilated and she has
focu sed on the air in front of him . notable piloerection.

122. A 45-year-old separated male ad mitted for 129. A 23-year-old m an is fou nd u nresponsive w ith
d epression and suicid al id eation is irritable, slow ed breathing. H e has m ultiple scars on his
asks for extra food , and spend s m ost of the first arm s and he has severe m iosis.
d ay sleeping. The chest pain he had on ad mis-
sion has subsid ed and he has no ECG changes.
130. On the third postop erative d ay you r 63-year-
old patient becom es agitated , d em and s you
123. After recess, a ju nior high school stu d ent rem ove the snakes from his room , and asks
sm ells “fu nny,” is stu m bling, feels d izzy and w hy it is so lou d at night (it is d aytim e). H e is
nauseated , yet rem ains sm iling and says she tachycard ic, hypertensive, and trem u lous.
feels “su ch a ru sh.” By the m id d le of her next
class she has a head ache but otherw ise feels
like she d id this m orning. Questions 131 through 136: Match each scenario
w ith the most likely receptor responsible from the
follow ing list.
124. A 17-year-old high school stu d ent attend s a
college fraternity party hosted by his eld er (A) 5-H T2 A recep tor
brother. After several hou rs, he feels m ore (B) Alp ha recep tor
courageou s and approaches lad ies he w ou ld (C) Beta recep tor
norm ally be too shy to engage, his w ord s are (D) Dop am ine recep tor
slightly slu rred , and he has d ifficulties m ov-
(E) H istam inergic recep tor
ing in a straight line. H e is slightly flu shed and
notes m ild m em ory problem s for events ear- (F) Mu scarinic recep tor
lier in the night.
Que s tions : 121–145 57

131. A 32-year-old m an on clozapine exp eriences 138. You r p atient is angry w ith you and claim s you
constipation. are the w orst d octor ever, as op p osed to her
form er clinician, w ho actu ally listened to her.
132. A 29-year-old w om an on qu etiap ine experi-
ences d izziness on stand ing. 139. You r bord erline p atient m isses seven ap p oint-
m ents in a row resu lting in you r term inating
133. A 52-year-old m an on risperid one exp eriences her treatm ent contract.
a m ilky d ischarge from his breasts.
140. After slip p ing in front of you r boss on a fro-
134. A 41-year-old m an on haloperid ol experiences zen pu d d le, you exclaim that “in a form er life
resolu tion of his au d itory hallu cinations and I w as actu ally an elite figu re skater!”
his d elu sions.
141. You r 69-year-old p atient states he ju st has a
135. A 22-year-old w om an on olanzap ine exp eri- bad cold after being d iagnosed w ith m eta-
ences significant sed ation and w eight gain. static lung cancer, and states all he need s is
som e hot tea and rest.

136. A 53-year-old m an on quetiap ine exp eriences


im p rovem ent and stabilization of m ood . 142. You argu e w ith the boss at w ork, and w hen
you com e hom e you harshly groom you r cat
so that she actu ally w riggles aw ay ou t of your
Questions 137 through 145: Match each scenario arm s.
w ith its most likely defense mechanism from the
follow ing list.
143. A 68-year-old w id ow volu nteers at the local
(A) Altru ism veteran’s nursing hom e.
(B) Denial
(C) Disp lacem ent 144. After rep eated ly failing to bring you r p or-
(D) H u m or tion of the grou p project to class, you accu se
(E) Projection the group lead er of forgetting to e-m ail you a
(F) Projective id entification rem ind er.
(G) Reaction form ation
(H ) Rationalization 145. After not m atching into you r chosen sp ecialty,
(I) Splitting you say that it w as fu ll of boring nerd s any-
w ay and that you really have w ay too m u ch
137. A 37-year-old married male is just told by his personality to be p art of them .
wife that she has been having an affair. He
immediately hugs her and tells her he loves her.
CHAPTER 3

S o matic Tre atme nt and


Ps yc ho pharmac o lo g y
Que s tions

D IRECTION S (Questions 1 through 73): For each (D) O isthotonos


of the multiple-choice questions in this section (E) Serotonin syn ro e
select the lettered answ er that is the one best
response in each case. Questions 3 and 4

A 24-ye r-o w o n resents to her ri ry c re


Questions 1 and 2 hysici n for regu r checku . During the ex in -
A 32-ye r-o n is itte to gener hos it tion, her octor notes 15- b w eight g in. The tient
fter ingesting n u nknow n qu ntity of hene zine co ins th t she w ys fee s tire uring the y
in su ici e tte t. After g stric v ge n in- es ite s ee ing fro 8 pm to 9 a m i y. On further
istr tion of ch rco s u rry, he is tr nsferre to the questioning, the hysici n e rns th t the tient
e ic intensive c re u nit for onitoring. Tw enty- fee s s n e ty, often thinks bout e th, c nnot
fou r hours ter, he begins to see horses running in concentr te t w ork, n cks the energy to c re for
the h n u s out his intr venous (IV) ines. her tw o chi ren. H ow ever, her oo icks u w hen
her hysici n questions her bout her chi ren.
1. Which of the fo ow ing tre t ents w ou be
the ost i ort nt t this ti e? 3. Which of the fo ow ing e ic tions w ou be
the ost ro ri te to tre t her i ness?
(A) Ch or ro zine
(A) F u oxetine
(B) Cy rohe t ine
(B) N ortri ty ine
(C) Lor ze
(C) P roxetine
(D) Me eri ine
(D) Phene zine
(E) Phenytoin
(E) Tr zo one
2. Tw e ve ys fter his su ici e tte t, he
receives ven f xine (Effexor) to tre t his 4. H er hysici n initi tes tre t ent w ith n nti-
e ression. One hour fter ingestion of the e ress nt t ther eu tic ose. Which of the
ven f xine, he beco es t chyc r ic n i - fo ow ing ti es w ou be consi ere ini-
horetic n eve o s yoc onic jerks. Which u equ te tri for the e ic tion?
of the fo ow ing con itions h s he ost ike y (A) 1 w eek
eve o e ? (B) 2 w eeks
(A) Acute ystoni (C) 6 w eeks
(B) Ak thisi (D) 3 onths
(C) N eu ro e tic ign nt syn ro e (E) 6 onths

81
82 3: S oma tic Tre a tme nt a nd P syc hop ha rma c ology

Questions 5 and 6 bec u se she is w orrie th t her foo y be


oisone . U on interview, she is ys oor
Whi e on c in gener hos it , you re c e t eye cont ct, sycho otor ret r tion, n
4:30 a m to ev u te tient w ho w nts to be is- te rfu n constricte ffect. Which of the
ch rge . U on rriving on the f oor, the nurse te s fo ow ing w ou be the ost ro ri te
you he ju st u e ou t his thir IV of the y n e ic tion regi en for this tient?
st rte to sw ing the IV o e in the ir w hi e ye ing
rof nities. H e te s nu rse’s i e he u st c tu re (A) C oz ine
the tiger th t is oose in the rking ot. You qu ick y (B) F u oxetine n ris eri one
ook t his ch rt n see th t he is 55-ye r-o n (C) F u oxetine n c on ze
w ith history of coho u se isor er, itte 36 (D) F u oxetine onother y
hou rs e r ier for b o in in, n u se , n vo -
(E) Ris eri one onother y
iting. H is tr ns in ses w ere e ev te on is-
sion, bu t his he titis rofi e is sti en ing. H e is
8. A 40-ye r-o w o n w ith bi o r isor er
sche u e for u er en osco y in the orning.
resents to the e ergency roo 2 w eeks fter
st rting new e ic tion. She re orts th t
5. Which of the fo ow ing shou be the ost
she w s oing “w e ” u nti she got vir
ro ri te e ic tion given t this ti e?
g stroenteritis n w s u n b e to e t for sev-
(A) Ch or i ze oxi e er ys. Whi e the vir sy to s reso ve ,
(B) Ch or ro zine she now co ins of n u se n vo iting,
(C) Disu fir t xi , n tre or. Which of the fo ow ing
stu ies shou be or ere first?
(D) H o eri o
(E) Ox ze (A) C rb ze ine eve
(B) De kote eve
6. You s e k brief y w ith this tient n re b e (C) H e CT
to sett e hi ow n. H is te er tu re is 102.1°F, (D) Lithiu eve
u se is 130 be ts/ in, n b oo ressu re is
(E) Urine toxico ogy screen
220/ 120 H g. Which of the fo ow ing is
the ost ro ri te e ic tion n rou te of
9. A 25-ye r-o n resents to the e ergency
inistr tion to give?
e rt ent cco nie by his f i y w ho
(A) H o eri o intr u scu r y (IM) re ort th t the tient h s not eft his be -
(B) H o eri o intr venou s y (IV) roo in 3 w eeks. They re ort th t he s en s
(C) L bet o IV his y re ing science fiction nove s th t they
rovi e for hi . On ex in tion, you fin
(D) Lor ze IM
hi to be o orou s n isheve e . H e is
(E) Lor ze IV gu r e w hen you s e k w ith hi . H e oes
not ke eye cont ct n he voi s ooking
7. A 32-ye r-o sing e w o n resents to the t the te evision in the w iting roo bec u se,
e ergency e rt ent escorte by her f i y he s ys, he w nts to revent intru ers fro
fter she reve e to the her su ici e n to contro ing his thoughts. After co eting
t ke fu bott e of cet ino hen n rink the ex in tion, you it hi to the in -
bott e of w ine. She st tes th t she c nnot tient sychi tric u nit n begin ri i r zo e.
h n e w ork ny onger bec u se her boss is After 1 w eek, he is no onger fr i to ook t
trying to h ve her fire n her cow orkers re the te evision bu t he s en s uch of the y
he ing hi fin f u t in her w ork. She c i s cing the f oor, st ting th t he fee s rest ess.
they h ve even t e her hone n re h v- After re u cing the ri i r zo e to the ow est
ing her fo ow e . She re orts ifficu ty f ing ose you be ieve is ro ri te, he sti ces
s ee n oor concentr tion t w ork. She the h s. Which of the fo ow ing is the ost
h s ost 10 b over the st 2 onths, in rt ro ri te next ste ?
Que s tions : 5–13 83

(A) A benztro ine 11. Which of the fo owing is the ost ike y serious
(B) A i henhy r ine co ic tion this tient y ex erience?
(C) A or ze (A) Diffu se intr v scu r co gu tion
(D) A ro r no o (B) Myoc r i inf rction
(E) Discontinu e ri i r zo e n try (C) Pu on ry e bo is
nother nti sychotic (D) Res ir tory f i u re
(E) Rh b o yo ysis
10. A 35-ye r-o w o n resents to the office
st ting th t she is e resse . She w s i
12. Which of the fo ow ing ong-ter si e effects
off fro her w ork 5 w eeks go, n is h v-
of high- ose thiori zine ost ike y e his
ing trou b e fin ing new job. She escribes
hysici n to sw itch to h o eri o ?
fee ing very ow, w ith fee ings of ho e essness
bout the fu tu re, n ifficu ty getting herse f (A) Agr nu ocytosis
u n resse in the orning. She h s ost (B) H y er ro ctine i
5 b n is s ee ing oor y. She re orts one re- (C) Pri is
vious e iso e of e ression. A ition y, she
(D) Retin ig ent tion
escribes erio bou t ye r go in w hich
she st ye u for sever ys in row n- (E) T r ive yskinesi
ning big rty for her hu sb n ’s birth y,
c e ning the hou se n st rting nu erou s Questions 13 and 14
other rojects; t th t ti e, others co ente A 45-ye r-o u ne oye invest ent b nker res-
th t she w s t king qu ick y n cting “ ike ents to you r office cco nie by his w ife. H e h s
the energizer bu nny.” Which of the fo ow ing been ou t of w ork for 1 onth bec u se he is fr i
e ic tions w ou be the ost ro ri te to t ke the tr in into w ork. H e never h ifficu ty
tre t ent for her e ression? tr ve ing u nti 2 onths go, w hen on tr in ur-
(A) Bu ro ion ing f i y v c tion, he su ffere chest in, i-
t tions, shortness of bre th, n use , ting ing in his
(B) F u oxetine
extre ities, n fe r of ying. These sy to s
(C) L otrigine
ste bou t 15 inu tes. A w orku by c r io o-
(D) V roic ci gist w s neg tive. H ow ever, these sy to s h ve
(E) Ven f xine continu e , so cco nie by intense fe r n
fee ing of being et che fro hi se f. H e h s h
Questions 11 and 12 roxi te y tw o tt cks er w eek bu t so h s
severe nxiety w hen just thinking bout returning
A 33-ye r-o inte ectu y is b e e resi ent
to w ork s he is fr i he w i ie on the tr in. H e
of grou ho e resents to the e ergency e rt-
te s you th t he cou not h ve co e to see you
ent escorte by st ff e bers w ho be ieve he h s
w ithou t his w ife’s he .
beco e confu se n isoriente over the st few
ys. H e h s been resi ent there for sever ye rs
13. Which of the fo ow ing e ic tions w ou
w ithou t inci ent, bu t st w eek his thiori zine w s
be the ost ro ri te first choice for this
ch nge to h o eri o bec use of concerns bout
tient?
the ong-ter u se of high oses of thiori zine.
On ex in tion, you fin hi to be isoriente to (A) Bu ro ion
ce n ti e; he is i horetic w ith te er tu re (B) Bu s irone
of 105.8°F, h s he rt r te of 130 be ts/ in, n (C) F u oxetine
res ir tory r te of 20 bre ths/ in. H is extre ities
(D) Pro r no o
re stiff. Rou tine bor tories reve w hite b oo
ce (WBC) cou nt of 15,000/ L, b oo u re nitro- (E) Tr zo one
gen (BUN ) 75 g/ L, cre tinine 1.2 g/ L, n
cre tinine hos hokin se 2,300 Iµ/ L.
84 3: S oma tic Tre a tme nt a nd P syc hop ha rma c ology

14. The tient is nxiou s bou t w iting u nti the (A) Continu e ibu rofen t the rescribe
e ic tion t kes effect, n sks if you c n ose.
give hi so ething th t w i rovi e ore (B) Decre se the ose of ibu rofen.
i e i te re ief. You eci e to rescribe (C) Incre se the ose of ibu rofen.
benzo i ze ine in the interi . Which of the
(D) Sto the ithiu .
fo ow ing e ic tions w ou be the ost
otent? (E) Sw itch fro ibu rofen to s irin.

(A) A r zo Questions 17 and 18


(B) Ch or i ze oxi e
A 9-ye r-o boy is brou ght to the hysici n bec use
(C) Te ze
his rents h ve receive note fro his secon -
(D) Di ze gr e te cher co ining th t he is isru tive in
(E) Ox ze c ss. H is te cher be ieves he cou chieve better
gr es if he cou sit sti n y ttention. H is r-
Questions 15 and 16 ents re ort he h s w ys been n ctive chi bu t
oes h ve ifficu ty getting ong w ith other chi -
A 22-ye r-o sing e e co ege u sic stu ent
ren in the neighborhoo . In the ex in tion roo ,
re u ct nt y resents to you r office t the requ est of
the boy see s un b e to sit sti n h s ifficu ty
his rents. H is rents re w orrie bec u se he h s
co eting t sk you h ve ssigne . You eci e to
not been e ting, t ks const nt y, n never see s
rescribe hi ethy heni te.
to s ee . The tient st tes he h s never fe t bet-
ter, th t he’s been co osing sever songs y,
17. Which of the fo ow ing is the ost co on
n requ ires on y few hou rs of s ee to fu nction.
si e effect of this tre t ent?
H e be ieves he w i be the next gre t rock st r n
w nts to finish the interview in or er to get b ck to (A) D yti e row siness
w riting, bu t he c nnot st y focu se ong enou gh to (B) Decre se in systo ic b oo ressu re
nsw er ny of you r qu estions. H e oes not h ve ny (C) Difficu ty f ing s ee
rior or f i y sychi tric history n enies u sing
(D) Incre se in etite
ny coho or i icit ru gs.
(E) S ow e grow th
15. You eci e to begin tri of ithiu . Before
initi ting tre t ent, w hich of the fo ow ing 18. Which of the fo ow ing sy to s w ou ost
w ou be the ini u bor tory infor tion ike y be ex cerb te by the e ic tion?
th t shou be obt ine ? (A) Be w etting
(A) Co ete b oo cou nt (CBC) (B) Myo i
(B) F sting g u cose n i i ne (C) Poor i u se contro
(C) Liver fu nction tests (D) Re ing ifficu ties
(D) Seru oni (E) Tics
(E) Seru cre tinine, BUN n e ectro ytes,
thyroi stu ies 19. An 8-ye r-o boy is brou ght to you r office
by his other bec u se of his “h bits,” w hich
16. One w eek fter st rting the ithiu , the tient h ve w orsene over the st ye r. She first
s r ins his nk e w hi e exercising. H is ri- notice the st ye r w hen sever ti es
ry c re octor ex ines hi n te s hi y he w ou re e te y b ink his eyes n
to use over-the-counter ibu rofen, w hich he frow n w hi e c e ring his thro t. H e continu es
w i h ve to t ke for ore th n 1 w eek. Which to o this es ite trying not to, n , in i-
of the fo owing wou be the ost ro ri te tion, he often sticks ou t his tongu e n s e s
course of ction? his shirt w hi e s e king w ith c ss tes.
These beh viors h ve resu te in his being
Que s tions : 14–24 85

te se n osing frien s t schoo . Which 22. Sw itching to w hich of the fo ow ing e ic -


of the fo ow ing gents w ou be the ost tions w ou be the ost ike y to i rove
effective to tre t his isor er? these sy to s?
(A) A r zo (A) Ari i r zo e
(B) A itri ty ine (B) C oz ine
(C) C oni ine (C) H o eri o
(D) P roxetine (D) O nz ine
(E) Ris eri one (E) Ris eri one

20. A 45-ye r-o n w ith history of hy er- 23. A 24-ye r-o rrie secret ry h s co -
tension, yoc r i inf rction, n chronic ine of izziness, it tions, n sw e ty
inso ni is referre to you by his ri ry s for the st 10 onths. She so occ -
c re hysici n for ev u tion of his jor sion y h s extre e u sc e tension in her
e ressive e iso e. H e h s been tre te for neck th t occu rs both t w ork n t ho e.
3 onths w ith f u oxetine 60 g i y, w ith She h s ifficu ty f ing s ee n fee s
no i rove ent. H e h s no history of rior “e gy” ost of the y. In ition, she h s
jor e ressive e iso es. Which of the fo - h ifficu ty concentr ting t w ork resu t-
ow ing w ou be the ost ro ri te next ing in her king ist kes. She h s gone to
ste in his tre t ent? oc e ergency roo three ti es bec u se of
the it tions bu t “nothing w s foun .” H er
(A) A ithiu .
f i y r ctitioner ce her on hy og yce-
(B) A thyroi hor one (T3). ic iet n referre her to neu ro ogist w ho
(C) Sw itch to cit o r . fou n no foc bnor ities. She te s you
(D) Sw itch to nortri ty ine. she h s ny w orries bout her f i y but
(E) Sw itch to ven f xine. oes not fee rticu r y e resse . Which of
the fo ow ing e ic tions w ou rovi e the
Questions 21 and 22 ost i e i te re ief of her sy to s?

You h ve been tre ting 55-ye r-o n w ith (A) Bu ro ion


schizo hreni for 20 ye rs w ith h o eri o n (B) Bu s irone
benztro ine. Gener y, he h s one w e n h s (C) C o i r ine
not requ ire jor e ic tion ch nges or hos i- (D) F u oxetine
t iz tions. Abou t 2 ye rs go, you notice so e (E) Lor ze
i s cking n tongu e rotru sions th t i not
bother the tient; how ever, now he so h s o , 24. A 22-ye r-o w o n is itte to sychi-
irregu r ove ents of his r s w hich ke it tric u nit fter seriou s su ici e tte t. She
ifficu t for hi to e t. h s h ny su ici e tte ts in the st
w ith v rying severity. H er r s re sc rre
21. Which of the fo ow ing con itions is ost fro rior tte ts t hu rting herse f. She
ike y the c u se of these sy to s? h been goo stu ent u nti high schoo ,
(A) Anticho inergic toxicity w hen she took u w ith “f st” crow , beg n
(B) H u ntington ise se bu sing coho n riju n , n r n w y
fro ho e sever ti es. She h s h sev-
(C) Meige syn ro e
er intense, stor y re tionshi s w ith en.
(D) Sy enh chore
Ou t tient tre t ent h s ost y consiste
(E) T r ive yskinesi (TD) of her co ints to her ther ist bou t her
f i y. She u su y c s her ther ist iy
bou t ifferent crises; how ever, her ther ist
w s on v c tion u ring her ost recent crisis.
86 3: S oma tic Tre a tme nt a nd P syc hop ha rma c ology

She escribes fee ings of ho e essness n (C) N o e ic tion


he essness consistent y for the st sever (D) O nz ine
w eeks; in ition, she is s ee ing oor y, tire (E) To ir te
throu ghou t the y, istr cte , n h s ost
so e w eight ue to ck of etite. Which of 27. A 36-ye r-o n w ith his first e iso e
the fo ow ing e ic tions w ou be the ost of jor e ression h s been tre te for
he fu for this tient? 8 onths w ith roxetine n is cu rrent y in
(A) C on ze re ission. Bec u se of sexu ysfu nction, he
(B) F u oxetine eci e to sto t king the e ic tion 3 ys
go w ithou t first infor ing you . Yester y, he
(C) H o eri o
bec e cu te y irrit b e n even h crying
(D) Lithiu s e . H e so co ins of bo y ches, chi s,
(E) Zi r si one n gener eth rgy. Which of the fo ow ing
ste s w ou best evi te his sy to s?
25. A 30-ye r-o w o n, 10 w eeks regn nt w ith
her first chi , resents to you r office st ting (A) Rest rt roxetine.
th t she is e resse n is h ving oubts (B) St rt cet ino hen.
bou t beco ing other. She h s been ex e- (C) St rt bu ro ion.
riencing i y e esis n is h ving trou b e (D) St rt v roic ci .
fu nctioning t w ork. She re orts th t she w s (E) St rt ven f xine.
e resse throu ghou t her 20s, but iscontin-
u e her nti e ress nts w hen she bec e 28. A 20-ye r-o w o n is st rte on otrig-
regn nt bec u se of concerns bout h r ing ine for bi o r isor er, cu rrent y e resse .
her chi . She w ou ike you r he in ev u t- Tw o onths ter, she co es b ck re orting
ing risks n benefits of rest rting sertr ine, th t her sy to s re unch nge , es ite
w hich w orke w e for her in the st. Which her being co i nt w ith the e ic tion. You
of the fo ow ing risks w ou be the ost ike y review her e ic tion ist n iscover th t
to occu r? she w s recent y st rte on new e ic tion(s).
(A) Ebstein no y Which of the fo ow ing e ic tions w ou
(B) Gest tion i betes ost ike y be res onsib e for the ck of
effic cy?
(C) Inf nt eve o ent e y
(D) N eur tu be efects (A) Dros irenone n ethiny estr io
(E) Persistent u on ry hy ertension of (B) Ibu rofen
the new born (C) Lithiu
(D) V roic ci
26. A 19-ye r-o w o n is itte to n in - (E) Zi r si one
tient sychi tric f ci ity for her e ting isor-
er. She w eighs 82 b n st n s 5 ft 6 in t . Questions 29 and 30
She beg n ieting in high schoo to ose n
unw nte 5 b. Encour ge by co i ents A 68-ye r-o n resents to you r office on refer-
on her figu re, she continu e to ose nother r fro his ri ry c re hysici n for ev u tion
10 b. H er e ting h bits re ritu ize : She cuts of e ression. H e fee s e resse n ho e ess, n
her foo into s ieces n oves it rou n he h s ost his etite. H e no onger enjoys seeing
on her te. She h s n intense fe r of being his gr n chi ren. It t kes hi sever hours to f
overw eight. Which of the fo ow ing e ic - s ee t night, n he st ys s ee for on y bou t
tions w ou be the ost usefu for this tient? 3 hou rs. H e occ sion y oes not bu y foo bec u se
he is concerne bou t going broke; how ever, his
(A) Cy rohe t ine
f i y infor s you th t he is fin nci y w e -off.
(B) F uoxetine In ition, the tient fee s signific nt gu i t bou t
Que s tions : 25–34 87

beco ing bu r en on his f i y. H is height is 6 ft (A) As irin


w ith w eight of 155 b, ecre se fro his u su (B) Decre se f u i int ke
175 b. (C) H y roch orothi zi e
(D) N itrog ycerin
29. Which of the fo ow ing e ic tions w ou be
the ost ro ri te for tre ting his inso ni ? (E) Pro r no o

(A) Bu ro ion Questions 33 and 34


(B) Di ze
A 47-ye r-o e nove ist h s fe rs of cont in -
(C) Di henhy r ine
tion fro nything he be ieves is “ irty.” In rest u -
(D) F uoxetine r nts, he u ses his ow n stic u tensi s th t he c rries
(E) Tr zo one w ith hi . E sew here, he w e rs g oves or u ses er
tow e s to voi tou ching “ irty objects.” U on
30. Which of the fo ow ing si e effects of this retu rning ho e, he then rocee s to w sh his h n s
e ic tion w ou be the ost i ort nt to 10 ti es before going bou t his ctivities. If he cci-
e u c te the tient bou t? ent y tou ches nything rior to the co etion
(A) Anorg s i of w shing his h n s, he ex eriences nxiety n is
then u n b e to erfor his u su night y t sks. As
(B) Gyneco sti
resu t of these sy to s he re ins u n b e to h ve
(C) I otence e ningfu soci ife.
(D) Pre tu re ej cu tion
(E) Pri is 33. Which of the fo ow ing e ic tions w ou be
the ost effective tre t ent for his con ition?
Questions 31 and 32
(A) C o i r ine
A 57-ye r-o n w ith bi o r isor er, w ho (B) C on ze
you h ve tre te su ccessfu y for 20 ye rs w ith ith- (C) N ortri ty ine
iu 300 g ti , is itte to the gener hos it (D) O nz ine
w ith chest in.
(E) Phene zine
31. Which of the fo ow ing fin ings w ou be
34. After 12 w eeks of ther eu tic ose of the
ost ike y seen on his e ectroc r iogr
e ic tion, the tient no onger nee s to
(ECG)?
w sh his h n s qu ite so often bu t sti h s
(A) First- egree trioventricu r (AV) b ock signific nt istress w hen he touches “ irty
(B) PR interv ro ong tion things.” Which of the fo ow ing tre t ents
(C) QT interv ro ong tion w ou be the ost ro ri te next ste ?
(D) Sino tri b ock (A) A bu s irone.
(E) T-w ve e ression (B) A ithiu .
(C) A ris eri one.
32. The tient is eter ine to h ve su ffere (D) Ch nge to ris eri one.
yoc r i inf rction (MI) n qu ick y u n er-
(E) Refer for cingu oto y.
goes ngio sty. H e is eventu y st bi ize
n isch rge fro the hos it on ro r n-
Questions 35 and 36
o o , to ic nitrog ycerin, hy roch orothi -
zi e, n s irin, in ition to his ithiu . A 21-ye r-o co ege stu ent h s been ex eriencing
Prior to his MI, his ithiu eve w s u su y u itory h u cin tions w ith co n s te ing hi
0.8 to 0.9 o / L. N ow it is 1.3 o / L on to hurt hi se f for ore th n ye r. H e h s h tri-
the s e ose. Which of the fo ow ing is the s w ith the fo ow ing gents since th t ti e: qu e-
ost ike y c u se for this ch nge? ti ine 400 g bi for 2 onths, then h o eri o
88 3: S oma tic Tre a tme nt a nd P syc hop ha rma c ology

10 g bi for 6 w eeks, then o nz ine 30 g i y re ort th t she f i e sever c sses bec u se


for 2 onths. Des ite inor re u ction in the she h s not been going to c ss or tu rning in
voices, he re ins iso te fro f i y n frien s. ssign ents. She is not t king ny e ic -
tions n her toxico ogy screen is neg tive.
35. Which of the fo ow ing gents w ou be the On ent st tus ex in tion, her s eech is
ost ro ri te to begin? ressu re n her thou ght rocess is t ngen-
ti . You eci e to it her to the hos it .
(A) Ari i r zo e
Which of the fo ow ing e ic tions w ou be
(B) C oz ine the ost ro ri te to begin?
(C) Per hen zine
(A) Cit o r
(D) Ris eri one
(B) L otrigine
(E) Zi r si one
(C) O nz ine
36. Which of the fo ow ing rece tors is ost (D) To ir te
ike y res onsib e for the bove e ic tion’s (E) V roic ci
effic cy?
39. A 28-ye r-o w o n w ho you re tre t-
(A) D2
ing for bi o r isor er infor s you th t she
(B) D3 is 3 w eeks regn nt n is nxiou s bou t the
(C) D4 b by’s he th. She is cu rrent y rescribe ith-
(D) 5-hy roxytry t ine-1 (5-H T1) iu 300 g bi , n she h s not h nic
(E) 5-hy roxytry t ine-3 (5-H T3) e iso e in 3 ye rs. Wh t w ou be best to
vise her reg r ing the u se of ithiu uring
37. A 65-ye r-o n w ith jor e ression regn ncy?
n hy ertension resents to the e ergency (A) Lithiu is highest risk u ring the thir
e rt ent bec u se he fe to the grou n fter tri ester of regn ncy.
rising fro ch ir. H e h s no signific nt c r-
(B) Lithiu incre ses the risk of neu r tu be
i c history n his ECG, e ectro ytes, n
efects.
neu ro ogic ex in tion re u nre rk b e.
H is f i y re orts th t his hysici n recent y (C) Lithiu c u ses eve o ent e y in
st rte new nti e ress nt bu t they o not ex ose inf nts.
know its n e. Which of the fo ow ing nti e- (D) Lithiu incre ses the risk of c r i c
ress nts w ou ost ike y be res onsib e for for tion.
his resent tion? (E) Lithiu incre ses the risk of c eft i
n te.
(A) Bu ro ion
(B) F u oxetine 40. You re c e to consu t on n 85-ye r-o
(C) I i r ine surgic tient w ho h s beco e co b tive,
(D) Mirt z ine ye ing, u nching st ff, n u ing ou t her
(E) N ortri ty ine IVs. She e n s to e ve bu t is too w e k
to get ou t of her hos it be . Which of the
38. A 21-ye r-o w o n w ith history of bu i- fo ow ing w ou be the ost ro ri te
i nervos resents to the e ergency intervention t this ti e?
e rt ent w ith her rents. She returne (A) Di henhy r ine
fro co ege st w eek, n since th t ti e,
(B) Done ezi
h s been st ying u very te, c ing frien s
roun the w or n “t king nonsense” (C) Lor ze
bout n internet st rt-u th t she is nning. (D) Orient tion to her su rrou n ings
She sks you to hu rry u w ith you r interview, (E) Ris eri one
since she h s w ork to get b ck to. H er rents
Que s tions : 35–46 89

Questions 41 and 42 43. Which of the fo owing gents wou be the


ost ro ri te to inister for his git tion?
A 67-ye r-o w o n resents to your office for
ev u tion of 20- b w eight oss over 4- onth (A) Ch or ro zine
erio n inso ni for the st 4 w eeks. H er (B) H o eri o
ughter infor s you th t her other no onger (C) Lor ze
enjoys ny of her hobbies n h s been s e king (D) Ris eri one
te y bou t ying. They h ve re y consu te
(E) Thiori zine
n internist w ho erfor e n extensive e ic
w orku , inc u ing che istries, b oo cou nt, thy-
44. Which of the fo ow ing gents w ou be the
roi stu ies, g strointestin i ging, n en os-
ost ro ri te to give in or er to ini ize
co y, of w hich w ere nor .
the tient’s risk of f ing?
41. You w ou ike to begin n nti e ress nt (A) H o eri o
e ic tion bu t w ou ike to ini ize (B) Per hen zine
verse si e effects w hi e “ex oiting” other (C) Qu eti ine
si e effects. Which of the fo ow ing gents
(D) Ris eri one
w ou be the ost ro ri te?
(E) Thiori zine
(A) Bu ro ion
(B) F u oxetine Questions 45 and 46
(C) Mirt z ine
A 32-ye r-o n resents to your c inic fter osing
(D) N ortri ty ine his job bec use he w s intoxic te w hi e w orking. H e
(E) Sertr ine h s been rinking i y since he w s 16 ye rs o . H e
w s b e to co ete co ege n w ent to w ork fu -
42. U on fu rther history, you e rn th t she h s ti e right fter gr u tion, but h s ost sever jobs
h sever f i e tri s of nti e ress nts since. H is w ife h s thre tene to e ve hi if he oes
n w ou ike to st rt tricyc ic nti e res- not get he .
s nt. Which of the fo ow ing con itions w ou
be the ost ike y to rec u e u sing su ch n 45. Which of the fo ow ing e ic tions w ou
gent? be the ost u sefu s beh vior o ifier to
ecre se his coho use?
(A) Estrogen re ce ent ther y
(B) L cu n r inf rcts (A) Ac ros te
(C) Left bu n e br nch b ock on ECG (B) Disu fir
(D) Tre t ent w ith o i ine (C) F u zeni
(E) Urin ry retention (D) N oxone
(E) N trexone
Questions 43 and 44
46. Further history reve s th t he oes not h ve
A 72-ye r-o n is itte to gener hos i-
oo isor er. He te s you th t often he si y
t ’s intensive c re u nit bec u se of tere ent
c nnot contro his cr ving to h ve nother rink.
st tu s. H is e ic w orku h s reve e neu o-
Which of the fo owing gents wou be the
ni n congestive he rt f i u re (CH F). On the sec-
ost ike y to ecre se his cr vings for coho ?
on hos it y, he is git te n u s out his
IV ccess. H e so h s been note to s e k ou t ou (A) Bu ro ion
w ith no one in the roo . H is eve of consciou sness (B) Disu fir
see s to w x n w ne. H e oes not h ve sychi- (C) F u zeni
tric history n is not ergic to ny e ic tions. (D) N oxone
Besi es his CH F n neu oni , he oes not h ve
(E) N trexone
other co orbi con itions.
90 3: S oma tic Tre a tme nt a nd P syc hop ha rma c ology

Questions 47 and 48 bou t 1 onth go w hen he beg n h ving if-


ficu ty s ee ing. The st ff hysici n rescribe
A 36-ye r-o w o n is referre to you by her ri- or ze 2 g t be ti e; how ever, he w s
ry c re hysici n for n ge ent of nxiety sti u n b e to s ee . H e st rte w n ering
n fe r th t h s ersiste for sever onths. She u ring the y n h s since been restricte
re orts th t she w s r e n he host ge on to his roo . Bec u se he re ins git te u r-
bo t for sever ys by tw o r e en. Du ring ing the y, they h ve been giving hi or z-
this, she ex erience intense fe r for her ife. Since e 2 g every 6 hou rs for git tion. H is
then, she h s h intense stress w henever she is other e ic tions inc u e hy roch orothi -
ne r the w ter n h s frequ ent night res. She zi e 25 g/ , igoxin 0.125 g every other
c nnot rec et i s of the or e bu t tries to voi y, i ti ze 240 g su st ine re e se i y,
riving her c r w ithin sight of the oce n, w hich h s n tr ns er nitrog ycerin 0.4 g/ h s
been ifficu t bec u se of the oc tion of her ho e. nee e . H is e ectro ytes re nor exce t
She fee s et che fro her hu sb n n f iy for ot ssiu eve of 3.4 o / L. A recent
n h s b n one ns to u rsue her c reer s igoxin eve is not v i b e. Which of the
inter. She h s ifficu ty f ing s ee n is e si y fo ow ing interventions shou be consi ere
st rt e by hone c s. As resu t of her ex erience, first?
she no onger goes to u b ic ces one.
(A) Checking the hy roch orothi zi e eve .
47. Which of the fo ow ing e ic tions w ou be (B) Monitoring orthost tic vit s.
ost he fu in tre ting the sy to s she is (C) Re cing the ot ssiu .
ex eriencing? (D) Sto ing the roo sec u sion.
(A) A r zo (E) T ering the or ze .
(B) C rb ze ine
(C) F u oxetine Questions 50 and 51
(D) N trexone A 64-ye r-o n, cco nie by his w ife, res-
(E) Thiori zine ents to you r office co ining of e ory oss. H e
is retire stockbroker w ho c n no onger rec
48. She is tre te w ith the ro ri te e ic - stock qu otes ike he i sever ye rs go. H is w ife
tion n h s signific nt re u ction in ost h s beco e both concerne n nnoye bec u se
of her sy to s. H ow ever, she continu es he see s to i e i te y forget w h tever she te s
to ex eriences frequ ent, intense night res, hi . These e ory rob e s h ve s ow y ro-
resu ting in her h ving oor s ee n f tigu e gresse over sever ye rs. Recent y, he h iffi-
the next y. Which of the fo ow ing gents cu ty getting resse , n ore th n once, he u t
w ou be the ost benefici in re u cing these his u n erw e r on over his nts. H is w ife h s so
sy to s? note th t he s e ks u ch ess u ring inner th n
he once i . H e oes not h ve ny other signific nt
(A) Lor ze
e ic i nesses.
(B) N trexone
(C) O nz ine 50. Which of the fo ow ing e ic tions w ou be
(D) Per hen zine ost ike y to i rove his con ition?
(E) Pr zosin (A) A r zo
(B) Done ezi
49. You h ve been ske to ev u te n 80-ye r-
o e nu rsing ho e resi ent bec u se of (C) F u oxetine
ch nge in ent st tu s. The nu rsing st ff (D) Methy heni te
re orts th t he h s been tient there for (E) Tr zo one
bou t ye r w ithout ny rob e s, u nti
Que s tions : 47–56 91

51. U on fu rther questioning, his w ife re orts th t 54. A 48-ye r-o w o n resents to you r r ctice
he oes not s ee w e t night n w n ers w ith co ints of e ression. H er other
rou n the hou se i ess y. Su bsequ ent y, he sse w y 6 onths go fro u ng c ncer
s ee s for rge rt of the y. Which of n her f ther ie 4 ye rs go fro the s e
the fo ow ing e ic tions w ou be the ost ise se. She h s seen ther ists to he w ith
u sefu to contro this beh vior? her grief but she continu es to h ve ongoing
s ness, ter in inso ni , ow energy, oor
(A) H o eri o
etite, n ifficu ty concentr ting. She
(B) Lor ze h s ssive thou ghts of su ici bu t no n
(C) Tr zo one or intent. She enies rior su ici e tte ts.
(D) Tri zo In ition, es ite sever tte ts to qu it
(E) Qu eti ine s oking, she sti s okes tw o cks of cig -
rettes y n h s h w orsening sth s
Questions 52 and 53 resu t. She h s no ition e ic rob-
e s. Which of the fo ow ing e ic tions
A 25-ye r-o n is brou ght into the e ergency w ou be the ost ro ri te to u se in this
e rt ent eth rgic n stu orou s. H e res on s tient?
on y to infu sti u i, w kes u brief y n ye s, then
goes b ck to s ee . A bu nce ersonne re ort (A) Bu ro ion
th t they fou n hi ne r hou se know n for ru g (B) Bu s irone
tr fficking. There is no evi ence of hysic inju ry. (C) I i r ine
(D) Phene zine
52. Which of the fo ow ing e ic tions shou he (E) Tr zo one
receive first?
(A) Dextrose n f u zeni 55. A 52-ye r-o n w ho h s oor y contro e
(B) Dextrose, f u zeni , n n oxone i betes is referre to you for ev u tion of
e ic tion nonco i nce. You e rn th t he
(C) Dextrose, f u zeni , n oxone, n
c nnot f s ee t night n h s no energy
thi ine
u ring the y. H is etite is gone n he
(D) Dextrose n n oxone
oes not enjoy ctivities s he once i . Over-
(E) Dextrose, n oxone, n thi ine he fee s e resse . A ition y, he h s
been ex eriencing chronic in in his feet,
53. The tient is su bsequent y tre te n u rine ee e to be u e to neu ro thic in, w hich
toxico ogy screen is ositive for the resence of is not re ieve by n gesics. Which of the fo -
o ioi s. When he is ore ert, he infor s you ow ing e ic tions w ou be the ost u sefu
th t he h s been using intr venous (IV) heroin in tre ting this tient’s sy to s?
i y for sever w eeks. H is st use w s bout
8 hours go. H is heroin u se esc te fro (A) A r zo
snorting to IV use fter he no onger fe t goo (B) A itri ty ine
“high” fro snorting. H e h s never etoxifie (C) Bu ro ion
before. H e h s no co orbi e ic con i- (D) Cit o r
tions n he oes not use coho . Which of (E) Du oxetine
the fo ow ing e ic tions w ou be the ost
ro ri te to etoxify this tient? 56. A 47-ye r-o w o n resents to your office
(A) Bu renor hine for sychi tric c re bec u se she h s ju st re o-
(B) Ch or i ze oxi e c te to you r tow n. She h s history of bi o-
r isor er, bu t her rior sychi trist recent y
(C) C oni ine
sto e her ithiu bec u se she eve o e
(D) N oxone
thyroi uto ntibo ies. She co ins of being
(E) Pro r no o irrit b e n not b e to s ee for sever weeks.
92 3: S oma tic Tre a tme nt a nd P syc hop ha rma c ology

She h s “ ots” of energy bu t is king is- (D) Sw itch to o nz ine.


t kes t w ork s she is istr cte . She te s you (E) Sw itch to ris eri one ong- cting
th t her thou ghts re r cing n you h ve injection.
ifficu t ti e re irecting her nsw ers. H ow -
ever, she be ieves her biggest rob e is her Questions 59 and 60
f ci in, w hich w s i gnose s trige i-
n neur gi n h s not i rove w ith n - A 50-ye r-o n w ho h s fe t e resse for sev-
gesics. Which of the fo ow ing e ic tions er onths, resents to you r office for ev u tion.
w ou be the ost ro ri te in tre ting this He h s h oor s ee , ecre se etite w ith
tient’s sy to s? w eight oss, no interest in his hobbies, n c nnot
concentr te u ring you r cognitive ex in tion. H e
(A) A itri ty ine h s ssing thou ghts of ying bu t no ctive n or
(B) C rb ze ine intent. H e re u ct nt y its to frequ ent y he ring
(C) F u oxetine voices te ing hi th t he is going to ie soon. You
(D) G b entin su bsequ ent y eci e to st rt n nti e ress nt n
(E) Lithiu ris eri one t 0.5 g bi .

57. A 33-ye r-o w o n w ith history of schizo- 59. Which of the fo ow ing si e effects of ris eri-
hreni h s recent y st rte tre t ent w ith one w ou be the ost ike y?
o nz ine. She h s to er te the e ic tion (A) Agr nu ocytosis
w e n is iving in grou ho e, ntici t- (B) Anticho inergic si e effects
ing oving into her ow n rt ent. She no
(C) Leu kocytosis
onger he rs voices n no onger h s esire
to hurt herse f. Which of the fo ow ing si e (D) Orthost tic hy otension
effects w ou be ost ike y to interfere w ith (E) Weight oss
her co i nce?
60. As his ose of ris eri one is incre se , the
(A) Agr nu ocytosis risk of w hich of the fo ow ing si e effects w i
(B) Deve o ent of c t r cts so incre se?
(C) G ctorrhe
(A) Agr nu ocytosis
(D) Incre se excit bi ity
(B) Anticho inergic effects
(E) Weight g in
(C) Extr yr i sy to s
58. A 32-ye r-o e w ith schizo hreni is (D) Leu kocytosis
being tre te for n cu te ex cerb tion of his (E) Weight oss
u itory h u cin tions. As is his ttern, he
res on s very f vor b y to e ic tions w hi e 61. A 33-ye r-o rrie fe e w ith history
hos it ize , bu t w hen he is isch rge for- of bi o r isor er, w ith eri rtu onset,
gets to t ke the regu r y. This resu ts in h s been st b e since being int ine on ris-
cyc e of subsequent w orsening of his sy- eri one for the st sever ye rs. H er son is
chosis n rehos it iz tion. H e is current y oing w e , n the tient n hu sb n w ish
t king ris eri one 5 g/ or y, n he is to h ve nother chi . For the st ye r she
to er ting it w e w ith goo effic cy. Which of h s been tte ting to conceive w ithou t su c-
the fo ow ing w ou be the ost ro ri te cess. She is on no other e ic tions besi es
next ste in his tre t ent? the nti sychotic, n she h s not been u sing
birth contro . She h s no other e ic rob-
(A) Continu e the cu rrent regi en. e s, n she is current y sy to tic, w ith-
(B) Sw itch to c oz ine. ou t e ression or sychotic sy to s. She
(C) Sw itch to h o eri o . rinks 1 to 2 g sses of w ine er w eek, enies
i icit rugs, n oes not use tob cco ro ucts.
Que s tions : 57–66 93

Invo ve ent of w hich of the fo ow ing br in 64. A 45-ye r-o fe e execu tive w ho tr ve s
thw ys is ost ike y res onsib e for her often to Eu ro e fro the Unite St tes res-
inferti ity? ents to you r office co ining of signific nt
inso ni n yti e f tigu e fter tr ns t n-
(A) Locus ceru eu s
tic f ights. Which of the fo ow ing e ic tions
(B) Mesocortic tr ct w ou be the ost ro ri te tre t ent for
(C) Meso i bic tr ct her co ints?
(D) N igrostri t tr ct
(A) A itri ty ine
(E) Tu beroinfu n ibu r tr ct
(B) Di henhy r ine
Questions 62 and 63 (C) Me tonin
(D) Qu eti ine
A 19-ye r-o co ege stu ent resents to the office (E) Te ze
t the u rging of her frien s, w ho re concerne
bout her w e -being. For the st sever onths, Questions 65 and 66
she h s been incre sing y reoccu ie w ith her
w eight n bo y i ge, n h s been regu r y A 60-ye r-o n resents to you r office co in-
c nce ing ns w ith frien s bec u se she oes not ing of the su en onset of it tions, sw e ting,
w nt to e t w ith other eo e. She st tes th t she is sh king, shortness of bre th, choking, n fee ing
“f t” n w ou ike to ose 10 b. When qu erie fu r- th t he is “going cr zy.” These sy to s st 5 to
ther, she its th t she is bingeing n throw ing 10 inu tes n h ve occu rre sever ti es er
u ost every y. She enies ny x tive use, w eek for the st 3 onths. H e enies ny rtic-
n she h s not been restricting her e ting. She is u r stressor or event w hich rovokes the tt cks,
5 ft 4 in n w eighs 110 b. H er ot ssiu is 3.4; thou gh he is “const nt y” fr i of h ving futu re
other e ectro ytes re w ithin nor i its. You e iso es. H e h s recent y been seen by his c r io o-
re ete her ot ssiu n refer her to sychi trist gist w ho h s ru e ou t c r i c etio ogy.
for fu rther ev u tion.
65. Which of the fo ow ing w ou be the ost
62. Which of the fo ow ing w ou be the ost ro ri te e ic tion to tre t this tient’s
effective e ic tion for her con ition? isor er?
(A) Bu ro ion (A) Bu ro ion
(B) F u oxetine (B) I i r ine
(C) L otrigine (C) Phene zine
(D) Lithiu (D) Qu eti ine
(E) Mirt z ine (E) Sertr ine

63. You ter e rn th t she w s tre te in high 66. If the tient’s sy to s w ere u e to c r-
schoo for the s e sy to s w ith sertr ine i c etio ogy n he su bsequ ent y bec e
n cit o r , both t high oses for over e resse , w hich e ic tion w ou ose the
3 onths e ch, bu t w ith itt e i rove ent in highest risk?
her sy to s. Which of the fo ow ing e i-
(A) A itri ty ine
c tions w ou be the ost ro ri te next
ste ? (B) Bu ro ion
(C) Methy heni te
(A) Bu ro ion
(D) Sertr ine
(B) Lithiu
(E) Ven f xine
(C) P roxetine
(D) To ir te
(E) V roic ci
94 3: S oma tic Tre a tme nt a nd P syc hop ha rma c ology

67. A 33-ye r-o w o n w ith i gnosis of of the fo ow ing si e effects o you infor hi
schizo hreni h s been int ine on h o- is the ost ike y?
eri o since she w s i gnose t ge 24. In
(A) He che
you r office, she s ys she h s not h her en-
stru erio in sever onths, h s i in- (B) H e ring oss
ishe sex rive, h s been isch rging i k (C) Me ory oss
fro her bre sts, n h s in u ring sexu (D) Seizures
intercou rse. Which of the fo ow ing w ou (E) Tooth in
ost ike y be incre se in this tient?
(A) A renocorticotro ic hor one (ACTH ) 70. You h ve been tre ting 33-ye r-o w o n
w ith bi o r isor er for 4 ye rs. In re r -
(B) Do ine
tion for nne regn ncy, you h ve gree
(C) N ore ine hrine to t er her e ic tions but see her fre-
(D) Pro ctin qu ent y in c se of nic e iso e. Six w eeks
(E) Serotonin into her regn ncy, she resents to you r office
w ith ressu re s eech n the be ief th t her
68. A 24-ye r-o n is hos it ize bec use b by is Jesu s Christ. H er hu sb n s ys he is
he h s been he ring voices for 6 onths te - concerne bec u se she h s sto e t king
ing hi to ki hi se f n now fe rs th t he her ren t vit ins n is cting biz rre y.
y ct on the . H e ost his job 2 onths go Which of the fo ow ing tre t ents w ou be
n on ent st tu s ex in tion e rs the s fest to tre t her isor er?
u nke t n isorg nize , w ith f t ffect.
(A) C rb ze ine
H e is begu n on ri i r zo e, n his sy -
to s eventu y ecre se enou gh for hi to (B) Di ze
be isch rge . On fo ow -u visit to you r (C) Div roex so iu
office, he re orts th t his h ucin tions h ve (D) E ectroconvu sive ther y (ECT)
i rove bu t th t he “c n’t sit sti ” n fee s (E) Lithiu c rbon te
ike he nee s to be const nt y in otion. The
tient inter ittent y st n s u n w ks 71. You re the consu t- i ison sychi trist on c
rou n you r office s you interview hi . for the tr u su rgery te . You re ske to
Which of the fo ow ing con itions ccount for ev u te 39-ye r o n w ho w s hit by
his new co ints? c r n severe y inju re . H e is un b e to give
(A) Acu te ystonic re ction history, bu t n cqu int nce w ho cco -
nie the tient to the hos it re orts th t he
(B) Ak thisi
is i y, he vy ru g u ser. On ex in tion,
(C) M nic e iso e the tient is note to h ve tr ck rks on his
(D) N eu ro e tic ign nt syn ro e r s. Which of the fo ow ing c sses of ru gs
(E) T r ive yskinesi o you te the surgeons is the ost ngerou s
if bru t y w ith r w n?
69. A 55-ye r-o n w ith ong history of jor
(A) C nn binoi s
e ression resents to your office fter u ti-
e f i e nti e ress nt tri s. Since the ge of (B) H u cinogens
20, he h s h equ te tri s of nortri ty ine, (C) O i tes
hene zine, f uoxetine, sertr ine, bu ro ion, (D) Se tive-hy notics
irt z ine, ven f xine, n u oxetine, w ith (E) Sti u nts
ini res onse. Other sychi trists h ve
suggeste ECT in the st, bu t he is concerne Questions 72 and 73
bout e ory oss. H e w ou ike to try tr n-
scr ni gnetic sti u tion (rTMS), n A 44-ye r-o n w ith history of chronic, severe
w nts to know ore bout si e effects. Which coho u se isor er is brou ght to the e ergency
Que s tions : 67–81 95

roo . You get re ort fro f i y e ber th t st rte on cit o r n h s n incre se of


he “h s been iving off booze n ru gs n nothing 11 sec in his QTc v u e on ECG.
e se for w eeks.” The e ergency e rt ent nu rse
is bou t to go in to r w b oo fro the tient n 75. A 51-ye r-o w o n being tre te for schizo-
offer hi foo . hreni eve o s i betes.

72. Which of the fo ow ing is the ost ro ri te


next ste in the tre t ent of this tient? 76. An 11-ye r-o gir being tre te for jor
e ressive isor er re orts frequ ent su ici
(A) A inister or ze . i e tion.
(B) A inister thi ine.
(C) Obt in co ete b oo cou nt. 77. A 34-ye r-o w o n being tre te for bi o-
(D) Obt in iver fu nction tests. r I isor er eve o s cu te in in her b o-
(E) Obt in vit in B12 eve . en n is i gnose w ith ncre titis.

73. After he is itte to the e icine service, 78. The WBC count of 27-ye r-o n being
w hich of the fo ow ing e ic tions w ou be tre te for the tre t ent refr ctory schizo-
the ost ro ri te to initi te? hreni ro s to 3,000/ L.
(A) Ac ros te
(B) C oni ine 79. A 42-ye r-o n sto s his nti e ress nt.
(C) Disu fir Tw o ys ter, he ex eriences fee ing of
(D) Lor ze ho e essness, h s ifficu ty s ee ing, n fee s
qu ite nxious.
(E) N trexone

D IRECTION S (Questions 74 through 83): The fol- 80. A 36-ye r-o wo n being tre te for bi o r
low ing group of numbered items are preceded by isor er fin s herse f urin ting very frequent y.
a list of lettered options. For each vignette, select
the one lettered option that is most closely associ- 81. Sever onths fter st rting on new nti-
ated w ith it. Each lettered option may be used sychotic, 32-ye r-o tient’s tot cho es-
once, multiple times, or not at all. tero incre ses fro 200 to 250 g/ L.

Questions 74 through 81
Questions 82 and 83
For e ch vignette, choose the e ic tion ost ike y
For e ch vignette, choose the e ic tion ost
to be ssoci te w ith the si e effect.
ro ri te to rescribe.
(A) C rb ze ine
(A) A r zo
(B) C oz ine
(B) Cit o r
(C) Div roex so iu
(C) Div roex so iu
(D) H o eri o
(D) F u oxetine
(E) Lithiu
(E) H o eri o
(F) P roxetine
(F) Lithiu
(G) Per hen zine
(G) C oz ine
(H ) Pi ozi e
(H ) P roxetine
(I) Zi r si one
(I) Ris eri one
74. A 14-ye r-o boy being tre te h r co- (J) Sertr ine
ogic y for Tou rette isor er is ition y (K) Tr zo one
96 3: S oma tic Tre a tme nt a nd P syc hop ha rma c ology

82. A 34-ye r-o n w ith schizo ffective isor- (C) C o i r ine


er h s been nonco i nt w ith e ic tions (D) F u oxetine
in the st. You w nt to be b e to give hi (E) F u vox ine
e ot injection of n nti sychotic.
Questions 86 and 87
83. You re tre ting 45-ye r-o veter n w ith
You re tre ting 28-ye r-o n w ith 6-ye r
history of coho use isor er for osttr u -
history of schizo hreni w ho ives w ith his rents.
tic stress isor er (PTSD), n he sks you
The tient h s h nu ber of ebi it ting si e
for so ething to he hi s ee .
effects, su ch s severe extr yr i sy to s,
fro tr ition nti sychotics inc u ing h o eri-
D IRECTION S (Questions 84 through 105): For o . H is f i y is w orrie bou t w eight g in, w hich
each of the multiple-choice questions in this sec- the tient ex erience on o nz ine. Des ite the
tion, select the lettered answ er that is the one best nu erous e ic tions, his sychotic sy to s
response in each case. continu e. You iscu ss c oz ine w ith the tient
n his f i y n they gree to try the e ic -
Questions 84 and 85 tion. You h ve initi te tre t ent s ow y n he h s
A 28-ye r-o sing e w o n resents w ith 8 w eeks show n signific nt i rove ent; how ever, he h s
of e ression, inso ni , ow energy n f tigu e, been ro u cing rge ou nt of s iv n roo
istr ctibi ity, n oor etite. She h s no rior so king his shirts.
sychi tric history n no f i y sychi tric his-
tory. After further history n bor tory ev u - 86. Which of the fo ow ing is the ost ro ri te
tions re co ete , she is begu n on sertr ine, next ste ?
w hich is eventu y incre se to 100 g/ . After (A) A nt ine.
2 onths on th t ose, the tient re orts th t she (B) A ro r no o .
fee s “b ck to nor ” n sks w hen she c n sto
(C) A su b ingu tro ine.
the e ic tion.
(D) Discontinu e c oz ine.
84. H ow ong shou you reco en her to (E) Incre se the ose of c oz ine.
continu e the e ic tion?
87. In ition to the bove, the tient co ins
(A) 1 onth of it tions. An ECG reve s sinu s t chy-
(B) 2 onths c r i , w hich is consistent w ith rior ECGs
(C) 4 onths erfor e over the st onth. Which of the
(D) 10 onths fo ow ing is the ost ro ri te next ste ?
(E) 2 ye rs (A) A benztro ine.
(B) A bet o .
85. She returns for fo ow-u visit 1 onth ter.
(C) A or ze .
Her sy to s h ve continue to re in in
(D) A ro r no o .
re ission, though she escribes recent con-
f ict with her boyfrien ue to h ving “ itt e sex (E) Discontinu e c oz ine.
rive.” The tient st tes th t it h s gotten to the
oint where she h s eci e to sto the e ic - 88. A 21-ye r-o sing e fe e co ege stu ent
tion un ess so ething c n be one. She is wi - is brou ght to the u niversity he th service by
ing to switch to nother e ic tion es ite the her roo tes. She kes it c e r to you th t
risk of re se. Which of the fo owing gents she oes not think there’s nything w rong
wou be the ost ro ri te choice? w ith her, bu t her roo tes te you th t they
“c n’t e w ith her ny ore” bec u se she
(A) Bu ro ion never s ee s n “never shu ts u .” The tient
(B) Cit o r insists this is bec u se she h s iscovere n
Que s tions : 82–93 97

i ort nt the tic roof th t she u st 91. The tient is 59-ye r-o fe e w ith schizo-
finish w riting. She’s convince th t the uni- hreni , s w e s oor y contro e i betes
versity w i i e i te y gr nt her tenure s e itu s, obesity, hy er i i e i , n hy er-
soon s they see the roof, thou gh she h s tension. She h s been tre te for over 35 ye rs
troub e st ying on to ic w hen you sk qu es- w ith v riou s first-gener tion nti sychotics,
tions bout it. She h s not seen sychi trist n thou gh she is on n equ te ose, she
before, n u rine toxico ogy screen, w hich sti h s resi u sychotic sy to s n h s
she consents to “to rove I’ not cr zy,” is now begu n to eve o invo u nt ry b inking
neg tive. Which of the fo ow ing is the ost n tongu e-ro ing ove ents. She is gree-
ro ri te gent for initi n inten nce b e to sw itching to n ty ic / secon -gen-
tre t ent of this tient? er tion nti sychotic. Which of the fo ow ing
e ic tions w ou be the ost ro ri te
(A) H o eri o
for this tient?
(B) L otrigine
(C) Lithiu (A) Ari i r zo e
(D) Lor ze (B) C oz ine
(E) O nz ine (C) O nz ine
(D) Qu eti ine
89. A 34-ye r-o e ectric engineer co ins to (E) Ris eri one
his octor th t he h s been fee ing e resse
for sever onths fter the bre ku of his 92. You ev u te 34-ye r-o ivorce , e, w ith
rri ge. Since th t ti e, he h s h signifi- history of coho use isor er w ho resents
c nt inso ni , ifficu ty concentr ting, h s w ith the chief co int of “I c n’t sto rink-
ost 25 b in the st 3 onths bec u se his ing, oc.” He h s h sever reh bi it tion
etite isn’t the s e, fee s f tigu e , h s sig- issions but is un b e to int in sobriety.
nific nt fee ings of gu i t reg r ing the se r - He is current y rinking one-fifth of vo k
tion, fee s ho e ess n he ess, n h s fe t ost i y. Whi e he is very iscour ge n
su ici w ithou t s ecific n. A i gnosis of gui ty bout his coho use, he is ho efu bout
jor e ressive isor er is e n he is getting better, n he enies erv sive e res-
begu n on n nti e ress nt. Which of the fo - sive sy to s or suici i e tion. H e h s
ow ing sy to s w ou be ex ecte to t ke rior history of signific nt coho w ith r w ,
the ost ti e to i rove? inc u ing one seizure n e iriu tre ens,
both requiring n ICU ission. Which of the
(A) Decre se etite
fo ow ing e ic tions w ou be ost ro-
(B) Decre se concentr tion ri te to rescribe for this tient?
(C) Decre se energy
(A) Disu fir
(D) Inso ni
(B) Meth one
(E) Su ici ity
(C) N trexone
90. A 58-ye r-o n w ith history of cirrho- (D) Sertr ine
sis, he titis C, n coho is w ishes to qu it (E) V roic ci
rinking bu t w nts to etox first. Which of
the fo ow ing e ic tions w ou be the ost 93. A 35-ye r-o sing e n resents to you r
ro ri te to rescribe in this tient? office co ining of inso ni . H e re orts
th t he is b e to f s ee t night, bu t st ys
(A) A r zo
s ee for on y 2 or 3 hou rs, then w kes u fre-
(B) Ch or i ze oxi e quent y throu ghout the night. H e is exh uste
(C) C on ze n w ou ike so ething to he hi s ee .
(D) Di ze Other th n fee ing fru str te n f tigue
(E) Lor ze u ring the y, he enies other e ressive
98 3: S oma tic Tre a tme nt a nd P syc hop ha rma c ology

sy to s or ny st sychi tric history. e ic tions w ou be the ost ro ri te to


Which of the fo ow ing e ic tions w ou be rescribe?
the ost ro ri te for this tient?
(A) Bu s irone
(A) A itri ty ine (B) F u oxetine
(B) R e teon (C) F u vox ine
(C) Tr zo one (D) Lor ze
(D) Z e on (E) Pro r no o
(E) Zo i e
97. A 30-ye r-o e resents w ith 4 w eeks
94. A 40-ye r-o rrie , regn nt w o n w ith of ow energy, oor s ee , oor etite,
history of hy ertension, yoc r i inf rc- ecre se interest in e su r b e ctivities,
tion, egener tive isc ise se, n jor n ow oo . H e re orts being hos it -
e ressive isor er w ith sychotic fe tu res ize s you ng u t for 2-w eek erio of
is being consi ere for e ectroconvu sive gr n iosity, hy ersexu ity, ecre se nee
ther y (ECT) fter f i ing u ti e nti e- for s ee , n eng ging in risky beh viors. H e
ress nt n nti sychotic tri s. Which of w s st rte on ithiu bu t i not to er te the
the fo ow ing con itions w ou be re tive e ic tion u e to eve o ent of i betes
contr in ic tion to ECT? insi i u s n h s not t ken e ic tion since.
H e w orks s n ccou nt nt, his BMI is 35, n
(A) Degener tive isc ise se
his st A1c w s 5.2. Which of the fo ow ing
(B) H y ertension e ic tions w ou be the ost ro ri te to
(C) Pregn ncy rescribe for this tient?
(D) Psychotic e ression
(A) Lithiu
(E) Recent yoc r i inf rction
(B) Lu r si one
95. You re c e to consu t on 70-ye r-o (C) Qu eti ine
w o n, hos it ize in the ICU fter yo- (D) Sertr ine
c r i inf rction. The tient is ye ing t (E) V roic ci
st ff, u ing ou t her IV n trying to get out
of be . She refuses to t ke ny e ic tion by 98. A 29-ye r-o w o n w ith history of sei-
outh. H er ECG show s QTc of 480. Which zu res is i gnose w ith bi o r isor er,
of the fo ow ing e ic tions w ou be best to ost recent e iso e e resse , n st rte
he her git tion? on otrigine. Abou t w hich of the fo ow ing
si e effects w ou be the ost i ort nt to
(A) Ari i r zo e IM
e u c te the tient?
(B) Di ze IV
(C) H o eri o IV (A) Acu te ystoni
(D) Lor ze IV (B) Ak thisi
(E) Ris eri one Const IM (C) A stic ne i
(D) R sh
96. A 30-ye r-o w o n w ith no reviou s sy- (E) Ren f i u re
chi tric history resents to you r office sking
for he in n ging erfor nce nxiety. 99. A 19-ye r-o n w ith bi o r isor er is
She oves to ct n sing in her co unity brought in to the e ergency roo by the
the tre, but she beco es terrifie w hen get- o ice. H e is gr n iose n e usion , s w e
ting on st ge in front of crow s. As resu t, s git te n co b tive. H e is given n
her voice sh kes n she h s been un b e intr uscu r injection of h o eri o . After
to get “goo rts.” Which of the fo ow ing 30 inutes, the tient is fou n rching for-
w r . On hysic ex in tion, stiffness of the
Que s tions : 94–104 99

neck n b ck usc es re note . Which of the 102. A 58-ye r-o n w ith tri fibri tion,
fo ow ing ter s best escribes this si e effect? hy ercho estero e i , n hy ertension is
sent to you by his ri ry c re hysici n
(A) L ryngos s
for he in n ging his nxiety. H e re orts
(B) Ocu ogyric crisis th t for ny ye rs he h s w orrie “ bout
(C) O isthotonos everything,” n th t he occ sion y h s to
(D) P eu rothotonos st y ho e fro w ork bec use he beco es so
(E) Tortico is fe rfu . H e often h s ifficu ty f ing s ee
n not u nco on y gets tension he ches
100. The tient is 59-ye r-o w o n w ith st w hen rticu r y stresse . H is e ic tions
e ic history of hy ercho estero e i , i - inc u e w rf rin, torv st tin, n hy ro-
betes, n i betic neu ro thy, w ho resents ch orothi zi e. H e oes not rink coho or
to her ri ry c re octor w ith the chief co - c ffeine. Which of the fo ow ing e ic tions
int of “he ches.” U on further interview, w ou be the ost ro ri te to tre t this
she st tes her sy to s beg n fter the e th tient’s sy to s?
of her hu sb n 5 onths go. Since th t ti e, (A) Bu ro ion
she h s fe t “e ty,” w ith oor s ee , f tigu e,
(B) Bu s irone
ecre se concentr tion, n w ishes th t she
“w ou ie so I c n join hi .” H er review of (C) Cit o r
syste s is re rk b e for ongoing “sh r ins (D) C on ze
n nee es” in her extre ities. H er hysic (E) Ven f xine
ex in tion is u nre rk b e, n her b oo
su g rs h ve been re tive y w e contro e . 103. A 40-ye r-o n is st rte on f u oxetine for
Which of the fo ow ing w ou be the ost jor e ressive isor er. H e notes i rove-
ro ri te tre t ent for her sy to s? ent in his oo , bu t h s ifficu ty s ee ing.
Tr zo one is su bsequ ent y e to his regi-
(A) Bu ro ion
en. Three ys ter, he c s co ining of
(B) Du oxetine infu erection th t h s not su bsi e fter
(C) F u oxetine sever hou rs. Which of the fo ow ing best
(D) Mirt z ine escribes this tient’s con ition?
(E) Re ssu r nce (A) Erecti e ysfu nction
(B) Ny ho ni
101. A 44-ye r-o rrie e tient w ith his-
tory of recurrent jor e ressive isor er (C) P r r xis
resents to sychi trist fter sto ing his ser- (D) Pri is
tr ine 1 onth go. A though it h s been effec- (E) S tyri sis
tive in the tre t ent of his e ression, he h s
h incre sing ifficu ty int ining n erec- 104. A 35-ye r-o rrie fe e is being fo -
tion, n this h s c use friction n conf icts ow e by sychi trist for her first e iso e
w ith his w ife. H e is concerne , how ever, th t of jor e ression. She is rescribe cit o-
he w i h ve re se of his e ressive sy - r 40 g i y n h s been in re ission
to s if not e ic te . Which of the fo ow ing for 3 onths. She now co ins of the new
w ou be the ost ro ri te h r co ogic onset of ecre se ibi o w hich is c u sing her
tre t ent for this in ivi u ? o er te ou nt of istress. Which of the
fo ow ing w ou be the ost ro ri te to
(A) Begin irt z ine.
to her cu rrent regi en?
(B) Begin roxetine.
(C) Begin ven f xine. (A) Bu ro ion
(D) N o e ic tion. (B) Desi r ine
(E) Rest rt sertr ine. (C) Lithiu
100 3: S oma tic Tre a tme nt a nd P syc hop ha rma c ology

(D) Tr ny cy ro ine 107. A 16-ye r-o boy is brou ght to you by his
(E) Ven f xine other. She st tes th t, es ite h ving high
IQ, he oesn’t co ete his ho ew ork, h s
105. A 41-ye r-o w o n resents to you r office been f i ing in schoo , n h s sto e going
sking for he in n ging her binge e t- out w ith frien s. H e its to s oking tw o to
ing. She e ts rge ou nts of ju nk foo three three joints of riju n i y.
or fou r ti es w eek, hi es foo , n su bse-
qu ent y fee s gu i ty bou t her ctions. H er 108. A 21-ye r-o n is brou ght into the e er-
BMI is 31. After erfor ing thorou gh his- gency e rt ent by the o ice fter getting
tory n hysic ex in tion, you eci e to rreste for ste ing g so ine. The tient fee s
rescribe to ir te. Which of the fo ow ing isconnecte fro re ity n frightene . On
si e effects o you te her is ost ike y? hysic ex in tion, he is note to h ve
(A) I rove ttention r sh rou n his ou th.
(B) P ncre titis
(C) Po yu ri 109. A 54-ye r-o n w ith o i te u se isor er
re se on heroin fter sever onths of
(D) Ren stones
sobriety. Whi e he beg n u sing $10 to $20 er
(E) Weight g in
y t first, he qu ick y incre se his u se to $50
to $60 er y bec u se “I on’t get high now
D IRECTION S (Questions 106 through 109): The
u n ess I u se ore.”
follow ing group of questions are preceded by a
list of lettered options. For each question, select
the one lettered option that is most closely asso- D IRECTION S (Question 110): Select the lettered
ciated w ith it. Each lettered option may be used answ er that is the one best response in this case.
once, multiple times, or not at all.
110. A 27-ye r-o n w ith schizo hreni notices
Questions 106 through 109 the eve o ent of bre sts, ecre se ibi o,
n not ej cu ting w hen he h s n org s .
For the fo ow ing vignettes, choose the correct ter Which of the fo ow ing e ic tions is ost
fro the ist be ow . ike y res onsib e for these si e effects?
(A) A otiv tion syn ro e (A) Ari i r zo e
(B) C nn bis (B) C oz ine
(C) For ic tion (C) Qu eti ine
(D) H u cinogen ersisting erce tion (D) Ris eri one
isor er (E) Zi r si one
(E) Inh nts
(F) Intoxic tion D IRECTION S (Questions 111 through 121): The
(G) Psycho ogic e en ence follow ing group of questions are preceded by a
(H ) Su bst nce u se isor er list of lettered options. For each question, select
the one lettered option that is most closely asso-
(I) To er nce
ciated w ith it. Each lettered option may be used
(J) With r w once, multiple times, or not at all.

106. A 45-ye r-o n w ith chronic coc ine u se Questions 111 through 121
isor er resents to the e ergency e rt-
ent w ith co ints of bugs cr w ing on his For the fo ow ing vignettes, choose the correct si e
skin. effect fro the ist be ow .
(A) Agr nu ocytosis
(B) Ak thisi
Que s tions : 105–122 101

(C) B e h ros s gener ize rigi ity, i horesis, n tere


(D) Consti tion ent st tu s.
(E) G ctorrhe
(F) Gyneco sti 117. A 36-ye r-o n w ith schizo ffective is-
(G) N eu ro e tic ign nt syn ro e or er, e resse ty e h s been receiving f u -
hen zine ec no te 50 g intr u scu r y
(H ) Obstru ctive j u n ice
every 2 w eeks. Whi e his sy to s re w e
(I) O isthotonos
contro e , he h s recent y notice w hite
(J) Orthost tic hy otension isch rge fro his ni es, bi ter y.
(K) Pig ente retino thy
(L) Pseu o rkinsonis 118. A 26-ye r-o n being tre te w ith high-
(M) R bbit syn ro e ose thiori zine is fou n on o hth o ogic
(N ) Retrogr e ej cu tion ex in tion to h ve e ery ig ent tion
(O) T r ive yskinesi of the retin . Des ite iscontinu tion of the
(P) Tortico is e ic tion, the ig ent tion continu es n
the tient eventu y suffers tot b in ness.
111. A 35-ye r-o n w ith chronic schizo hreni
t king thiori zine notices th t, if he urin tes 119. A 22-ye r-o n co ins of severe rest-
fter sturb ting, his u rine e rs c ou y. essness n git tion fter st rting on ri i -
r zo e. H e co ins th t he c nnot sit sti
112. A 46-ye r-o C uc si n w o n w ith schizo f- n ces const nt y.
fective isor er, bi o r ty e h s been n-
ge for ny ye rs on ris eri one 3 g tw ice 120. A 26-ye r-o tient w ith schizo hreni is
i y or y. She enies ny co ints but on st rte on ch or ro zine. After 1 w eek, he
ent st tus ex in tion is note to h ve co ins of f u - ike sy to s. A CBC is
re etitive, r i ove ents of her ou th (bu t obt ine n is nor . After nother w eek,
not tongue) w ith s cking of her i s. the tient eve o s ye ow ish co or to his
skin n sc er .
113. A 22-ye r-o Afric n-A eric n n on c o-
z ine for 3 onths re orts sore thro t n 121. A 45-ye r-o tient w ith schizo hreni is
fever. A CBC is obt ine w ith WBC cou nt of st rte on high ose of ch or ro zine. The
3
1,000/ w ith 1% gr nu ocytes. next y, he co ins of being ight-he e
every ti e he st n s. On ex in tion, he is
114. A 43-ye r-o n w ith schizo hreni is note to h ve signific nt ecre se in b oo
on h o eri o . H e is note to h ve f exe ressu re n n incre se in he rt r te.
osture, festin ting g it, resting tre or, n
br ykinesi . D IRECTION S (Questions 122 and 123): For each of
the multiple-choice questions in this section, select
115. The s e 43-ye r-o n w s continu e the lettered answ er that is the one best response in
on h o eri o es ite his si e effects n each case.
fter sever ye rs eve o e choreo thetoi
ove ents of the trunk n i bs, ong w ith 122. You re c e in the i e of the night by the
i s cking n tongu e ove ents. su rgic service to see 52-ye r-o n w ho
w s itte to the hos it for n e ergency
en ecto y. Review of the ch rt ocu ents
116. A 21-ye r-o n w ith sychosis w s nu erou s st hos it iz tions for coho
st rte on h o eri o t high oses. Eight intoxic tion n w ith r w . The su rgic
ys into his tre t ent, he eve o e fever, te is concerne bout his w ith r w ing
102 3: S oma tic Tre a tme nt a nd P syc hop ha rma c ology

w hi e in recovery. H ow ong fter this tient’s tri s of ithiu , v roic ci , n o nz -


st rink w ou you ost ike y ex ect to see ine, he is st rte on oxc rb ze ine. Three ys
serious w ith r w sy to s eve o ? ter, he co ins of n u se n w e kness;
he e rs confu se bou t w hy he is in the
(A) 6 hou rs
hos it . Which of the fo ow ing bor tory
(B) 12 hou rs stu ies w ou be the ost i gnostic in this
(C) 1 y c se?
(D) 3 ys
(A) BUN n cre tinine
(E) 7 ys
(B) CBC
123. A 33-ye r-o n w ith history of bi o r I (C) E ectro ytes
isor er is itte to the in tient u nit w ith (D) Liver fu nction tests
cu te ni . Bec u se he h s f i e reviou s (E) Oxc rb ze ine eve
CHAPTER 4

Ps yc ho lo g ic al Tre atme nt and


Manag e me nt
Que s tions

DIRECTIONS (Questions 1 through 42): For each of (C) Ego strengths


the multiple-choice questions in this section, select (D) Projections
the lettered answ er that is the one best response in (E) Unconsciou s thou ghts
each case.
3. A 43-year-old m an is chastised at w ork. When
Questions 1 and 2 he com es hom e, his friend asks him how his
d ay w ent. H e respond s angrily saying that a
A 38-year-old m ale thinks that he is a “failu re and “real friend w ou ld n’t be so nosy.” Which of
w ill never find a p artner” w henever he feels rejected . the follow ing d efense m echanism s m ost likely
H e often find s him self getting d epressed and isolat- characterizes this reaction?
ing him self w hen he thinks like this. H is therap ist
thinks that it w ould be a good id ea for him to m ake (A) Denial
a log of the situ ations, his im m ed iate thou ghts, his (B) Disp lacem ent
im m ed iate feelings, alternate resp onses to those (C) H u m or
thoughts and feelings, and re-rate his feelings after (D) Intellectu alization
com pleting the exercise so they can be exam ined (E) Isolation of affect
d uring their sessions.
4. A 20-year-old w om an d iagnosed w ith bord er-
1. Which of the follow ing types of therapeu tic line p ersonality d isord er thinks of her thera-
ap proaches is the therapist m ost likely u sing? p ist as “the best person I’ve ever know n.” The
(A) Cognitive–behavioral next w eek, the therapist announces that he
(B) Exp ressive w ill be going on vacation. The patient becomes
enraged and states that he is “the cru elest
(C) H yp notherap y
d octor in the w orld !” Which of the follow ing
(D) Psychoanalysis d efense mechanism s best characterizes this
(E) Psychod ynam ic reaction?

2. Which of the follow ing term s refers to the (A) Devalu ation
im m ed iate thou ghts (e.g., “I am a failu re”) (B) Id ealization
that the p atient exp eriences and record s in this (C) Intellectu alization
type of therapy? (D) Repression
(A) Au tom atic thou ghts (E) Sp litting
(B) Core beliefs

123
124 4: P syc holog ica l Tre a tme nt a nd Ma nag e me nt

5. A 16-year-old boy has just lost his mother and m ind .” Which of the follow ing analytic tech-
father in a car accident. In your office, he talks niques d oes this best rep resent?
philosophically about death and its implica-
(A) Free association
tions. When asked how he feels about his par-
ents’ death, he responds by saying that “it is the (B) H yp nosis
nature of things to pass away.” Which of the fol- (C) Rep ression
lowing defense mechanisms best d emonstrates (D) Thou ght record s
this patient’s inability to talk directly about his (E) Transference
emotional experience concerning personal loss?
(A) Denial 9. A psychotherapist has been m eeting regu larly
w ith a 10-year-old boy for 18 months. During
(B) Intellectu alization
one of the therapy sessions, she find s herself
(C) Projection feeling very angry at the patient after he reports
(D) Su ppression “beating up” his little brother. Which of the
(E) Und oing follow ing phenomena is the psychotherapist
most likely exhibiting?
Questions 6 and 7
(A) Countertransference
A 44-year-old w om an w ith schizop hrenia is stru g- (B) Extinction
gling w ith p aranoia, au d itory hallu cinations, and (C) Interp retation
d elu sions. She lives w ith her m other but has a poor (D) Resistance
relationship w ith her. She tells you that everyone
(E) Transference
w ants her to sp end a lot of m oney and bu y d ru gs.
10. In an acu te inp atient p sychiatric u nit, the
6. Which of the follow ing resp onses w ou ld be
resid ent p sychiatrist is in charge of lead ing a
the m ost usefu l?
grou p consisting of p atients new ly ad m itted
(A) “Well, d ecid ing for you rself is best.” to the w ard . Du ring group d iscu ssion she asks
(B) “If you bu y d ru gs, I’ll call the p olice.” tw o participants to act out a scenario, d escribe
(C) “Perhap s w e shou ld look at w hat you r their feelings abou t the situ ation, and then
m other w ou ld think abou t that.” exp lore the ind ivid u al conflicts w hich arose.
Which of the follow ing techniques of p sycho-
(D) “Why d o you think everyone w ants you
therapy d oes this m ost represent?
to d o that?”
(E) “N o, they d on’t.” (A) Feed back
(B) Free association
7. You d ecid e that su p p ortive therap y w ou ld be (C) Go-arou nd
helpfu l to this patient. Which of the follow ing (D) Psychod ram a
w ou ld be the m ost ap p rop riate goal of su p -
(E) Resistance
p ortive therap y for this p atient?
(A) Correcting fau lty id eas Questions 11 and 12
(B) Exp loring the feeling of m eaninglessness
A 28-year-old fem ale p atient w ho is ad d icted to opi-
in life
oid analgesics states that she has been up set w ith
(C) Investigating the freed om of ind ivid u als fam ily for not continu ing to su p p ort her financially
(D) Personality change and allow ing her to stay w ith them . She has been
(E) Strengthening of d efenses u nem ployed for m any years since leaving college.
They recently told her that she shou ld be m ore ind e-
8. A 32-year-old p sychiatry resid ent is in p sycho- p end ent; they w ant her to start w orking and find a
analysis 4 d ays p er w eek. H e is encouraged to p lace to live. In her next therap y session, she tells
lie on the cou ch and say “w hatever com es to her therapist that she u nd erstand s w hy they w ou ld
Que s tions : 5–16 125

say that, bu t then goes on to d escribe a fight w ith her 14. A 34-year-old male is referred by his job because
boyfriend , stating she w ants to break u p w ith him of ongoing interpersonal conflicts. During
becau se he is not su p p ortive of her. H er therap ist the interview, he appears very focused on his
asks if it is p ossible she is u pset at her parents for health, his attractiveness to others, and his suc-
w anting her to be m ore ind ep end ent bu t exp ressing cess at work. When confronted with his diffi-
this anger tow ard her boyfriend instead . culties, he becomes defensive, blaming others,
and accusing them of being “jealous of me.” He
11. Which of the follow ing therapeu tic techniqu es describes himself as “better” than his friends
best d escribes the therap ist’s resp onse? and colleagues and admits to taking advantage
of others in order to “get what I deserve.” Which
(A) Cou ntertransference
of the following treatment modalities would be
(B) Em p athy the most appropriate for this patient?
(C) Interp retation
(A) Anger m anagem ent
(D) Parad oxical intervention
(B) Com bined ind ivid u al and grou p
(E) Working throu gh
therapy
12. The p atient’s fam ily m em bers and close (C) Psychoanalysis
friend s w ork w ith the therap ist and join the (D) Psychopharm acotherap y
p atient in a session. Du ring the m eeting, they (E) Social skills training
tell her about her m alad aptive behaviors and
how they have affected her and her fam ily 15. A 29-year-old w om an has been d epressed for
negatively. They then give her an u ltim atu m , 2 months prior to seeking med ical attention.
w hereby if she d oes not get treatm ent for her She believes that nobod y likes her even though
ad d iction they w ill not continu e to su pp ort she is alw ays cord ial, and that there is noth-
her. Which of the follow ing techniqu es best ing she can d o to change the w ay other people
d escribes this scenario? perceive her. Therefore, w hen she is around
peop le she feels tense and u ncomfortable and
(A) Behavioral therap y
has started avoid ing leaving the hou se. Which
(B) Fam ily therap y of the follow ing cognitive–behavioral therapy
(C) Grou p therap y (CBT) interventions w ould most d irectly target
(D) Intervention the physical feelings she has around people?
(E) Relap se p revention (A) Cognitive restru cturing
(B) Exp osu res
13. A 24-year-old female patient with a history of
generalized anxiety disorder presents for therapy. (C) Keep ing a m ood d iary
In a previous session she reluctantly discussed (D) Keep ing a thou ght record
how much she did not like her new step-mother (E) Progressive m u scle relaxation
and didn’t approve of her father marrying her.
This session she reports that her step-mother is 16. After several weeks, a 40-year-old female patient
in the hospital and she has been spending most who is in psychodynamic psychotherapy begins
of the past few days at the hospital caring for her, to show up late and miss appointments; each
buying many get-well gifts and inserting herself time this happens she has a reason for doing
in her step-mother ’s care. Which of the following so. However, the therapist begins to feel that
concepts best describes this behavior? her behavior is interfering with her treatment.
The therapist is not sure if the patient under-
(A) Abreaction
stands that her behavior is inappropriate and
(B) Dep end ency potentially damaging to her therapy. Which of
(C) Projection the following topics would be the most impor-
(D) Reaction form ation tant to review with the patient before making
(E) Sp litting an interpretation of her behavior?
126 4: P syc holog ica l Tre a tme nt a nd Ma nag e me nt

(A) Bou nd aries and ru les d efining the w ay (D) Interp retation
in w hich therap y is cond u cted (E) Refram ing
(B) Focu s on the therapeu tic goals
(C) Method u sed to change the p atient’s Questions 19 and 20
m alad aptive thou ghts
An 18-year-old , single m an p resents to you r office
(D) Patient’s need for attention com plaining that he cannot pass a m ovie theater
(E) Reasons the p atient is late to the w ithou t stop p ing, going insid e, and bu ying cand y.
ap pointm ents This behavior is trou blesom e to him and interferes
w ith his d aily activities, resu lting in his being con-
17. A 45-year-old m an p resents to you r ou tp atient stantly late to other app ointm ents. You d ecid e to
clinic hand icap ped by a fear of parking lots help the p atient w ith behavior therap y.
and field s. The fear started 4 m onths p rior to
this visit. At the beginning of cognitive ther- 19. Which of the follow ing w ould be the m ost
ap y, he tells you that his behavior is constantly ap p rop riate focu s of the therapy?
being scru tinized and criticized by other
p eop le. H e claim s that he cannot change his (A) Analyzing the p atient’s relationship s
behavior, becau se if he d oes, others w ill think (B) Decreasing the m alad ap tive behavior of
he is a fool. Which of the follow ing w ou ld be stopping by m ovie theaters
the m ost appropriate response? (C) Exam ining the p atient’s negative eating
habits
(A) “That’s silly. People w ill not think you
are a fool.” (D) Exp loring the p atient’s child hood
trau m as
(B) “Yes, I can see that.”
(E) Working on resolving the p atient’s
(C) “What m akes you think th at others
u nconsciou s conflicts
are con stantly scru tinizing you r
beh avior?”
20. Which of the follow ing behavioral interven-
(D) “H ow can you possibly think other tions w ould be the m ost help fu l in this case?
p eople care enou gh abou t you to
constantly scru tinize your behavior?” (A) Aversion therap y
(E) “Well, m aybe you are a fool. H ave you (B) Environm ental m od ification
ever thou ght of that?” (C) Exp osu re and resp onse training
(D) Mod eling
18. A 32-year-old fem ale in therap y d escribes (E) Relaxation training
how she feels that she should have a life p art-
ner; and since she is not currently d ating, she 21. A 39-year-old m an p resents to you r office
is u nlikely to ever get m arried and is therefore com plaining of chronic “stress,” anxiety, and
u nlovable. H er therapist points ou t to her that p oor sleep. H e also ad m its to interm ittent
she has had a few m eaningful long-term rela- head aches, w hich you d iagnose as tension
tionships in the past and that she has d ated as head aches. You recom m end behavioral ther-
recently as a cou p le of m onths ago. The thera- ap y for red u ction of his head aches. Which of
p ist also explains that m any p eople m ay not the follow ing behavioral techniqu es w ould be
d ate, bu t this d oes not m ean they are u nlov- m ost appropriate for this patient?
able or w ill never get m arried . Which of the
follow ing term s best d escribes the therap eu tic (A) Aversive stim u li
intervention d escribed above? (B) Biofeed back
(C) N egative reinforcem ent
(A) Clarification
(D) Stim u lu s control
(B) Confrontation
(E) System atic d esensitization
(C) Em p athy
Que s tions : 17–26 127

Questions 22 and 23 (C) Com p lete hom ew ork frequ ently to


better learn abou t their issu es.
A 32-year-old m ale com p lains of ongoing d epression
(D) Prevent conflicts from arising betw een
for the past m onth w hich he attribu tes to the break-
the therapist and p atient.
up w ith his fiancée. H e has been having insom nia,
d ecreased appetite, and low energy, althou gh he (E) Requ ire m ore intensive treatm ent d u e to
d enies anhed onia or problem s concentrating. H e severity of m ental illness.
also d enies any su icid al id eation. H e has ju st started
to d ate som eone else w ithin the last w eek. After d is- 25. Which of the follow ing w ou ld be the m ost
cussion w ith the patient, it is d ecid ed to u tilize both im p ortant tool in his p sychoanalysis?
m ed ication and interp ersonal psychotherap y for his (A) Altering cognitive d istortions
d ep ression. (B) Altering m alad ap tive behaviors
(C) Interp retation of transference
22. Which of the follow ing goals w ou ld be the
m ost likely focu s of his p sychotherapy? (D) Interp reting d ream s
(E) Solving interp ersonal p roblem s
(A) Character change
(B) Clarifying com m u nication p atterns 26. A m ale th erap ist h as been w orkin g w ith a
(C) Strengthening d efenses fem ale p atient for over 3 years, enabling her
(D) Interpreting transference to im p rove her self-esteem and confid ence,
(E) Pointing ou t resistance and to eventu ally d ivorce her em otion ally
abu sive h u sban d 1 year ago. The p atient con-
23. Which of the follow ing techniqu es w ou ld tinu ally thanks the therap ist and exp lains
be m ost likely u tilized d u ring this p atient’s that she is sexu ally attracted to the therap ist.
therapy? H e is initially taken aback and then som e-
w hat flattered by her sexu al interest. H ow -
(A) Dream interp retation ever, he also realizes that he feels a strong
(B) Defense analysis attraction to her and feels the n eed to care
(C) Free association for her. Which of the follow ing w ou ld be the
(D) H ypnosis m ost ap p rop riate n ext step for the th erap ist
to take?
(E) Role p laying
(A) Act cold , show less em p athy, and set
Questions 24 and 25 m ore rigid bou nd aries d u ring sessions.
A 22-year-old college stud ent presents com plaining (B) Exp lore the p atient’s feeling in ord er to
of having “no goals” in his life, not d oing as w ell as he increase her erotic transference.
could in school, and d esiring but having no serious (C) Ignore the p atient’s d isclosu re of her
long-term romantic relationships. H e has not chosen sexu al attraction.
a m ajor and states that his ind ecision is paralyzing (D) Seek su p ervision w ith a m entor or
him from “m oving on w ith the rest of m y life.” After colleague.
d iscussing various form s of psychotherapy w ith the (E) Term inate or fire the p atient from his
patient, you recomm end psychoanalysis fou r times care.
per w eek.
Questions 27 and 28
24. Which of the follow ing is the m ost im p ortant
reason for the frequ ency of p sychoanalysis? You are seeing a 25-year-old man in cognitive–behav-
ioral therapy w ho suffers from generalized anxiety
(A) Allow the p atient to get enou gh su p p ort d isorder, as w ell as epilepsy. H e often talks about
d u ring the course of their treatm ent. the many dreams he has w hile in his sessions, and
(B) Allow transference to bu ild betw een he is having a hard time w ith the restructuring of
p atient and therapist.
128 4: P syc holog ica l Tre a tme nt a nd Ma nag e me nt

malad aptive thoughts. You d ecid e that hypnosis may relationships. He blames the problem on “his
be helpful as an adjunctive therapy for this patient, sabotaging things” when the women “start to
as he has been hypnotized before w ith good results get serious.” He is clearly ambivalent about his
and you have a skillful hypnotist in your clinic. The desire to “settle down” with someone. Accord-
patient is hopeful that hypnosis w ill be beneficial for ing to Erik Erikson, which of the following stages
him again. of development is the patient most likely in?
(A) Tru st versu s m istru st
27. Which of the follow ing characteristics is the
best p red ictor of resp onse to hyp nosis in this (B) Au tonom y versu s sham e
p atient? (C) Initiative versu s gu ilt
(D) Intim acy versu s isolation
(A) Patient has a lot of d ream s.
(E) Id entity versu s role confu sion
(B) Patient is resp onsive to su ggestion.
(C) Patient’s d iagnosis. 31. A 9-year-old boy w ith attention-d eficit/
(D) Patient w ith low seizu re threshold . hyperactivity disorder (ADHD), predominantly
(E) Trained hyp notist w ill be d oing his hyp eractive/ im p u lsive presentation, presents
treatm ent. to you r office accom panied by his m other. H e
is cu rrently on m ethylp henid ate (Ritalin) and
28. The p atient is started in hyp nosis, bu t there they both w ond er if there is som ething else
is a concern as to w hether he is in a trance that can be d one to help the patient. Despite
d u ring the sessions. Which of the follow ing som e benefit and tolerability, the child contin-
ind icators w ou ld be m ost likely increased in a u es to have d ifficulty w ith com pleting hom e-
trance state? w ork and follow ing instru ctions at school
and at hom e. H e has also recently gotten
(A) Am nesia
into a m inor p hysical fight w ith a p eer. Par-
(B) Pain p ercep tion ent m anagem ent training is recom m end ed for
(C) Pu lse the parents. Which of the follow ing aim s best
(D) Reflexes illu strates how p arent m anagem ent training
(E) Resp irations w ill help the fam ily?
(A) Focu sing on m alad ap tive
29. You are treating an 11-year-old boy for op p osi- com m unication in the fam ily
tional d efiant d isord er and have heard from
(B) Focu sing on the fam ily’s interp ersonal
the p arents that his d isord er has had a sig-
relationship s
nificant im p act on the fam ily d ynam ics. They
have tw o old er child ren w ho often feel left (C) Increasing p arental u se of p ositive
ou t and feel d istanced from the fam ily u nit. reinforcem ent to increase d esired
You recom m end a trial of fam ily therap y. behaviors
In this case, w hich of the follow ing w ou ld (D) Interp reting p arent’s negative cognitions
be the m ost ap p rop riate focu s of the fam ily abou t the p atient
therap y? (E) Teaching parents to p ositively refram e a
negative exp erience
(A) Assigning roles in a hou sehold
(B) Early child hood exp eriences 32. A 14-year-old boy w ith attention-d eficit/ hyper-
(C) Im p u lsive behavior activity d isorder (ADH D) is frequently late for
(D) Relationship p atterns school and forgets to d o his chores around the
(E) Unconsciou s conflicts house d ue to his d isorganization. H is parents
are fed up w ith him and ask you w hat they can
30. A 32-year-old man presents for psychother- d o to help change his behavior. Which of the
apy. He denies pervasive depression or anxi- follow ing represents the most helpful behav-
ety symptoms, but he has had a string of failed ioral tool for patients w ith ADH D?
Que s tions : 27–37 129

(A) A chart of behaviors to be rew ard ed by d iagnosed w ith schizop hreniform d isord er.
p arents H e recovers after 4 m onths bu t has another
(B) Aversion therapy relap se 6 m onths later in the absence of an
(C) Interp retation of m alad ap tive behaviors elevated , irritable, or d ep ressed m ood . H e is
then given a d iagnosis of schizop hrenia and
(D) Positively refram ing a negative
referred for m ed ication m anagem ent and
exp erience
therap y. After a few w eeks of therap y, the
(E) Pu nishm ent p atient exp eriences a w orsening of his p sy-
chotic sym p tom s. Which typ e of therap y w as
Questions 33 and 34 he m ost likely referred to?
A 25-year-old u nm arried m an in his fou rth year of (A) Behavioral therap y
m ed ical school presents to you r office com p laining
(B) Fam ily therap y
of not being able to rem ain in a relationship w ith
(C) Grou p p sychotherap y
a girlfriend for longer than 3 m onths. H e says that
he w ould like an insight-oriented therap y, and you (D) Psychoanalytic p sychotherap y
agree that this form of therapy w ou ld help him . You (E) Su p p ortive p sychotherap y
decide to treat this patient with brief psychod ynamic
p sychotherap y. 36. A 50-year-old m an is in p sychoanalysis
becau se he feels stagnant in his p ersonal
33. Which of the following factors would best predict life. H e notes he is not having any p roblem s
a positive outcome with brief psychodynamic w ith his w ork or relationship w ith his w ife,
psychotherapy in this particular patient? althou gh he feels that he has never had m ore
than a p latonic relationship w ith her. While he
(A) Age feels cap able and confid ent in m ost sp heres
(B) Gend er of his life, inclu d ing his relationship s, he feels
(C) Marital statu s like he is at a “road block” in therap y and d oes
(D) Motivation for change not know how to p roceed . H e has, how ever,
(E) Socioeconom ic statu s m entioned a reoccu rring d ream about his
w ife tu rning into his m other and taking care
34. Which of the follow ing asp ects best d istin- of him . The therapist thinks that interpreting
gu ishes his sp ecific typ e of p sychotherap y his d ream m ay be a good tool to u se in ther-
from other form s of therap y? ap y. Which of the follow ing u ses of his d ream
w ou ld be the m ost ap p rop riate in this case?
(A) Correcting cognitive errors
(B) Id entification of a focal conflict (A) Interp reted by the analyst to
su blim inally influ ence the m ind of the
(C) Interp reting d ream s
p atient.
(D) Interp reting resistance
(B) Provid e inform ation abou t p sychic
(E) Mod ifying m alad ap tive behaviors conflicts.
(C) Rep resent the consciou s fram ew ork for
35. A 20-year-old m an is cu rrently in college and
behavior.
has com p leted the first sem ester of his sop ho-
m ore year. H alfw ay into the second sem ester, (D) Used to alter cognitive errors.
his grad es d rop from As to Fs over a 3-m onth (E) Used to change behavior.
p eriod . H e becom es increasingly isolated
and p aranoid , believing that the governm ent 37. You have been assigned to lead a d ialectical
is after him becau se he has solved “all theo- behavioral therap y grou p for p atients w ith
logical p roblem s” throu gh d irect com m u ni- bord erline p ersonality d isord er. Du ring the
cation w ith God . H e is started on olanzap ine grou p one of the p atients relates a story abou t
10 m g by m ou th (PO) d aily and is tentatively getting into a fight w ith her m other. Afterw ard
she w as so d espond ent she had thou ghts
130 4: P syc holog ica l Tre a tme nt a nd Ma nag e me nt

abou t cu tting herself. Instead of resp ond - Which of the follow ing stages of grou p form a-
ing, she w atched her favorite com ed y m ovie. tion is this grou p m ost likely in at this tim e?
Which of the follow ing skills d id the p atient
(A) Form ing
m ost likely em p loy?
(B) Storm ing
(A) Cognitive restructu ring (C) N orm ing
(B) Distress tolerance (D) Perform ing
(C) Em otion regu lation (E) Ad jou rning
(D) Interpersonal effectiveness
(E) Mind fu lness 41. A 38-year-old law yer is referred for behavioral
therapy because of extrem e social anxiety
Questions 38 and 39 w hich is interfering in his ability to litigate. In
his early therap y sessions, he is told to m ake
A 33-year-old w om an p resents to the em ergency a hierarchy of situ ations that m ake him anx-
d epartm ent saying that she w ants to kill herself. She iou s. The therap ist then begins w orking w ith
has felt increasingly d epressed for m any m onths, him by first exposing him to the least anxiety-
w ith late insom nia, p oor ap p etite, w eight loss, d is- p rovoking item s on the list and then grad u ally
tractibility, and fatigu e. She cu rrently feels hopeless increasing the severity. Which of the follow ing
and help less, and has a p lan to overd ose on p ills therapeu tic techniques is the therapist m ost
that she has stockpiled at hom e. likely u tilizing?

38. Other than her feelings of helplessness and (A) Aversion


hopelessness, w hich of the follow ing cognitive (B) Flood ing
d ifficulties w ou ld she m ost likely also exhibit? (C) Mod eling
(A) Begging the question (D) Su ggestion
(B) Circu lar reasoning (E) System atic d esensitization
(C) Personalization
42. A 47-year-old , d ivorced m ale has been
(D) Rigid , black-or-w hite thinking
attend ing p sychotherap y for 8 m onths d u e to
(E) Selective abstraction ongoing “stress” and conflicts at w ork. Du r-
ing m ost of his therap y sessions, the p sychia-
39. After hospitalization, w hich of the follow ing trist resp ond s to his com m ents w ith em p athic
form s of p sychotherap y w ou ld be the m ost resp onses. Which typ e of p sychotherap y is
ap p rop riate to help m inim ize this patient’s the p sychiatrist m ost likely u tilizing in this
su icid ality? case?
(A) Behavioral therapy (A) Behavioral
(B) Cognitive therap y (B) Cognitive
(C) H yp nosis (C) Dynam ic
(D) Interpersonal p sychotherap y (D) Existential
(E) Parad oxical therap y (E) Su p p ortive

40. In the context of a w om en’s grou p , one m em - D IRECTION S (Questions 43 through 47): The fol-
ber has been frequ ently sid e talking. As low ing group of questions is preceded by a list of
th e facilitator, you bring this u p d u ring the lettered options. For each patient scenario, select
grou p . Th e m em ber states she only d oes this the one lettered option that is most closely asso-
becau se you d on’t give enou gh tim e for each ciated w ith it. Each lettered option may be used
m em ber to talk. The other grou p m em bers once, multiple times, or not at all.
agree w ith h er.
Que s tions : 38–50 131

(A) Biofeed back 46. To train a 4-year-old boy to say “thank you,”
(B) Cou ntertransference his m other gives him a sw eet w henever he
(C) Flood ing d oes.
(D) H ypnosis
(E) N egative reinforcem ent 47. To train a 4-year-old boy not to bite his class-
m ates, his m other p uts soap in his m ou th
(F) Positive reinforcem ent
w henever he d oes.
(G) Projection
(H ) Projective id entification
D IRECTION S (Questions 48 through 50): The fol-
(I) Punishm ent
low ing group of questions is preceded by a list
(J) Real relationship of lettered options. For each question, select the
(K) Schem a one lettered option that is most closely associated
(L) Sp litting w ith it.
(M) Therap eu tic alliance (A) Josef Breu er
(N ) Transference (B) Jean-Martin Charcot
(O) Und oing (C) Erik Erikson
(D) Sigm u nd Freu d
43. You are seeing a 26-year-old w om an w ho com -
plains of m igraines. She is otherw ise healthy. (E) H einz Kohu t
You exp lain to her that you w ou ld like to try a (F) Franz Anton Mesm er
techniqu e in w hich you give her inform ation
on m u scle tension and tem peratu re. 48. Mad e m any contributions to p sychiatry and
neu rology w ith a sp ecial interest in d escribing
hysteria.
44. You are seeing a 45-year-old m an in p sycho-
analysis fou r tim es p er w eek. Du ring one ses-
sion he appears less com fortable talking. H e 49. Know n as the inventor of the “cathartic” treat-
eventu ally tells you that you r new glasses m ent or talking therap y.
rem ind him of his au nt w ho cared for him as a
child bu t w as cru el to him . 50. Know n as the d evelop er of self-p sychology.

45. A p atient w ho is afraid of clow ns is taken to


the circus in the beginning of her therapy in
ord er to get over her fear.
CHAPTER 5

Le g al and Ethic al Is s ue s in
Ps yc hiatry and Me dic ine
Que s tions

DIRECTIONS (Questions 1 through 51): For each of (E) Resp ect the p atient’s confid entiality
the multiple-choice questions in this section, select and w ait u ntil you r next sched u led
the lettered answ er that is the one best response in ap p ointm ent w ith him to d iscuss his
each case. feelings.

2. The legal p reced ent that gu id es the ap p rop ri-


Questions 1 and 2
ate cou rse of action in this case is w hich of the
The case m anager for a 35-year-old m an that you are follow ing?
treating contacts you to d iscuss concerns regard ing (A) Durham v. United States
a change in your p atient’s behaviors. You learn from
(B) O’Connor v. Donaldson
the case m anager that the patient has been m ak-
ing p rovocative sexu al com m ents tow ard her over (C) Rogers v. Commissioner of the Department
the past m onth. Du ring your session, the p atient of M ental Health
exp resses concern that his case m anager is som ehow (D) Rouse v. Cameron
conspiring against him and he plans to d o som e- (E) Tarasoff v. Regents of University of California
thing abou t it. On fu rther qu estioning, he becom es
increasingly irritable and abru p tly storm s ou t of Questions 3 through 5
you r office. You hear him in the hallw ay exclaim ing,
“That w om an, I’m going to stab her and she w on’t Tw o years ago, you w ere the anesthesiologist
bother m e anym ore!” involved in a cesarean d elivery of a baby born w ith
cerebral palsy. You had heard from your obstetrical
1. You r first cou rse of action shou ld be w hich of colleagu e that the fam ily w as planning to sue, and
the follow ing? tod ay a process server d elivers papers notifying you
that the fam ily has brought an action against the
(A) Attem p t to contact the p atient over the p hysicians involved in the su rgery, inclu d ing you.
next several hou rs to d iscu ss the intent
of his threat tow ard his case m anager. 3. In m ed ical m alp ractice cases, the p laintiff
(B) Contact the p atient’s fam ily and let m ust establish w hich of the follow ing?
them know abou t his threats tow ard the
(A) Burd en of p roof beyond a reasonable
case m anager.
d ou bt
(C) Inform the p olice of the content of you r
(B) Clear and convincing evid ence of
p reviou s sessions and his recent threat
w rongd oing
tow ard the case m anager.
(C) Crim inal intent
(D) N otify the case m anager of the p otential
d anger. (D) Crim inal m ischief
(E) H arm or d am age

141
142 5: Le ga l a nd Ethica l Is s ue s in P sychia try a nd Me d ic ine

4. Which of the follow ing behaviors best p rotects 7. The p atient’s fam ily hires an attorney to chal-
a p hysician against a m alp ractice su it? lenge the valid ity of her w ill. Which of the
follow ing item s, if p resent, w ou ld u nd erm ine
(A) H aving the patient sign a “no harm ”
this patient’s com petence to m ake a w ill?
contract.
(B) H iring a m alp ractice attorney to (A) Inability to read and w rite.
exam ine the p hysician’s treatm ent (B) Paranoid d elu sions regard ing the
p ractices. p atient’s fam ily.
(C) Maintaining a good therap eu tic alliance (C) Patient’s nond elu sional exp lanation of
w ith the p atient. w hy she w ants to d onate her estate to
(D) Refusing to prescribe m ed ications that the local hum ane society.
m ay have ad verse effects. (D) Presence of a d iagnosable p sychiatric
(E) Seeing the p atient in clinic on a w eekly d isord er.
basis. (E) Refu sal to u nd ergo treatm ent
recom m end ed by her p rim ary care
5. When d eterm ining m ed ical m alpractice, the p hysician.
term “stand ard of care” refers to the use of
m ed ical and p sychiatric treatm ents that are: Questions 8 and 9
(A) End orsed by the Am erican Med ical A 23-year-old m an w ith no p rior p sychiatric history
Association is charged w ith m u rd ering his next-d oor neighbor.
(B) Evid ence based H is friend s note that he becam e increasingly iso-
(C) Safe and free from any p otential lative and su sp iciou s of others in the w eeks p rior
negative sid e effects to the crim e. The exam ining psychiatrist reported
(D) The m ost cu rrent treatm ents available that the m an harbored paranoid d elu sions regard -
ing his neighbor and noted that his thou ght p rocess
(E) Used by average, reasonable
w as too d isorganized to com p lete the exam ination.
p ractitioners in sim ilar circu m stances
8. Which of the follow ing legal stand ard s is
Questions 6 and 7
the cu rrent basis for establishing an insanity
An 80-year-old w id ow w ith a h istory sign ificant d efense?
for schizop hrenia w as recently d iagnosed w ith (A) Am erican Law Institu te test (Mod el
end -stage hep atic cancer. She is concerned abou t Penal Cod e)
the d isp osition of her estate and d oes not w ant
(B) Du rham ru le
her fam ily to receive any of h er m on ey. Sh e p lans
to d onate her entire estate to the local hu m ane (C) Irresistible im p u lse ru le
society. (D) M’N aghten ru le
(E) Prod u ct ru le
6. Which of the follow ing is m ost im p ortant in
establishing a legally valid w ill? 9. Crim inal resp onsibility requ ires d em onstra-
tion of the crim inal act along w ith w hich of the
(A) Conservatorship of estate
follow ing elem ents?
(B) Inform ed consent
(C) Lack of m ental illness (A) Dim inished m ental cap acity
(D) Testam entary cap acity (B) Mens rea
(E) Witnesses at the signing of a w ill (C) Mod u s op erand i
(D) Psychiatric d isord er
(E) Witness to the act
Que s tions : 4–13 143

Questions 10 and 11 12. A 19-year-old m an w ith a history significant


for bip olar I d isord er is charged w ith assau lt-
An obese 54-year-old w oman presents via ambu- ing a p olice officer. This is the m an’s fifth arrest
lance to the emergency d epartment complaining of in the p ast 4 m onths for aggression tow ard an
severe substernal chest pain lasting 40 minutes, pro- au thority figu re. The assigned attorney has
fuse sw eating, and nausea. Her vital signs are blood inform ed the forensic p sychiatrist involved
pressure 195/ 96 mm H g, heart rate 63 beats/ min, in the case that the p atient has been nonco-
respiratory rate 18 breaths/ min, temperature 98.8°F. operative and belligerent w hile attem p ting
An electrocardiogram reveals 3-mm ST-segment ele- to prepare for his d efense. The attorney also
vations in lead s V4, V5, and V6. Cardiac enzymes and questions w hether the patient has an und er-
laboratory w orkup are pend ing. You suspect a lateral stand ing of the severity of his crim e given his
w all myocardial infarction and recommend imme- acu te m ental state and lack of form al ed uca-
d iate thrombolytic therapy. The patient exclaims, tion. Which of the follow ing item s w ou ld be
“You’re crazy if you think I’m going let some intern consistent w ith the assertion that the patient is
care for me! My family w ill d rive me across tow n to incom p etent to stand trial?
the private hospital!” The patient then jumps from
the gurney and begins w alking tow ard the exit. (A) Failu re to com p rehend the crim inal
charges.
10. Which of the follow ing is the m ost ap prop riate (B) H istory of p rior assau lt charges.
next step?
(C) Inability to p rovid e inform ed consent.
(A) Ad m it the patient to the card iology (D) Inability to read and w rite.
service on a physician em ergency (E) Presence of a d iagnosable m ental
certificate. illness.
(B) Allow the p atient to leave against
m ed ical ad vice. 13. A 35-year-old m an is brou ght to the em er-
(C) Detain the p atient u ntil the resu lts of the gency room by his fam ily for threatening to
card iac enzym es are available. shoot his m other in the chest. Urine toxicology
(D) Detain the p atient u ntil you can assess is negative for cocaine, p hencyclid ine (PCP),
her ability to p rovid e inform ed consent. or op ioid s, and his blood alcohol level is 0.04.
(E) Sed ate the p atient and begin You learn from the p atient’s fam ily that for
throm bolytic therapy. several d ays he has been threatening to kill his
m other and has even w ritten notes about how
11. On further evalu ation, the patient d em on- he w ou ld execu te his p lan. On m ental statu s
strates a thorou gh u nd erstand ing of the infor- exam ination (MSE), the p atient’s sp eech is
m ation you have given to her and ap p reciates lou d , p ressu red , and threatening tow ard the
the consequences of not being treated im m e- em ergency d ep artm ent staff. Fam ily history
d iately. The ap propriate next step is w hich of is significant for a bip olar sp ectru m d isor-
the follow ing? d er. The patient states that he has not d one
anything w rong and d em and s to be released
(A) Ad m it the p atient to the card iology im m ed iately. Which of the follow ing next
service on a physician em ergency steps w ou ld be the m ost appropriate?
certificate.
(B) Allow the p atient to leave against (A) Arrange for a p olice hold given the
m ed ical ad vice. p atient’s p otential for violence.
(C) Detain the p atient u ntil the resu lts of the (B) Involu ntarily ad m it the p atient to an
card iac enzym es are available. inp atient p sychiatric w ard .
(D) Detain the p atient so that her fam ily can (C) Med icate the p atient w ith a m ood
m ake d ecisions on her behalf. stabilizer and arrange for outpatient
follow -u p .
(E) Sed ate the p atient and begin
throm bolytic therap y.
144 5: Le ga l a nd Ethica l Is s ue s in P sychia try a nd Me d ic ine

(D) N otify the police abou t the patient’s (C) Confid entiality
hom icid al threats. (D) Ju stice
(E) Warn the patient’s m other about the (E) N onm aleficence
p atient’s threats.
16. A 36-year-old woman with delusional disord er
Questions 14 and 15 is brou ght to the em ergency room by p olice
You are the psychiatrist on d uty w hen a man, arrested after she w as fou nd tresp assing on the Gov-
by the police for assaulting his w ife, is brought into ernor ’s p rivate estate. She claim s she has
the emergency room for an evaluation. The man’s been having an affair w ith the Governor and
behavior is w ild and unpredictable. H e refuses to d em and s that he acknow led ge he is the father
answ er your questions and accuses you of having of her 5-year-old son. This is the third tim e the
an affair w ith his w ife. At one point he picks up a p atient has been caught stalking the Gover-
chair and looks menacingly at a nurse before being nor. The patient is su bsequ ently ad m itted to
subd ued by police officers. Detectives tell you that the inp atient unit and d ischarged several d ays
the man has been arrested in the past for aggression later to her p arents w ith w hom she lives. The
tow ard family members, and that the only chance of Governor u ltim ately d ecid es to pu rsue charges
his avoid ing jail time that night is to be ad mitted to against the w om an. The w om an’s attorney
the psychiatric unit. H ow ever, the man makes it clear em p loys a forensic p sychiatrist to evalu ate the
that he d oes not w ant to stay in the hospital. w om an and rend er an op inion abou t her m en-
tal status at the tim e of the crim e. Which of
14. Which of the follow ing shou ld ju stify the the follow ing statem ents regard ing her incar-
p hysician’s d ecision to involu ntarily ad m it ceration is m ost accu rate if fou nd not guilty by
the patient? reason of insanity?

(A) H istory of prior p sychiatric (A) She w ill not serve any tim e and w ill
hosp italization. be released w ith close p sychiatric
(B) Im m inent threat to others. follow -u p .
(C) Med ication noncom p liance. (B) She w ill serve half her crim inal sentence
incarcerated and the other half in a
(D) Presence of a d iagnosable m ental
locked p sychiatric facility.
d isord er.
(C) She w ill serve her original crim inal
(E) Refu sal to follow -u p w ith p sychiatric
sentence once she is d eem ed sane.
ou tpatient services.
(D) She w ill serve less tim e than if fou nd
After being ad m itted to the psychiatric unit, the gu ilty.
patient becom es increasingly aggressive tow ard (E) She w ill serve m ore tim e in a locked
staff. H e ad am antly refu ses all m ed ications offered p sychiatric u nit than she w ou ld in
to help calm his agitated behaviors. The patient at p rison if she w ere fou nd gu ilty.
one point accu ses a staff m em ber of stealing m oney
from his w allet and attem p ts to p u nch him . You Questions 17 and 18
place a stat ord er for a H ald ol 5 m g intram u scu lar A 9-year-old girl is brou ght to the p ed iatric em er-
(IM) injection to sed ate the p atient and p revent fu r- gency d ep artm ent by her p arents for evalu ation of
ther escalation of his com bative behaviors. a p ersistent cough. She is w ithd raw n and com plains
of a “scratchy throat.” Vital signs are stable and the
15. Ad m inistering the H ald ol w ithou t the p atient is afebrile. On exam ination, the lungs are
p atient’s consent d irectly challenges w hich of clear to auscultation bilaterally and the posterior
the follow ing ethical principles? oropharynx is clear. There are m ultiple bruises on
(A) Au tonom y her bu ttocks and back. A chest x-ray d em onstrates
(B) Beneficence several rib fractu res in d ifferent stages of healing.
H er p arents rep ort that their d au ghter is qu ite active
Que s tions : 14–21 145

w ith her you nger siblings and often gets into fights (D) Find ou t the boyfriend ’s nam e and
w ith them . The p atient agrees w ith her p arents. telephone num ber to confirm the
history.
17. Which of the follow ing is the m ost likely (E) Maintain confidentiality by disclosing no
d iagnosis? information, but encourage the patient to
(A) Age-app ropriate “rou gh p lay” d iscuss this issue w ith her parents.
(B) Major d ep ressive d isord er
20. At the p atient’s next follow -u p visit, she
(C) Physical abu se requ ests to be tested for H IV. She rep orts that
(D) Sexu al abu se her boyfriend recently tested negative, bu t
(E) Som atic sym p tom d isord er given her ad m itted p rom iscu ity, she w ond ers
if she has contracted H IV. Which of the follow -
18. Which of the follow ing is the m ost ap p rop ri- ing shou ld be d one p rior to ad m inistering the
ate next step ? H IV test?
(A) Confront the siblings abou t their (A) Directly inform the p atient’s boyfriend
behavior. that he is at risk for H IV.
(B) Discharge the p atient w ith follow -u p in (B) Exp lain to the p atient how the test is
1 w eek to reevalu ate her bru ises. p erform ed and interp reted , along w ith
(C) N otify Child Protective Services. inform ation on confid entiality and how
(D) Refer for fam ily therapy to ad d ress the you w ill p roceed if the resu lt is p ositive.
issu e of rou gh p lay. (C) Inform the d ep artm ent of p u blic health
(E) Treat the p atient w ith p enicillin. of you r intent to ad m inister an H IV test.
(D) N otify her p arents abou t the patient’s
Questions 19 and 20 requ est for an H IV test.
(E) Perform a p regnancy test.
A 17-year-old girl p resents to you r office com plain-
ing of a bu rning vaginal d ischarge. She inform s you
21. A 44-year-old m an w ith insu lin-d ep end ent d ia-
that she has been involved in a consensual sexu al
betes m ellitu s and end -stage renal failu re has
relationship w ith her boyfriend and is w orried that
been on d ialysis for several years w hile aw ait-
she has contracted a sexu ally transm itted d isease
ing a kid ney transp lant. H e feels as thou gh he
(STD). You start the p atient on antibiotics. Three
has “w aited long enough” and d oes not w ant
days later, you discover that the culture of her vaginal
to continu e “living tied to a m achine.” After
d ischarge grew N eisseria gonorrhoeae.
several fam ily m eetings and consultations
w ith other p hysicians, he inform s you that he
19. A few d ays later, you receive a call from the
no longer w ishes to be d ialyzed . You obtain a
p atient’s p arents d em and ing to know w hy
p sychiatric consu lt that conclu d es no evid ence
their d au ghter w as seen in you r office. Which
of a m ood or thou ght d isord er. The fam ily is
of the follow ing is the m ost appropriate next
u p set w ith the p atient’s d ecision and d em and s
step?
that you continue to ad minister d ialysis u ntil a
(A) Alert the parents that their d au ghter is transplant is available. Which of the follow ing
at risk for hu m an im m u nod eficiency is the m ost ap p rop riate next step ?
virus (H IV) and shou ld be tested .
(A) Coerce the p atient into continu ing
(B) Ask the p arents abou t their d au ghter’s treatm ent.
sexu al history.
(B) Continu e d ialysis u ntil you convince the
(C) Exp lain to the p arents that their ethics com m ittee to su p p ort the fam ily’s
d au ghter has contracted an STD and d ecision.
requ ires im m ed iate antibiotic treatm ent.
(C) Encou rage the fam ily to d issu ad e the
p atient from w ithhold ing treatm ent.
146 5: Le ga l a nd Ethica l Is s ue s in P sychia try a nd Me d ic ine

(D) Resp ect the patient’s w ishes and 24. A 23-year-old w om an w ith a know n history
d iscontinu e d ialysis. of heroin u se is ad m itted for intravenou s (IV)
(E) Tell the patient that a transplant w ill antibiotic treatm ent for infective end ocard itis.
arrive soon and encou rage him to The nu rse inform s you that the p atient w as
rem ain in treatm ent. accid entally given the w rong antibiotic bu t
has su ffered no ad verse reaction. Which of the
22. A 40-year-old su rgical attend ing is ad m itted to follow ing is the m ost ap p rop riate next step ?
the m ed ical u nit after d eveloping severe right (A) Encourage the p atient to seek legal
flank pain. Fu rther w orkup confirm s a d iag- action becau se a critical m istake has
nosis of nep hrolithiasis. One of the su rgical occu rred .
resid ents asks you abou t the attend ing’s con-
(B) Enforce the “no harm , no d isclosu re”
d ition in ord er that ad equ ate coverage can be
ru le.
arranged . Which of the follow ing is the m ost
ap prop riate next step? (C) First rep ort the m istake to the hosp ital
ad visory com m ittee.
(A) Arrange a conference betw een you r (D) Inform the p atient that she w as given
m ed ical attend ing and the su rgical the w rong m ed ication.
hou se staff.
(E) Notify the patient’s family of the mistake.
(B) Inform the su rgical resid ent of the
attend ing’s cond ition and length of stay. 25. A 38-year-old w om an is ad m itted to the oncol-
(C) Reveal only the estim ated length of stay. ogy u nit w ith severe aplastic anem ia. She
(D) Tell the su rgical resid ent that you w ill ap p ears p ale and w eak. Vital signs ind icate a
not say anything, bu t she cou ld take a blood p ressu re of 110/ 75 m m H g and a p u lse
look in the attend ing’s chart. of 110/ m in. H em atocrit is 18%. On MSE, cog-
(E) Tell the su rgical resid ent to ad d ress all nition is intact and there is no evid ence of a
concerns d irectly to the attend ing. m ood or p sychotic d istu rbance. The p atient
states that she is a Jehovah’s Witness and
23. A 43-year-old m an is referred to you for refu ses any blood transfu sion on the basis of
continued treatm ent of d epression after his her religiou s beliefs. Which of the follow ing is
release from jail. The cou rt m and ated p sychi- the most appropriate next step in the treatment
atric treatm ent w hile he is on p robation. The of this patient?
p atient’s probation officer calls you for infor- (A) Ad m inister p acked red blood cells.
m ation regard ing his cond ition, p rogress, and
(B) Exp lain the im p lications of no treatm ent
treatm ent com pliance. Which of the follow ing
bu t resp ect the p atient’s refu sal for
is the m ost ap p rop riate resp onse?
treatm ent.
(A) Corresp ond w ith the p robation officer (C) Persu ad e the p atient that she m u st
only through w ritten d ocum ents. accep t the transfu sion.
(B) Discuss the case with the probation officer (D) Refer the case to the hosp ital’s ethics
because the treatment is court mandated. com m ittee.
(C) Ignore the requ est altogether becau se (E) Refer the p atient for involu ntary
p sychiatric treatm ent bears no relation p sychiatric treatm ent based on her
to law enforcem ent. life-threatening d ecision.
(D) Lim it you r d iscussion w ith the
p robation officer to only treatm ent 26. The p arents of a new born w ith Dow n syn-
com pliance because the rest of the d rom e find their d au ghter to be lethargic
inform ation is confid ential. and m inim ally resp onsive. Med ical evalu a-
(E) Obtain a confid entiality w aiver from the tion is significant for the follow ing cerebro-
p atient before speaking to the probation spinal flu id find ings: opening pressu re of
officer. 100 m m H g, w hite blood cell cou nt of 5,000/ µL
Que s tions : 22–29 147

(p red om inantly neu trop hils), protein m ore p hysician for years, is d iagnosed w ith hepato-
than 40 m g/ d L, glu cose content m ore than cellu lar carcinom a. The patient d oes not have
40 m g/ d L, and Gram stain p ositive for bac- an ad vance d irective and never d esignated a
teria. You suspect grou p B streptococcal m en- p ow er of attorney. Psychiatric consultation
ingitis and recom m end IV antibiotic therap y. conclud es that the patient is unable to m ake
The p arents feel as thou gh their child w ill u lti- an inform ed d ecision regard ing treatm ent
m ately have a p oor quality of life and requ est options, and you tu rn to his fam ily for gu id -
that treatm ent be w ithheld . Which of the ance. The p atient’s old est son is ad am ant that
follow ing is the m ost app rop riate next step ? his father receive chem otherapy, w hile the
tw o you nger d au ghters feel that he shou ld
(A) Refer the case to the ethics com m ittee
not su ffer the ad verse effects of chem otherapy
for review at their next sched u led
“especially becau se he’s so d em ented .” Which
m eeting.
of the follow ing is the next appropriate step?
(B) Rep eat the lum bar punctu re to verify
the d iagnosis. (A) Abid e by the d aughters’ w ishes becau se
(C) Resp ect the p arents’ w ishes becau se the patient’s quality of life is already poor.
they are the prim ary d ecision m akers. (B) Abid e by the son’s w ishes becau se he is
(D) Start intravenous antibiotics against the the old est.
p arents’ w ishes. (C) Ask the p atient w hich fam ily m em ber
(E) Threaten to rep ort the p arents to child he w ou ld like to d esignate p ow er of
p rotective services unless they change attorney.
their d ecision. (D) Consu lt the hosp ital’s ethics com m ittee.
(E) Let the p atient d ecid e w hether or not to
27. You are a p sychiatrist w ho hosts a m orning p roceed w ith treatm ent.
rad io show d ed icated to ed u cating the general
p u blic abou t m ental illness. Du ring a qu es- 29. A 27-year-old w om an w ho is 3 m onths post-
tion and answ er segm ent, a caller p hones in p artu m is brou ght to the em ergency room by
to ask abou t you r op inion regard ing a prom i- her hu sband and m other w ith concerns that
nent politician w hose recent erratic behaviors the patient “is not acting like herself.” The
have gained significant m ed ia attention. The hu sband inform s the consu lting p sychiatrist
caller asks you d irectly w hether you believe that for the past few w eeks the p atient has
the p olitician has a bipolar sp ectru m d isord er. been increasingly irritable, w ithd raw n, and
Which of the follow ing resp onses is the m ost crying alm ost d aily. H e d oesn’t u nd erstand
ap prop riate? w hy the p atient is behaving like this given
that the p regnancy w as planned and the
(A) Com m ent in w ritten form only.
p atient w as looking forw ard to having a fam -
(B) Offering a p sychiatric d iagnosis in su ch ily. The p atient tearfu lly ad m its that this is her
instances is u nethical. first child and that she is overw helm ed w ith
(C) Provid e an “off-the-record ” or the responsibilities of being a new m other.
anonym ou s opinion. On fu rther qu estioning, she hesitantly con-
(D) State you r d iagnosis bu t ind icate that fesses to intru sive thou ghts of su ffocating
other problem s m ay account for the her child and som etim es w orries abou t being
sym p tom s. alone w ith the baby. She d enies su icid al id e-
(E) Stating you r op inion p u blicly is ation or aud itory/ visu al hallu cinations. The
legitim ate as long as the p olitician is not p atient is w illing to seek ou tp atient treatm ent
you r p atient. bu t ad am antly refu ses volu ntary ad m ission.
H er m other states that the p atient is “ju st
28. An 84-year-old w id ow ed m an w ith severe exhau sted ” and that “everything w ill be fine
m ajor neu rocognitive d isord er (d em entia), after she gets som e rest.” Which of the follow -
for w hom you have been the p rim ary care ing is the m ost ap p rop riate next step ?
148 5: Le ga l a nd Ethica l Is s ue s in P sychia try a nd Me d ic ine

(A) Ad m inister a stat d ose of lorazep am flattered by the p atient’s sexu al interest and are su r-
2 m g IM and reevalu ate w hen the p rised by you r ow n interest in the patient.
p atient is m ore calm .
(B) Ad m it the patient involuntarily. 31. Which of the follow ing is the most appropriate
(C) Call Child Protective Services becau se cou rse of action?
the infant is at risk of harm . (A) Decline p articip ation in the relationship
(D) Discharge the p atient and inform becau se sex w ith a form er p sychiatric
the hu sband and m other to bring p atient is u nethical.
the patient back to the hosp ital if her (B) Engage in sexu al relations becau se sex
sym ptom s d o not im prove in the next w ith a cu rrent or form er p sychiatric
few w eeks. p atient is ethical.
(E) Give the p atient a 2-w eek sup ply of (C) Engage in sexu al relations becau se sex
flu oxetine to treat her d ep ression and w ith a form er p sychiatric p atient is
then arrange ou tpatient follow -up. p erm issible only if you d o not exp loit
you r p ast p osition of au thority.
30. A physician is at a comm unity fair w ith her (D) Engage in sexu al relations becau se there
spouse w hen a patient approaches the p sychi- is no established p rofessional cod e of
atrist to say hello. The spou se d oes not recog- ethics regard ing sex w ith p sychiatric
nize the patient. The patient d oes not introd u ce p atients.
him self to the sp ou se, nor d oes the physician
(E) Inform the p atient that the p rofessional
acquaint the tw o. After a brief conversation,
cod e of ethics requ ires that you w ait
the patient politely excu ses him self and leaves.
1 year after term ination before you can
On the w ay hom e, the spouse asks, “Who w as
ethically engage in sexu al relations.
that man you w ere talking to earlier?” The
physician should d o w hich of the follow ing?
32. Which of the follow ing w ou ld be the m ost
(A) Answ er the sp ou se’s qu estion tru thfu lly. ap p rop riate cou rse of action if the above
(B) Ask the sp ou se to gu ess the id entity of p atient w ere you r m ed ical or su rgical p atient?
the person. (A) Decline particip ation in the relationship
(C) Inform the sp ou se that he m u st first becau se sex w ith a cu rrent p atient is
p rom ise not to reveal the id entity of the u nethical.
p atient before answ ering the qu estion. (B) Engage in sexu al relations becau se sex
(D) Inform the sp ou se that revealing w ith a cu rrent nonp sychiatric p atient is
su ch inform ation w ou ld com prom ise ethical.
confid entiality. (C) Engage in sexu al relations becau se sex
(E) Lie to the sp ou se to p rotect the id entity w ith a cu rrent nonp sychiatric p atient is
of the patient. p erm issible only if you d o not exp loit
you r p ast p osition of au thority.
Questions 31 and 32 (D) Engage in sexu al relations becau se there
You have been treating a fem ale patient w ho has is no established p rofessional cod e of
been seeing you for p sychod ynam ic p sychotherap y ethics regard ing sex w ith nonp sychiatric
for app roxim ately 6 m onths. N ear the end of the p atients.
su ggested cou rse of treatm ent, the p atient reports (E) Inform the p atient that the p rofessional
that she feels m arked ly better abou t her progress cod e of ethics requires that you w ait
and attributes her im p rovem ent to you r exp ertise. 1 year after term ination before you can
Prior to the last session, she confesses that she has ethically engage in sexu al relations.
alw ays found you attractive and that she w ou ld like
begin an intim ate relationship w ith you . You feel 33. A 52-year-old man, for w hom you have been
the primary care physician for the last 20 years,
Que s tions : 30–36 149

w as recently diagnosed w ith amyotrophic (B) Ensu re that the p atient is established
lateral sclerosis. The disease has rapid ly pro- w ith a casew orker to su p ervise the
gressed and he has experienced multiple respi- p atient in the com m unity before
ratory complications that likely w ill require a d ischarge.
tracheotomy. Severe muscle w eakness and atro- (C) Establish follow -u p at the p atient’s
phy are apparent in all limbs. The patient states outpatient com m u nity m ental health
that there is no meaning in continuing w ithout facility p rior to d ischarge.
his physical capacities. H e asks for your help (D) Keep the p atient on the u nit as long as it
in ending his life in a humane and d ignified is m ed ically necessary.
manner. H is MSE is unremarkable and there
(E) Sp eak w ith the inp atient social w orker
is no evidence of any psychiatric d isorder. You
to d eterm ine w hether the patient is
d iscuss the patient’s request w ith his family and
eligible for a loan.
they unanimously support his d esire to “end
the suffering.” Which of the follow ing w ould
Questions 35 and 36
be the most appropriate course of action?
You receive a subp oena from an attorney rep resent-
(A) Ignore the p atient’s requ est.
ing a p arty that has filed a law su it against one of
(B) Provid e the p atient w ith inform ation
you r p atients. The su bp oena p ertains to releasing
regard ing how to effectively end his
the m ed ical record s of you r patient.
life.
(C) Provid e the p atient w ith enou gh 35. Which of the follow ing is the m ost ap p rop riate
m ed ication refills to p rovid e a lethal next step ?
d ose.
(D) Refu se to p articip ate in assisting the (A) Contact the attorney w ho obtained
p atient w ith su icid e and focu s on the su bpoena to d iscu ss the p rocess of
resp ond ing to the p atient’s end -of-life releasing the m ed ical inform ation.
issu es. (B) Do not release the inform ation and
(E) Resp ect the p atient’s w ishes by help ing contact your patient regard ing the
him end his life in a p ainless and su bpoena.
resp ectfu l m anner. (C) Release the m ed ical record s d irectly to
the presid ing jud ge.
34. You are an inp atient p sychiatrist treating a (D) Release the m ed ical record s u p on
p atient w ith bip olar I d isord er. The p atient receiving the su bp oena.
has a long history of m ed ication noncom p li- (E) Requ est that the p atient sign a release
ance resu lting in severe, persecu tory d elu - of inform ation form and tu rn over the
sions d u ring his m anic ep isod es. After 1 w eek record s.
of treatm ent, you receive a p hone call from
hospital ad m inistration inform ing you that 36. A cou rt hearing has been organized by you r
the p atient’s insu rance w ill not cover the cost p atient’s attorney to qu ash the su bpoena you
of an ad d itional inpatient stay. You are encou r- have been issu ed . At the hearing, the ju d ge
aged to d ischarge the p atient so that the hosp i- rules that you should release the entire m ed i-
tal w ill not have to incu r these costs. You feel, cal record even thou gh your patient has not
how ever, that the p atient requ ires m ore tim e consented to the release of information. Which
on the inpatient u nit because of safety con- of the follow ing w ould be the most appropriate
cerns. Which of the follow ing actions w ou ld next step?
be the m ost ap p rop riate?
(A) Ap p eal to the state’s Sup rem e Cou rt to
(A) Contact the insu rance com pany w ithou t block the release of the m ed ical record .
the patient’s perm ission and request (B) Refu se to d isclose the p atient’s m ed ical
coverage for ad d itional d ays. record regard less of the cou rt ord ers.
150 5: Le ga l a nd Ethica l Is s ue s in P sychia try a nd Me d ic ine

(C) Release only inform ation that w ill not (A) “Becau se you r d ep ression is im p roved ,
incrim inate you r p atient. it w ou ld be ap p rop riate for u s to be
(D) Release the com p lete m ed ical record to bu siness p artners.”
the jud ge. (B) “I can invest w ith you only w hen ou r
(E) Work ou t a p lan of legal action w ith treatm ent is nearing its end .”
you r p atient. (C) “I have to d ecline; it p otentially
m ay interfere w ith ou r treatm ent
37. A cou rt-ap pointed forensic psychiatrist is relationship .”
evalu ating a m an charged w ith arm ed robbery (D) “It is p robably a bad id ea; I’m alread y
to d eterm ine w hether he is com petent to stand com m itted in other investm ents.”
trial. Du ring the interview, the d efend ant con- (E) “Thank you for thinking abou t m e. I’d
fesses to m u rd ering a w om an 3 years ago be honored to invest w ith you .”
and hid ing her bod y in an u nd isclosed area.
Which of the follow ing actions w ou ld be the Questions 40 and 41
m ost appropriate?
A 23-year-old Cau casian Catholic w om an w ith a
(A) Avoid d etails of the d efend ant’s prior
history of m ajor d epressive d isord er and m u ltiple
crim inal history in the report.
m ed ical illnesses p resents to your office after argu -
(B) Encou rage the d efend ant to sp eak w ith ing w ith her hu sband . She is a stay-at-hom e m other
his attorney about the m u rd er. of fou r child ren and ad m its that she is u nable to
(C) Im m ed iately notify au thorities of the w ork becau se of her m ed ical lim itations. She rep orts
d efend ant’s confession. that her hu sband d oes not allow her to visit w ith
(D) Inclu d e this d etail in the final rep ort. friend s or fam ily w ithou t his consent and generally
(E) Withd raw from the case. d oes not help w ith child care or other household
d u ties and obligations. As a result, she is becom -
38. A 28-year-old w om an w ith m ajor d ep ressive ing increasingly resentfu l, d ep ressed , and isolative.
d isord er has been seeing you for w eekly psy- Du ring the session, she ad m its to thou ghts of killing
chotherap y and has failed to p ay her bill for herself. Of note, she has a p ast history of an over-
2 m onths. Which of the follow ing is the m ost d ose attem pt w ith aspirin.
ap p rop riate next step ?
40. Which of the follow ing is her strongest risk
(A) Contact the patient’s family to d etermine
factor for su icid e?
if the patient is financially stable.
(B) Contract w ith a billing collector to (A) Age
d em and im m ed iate p aym ent. (B) Gend er
(C) Inform the p atient that you w ill not see (C) Marital statu s
her if she d oesn’t pay for her treatm ent. (D) Past history of su icid e attem p t
(D) Inqu ire as to the reasons she has been (E) Term inal m ed ical illness
avoid ing p aym ent at the p atient’s next
visit. 41. Which one of the follow ing, if d ocu m ented ,
(E) Term inate the treatm ent. w ould mostly likely legally protect a physician
in the event of a patient suicid e?
39. A wealthy 46-year-old male banker is in psy-
(A) A w ritten “no self-harm ” contract
chotherapy with you for treatment of a single
signed by the p atient.
episode of major depression. After significant
improvement in his symptoms, he offers you the (B) An assessm ent of su icid e risk and
opportunity to take part in one of his financial p rotective factors.
ventures. The investment appears to be sound (C) The p atient’s m issed ap p ointm ents.
and fairly lucrative. Which of the following is (D) The p atient’s refu sal to consid er
the most appropriate response to the banker? p harm acological treatm ents.
Que s tions : 37–45 151

(E) The patient’s verbal p rom ise to seek ative on the u nit. H e becom es intru sive, not able to
m ed ical attention if feeling su icid al. be red irected , and d em and s im m ed iate release. You
exp lain to him that you feel he is gravely d isabled
42. A 36-year-old m an w ith a history of bip olar I and u nable to care for him self. H e d isagrees w ith
d isord er is brought to the em ergency d epart- you and d em and s “d u e p rocess.”
m ent by p olice after stabbing a patron in a
bar room braw l. H is blood alcohol level w as 44. On w hich of the follow ing legal princip les is
0.320 u pon arrival, and the p atient requ ired the patient’s request for a hearing based ?
intram u scu lar (IM) H ald ol for agitation. The (A) Actu s reu s
p atient has no recollection of the event, and
(B) H abeas corp us
the victim d ied 2 hours later. The p atient has a
(C) Mens rea
history of assau lt w hile being m anic. H e cu r-
rently p articip ates in w eekly p sychotherap y (D) Parens p atriae
sessions and attend s his m ed ication m anage- (E) Rights u nd er the Fou rth Am end m ent
m ent ap pointm ents every m onth. H is law yer
has chosen to assert an insanity p lea in d efense 45. The cou rt agrees that the p atient is severely
of the patient. Which of the follow ing factors d isabled and in need of acu te m ed ical m anage-
is m ost likely to u nd erm ine his assertion of the m ent. You initiate treatm ent w ith qu etiapine
insanity d efense? and his sym ptom s of m ania m arked ly d im in-
ish. The p atient requ ests im m ed iate d ischarge
(A) Inability to recall the event. and agrees to follow u p w ith a p artial hospi-
(B) Mental d isease or d efect. talization program . You feel that he w ou ld
(C) Prior history of assau lt. benefit from fu rther inp atient treatm ent bu t is
(D) Voluntary intoxication. no longer gravely d isabled or a threat for self-
(E) Violent nature of the crim e. harm . Which of the follow ing is the next m ost
ap p rop riate step ?
43. You are a forensic p sychiatrist hired as an
(A) Ap pease the patient by increasing
exp ert w itness by the d efense attorney of a
sm oking privileges.
m entally ill p atient charged w ith crim inal m is-
(B) File for another cou rt hearing to d etain
cond u ct. The attorney is seeking you r help in
the patient fu rther.
convincing the ju ry that the patient w as m en-
tally ill at the tim e of the crim e. Which of the (C) Ignore the p atient’s requ est becau se he
follow ing is your prim ary resp onsibility as an has been com m itted by the court.
exp ert w itness? (D) Persu ad e the patient to stay for a few
m ore d ays.
(A) Cou ntering evid ence of crim inal
(E) Release the patient to the partial
resp onsibility.
hosp italization p rogram .
(B) Establishing reasonable d ou bt.
(C) Evalu ation, d iagnosis, and initiation of Questions 46 and 47
treatm ent of the accu sed .
(D) Obtaining a not guilty by reason of You are consu lted by the m ed ical team to evalu -
insanity verd ict. ate a w om an on the m ed ical u nit w ho su ffers from
a m ajor neurocognitive d isord er (d em entia). The
(E) Rend ering an opinion based on
internist believes that she requ ires a central line for
reasonable m ed ical certainty.
flu id s and m ed ication, bu t is u nsu re if she is able to
Questions 44 and 45 fu lly com p rehend the risks and benefits of the p ro-
ced ure. The team is requ esting help in d eterm ining
A 48-year-old m an is involu ntarily ad m itted to the her capacity to give inform ed consent.
hosp ital after an acute m anic episod e. The patient
is hyperactive, d em and ing, and increasingly talk-
152 5: Le ga l a nd Ethica l Is s ue s in P sychia try a nd Me d ic ine

46. Which of the follow ing com ponents is the d enies any hallu cinations or d elu sions. Which
m ost im portant in obtaining inform ed consent of the following items from your evaluation of
in this case? the patient most raises your suspicion regarding
a d iagnosis of m alingering?
(A) Ability to read and w rite.
(B) Absence of m ental illness. (A) Du ration of sym p tom s.
(C) Involving fam ily m em bers in this (B) Flashbacks w hile aw ake.
d iscu ssion. (C) H istory of m ajor d ep ressive d isord er.
(D) Petitioning a court to establish the (D) Lack of physical inju ries.
p atient’s com petence. (E) Prior incarcerations.
(E) Raising alternative treatm ent op tions.
49. One of you r p atients consistently m isses
47. The fam ily is su bsequ ently contacted abou t ap p ointm ents w ithou t giving you ad vance
her cond ition, and they requ est a m eeting notice. After num erou s failed attem p ts at
w ith the treatm ent team . Du ring the m eeting, resolving this issu e w ith the patient, you real-
the patient’s eld est son notifies you that the ize that the p atient’s behavior is not changing
p atient d id create a living w ill ap proxim ately and you d ecid e to d ischarge the p atient from
1 year ago, but they are unsure whether it would you r care. Which of the follow ing m ost ap p ro-
be useful. Which of the follow ing w ould you p riately d escribes w hat you shou ld d o?
tell them best d escribes the pu rpose of a living
(A) Contact the patient’s fam ily/ friend s
w ill?
to requ est their help in im p roving the
(A) Absolves personal resp onsibility. p atient’s attend ance.
(B) Arranges for fu neral services and (B) N otify the p atient’s insu rance com p any
d istribu ting her estate. that she is d ischarged from you r care.
(C) Establishes p ersonal p references (C) N otify you r staff that the p atient is not
regard ing end -of-life issu es. to be given fu rther appointm ents.
(D) Prevents the p atient from changing her (D) Write a letter to the p atient notifying her
m ind about life su pport if she becom es that she has been d ischarged effective
term inally ill. im m ed iately d u e to her failu re to
(E) Requ ests p hysician-assisted su icid e if com ply w ith treatm ent.
she becom es term inally ill. (E) Write a letter to the p atient stating that
she w ill be d ischarged in 60 d ays d ue to
48. A 28-year-old m an involved in a m otor vehicle her failu re to com p ly w ith treatm ent.
accid ent brings a law su it against the d river.
Em ergency d ep artm ent record s d o not show 50. You are asked by the cou rt to p rovid e a foren-
any p hysical inju ries, but the p atient is claim - sic evaluation for a child cu stod y case. The
ing to su ffer from p osttrau m atic stress d is- child had been living w ith his biological
ord er (PTSD). You are asked to evalu ate the m other and stepfather since birth. Recently,
p atient’s sym p tom s. H e com p lains of d istress- the child ’s stepfather passed aw ay from lu ng
ing d ream s of the accid ent, having flashbacks cancer leaving the u nem ployed m other alone
w hile aw ake, and avoid ing the intersection to raise the child . The child ’s biological father,
w here the accid ent occu rred . H is sym p tom s w ho is a renow ned orthop ed ic su rgeon in the
have lasted for 3 m onths. Past psychiatric his- com m unity, is requ esting full cu stod y becau se
tory is significant for m ajor d epressive d isor- he asserts that he is better able to financially
d er, im p u lsivity, and violent behavior. H e has su pport the child . The child ’s m other refu ses
been incarcerated nu m erou s tim es, show ing to relinquish cu stod y d u e to concerns that
a blatant d isregard for the law. On MSE, the leaving her hou sehold w ill em otionally im p act
p atient is likable and coop erative. H is m ood is the child . She also im p lies that the father has
rep orted as being d ep ressed and he cu rrently “anger p roblem s” and voices su sp icion as to
Que s tions : 46–51 153

w hether he is, in fact, the biological father. that your colleagu e has been m aking increas-
Which of the follow ing factors w ou ld be the ingly careless m istakes and m issing im p ortant
m ost im p ortant in d eterm ining w hich p arent m eetings and d iscu ssions. Occasionally, you
shou ld get custod y? observe that his hand s shake w hen he is d ocu -
m enting his notes or hold ing his charts. You
(A) Who can p rovid e for the best interests
are concerned that he m ay have an alcohol use
of the child ?
d isord er. Which of the follow ing is the m ost
(B) Who has the highest level of ed u cation? ap p rop riate cou rse of action?
(C) Who is in the best m ental and p hysical
health? (A) Confront the colleagu e and d em and that
he seek ad d iction cou nseling services.
(D) Who is the m ost financially stable?
(B) Contact the Dru g Enforcem ent
(E) Who is the biological p arent?
Ad m inistration to rescind his license.
51. You are an internal m ed icine resid ent w ho has (C) Do nothing so as to avoid p ersonal
been assigned to rotate throu gh the inp atient liability.
m ed ical service for the next 2 m onths. Shortly (D) N otify the hosp ital’s com m ittee for
after you begin, you notice that one of you r im p aired p hysicians.
colleagu es consistently com es to w ork late and (E) N otify the local p olice.
sm ells of alcohol. Som etim e later, you notice
CHAPTER 6

Diffe re ntial Diag no s is and


Manag e me nt
Que s tions

D IRECTION S (Questions 1 through 60): For each of (C) H lop erid ol (H ld ol) nd p erp hen zine
the multiple-choice questions in this section, select (Tril fon)
the lettered answ er that is the one best response in (D) Mirt z p ine (Re eron) nd cit lop r
each case. (Celex )
(E) Zip r sid one (Geod on) nd sertr line
Questions 1 and 2 (Zoloft)

A 43-ye r-old w o n p resents to you r office tell- Questions 3 and 4


ing you th t recently she h s been exp eriencing n
incre se in the volu e of voice th t she h s been A 29-ye r-old w o n w ho ju st d elivered 3 w eeks
he ring for ye rs. It const ntly criticizes her beh v- go is referred bec u se her obstetrici n noticed th t
iors reg rd less of her ood . She lso notes th t for she ppe red to be d isheveled . Upon initi l inter-
ost of the p st ye r, her ood h s been very low . view , the p tient tells you th t she h s been feel-
She is no longer ble to get ny ple su re fro w tch- ing d ow n since d elivering her son. She tells you
ing television. H er sleep is p oor nd her energy is th t w hile she continu es to c re for her son, she is
low . In d d ition, she d escribes 20-lb w eight loss in grow ing incre singly d ep ressed bec u se she d oes
the p st ye r bec u se she no longer feels the need to not get ny p le sure fro t king c re of hi . She is
e t. She d enies the u se of ny d ru gs or lcohol. u n ble to sleep nd h s not been e ting uch either.
She feels tired ll the ti e. She lso tells you th t t
1. Which of the follow ing d i gnoses best ti es, she w ill he r b by in the b ckground cry-
ccou nts for this p tient’s sy p to s? ing, bu t w hen she checks on her son, he is sleep ing
sound ly. She d enies ny thoughts of w nting to hu rt
(A) Bip ol r II d isord er her son or herself.
(B) M jor d epression w ith psychotic fe tures
(C) Schizo ffective d isord er 3. Which of the follow ing d i gnoses is the ost
(D) Schizop hreni likely?
(E) Schizoid p erson lity d isord er (A) Ad ju st ent d isord er
(B) Brief p sychotic d isord er
2. Which of the follow ing ed ic tion co bin -
(C) M jor d ep ressive d isord er w ith
tions w ou ld best tre t this p tient’s sy pto s?
p erip rtu onset nd p sychotic fe tu res
(A) Div lp roex sod iu nd lor zep (D) Postp rtu obsessive-co p u lsive
(Ativ n) d isord er
(B) Flu oxetine (Proz c) nd d i zep (E) Schizo ffective d isord er, d ep ressed typ e
(V liu )

165
166 6: Diffe re ntia l Diag nos is a nd Ma na g e me nt

4. Which of the follow ing is the ost pp rop ri- Questions 7 and 8
te first step in her tre t ent?
A 62-ye r-old w o n p resents to you r office long
(A) Flu oxetine (Proz c) nd qu eti p ine w ith her d u ghter. H er d u ghter exp l ins to you
(Seroqu el) th t l tely, her other h s been extre ely con-
(B) H lop erid ol (H ld ol) cerned bout her bod y od or. Despite ultiple re s-
(C) H osp it lize the p tient su r nces fro her d ughter th t she d oes not s ell,
(D) Lithiu she continues to be concerned th t other people find
her bod y od or extre ely offensive. The p tient tells
(E) Su p p ortive ther p y
you th t she know s th t she s ells bec u se she c n
s ell her ow n bod y od or ll the ti e d espite t king
Questions 5 and 6
u ltip le show ers throu ghou t the d y. Despite being
A 64-ye r-old w o n is brou ght to the e ergency concerned bout her od or, she continues to w ork
d ep rt ent by her neighbor, w ho s ys “ y friend fro ho e s she h s d one for the p st 30 ye rs
isn’t cting right.” The p tient requ ires the su p p ort nd she continu es to sp e k w ith friend s on the tele-
of nurse w hile w lking to n ex in tion t ble. p hone. She p ys her onthly bills w ithou t the help
Ex in tion reve ls th t she c nnot correctly id en- of her f ily. H er thinking otherw ise see s logic l
tify the se son or the tow n she is in. She d oes not nd pp rop ri te.
recognize her neighbor. She is in ttentive nd see -
ingly p thetic to the ctivity rou nd her. She d ozes 7. Which of the follow ing is the ost likely
off repe ted ly d uring the interview , bu t e ch ti e d i gnosis?
is rou s ble nd resu es nsw ering qu estions. H er (A) Delu sion l d isord er
nsw ers re illogic l nd inconsistent.
(B) M jor d ep ressive d isord er w ith
Vit l signs re w ithin nor l li its nd she
p sychotic fe tu res
is neither tre u lou s nor d i p horetic. N eu rologic
ex in tion find s bil ter l sixth nerve p lsy nd (C) Obsessive-co p u lsive d isord er
horizont l nyst g u s. Urine toxicology screen nd (D) P r noid p erson lity d isord er
blood lcohol level re neg tive. (E) Schizop hreni

5. Which of the follow ing is the ost likely 8. Which of the follow ing w ou ld be the ost
d i gnosis? p p rop ri te tre t ent?
(A) Acu te su bd u r l he to (A) C rb zep ine (Tegretol)
(B) Alcohol w ithd r w l (B) Electroconvu lsive ther p y (ECT)
(C) Folic cid d eficiency (C) Flu oxetine (Proz c)
(D) N or l p ressu re hyd rocep h lu s (N PH ) (D) Lithiu (Esk lith)
(E) Wernicke encep h lop thy (E) Ol nz p ine (Zyp rex )

6. Which of the follow ing is the ost i p ort nt Questions 9 and 10


first step in n ging this p tient?
A 29-ye r-old n w ith history of chronic schizo-
(A) Ad inistr tion of benzod i zep ine p hreni co es to the e ergency d ep rt ent w ith
(B) Ad inistr tion of folic cid te p er tu re of 102.9°F, l bile blood p ressu re rising
(C) Ad inistr tion of thi ine to 210/ 110 H g, p u lse of 110/ in, nd resp i-
(D) Co p u ted to ogr p hy (CT) sc n of the r tory r te of 22 bre ths/ in. This p tient’s ed ic -
he d tions includ e h lop erid ol, benztrop ine (Cogentin),
nd clon zep (Klonop in). H e c nnot correctly
(E) Intr venou s (IV) flu id s nd observ tion
id entify the d y, d te, or ye r, nd believes hi self
to be in city fro w hich he oved 10 ye rs go.
A f ily e ber ind ic tes th t 3 d ys go he w s
Que s tions : 4–13 167

he lthy nd co pletely oriented nd th t he h s no 11. Which of the follow ing shou ld be the ost
signific nt ed ic l or surgic l history. i ed i te n ge ent?
Physic l ex in tion reve ls th t he is in cute
(A) Ad ission to ed ic l u nit
d istress w ith hypertonicity. L bor tory ex in tion
reve ls cre tinine phosphokin se (CPK) of 45,000 (B) Ad ission to p sychi tric u nit
IU/ L, w hite blood cell cou nt of 15,000/ µL nd no (C) Med ic tion nd d isch rge to close
left shift, sod iu of 145 Eq/ L, nd cre tinine of f ily e ber
2.5 g/ d L. Lu b r p u nctu re prod u ces cle r fluid (D) Referr l to n ou tp tient p sychi trist
w ith slightly elev ted p rotein cou nt. (E) Restr ints nd ed ic tion in the
e ergency d ep rt ent
9. Which of the follow ing is the ost likely
d i gnosis? 12. Which of the follow ing ed ic tion(s) w ou ld
(A) Anticholinergic synd ro e be the ost p p rop ri te?
(B) Centr l nervou s syste (CN S) infection (A) Benzod i zep ine only
(C) M lign nt hyp erther i (B) Lithiu nd selective serotonin
(D) N eu rolep tic lign nt synd ro e (N MS) reu p t ke inhibitor (SSRI)
(E) Prolonged i obiliz tion (C) Lithiu only
(D) SSRI nd n ntip sychotic
10. With ppropri te tre t ent, the p tient recov- (E) Tricyclic ntid ep ress nt (TCA) only
ers co pletely nd returns ho e. In onth’s
ti e, he co es to the e ergency dep rt ent Questions 13 and 14
st ting th t the “voices in the w lls” re telling
hi to kill hi self. He h s t ken no edic tions A 22-ye r-old n is brou ght to the e ergency
since he left the hospit l. His vit l signs re st - d ep rt ent by p olice fter n ep isod e in w hich he
ble nd edic l workup is neg tive. Which of r ns cked the office w here he w orks looking for
the following ther pies should be initi ted first? “evid ence.” H e st rted this job 2 onths go fter
gr d u ting fro college. H e lives w ith fou r roo -
(A) ECT tes, nd he believes they re je lous of hi
(B) H lop erid ol d ep ot injections bec u se of his job nd h ve therefore been p oison-
(C) Ol nz p ine ing his food . H is f ily reve ls th t once before
(D) Physic l restr ints w hen he beg n college he w ent throu gh p eriod
(E) S fety onitoring only of “ cting cr zy” but got better w ithout tre t ent
nd h s d one w ell since. In the e ergency d ep rt-
Questions 11 and 12 ent, he is shou ting th t he h s been u p for w eek
w riting “cl ssic” book bou t ccou nting bu t th t
A 39-ye r-old w o n co es to the e ergency so eone t the office stole it fro hi . H e need s
d ep rt ent nd co pl ins th t since her boyfriend to be p hysic lly restr ined by e ergency d ep rt-
broke u p w ith her 3 onths go, she h s been sleep - ent secu rity. Physic l ex in tion nd co plete
ing nd e ting p oorly, h s lost ll interest in her l bor tory w orku p nd toxicology screen p rove to
w ork, nd feels gu ilty th t she d rove her boyfriend be neg tive.
w y. In the p st onth, she h s begu n to feel hop e-
less, help less, nd th t “life y not be w orth it.” In 13. Which of the follow ing ed ic tion(s) w ould
the p st 2 w eeks, she h s d eveloped belief th t be the ost p p rop ri te to initi te?
r re d ise se is rotting her he rt, nd over the p st
(A) Antip sychotic nd benzod i zep ine
w eek, voice h s been telling her she is no good nd
should t ke n overd ose of he rt ed ic tion she is (B) Bu sp irone (Bu Sp r)
prescribed . At first she w s ble to ignore the voice, (C) C rb zep ine (Tegretol)
how ever, she is now t the point th t she believes (D) Lithiu
she shou ld ct on it. (E) SSRI
168 6: Diffe re ntia l Diag nos is a nd Ma na g e me nt

14. Three onths l ter, the p tient sees his d octor (A) Acu te stress d isord er
for follow -u p . H e is t king lithiu nd h lo- (B) Ad ju st ent d isord er
p erid ol. H e is d oing w ell, except he co pl ins (C) MDD
of p inful u scle cr p ing. H is lithiu level
(D) P nic d isord er
is 0.8 Eq/ L. Which of the follow ing w ou ld
be the ost p p rop ri te next step in his (E) PTSD
n ge ent?
17. A p tient w ith history of bip ol r d isord er is
(A) Decre se the h lop erid ol d ose. d itted to p sychi tric hosp it l in n cute
(B) Decre se the lithiu d ose. nic ep isod e. H er ed ic tions inclu d e n
(C) Incre se the h lop erid ol d ose. SSRI nd benzod i zepine, w hich re both
(D) Incre se the lithiu d ose. d iscontinu ed on d ission. An ntipsychotic
nd ood st bilizer re st rted . Tw o d ys
(E) St rt b clofen (Liores l).
fter d ission, she c lls the nu rsing st ff to
her bed . She is extre ely frightened nd co -
Questions 15 and 16
p l ins excited ly th t she c nnot stop looking
A 29-ye r-old w o n tells her d octor th t bou t u p . On ex in tion, her eyes re noted to be
3 w eeks go she w s c rin g for ch ild w ho r n d evi ted up w rd , bil ter lly. Which of the fol-
into the street nd w s killed by bu s. Since then, low ing sid e effects is ost consistent w ith her
she c nn ot get th e i ge of the ccid ent ou t of h er p resent tion?
ind . Even in sleep , she d re s bou t it nd it p re- (A) N MS
vents her fro sleep ing ore th n few hou rs t
(B) Ocu logyric crisis
nigh t. She u sed to t ke bu s to w ork, bu t she n ow
d rives bec u se she c nnot be r to be ne r bu ses s (C) Retrocollis
this c u ses her to think bou t the ccid ent. In the (D) Torticollis
p st w eek, sh e h s begu n issin g w ork bec u se (E) Tris u s
she is u n co fort ble le ving her h ou se. Sh e feels
extre ely gu ilty, believing the ccid ent w s h er Questions 18 and 19
f u lt.
A 59-ye r-old w o n w ith long history of gener-
15. Which of the follow ing is the ost likely lized nxiety d isord er (GAD) tells her p ri ry c re
d i gnosis? d octor th t 2 d ys go w hile in crow d ed su p er-
rket she felt d izzy, long w ith ssoci ted he rt
(A) Acu te stress d isord er p lp it tions, p ressu re on her chest, nd fright-
(B) Ad ju st ent d isord er ening sense of d oo . Shortly there fter, she fell
(C) M jor d ep ressive d isord er (MDD) u nconsciou s nd w oke u p inu tes l ter to crow d
(D) P nic d isord er nd gor p hobi rou nd her. She felt so ew h t better nd rejected
others’ d vice th t n bu l nce be c lled . She
(E) Posttr u tic stress d isord er (PTSD)
qu ickly d e her w y ho e.
16. The p tient d ecid es g inst ny ed ic tion
18. Which of the follow ing w ou ld be the ost
bu t follow s u p w ith p sychother p y. A ye r
p p rop ri te next step in her n ge ent?
l ter, lthou gh she is no longer h ving d is-
tressfu l sy pto s rel ting to the ccid ent, she (A) Cognitive-beh vior l ther p y
feels s d nd te rfu l ost of the ti e, is h ving (B) Electroc rd iogr (ECG)
trou ble e ting, h s lost interest in g rd ening, (C) Re ssu r nce th t her cond ition is benign
nd w kes u p t 4 a m every orning, u n ble
(D) Short- cting benzod i zep ines
to get b ck to sleep. She const ntly feels tired
throughout the d y. Which of the follow ing is (E) SSRI
the ost likely d i gnosis?
Que s tions : 14–24 169

19. Which of the follow ing d i gnoses is the ost to feel d epressed , helpless, nd hopeless, nd
i p ort nt to ru le ou t first? he su bsequ ently overd oses on his ed ic tion.
H is w ife find s hi u nconsciou s nd p roceed s
(A) Acu te stress d isord er
to c ll 911. Which of the follow ing w ou ld be
(B) C rd iov scu l r d ise se the ost likely c u se of d e th?
(C) GAD
(A) C rd i c rrhyth i
(D) Illness nxiety d isord er
(B) Resp ir tory f ilu re
(E) P nic tt ck
(C) Seizu re
20. A 28-ye r-old le p resents to you r office (D) Shock
bec u se he w nts to stop p rticu l rly d is- (E) Stroke
turbing beh vior. H e tells you th t he often
goes on p u blic tr ins w ith the intention of try- Questions 23 and 24
ing to ru b his genit ls on other p eop le w ithou t
their consent. H e continu es to h ve f nt sies A 19-ye r-old n w ith no p reviou s psychi tric his-
bou t this, bu t he w ishes to stop bec u se he tory is noted by his college roo te to be cting
is fr id of getting into trou ble. Which of the biz rrely for the p st onth nd h lf, h ving con-
follow ing is his ost likely d i gnosis? vers tions w ith people w ho re not there, w lking
rou nd the d or itory roo n ked , nd ccusing the
(A) Exhibitionistic d isord er roo te of c lling the Dep rt ent of H o el nd
(B) Frotteu ristic d isord er Secu rity to h ve hi onitored . The p tient’s vit l
(C) Sexu l sochis d isord er signs re ll w ithin nor l li its nd his neurologic
(D) Sexu l s d is d isord er ex in tion show s no d eficits or bnor lities.
(E) Voyeu ristic d isord er
23. Which of the follow ing tests w ou ld be ost
usefu l in the initi l d i gnosis of this p tient?
Questions 21 and 22
(A) Co p lete blood cou nt (CBC)
A 53-ye r-old n presents to your office co pl in-
(B) Erythrocyte sed i ent tion r te (ESR)
ing of w orsening d epression. H e st tes th t he no lon-
ger enjoys spend ing ti e w ith his f ily, he is only (C) Liver fu nction tests
getting few hours of sleep t night, nd his ppetite (D) N oncontr st CT sc n of the br in
is uch low er th n it used to be. H e feels tired ll d y (E) Toxicology screen
long. You d ecid e to initi te n SSRI.
24. If the bove test w ere neg tive, w hich of the
21. Which of the follow ing sid e effects is he follow ing w ou ld be the ost likely d i gnosis?
ost likely to exp erience fter 3 onths of
(A) Delu sion l d isord er
tre t ent?
(B) M jor d ep ression w ith p sychotic
(A) Di rrhe fe tu res
(B) H e d ches (C) Schizop hreni
(C) N u se (D) Schizop hrenifor d isord er
(D) Sed tion (E) Su bst nce-ind u ced p sychosis
(E) Sexu l d ysfu nction
Questions 25 and 26
22. The p tient retu rns few ore ti es, nd
d u ring e ch visit, you incre se the d os ge of You h ve been sked by the su rgery te to ev lu -
the SSRI. H ow ever, he d oes not report ny te 35-ye r-old n w ho h d surgery to rep ir his
d ecre se in his d epressive sy pto s. Bec use fr ctu red right w rist 24 hou rs go nd is now co -
of his l ck of response to n SSRI, you d ecid e to pl ining of nxiety. The p tient h s been in the hos-
initi te tricyclic ntid ep ress nt. H e continu es pit l for 2 d ys. H is he rt r te is 120 be ts/ in, nd
Que s tions : 25–35 171

31. Which of the follow ing w ould be the ost Questions 34 and 35
d ngerou s sid e effect of bru p tly stop p ing the
bove ed ic tion? A 34-ye r-old fe le p resents to you r office s she
h s been feeling d epressed nd ngry for the p st
(A) Au tono ic hyp er ctivity cou ple of d ys bec u se she believes th t her boy-
(B) Seizu res friend is going to le ve her. As resu lt, she h s been
(C) H llu cin tions c lling hi every hour ju st to confir th t he is not
(D) Worsening nxiety le ving her. She tells you th t she h s h d ny rel -
tionships in the p st w hich h ve lw ys been very
(E) Vo iting
rocky, c u sing her e otions to const ntly go u p nd
d ow n. When ny of these rel tionships end ed ,
Questions 32 and 33
she cu t on her r s sup erfici lly to “relieve stress.”
A 73-ye r-old n is d itted to the hosp it l for At this ti e, she tells you th t she d oes h ve so e
co u nity- cqu ired pneu oni nd d ehyd r - thou ghts of w nting to cut herself, bu t no thoughts
tion. On d y 2 of his hosp it liz tion, you re sked of w nting to end her life.
to ev lu te the p tient for d ep ression; the st ff h s
noted th t he see s very w ithd r w n. H e is not e t- 34. Which of the follow ing is the ost likely
ing or sleeping w ell. The nu rsing st ff reports th t d i gnosis?
l st night he w s ngry nd requ ested to le ve the (A) Bip ol r d isord er
hospit l. You t lk w ith the p tient’s f ily nd find
(B) Bord erline p erson lity d isord er (PD)
th t the p tient h s no previou s psychi tric history.
Prior to the onset of this illness 5 d ys go, he h d (C) H istrionic PD
no d ep ressive sy pto s nd no d ifficu lties w ith (D) M jor d ep ressive d isord er
cognition. On ex in tion, his vit l signs re te - (E) N rcissistic p erson lity d isord er
per tu re of 98.2°F, he rt r te of 87 be ts/ in,
blood pressure of 130/ 86 H g, nd p erip her l 35. Which of the follow ing is the ost pp rop ri-
oxygen s tur tion of 95% on roo ir. H e is d row sy te tre t ent for this d i gnosis?
nd oriented only to p erson. H is Mini-Ment l St te
(A) Arip ip r zole (Abilify)
Ex in tion (MMSE) score is 23/ 30.
(B) Cit lop r (Celex )
32. Which of the follow ing is the ost likely (C) Cognitive-beh vior l ther p y
d i gnosis? (D) Di lectic l beh vior l ther p y
(E) Psycho n lysis
(A) Anxiety d isord er
(B) Deliriu Questions 36 and 37
(C) F ctitiou s d isord er
(D) M jor d ep ressive d isord er (MDD) A 65-ye r-old w o n w ith p st ed ic l history
of non–insu lin-d ep end ent d i betes ellitu s nd
(E) M jor neu rocognitive d isord er
d epression is d itted w ith incre singly d epressed
(d e enti )
ood over the l st onth. She is u n ble to co -
plete her crossw ord pu zzles bec use of d ifficu lty
33. The ed ic l te sks for ed ic tion reco -
concentr ting. She h s trouble f lling sleep nd
end tions if the p tient beco es git ted .
lso w kes u p in the id d le of the night. She d enies
Which of the follow ing w ou ld be the ost
su icid l id e tion, but d oes feel gu ilty th t she is
pp rop ri te ed ic tion to reco end ?
d epressed . Prior to this episod e, she w s d oing w ell
(A) Benztrop ine nd w s ctively eng ged in co u nity volunteer
(B) Dip henhyd r ine grou p s. In the l st onth, she h s lost 13 lb d ue to
(C) Lor zep poor int ke. When sked w hy she is not e ting, she
st tes she is w orried th t she w ill beco e infected
(D) H lop erid ol
w ith b cteri . She h s been to her p ri ry p hysici n
(E) Thiorid zine
for n ev lu tion, bu t she cl i s everything w s
170 6: Diffe re ntia l Dia g nos is a nd Ma na g e me nt

his blood pressure is 160/ 106 H g. H e is febrile 28. Which of the follow ing w ou ld be the ost
nd reports th t he h s never su ffered nything like pp rop ri te tre t ent for this p tient?
this before. H e is not in ny p in nd h s no pre-
(A) A low -sti u lu s environ ent
viou s psychi tric history. H is ed ic tion inclu d es
only cet inophen for p in control. You note th t (B) Benztrop ine
he is d i phoretic, flu shed , nd tre u lou s. (C) Methylp henid te (Rit lin)
(D) Phenytoin (Dil ntin)
25. Which of the follow ing is the ost p p rop ri te (E) Prop r nolol (Ind er l)
ed ic tion to tre t this p tient?
(A) C rb zep ine 29. A 40-ye r-old d ivorced w o n is brought
in to the e ergency d ep rt ent fter being
(B) Clonid ine (C t p res)
fou nd sleep ing in p ile of le ves. Initi lly, she
(C) Lor zep is d ifficu lt to rou se nd h s trou ble nsw er-
(D) Meth d one ing you r qu estions d u e to u bling. She
(E) N ltrexone (ReVi ) d oes not ppe r to be in respir tory d istress,
bu t her p u p ils re noted to be p inp oint. After
26. Withou t the bove tre t ent, w hich of the few hou rs, she beco es ore inter ctive nd
follow ing co p lic tions is he ost t risk for is co p l ining of d iffu se, cr p y bd o in l
d evelop ing? p in nd sy p to s of “the flu .” H er pu p ils
re now slightly d il ted , nd she is y w ning.
(A) Abd o in l p in
H er vit l signs re te p er tu re of 99.3°F,
(B) Cirrhosis he rt r te of 99 be ts/ in, nd blood pressu re
(C) F tty liver of 142/ 90 H g. Cu rrently, w hich of the
(D) Mu scle cr p s follow ing is the ost likely d i gnosis?
(E) Seizu res (A) C nn bis bu se
(B) Coc ine intoxic tion
Questions 27 and 28
(C) Coc ine w ithd r w l
You re w orking in the psychi tric e ergency (D) Op i te intoxic tion
d ep rt ent of l rge etropolit n hospit l. A (E) Op i te w ithd r w l
20-ye r-old n w ith unknow n p sychi tric his-
tory is brou ght in by the p olice fter being fou nd Questions 30 and 31
stu bling n ked rou nd loc l college c p u s.
H e is rked ly git ted , p cing, nd pp e rs to be A 70-ye r-old w id ow is d itted for n ev lu tion
resp ond ing to intern l sti u li. On ex in tion, you of d epression nd nxiety. She tells you th t for
note th t he is t chyc rd ic w ith he rt r te in the the p st 15 ye rs her f ily d octor h s prescribed
110s, he h s tics or sp s s in his f ce, nd he h s “so e p ills” th t h ve help ed her sleep nd feel less
vertic l nyst g u s. nervou s. She s ys th t she r n ou t of the yesterd y
nd since th t ti e h s felt incre singly nxiou s nd
27. Which of the follow ing tests w ou ld be ost jittery. She lso notes th t she’s now h ving tre ors
helpfu l in the d i gnosis? in her h nd s th t h ve not been there before.
(A) CBC
30. Which of the follow ing ed ic tions w ou ld be
(B) Electroencep h logr (EEG) the ost d ngerou s to su d d enly d iscontinue?
(C) M gnetic reson nce i ge (MRI) of the
br in (A) Flu p hen zine (Prolixin)
(D) N oncontr st he d CT (B) I ip r ine (Tofr nil)
(E) Toxicology screen (C) N ortrip tyline (P elor)
(D) Thiorid zine (Mell ril)
(E) Tri zol (H lcion)
172 6: Diffe re ntia l Dia g nos is a nd Ma na g e me nt

nor l. H er hu sb nd confir s th t the p tient h s looks frightened nd sks, “Wh t is h p p ening to


been very w orried bou t “getting d ise se” to the e?” H is ood is “OK” nd his ffect is fl t. H e is
p oint w here she w ill e t only food in se led con- fu lly oriented . H e s ys th t he he rs nu ber of
t iners. H e lso confid es in you th t she h s been p eople t lking to hi , “ ybe in y he d ,” s ying
w orried th t she ight h ve c ncer, nd , d esp ite b d things bou t hi .
re ssu r nces fro her p ri ry c re p hysici n, she
continu es to voice her concerns to her hu sb nd th t 38. Which of the follow ing d i gnoses is ost
“ y intestines re not w orking.” likely?
(A) Brief p sychotic d isord er
36. Which of the follow ing is the ost likely
d i gnosis? (B) Delu sion l d isord er
(C) Schizop hreni
(A) M jor d ep ressive d isord er (MDD)
(D) Schizop hrenifor d isord er
w ithou t p sychotic fe tu res
(E) Su bst nce-ind u ced p sychotic d isord er
(B) MDD w ith p sychotic fe tu res
(C) Obsessive-co p u lsive d isord er (OCD) 39. Which of the follow ing l bor tory tests or pro-
(D) Persistent d ep ressive d isord er ced u res w ould be the ost helpfu l in n rrow -
(d ysthy i d isord er) ing the bove d ifferenti l?
(E) So tic sy p to d isord er
(A) CBC
37. Which of the follow ing ed ic tion(s) w ou ld (B) EEG
be the ost p p rop ri te tre t ent for this (C) Electrolytes
p tient? (D) N oncontr st he d CT
(A) Div lp roex sod iu (E) Toxicology screen
(B) Lithiu nd sertr line
Questions 40 and 41
(C) N ortrip tyline nd lor zep
(D) Sertr line A 35-ye r-old w o n w ith no previous person l or
(E) Sertr line nd risp erid one f ily psychi tric history is brought to the e er-
gency d ep rt ent by her husb nd, w ho reports th t
Questions 38 and 39 his w ife w s tte pting to kill herself by cutting her
w rists. H er husb nd tells you th t 6 onths go the
An 18-ye r-old n is brou ght to the p sychi tric p tient’s gr nd other d ied . Since th t ti e, her hus-
e ergency d ep rt ent by his p rents for ev lu tion b nd believes th t the p tient h s been beco ing
of his beh vior. Three onths go, the p tient st rted ore d epressed . She h s d ifficulty f lling sleep nd
cl sses t the st te university loc ted in d ifferent h s lost 15 lb in 2 onths. She w s recently fired fro
city, lthough he w ou ld co e ho e e ch w eekend her job s p r leg l bec use she w s un ble to con-
to visit. H is p rents h ve noticed th t over the p st centr te nd d e frequent ist kes. She feels guilty
3 w eeks he h s beco e incre singly w ithd r w n nd th t she is un ble to feel better, nd she end orses feel-
d oes not see to be t king c re of hi self. The p r- ings of hopelessness nd w orthlessness. She believes
ents w ere recently c lled by the p tient’s roo te th t her “only w y out of this” is to kill herself.
w ho infor ed the th t the p tient h s not been
going to cl sses for the l st w eek, h s not been e ting 40. Which of the follow ing w ou ld be the ost
or b thing, nd h s been spe king bout how peo- p p rop ri te tre t ent?
p le in his cl sses re trying to kill hi . The p tient
(A) Flu zenil (Ro zicon)
d enies ny d ru g u se nd is not on ny ed ic tion.
On ex in tion, the p tient’s vit l signs re w ithin (B) Flu oxetine
nor l p r eters. H is physic l (inclu d ing neu ro- (C) Flu p hen zine
logic ex in tion) is u nre rk ble. H e is p cing (D) Lor zep (Ativ n)
nd ppe rs to be resp ond ing to intern l sti uli. H e (E) Phenelzine (N rd il)
Que s tions : 36–45 173

41. After being begu n on the p p rop ri te ed ic - the d e n of her college is concerned bout her nd
tion, w hich of the follow ing sid e effects y w orried th t she y be d ep ressed . The p tient tells
i ic w orsening sy pto of her illness? you th t d esp ite being f r w y fro her ho e for
the first ti e, she w s enjoying school nd her new
(A) Ak thisi
friend s u ntil 2 onths go, w hen she le rned th t
(B) Constip tion her p rents w ere getting d ivorced . Since th t ti e,
(C) Di rrhe her gr d es h ve gone fro As nd Bs, to Bs nd Cs
(D) Inso ni bec u se she is concerned bou t her p rents nd her
(E) N u se sister t ho e. She feels “bu ed ou t” ost of the
ti e, nd her friend s note th t she see s unh ppy
Questions 42 and 43 nd occ sion lly beco es te rfu l w hen t lking
bou t her f ily. She d enies d ifficu lty sleep ing or
A 46-ye r-old n w ith no previou s p sychi tric his- ch nges in ppetite or w eight, nd she continu es to
tory p resents w ith co pl ints of feeling d epressed enjoy d ily trip s to the gy to exercise. She d enies
since his w ife d ied 2 w eeks go fro p ncre tic c n- ny suicid l id e tion.
cer. H e sh res th t lthough he know s she is d e d ,
he so eti es he rs her c lling his n e. This h s 44. Which of the follow ing is the ost likely
occu rred sever l ti es nd usu lly h pp ens ore d i gnosis?
often t night w hen he is f lling sleep. H e d enies
ny visu l h llu cin tions, p r noi , or d elu sion l (A) Acu te stress d isord er
beliefs. H e expresses gu ilt bou t not sp end ing ore (B) Ad ju st ent d isord er w ith d ep ressed
ti e w ith his w ife w hen she w s live bu t d enies ood
ny thou ghts of w nting to end his life. H e is now (C) GAD
sleeping only bou t 3 to 4 hou rs t night nd h s (D) MDD
h d slightly d ecre sed pp etite since her d e th.
(E) N or l grief
Desp ite the bove, he h s still been ble to go to
w ork nd focu s on his job.
45. Which of the follow ing tre t ents w ou ld be
ost p propri te?
42. Which of the follow ing is the ost likely
d i gnosis? (A) Di lectic l beh vior l ther p y
(A) Ad ju st ent d isord er (B) Flu oxetine (Proz c)
(B) M jor d ep ressive ep isod e (C) N ortrip tyline
(C) N or l grief (D) Psychother p y
(D) Persistent d ep ressive d isord er (E) Risp erid one (Risp erd l)
(d ysthy ic d isord er)
Questions 46 and 47
(E) Schizop hreni
A 26-ye r-old w o n w ith no p reviou s psychi tric
43. Which of the follow ing w ou ld be the ost history is referred to you by her p ri ry c re p hysi-
pp rop ri te tre t ent for this p tient? ci n for ev lu tion of “ nxiety tt cks.” She tells you
(A) A itrip tyline (El vil) th t p p roxi tely 2 onths go she beg n h ving
period s l sting 10 or 15 inu tes d u ring w hich, she
(B) Flu oxetine (Proz c)
s ys, “I feel like I’ going to d ie.” Du ring these epi-
(C) H lop erid ol (H ld ol) sod es, her he rt r ces, she feels s thou gh she c nnot
(D) Lithiu c tch her bre th, she is d izzy nd fr id she y p ss
(E) N o intervention t this ti e out, nd she h s tingling nd tre ors in her h nd s.
She is concerned bec use she is now h ving d iffi-
Questions 44 and 45 cu lty le ving her hou se d ue to w orry th t these ep i-
sod es w ill occu r, king it i p ossible for her to get
You re sked to see n 18-ye r-old w o n w ith
ho e. She is u n ble to id entify ny triggers le d ing
no p reviou s psychi tric or ed ic l history bec u se
174 6: Diffe re ntia l Dia g nos is a nd Ma nag e me nt

to the episod es. She is not on ny ed ic tion nd (C) Obsessive-co p u lsive p erson lity
h s no ed ic l proble s. d isord er (OCPD)
(D) P nic d isord er
46. Which of the follow ing is her ost likely (E) So tic sy p to d isord er
d i gnosis?
(A) GAD 49. Which of the follow ing tre t ents w ould be
(B) P nic d isord er nd gor p hobi ost ppropri te for this p tient?
(C) Sep r tion nxiety d isord er (A) Cognitive-beh vior l ther p y
(D) Soci l nxiety d isord er (soci l p hobi ) (B) ECT
(E) Sp ecific p hobi (C) Lithiu
(D) Psychod yn ic p sychother p y
47. Which of the follow ing w ou ld be the ost (E) Risp erid one
p propri te tre t ent to initi te?
(A) Alp r zol 50. A 47-ye r-old secret ry co es to your office
(B) Lithiu co pl ining, “I’ lw ys w orried .” She s ys
th t she w orries bout her job, her kid s, her
(C) Prop r nolol
hou sew ork, nd her hu sb nd . She is seeking
(D) Sertr line (Zoloft) help now bec u se she h s been h ving n
(E) Tr nylcyp ro ine (P rn te) incre singly d ifficu lt ti e concentr ting t
w ork nd h s been ore irrit ble w ith p eo-
Questions 48 and 49 ple rou nd her. H er sleep h s been “ok y,”
bu t she d oes not feel rested w hen she gets u p
A 36-ye r-old w o n w ith no for er p sychi t-
in the orning. She h s been ore w re of
ric history is referred to you by d er tologist
these feelings over the l st 2 ye rs nd they
for ev lu tion of her chronic lly ch pped h nd s.
occu r l ost every d y. She d enies ny d is-
She s ys she h s been seeing her d er tologist for
crete p nic tt cks. Which of the follow ing
p proxi tely 5 ye rs for this proble nd tre ted
d i gnoses is the ost likely in this p tient?
w ith v riety of top ic l gents w ith li ited su c-
cess. Over the p st 3 w eeks, her h nd s h ve beco e (A) GAD
w orse, to the p oint w here they re lw ys cr cked (B) OCD
nd bleed ing. Relu ct ntly, she confid es in you th t (C) Soci l nxiety d isord er (soci l p hobi )
she h s h d longst nd ing fe r of ger s, bu t since
(D) P nic d isord er
her colle gu e t w ork h s been sick, she h s been
w shing her h nd s t le st 40 ti es p er d y bec u se (E) Schizop hreni
she is fr id of contr cting the d ise se. She lso
refu ses to tou ch nything th t ight infect her w ith- 51. A 28-ye r-old w o n w ith no p revious psy-
ou t u sing h nd kerchief. She d its to being very chi tric or ed ic l history is d itted to the
tid y t ho e, s w ell. She spend s bou t 2½ hou rs neu rology service for ev lu tion of cu te onset
in the orning getting show ered . She re lizes her of nu bness nd w e kness of the right sid e of
fe rs of cont in tion re irr tion l, but every ti e her f ce nd right r nd leg. Physic l ex -
she tries to stop w shing her h nd s she beco es in tion show s sy etric l 2/ 4 reflexes in
incre singly nxiou s. ll d istribu tions, d ow ngoing p l nt r reflexes
bil ter lly, nd 2/ 5 strength in the right u p per
48. Which of the follow ing d i gnoses w ou ld be nd low er extre ity in ll u scle group s. N o
ost likely? trop hy or f scicul tions re noted . H er g it
is t xic nd st ggering w ith extre e ex g-
(A) GAD ger ted ove ents of her r s; how ever, she
(B) Obsessive-co p u lsive d isord er (OCD) d oes not f ll w hen bu l ting w ithou t ssis-
t nce. Given the severity of her d eficits, she
Que s tions : 46–55 175

see s u nconcerned by her level of d is bility. As.” On ex in tion, she is c l , thin w o n


Which of the follow ing d i gnoses w ou ld be d ressed in very b ggy jogging suit. You co -
the ost p propri te? ent on her thinness, nd she tells you she p rid es
herself on her pp e r nce nd tries to st y sli by
(A) Conversion d isord er
exercising bou t 4 hou rs d y long w ith good
(B) F ctitiou s d isord er d iet. She d enies proble s w ith e ting too u ch or
(C) M lingering too little, nd cl i s th t only d iet nd exercise help
(D) So tic sy pto d isord er her to control her w eight. Desp ite you r concerns,
(E) Und i gnosed neurologic d ise se she d its th t she w ou ld like to lose few ore
p ou nd s. Up on fu rther qu estioning, she infor s you
52. A 54-ye r-old w o n w ith p st ed ic l she h s not h d regu l r enstru l period for over
history of hypothyroid is is d itted w ith ye r.
septic right knee. The surgery te sks you
to ev lu te the p tient bec use they fou nd 53. Which of the follow ing w ou ld be the ost
th t the flu id sp ir te fro the knee w s likely w orking d i gnosis?
grow ing p thogen fou nd p ri rily in the (A) Anorexi nervos
hu n ou th. They su sp ect the p tient w s
(B) Anxiety d isord er
injecting s liv into her knee. You ev lu te the
p tient nd find her to be p le s nt nd coop - (C) Bod y d ys orp hic d isord er
er tive. She tells you th t she h s h d very (D) Bu li i nervos
tough ti e l tely bec use her husb nd h s (E) MDD
recently been sick. Fortu n tely, she is nu rse
nd h s been ble to c re for hi t ho e. 54. Which of the following tests would be ost help-
L tely, she d its to feeling overw hel ed ful in supporting your provision l di gnosis?
nd not pp reci ted . She h s no id e w h t h s
(A) ECG
c used the proble w ith her knee. You t lk to
the f ily nd they tell you the p tient is in (B) Seru yl se
no fin nci l d ifficu lty nd continu es to enjoy (C) Seru gnesiu level
w ork s nu rse. After w orking closely w ith (D) Seru p ot ssiu level
you for sever l w eeks, she eventu lly d its (E) Weight nd height
to injecting her knee lthou gh c nnot u nd er-
st nd w hy she d id it. Which of the follow ing 55. A 5-ye r-old boy is referred to you by his ped i-
is the ost p p rop ri te d i gnosis? trici n for ev lu tion of ggressive beh vior.
(A) Conversion d isord er H is ed ic l nd extensive neu rologic w orku p
w s neg tive. An interview w ith the p tient
(B) F ctitiou s d isord er
reve ls restless boy w ho is ble to eng ge in
(C) Illness nxiety d isord er convers tion. H e tells you he gets ngry nd
(D) M lingering fru str ted in school s he “thinks it’s boring.”
(E) So tic sy p to d isord er H is p rents rep ort th t he is cu rrently rep e t-
ing kind erg rten d u e to p oor p erfor nce
Questions 53 and 54 nd d ifficu lty soci lizing w ith other child ren.
H is ggressive ou tbu rsts t school see to
An 18-ye r-old w o n in her first ye r of college occu r t ti es w hen he d oes not und erst nd
co es to see you for ev lu tion of d epression, fter the schoolw ork. H is other tells you th t she
her roo tes encour ged her to seek help . She still h s to help hi pick ou t his clothes for
rep orts d ifficu lty f lling sleep nd e rly orning school nd get d ressed . H is f ily history is
w kenings, poor concentr tion, f tigue, nd nxi- p ositive for tw o p tern l u ncles w ith le rn-
ety over the p st onth since rriving for the f ll ing d is bilities. Which of the follow ing tests
se ester. She tells you th t her p rents re very strict w ou ld be ost help fu l in the ev lu tion of
nd she is w orried th t she w ill not “get str ight this p tient?
176 6: Diffe re ntia l Dia g nos is a nd Ma na g e me nt

(A) Good enough-H rris Dr w -A-Person p resents w ith od d beh vior, confu sion,
Test blood p ressure of 128/ 85 H g, nd he rt
(B) Kohs Block Test r te of 68 be ts/ in. H is p rents rep ort th t
(C) Minnesot Mu ltip h sic Person lity t ti es they h ve observed hi repetitively
Inventory-2 (MMPI-2) tou ching his sto ch nd r pid ly blinking his
eyes. On ex in tion, he p p e rs d zed nd
(D) Pe bod y Voc bu l ry Test
u n ble to concentr te. Which of the follow ing
(E) Wechsler Intelligence Sc le for Child ren is the ost likely d i gnosis?
(WISC)
(A) A p het ine toxicosis
Questions 56 and 57 (B) Ep ilep sy
(C) H yp oglyce i
An 8-ye r-old boy is brou gh t in to you r office
by h is oth er bec u se sh e is con cern ed th t h e (D) MDD
c nnot stop blinking his eyes nd shru ggin g his (E) So tic sy p to d isord er
sh ou ld ers. Sh e tells you th t this occu rs l ost
every d y n d h s been th is w y for the p st ye r 59. A 50-ye r-old n w ith long history of IV
nd h lf. Sh e tells h er son to stop d oin g this d rug u se is brought to the hosp it l by p olice
w h ile h e is in you r office, n d h e d oes stop . You fter loc l ho eless shelter w orker noted
notice, h ow ever, th t h e st rts b rkin g n d cou gh- hi to be confu sed nd “w lking funny.”
in g rep e ted ly. While t the shelter, he bec e very su spiciou s
of the w orkers nd ccu sed the of t king his
56. Which of the follow ing ed ic tions w ou ld be belongings. On ex in tion, you observe th t
the ost ppropri te in tre ting the p tient’s he h s left p u p il th t cco od tes but d oes
cond ition? not re ct, d epressed d eep tend on reflexes in
ll d istribu tions, nd loss of p osition sense t
(A) Ato oxetine the gre t toes bil ter lly. Althou gh the p tient
(B) Clonid ine w s tre ted w ith ntip sychotic ed ic tions in
(C) L otrigine the p st, he d enies cu rrently t king ny ed i-
(D) Lor zep c tion. Which of the follow ing d i gnoses best
(E) V lp roic cid ccou nts for this p tient’s sy p to s?
(A) Antip sychotic-ind u ced d yskinesi
57. You begin the p p rop ri te ed ic tion nd
(B) Kors koff p sychosis
2 onths l ter the boy nd his other retu rn
(C) N eu rolep tic lign nt synd ro e
for follow -u p visit. She tells you th t w hile
the eye blinking nd should er shrugging h ve (D) N eu rosyp hilis
gotten better, he is now f lling sleep in his (E) Wernicke encep h lop thy
cl sses t school. Wh t is the next ost p pro-
p ri te step in the n ge ent? 60. A 57-ye r-old w o n w ith no p revious p sy-
chi tric history co pl ins of incre sing nxi-
(A) Ad d nother ed ic tion to keep the boy ety over the l st 2 onths. Tod y she reports
w ke. th t it bec e “very b d .” She lso notes th t
(B) Ed u c te the other bou t p rop er sleep w ith these p eriod s of nxiety she gets p ou nd -
hygiene. ing he d che nd once f inted . She continu es
(C) Discontinu e the ed ic tion. to feel “sh ky.” When you check her vit l
(D) Red u ce the d os ge. signs, her he rt r te is 170 be ts/ in nd her
(E) Sw itch to nother ed ic tion. blood p ressu re is 230/ 130 H g. She is d i -
p horetic nd tre u lou s. Given this p tient’s
58. A 17-ye r-old boy w ith h istory of tten - sy p to s, w hich of the follow ing cond itions
tion-d eficit/ h yp er ctivity d isord er (ADH D) is ost likely?
Que s tions : 56–66 177

(A) Acu te lcohol intoxic tion incre singly te rfu l, irrit ble, restless, s w ell
(B) H yperc lce i s h ving d ifficu lty p ying bills nd b l nc-
(C) H yp othyroid is ing her checkbook. She h s occ sion l “hot
fl shes.” On ex in tion, she continuou sly
(D) Pheochro ocyto
ch nges position in her se t. She h s d ifficulty
(E) Posterior circu l tion stroke w ith si p le rith etic nd short-ter e -
ory. H er d eep tend on reflexes re brisk sy -
D IRECTION S (Questions 61 through 66): For each etric lly throu ghou t. She h s fine resting
patient vignette, select the one lettered option tre or of her h nd s nd d ifficu lty rising fro
that is most closely associated w ith it. Each let- se ted p osition.
tered option may be used once, multiple times, or
not at all.
64. A 33-ye r-old w o n p resents w ith 1-ye r
(A) Conversion (fu nction l neu rologic l
history of tension he d ches, recently w ors-
sy pto ) d isord er
ening nd now ssoci ted w ith blu rry vision.
(B) H ep tic enceph lop thy So f r, n extensive neu rologic ev lu tion h s
(C) H u n i u nod eficiency viru s (H IV) been u nreve ling. She volu nteers th t she h s
(D) H yp erthyroid is been sick ost of her life, beginning t ge 16
(E) H yp oglyce i w hen she h d su rgery for p resu ed end o-
(F) Pheochro ocyto etriosis, w hich h s left her u n ble to h ve
sexu l rel tions. Since the su rgery, she h s
(G) So tic sy p to d isord er
h d inter ittent bd o in l cr ping, blo t-
(H ) Syste ic lu p u s erythe tosu s (SLE) ing, nd d i rrhe . Review of her ch rt show s
th t 3 onths go, she s w rheu tologist
61. A 55-ye r-old n is d itted to the hosp it l. for knee, b ck, nd eye p in.
H is w ife reports th t he’s ch nged over the
l st ye r, h s beco e very forgetfu l, nd
h s period s w hen he beco es very u p set. 65. A 30-ye r-old w o n is brou ght to the hos-
On ex in tion, he h s signific nt e ory p it l by her fi ncé. H e s ys th t over the l st
i p ir ent, sterixis, p l r erythe , nd 2 w eeks, she h s been “ tot lly d ifferent p er-
l rge ecchy otic re on his right sc p u l . son.” She h s not been e ting or sleeping, nd
H e w s initi lly cooper tive, bu t now is very h s been irrit ble nd ngry. E rlier in the
git ted nd d e nd s to le ve. onth, she co pl ined of he d ches, p in in
her h nd s nd feet, nd fever. On ex in -
tion, she looks tired ; she is fully oriented bu t
62. A 52-ye r-old n w ith history of IV d ru g h s d ifficu lty rel ting the events of the l st
u se is brou ght to the e ergency d ep rt ent onth. Occ sion lly, she see s to beco e
by his soci l w orker fro ho eless shelter. confused .
She h s know n the p tient for 10 ye rs bu t h s
seen d r stic ch nge in hi over the l st ye r.
While previou sly jovi l nd inter ctive, he is 66. A 42-ye r-old w o n in the e ergency d ep rt-
now d iseng ged nd subd u ed . H e h s been ent pp e rs confu sed nd w ill not llow
incre singly forgetfu l, tod y h ving d ifficu lty blood to be d r w n. She is not oriented to ti e
u sing his e ting u tensils. H e h s lost t le st or pl ce. She t kes ed ic tions for “ cond i-
40 lb in the l st 6 onths nd co pl ined tion,” bu t c nnot el bor te fu rther. The only
bou t feeling w e k nd losing his b l nce. H e vit l signs th t w ere t ken show he rt r te
scores 20/ 30 on the MMSE. of 140 be ts/ in nd blood p ressure of 172/
98 H g. Physic l ex in tion is re rk-
ble for fine tre or of her h nd s bil ter-
63. A 33-ye r-old w o n is brou ght to the e er- lly, nd d i phoresis. Five inu tes l ter, the
gency d ep rt ent by her hu sb nd , w ho tells p tient h s seizu re.
you th t over the l st few onths she h s been
178 6: Diffe re ntia l Dia g nos is a nd Ma na g e me nt

DIRECTIONS (Questions 67 through 72): For each 70. An 86-ye r-old fe le is brou gh t to th e ED
patient vignette, select the one lettered option that is fter the nu rsing ho e st ff noticed su d d en
most closely associated w ith it. Each lettered option ch nge in h er beh vior over the l st sever l
may be used once, multiple times, or not at all. d ys. On history nd p hysic l ex in tion, it
(A) Deliriu is d ifficu lt to elicit infor tion s the p tient
(B) MDD (p seu d od e enti ) keep s f lling sleep . H er vit l signs show
te p er tu re of 101.2°F n d he rt r te of
(C) M jor neu rocognitive d isord er d u e to
101 be ts/ in. Urin lysis is p ositive for
Alzhei er d ise se
ketones.
(D) M jor neu rocognitive d isord er d u e to
frontote por l lob r d egener tion
(Pick d ise se) 71. A 59-ye r-old fe le is brou ght in by her son
w ith w ho she lives. H e st tes th t over the
(E) M jor neu rocognitive d isord er d u e
p st cou p le of onths his other h s been
to H u ntington d ise se
isol tive, no longer sp end s her ti e re d -
(F) M jor neu rocognitive d isord er d ue to
ing w hich she u sed to enjoy. She w ill re in
Lew y bod y d ise se
w ke l te t night nd e rly in the orning,
(G) M jor neu rocognitive d isord er d u e nd her p p etite h s d ecre sed . H e is lso con-
to Prion (Creutzfeld t–J cob) d ise se cerned bec use she w ill so eti es re e ber
(H ) M jor v scu l r neu rocognitive d isord er inor d et ils fro the d y before, bu t other
ti es w ill not rec ll w h t she h d for bre k-
67. A 71-ye r-old fe le is brou ght to the ER by f st. Ment l st tu s ex in tion is signific nt
her hu sb nd . Over the p st few ye rs, he h s for incre sed sp eech l tency, d ep ressed ood
noticed th t she h s been h ving e ory nd ffect, nd trou ble rec lling 2/ 3 ite s
p roble s. She often forgets w here she p u t fter sever l inu tes.
her keys or pu rse. H e no longer lets her d rive
bec u se she gets lost e sily. On MMSE, she h s
72. A 39-ye r-old le w ith no p reviou s p sychi-
p rticul r p roble s w ith the d y of the w eek,
tric history p resents w ith co p l ints of feel-
d te, three-ite rec ll, nd n ing pencil.
ing “very d ep ressed l tely.” H is f ily notes
th t he h s been u ch ore irrit ble recently,
68. A 68-ye r-old le w ith no p sychi tric history often yellin g w ith little p rovoc tion nd
is brou ght by his d u ghter bec u se, d esp ite see ingly t lking to hi self. On MSE, he is
u ltiple re ssu r nces, he continu es to insist noted to h ve qu ick, su d d en, bu t involu nt ry,
so eone else is in the house. The p tient st tes jerking ove ents of his r s.
he sees s ll n every d y ju st insid e of
the d oor. H is d ughter h s lso noticed th t
D IRECTION S (Questions 73 through 98): For each
he h s proble s w ith his e ory. On ex i-
of the multiple-choice questions in this section,
n tion, he h s shu ffling g it nd blu nted
select the lettered answ er that is the one best
ffect, w hich the d ughter st tes beg n t the
response in each case.
s e ti e s the visu l h llu cin tions.
73. A 56-ye r-old n p resents to you r office t
69. A 55-ye r-old le is brou ght by his w ife the requ est of his w ife, w ho s ys th t he d rinks
bec u se she h s noticed ch nges in his beh v- too uch. Wh t w ould be the ost i port nt
ior. L tely, his food p references h ve ch nged str tegy in ev lu ting this p tient for lcohol
nd he is now cu rsing lou d ly in pu blic. On u se d isord er?
ex in tion, he is e sily git ted . N eu rologic
ex in tion d e onstr tes p ositive B binski (A) Ascert in how often he d rinks.
resp onse nd snou t reflex. An MRI show s (B) Ask hi how frequ ently he gets d ru nk.
trophy in the front l nd te p or l lobes. (C) Ask hi w h t his f ily nd friend s
s y bou t his d rinking.
Que s tions : 67–78 179

(D) Perfor co plete l bor tory Questions 77 and 78


investig tion.
A 55-ye r-old le w ith history of lcohol u se
(E) Qu ntify the ver ge ou nt he d rinks.
d isord er presents to you r office bec use he w ould
like to stop d rinking lcohol. H e believes he d rinks
74. A 42-ye r-old d ivorced le p resents to you r
bec u se he is d ep ressed nd w ishes to lso get tre t-
office st ting th t for the p st 7 onths he
ent for his d epression. After fu rther history is
h s been w orrying bout fin nci l nd w ork-
obt ined , it is reco end ed th t the first step is to
rel ted p roble s. H e st tes th t ore often
bst in fro ll lcohol s it c n c u se d ep ression.
th n not, he is w orrying bou t these issues
H e grees to this pl n.
throu ghou t the d y. H e h s d ifficu lt ti e
st ying sleep, he feels tired ll d y long,
77. Wh t is the likelihood of his re ining
nd he c n no longer concentr te d equ tely
d ep ressed if he is ble to refr in fro u sing
t w ork d u e to the w orrying. H e lso tells
lcohol for 1 onth?
you th t he feels very tense in his shou ld er
nd b ck. Which of the follow ing is the ost (A) 5%
p propri te d i gnosis? (B) 15%
(A) Depend ent person lity d isord er (C) 25%
(B) Gener lized nxiety d isord er (GAD) (D) 33%
(C) M jor d ep ressive d isord er (MDD) (E) 50%
(D) P nic d isord er
78. The bove p tient retu rns fter 6 w eeks of
(E) Soci l nxiety d isord er
int ining sobriety fro lcohol. H e tells
you th t he h s continu ed to feel d ep ressed .
Questions 75 and 76
H e h s p roble s sleep ing t night, feels tired
You re interview ing 54-ye r-old rried w o n throughou t the d y, h s poor pp etite, nd he
w ho h s been u rged to “see shrink” by her f - no longer d erives ple su re fro pl ying w ith
ily. She d escribes sy p to s of feeling ineffectu l, his d ogs? Which of the follow ing ed ic tions
believing th t the w orld is lw ys hostile to her, nd w ou ld be the ost p p rop ri te to begin t this
know ing th t things w ill never ch nge. ti e?
(A) A itrip tyline (El vil)
75. This tri d of sy p to s is ost ssoci ted
(B) Bu sp irone (Bu Sp r)
w ith w hich of the follow ing d isord ers?
(C) Lithiu (Esk lith)
(A) Dep ressive d isord ers (D) Lor zep (Ativ n)
(B) Dissoci tive d isord er (E) Sertr line (Zoloft)
(C) GAD
(D) P nic d isord er Questions 79 and 80
(E) Schizop hreni
A 52-ye r-old le w ith longst nd ing history of
lcohol use d isord er is brought into the ED intoxi-
76. Which of the follow ing tre t ents w ou ld
c ted w ith lcohol. B sic l bs re d r w n nd he is
ost likely t rget these sy p to s?
found to be d ehyd r ted , so he is given IV fluid s
(A) Beh vior l ther p y long w ith glu cose. Abou t n hou r l ter, the physi-
(B) Cognitive ther p y ci n reev lu tes the p tient nd find s th t his speech
(C) Cou p les ther p y is ore slurred , he is very confused , nd he now h s
nyst g us.
(D) Interp erson l ther p y
(E) P r d oxic l ther p y
180 6: Diffe re ntia l Diag nos is a nd Ma na g e me nt

79. Which of the follow ing is the next ost (A) Ad ju st ent d isord er
p propri te step in the n ge ent of this (B) Bip ol r d isord er
p tient? (C) MDD
(A) Ad inister ntibiotics. (D) Persistent d ep ressive d isord er
(B) Ad inister benzod i zep ine. (E) Schizop hreni
(C) Ad inister hep rin.
(D) Ad inister thi ine. 83. A 45-ye r-old n is in the e ergency dep rt-
ent bec use of di betic foot ulcer. In g ther-
(E) Ad inister tissu e p l s inogen
ing history, the physici n le rns th t this n
ctiv tor.
lives lone nd works nights s security gu rd.
He s ys he h s no friends but th t this does not
80. After d in istr tion of th e correct ed ic -
bother hi . He h s never been hospit lized
tion the p tien t i p roves signific ntly. H is
or received ny psychi tric help. On MSE, his
f ily rrives th e n ext orn in g n d th ey
ffect is fl t. Although he nswers questions
tell you th t he h s recen tly been st rting
nd see s to trust the judg ent of the doctors,
to ke u p f cts bou t h is life. You ord er
he h s little interest in the interview. He exhibits
n MRI of his br in nd d iscover lesion s in
no signs or sy pto s of psychosis or depres-
h is ill ry bod ies. Wh ich of th e fol-
sion. Wh t is the ost likely di gnosis?
low in g d i gn oses w ou ld best exp l in th ese
sy p to s? (A) MDD
(A) Alcohol-ind u ced jor neu rocognitive (B) P r noid p erson lity d isord er
d isord er (Kors koff synd ro e) (C) Schizoid p erson lity d isord er
(B) Conversion d isord er (D) Schizop hreni
(C) F ctitiou s d isord er (E) Schizotyp l p erson lity d isord er
(D) M jor neu rocognitive d isord er d u e to
Alzhei er d ise se 84. The id entified p tient is 30-ye r-old sep -
r ted fe le brou ght into the e ergency roo
(E) Wernicke encep h lop thy
by her id entic l tw in sister. The p tient’s his-
tory is not ble for prior episod e of d epres-
Questions 81 and 82
sion 5 ye rs go su ccessfu lly tre ted w ith
A 27-ye r-old n co p l ins th t he h s felt “d ow n venl f xine. The p tient h s been st ying
in the d u ps” for onths nd is feeling gu ilty w ith her sister since her sep r tion 1 onth
bec u se he h s been h ving n extr rit l ff ir. go. For the p st 2 w eeks, she h s been p cing
In recent w eeks, he h s st rted to believe th t his rou nd the hou se, not sleep ing ore th n 2 to
w ife is p oisoning his food nd the rest of his f ily 3 hou rs per night. Desp ite her feeling “s d ”
is involved in n el bor te plot to d rive hi fro i ed i tely fter the sep r tion, the p tient
the hou se. now feels “w ond erfu l, like I c n cco p lish
nything!” In f ct, she h s been tte p ting to
81. Assu ing his thinking is d elu sion l, how re od el her sister ’s b throo , even thou gh
w ou ld his d elu sions be best ch r cterized ? she h s no tr ining or experience. H er sister
h s been extre ely concerned bou t her, bu t
(A) Biz rre she h s been u n ble to t lk to her bout it s,
(B) Ego-syntonic “I c n’t get w ord in ed gew ise, nd she
(C) Mood congru ent d oesn’t lw ys ke sense.” The p tient is
(D) Mood incongru ent only t king or l contr ceptives nd o epr -
(E) So tic zole for cid reflux. H er sister is concerned
th t she, herself, y eventu lly d evelop this
82. Which of the follow ing d i gnoses w ou ld ost illness. Wh t is her p p roxi te risk of d evel-
likely be resp onsible for the bove d elu sions? oping this d ise se?
Que s tions : 79–89 181

(A) 0% to 10% ed ic tions w ou ld be the best choice for this


(B) 10% to 20% p tient?
(C) 20% to 30% (A) Alp r zol (X n x)
(D) 50% to 70% (B) Cod eine (Brontex)
(E) 80% to 90% (C) Dip henhyd r ine (Ben d ryl)
(D) Qu eti p ine (Seroqu el)
85. A 33-ye r-old w o n w ith history of
(E) Zolp id e (A bien)
schizop h reni tells you she h s been h e ring
voices for the p st 5 ye rs. Sh e h s b se-
88. An 8-ye r-old girl p resents to her p ed i tri-
line u d itory h llu cin tion th t she c nnot
ci n w ith her p rents second ry to incre sed
u nd erst n d ost of the ti e. Du ring p eri-
p roble s t school. She recently entered the
od s w here the voices beco e w orse, she
third gr d e nd h s been h ving d ifficu lties
typ ic lly he rs co nd u d itory h llu ci-
w ith beh vior in cl ss. At ho e, her p r-
n tions. Fro s fety st n d p oint, w hich of
ents d escribe her s ctive nd lw ys ov-
the follow ing is the ost i p ort nt infor-
ing fro one ctivity to the next. At school,
tion to obt in reg rd ing her u d itory
the child is u n ble to sit still in her ch ir nd
h llu cin tions?
frequ ently interru p ts the lessons. On ex i-
(A) H ow long she h s been he ring voices. n tion, you notice th t she is fid gety nd
(B) H ow lou d the voices re. const ntly p l ying w ith her h nd s. When
(C) Wh t the voices re s ying. she tte p ts to co p lete t sk she is e sily
d istr cted . Which of the follow ing w ou ld be
(D) Whether she recognizes the voices.
the ost effective p h r cother p y for this
(E) Whether the voices co e fro insid e or p tient?
ou tsid e her he d .
(A) Clonid ine
86. A 38-ye r-old n co p l ins th t for the (B) I ip r ine
p st 2 ye rs he h s requ ired sever l n p s (C) Methylp henid te
over the cou rse of the d y; he find s the n ps (D) Risp erid one
qu ite refreshing, but sees his d octor bec use
(E) Sertr line
l tely, s he is w king u p , he feels o en-
t rily “p r lyzed .” H e is cu rrently not on ny
89. A 30-ye r-old le co es to n urgent c re
ed ic tions. H e d enies the u se of ny illicit
f cility 8 hou rs fter the onset of u scle ches,
su bst nces. Which of the follow ing is the ost
ru nny nose, runny eyes, d ifficu lty sleeping,
likely d i gnosis?
nd loose stools th t he ttribu tes to flu
(A) Circ d i n rhyth sleep –w ke d isord er, viru s. H e lives in d ifferent city nd is visit-
d v nced sleep -ph se type ing for conference. H e rep orts th t he forgot
(B) Circ d i n rhyth sleep –w ke d isord er, his ed ic tions t ho e. H e t kes orphine
d el yed sleep -p h se typ e IR every 6 hou rs for chronic low b ck p in,
(C) H yp erso nolence d isord er d ip henhyd r ine occ sion lly to help w ith
sleep , venl f xine 150 g d ily for d ep res-
(D) N rcolepsy
sion nd Lisinopril for blood pressu re control.
(E) P r so ni On ex in tion, he is sw e ting, y w ning, h s
d il ted pu pils nd is ss ging his u scles.
87. A 45-ye r-old le w ith history of op i te H is blood p ressu re is 160/ 95 H g, pu lse
u se d isord er p resents to you r office co p l in- of 102 bp , nd he is febrile. When sked to
ing of p roble s w ith sleep . H e cl i s th t cont ct his pri ry c re p hysici n he beco es
he h s been sober fro heroin for the p st incre singly git ted , st ting they w ou ld not
10 ye rs. H e sks to be st rted on ed ic - contribute ny infor tion to this visit. L b
tion for his inso ni . Which of the follow ing v lues re ll w ith in nor l li its. Which
182 6: Diffe re ntia l Diag nos is a nd Ma na g e me nt

of the follow ing ost likely exp l ins his etfor in, u ltivit in, n OTC herb l su pp le-
sy pto tology? ent, tr d ol for chronic b ck p in, nd recent
ntibiotic prescription for linezolid . H is vit l signs
(A) Anticholinergic toxicity
re: BP of 160/ 92, he rt r te of 110 bp , nd te -
(B) H ypertensive crisis p er tu re of 101.2°F. On ex in tion, he d i p horetic
(C) Influ enz nd h s clonu s in his bil ter l low er extre ities.
(D) Op i te w ithd r w l
(E) Serotonin synd ro e 92. Which of the follow ing is his ost likely
d i gnosis?
Questions 90 and 91
(A) Bipol r d isord er
A 28-ye r-old fe le w ith p st ed ic l history (B) H ypertensive crisis
of hypothyroid is nd syste ic lu pu s erythe - (C) Migr ine w ith u r
tosus (SLE) is hospit lized for lupu s fl re requ ir- (D) N eu rolep tic lign nt synd ro e (N MS)
ing IV steroid s. You re consu lted for the p tient
(E) Serotonin synd ro e
s she is co pl ining of ud itory h llu cin tions
nd h s been beco ing incre singly git ted . H er
93. Which of the follow ing is the best next step in
ed ic tions inclu d e IV ethylp red nisone, u lti-
the n ge ent of this p tient?
vit in, nd levothyroxine. On ex in tion, the
p tient’s ood is l bile nd she end orse u d itory (A) Continu e ll ho e ed ic tions.
h llucin tions. H er vit l signs re st ble. H er TSH is (B) Discontinu e linezolid , tr d ol,
0.7 IU/ L nd her T4 is 7.2 u g/ d L. p henelzine.
(C) Discontinu e etfor in.
90. Which of the follow ing is the ost likely
(D) Discontinu e over the cou nter herb l
d i gnosis?
sup ple ent.
(A) Bipol r d isord er d u e to hyperthyroid is (E) Incre se risp erid one.
(B) Brief psychotic d isord er
(C) MDD w ith p sychotic fe tu res 94. A 58-ye r-old le w ith no p st ed ic l his-
(D) Schizop hreni tory is brou ght to the d octor ’s office by friend .
(E) Steroid -ind u ced p sychotic d isord er The friend rep orts the p tient h s ccu u -
l ted too u ch “stu ff” in his one-story ho e
91. Which of the follow ing is the best next step in nd h s not been ble to le ve his ho e. The
the n ge ent of this p tient? p tient cl i s th t he y need these ite s t
so e point in the futu re nd refu ses to throw
(A) Decre se steroid d os ge nything w y. When sked if he h s ever
(B) Incre se levothyroxine tried to cle n out his hou se he gets d efensive,
(C) Initi te ECT fid gety nd st tes he cou ld “ bsolutely never
(D) Initi te lithiu throw nything ou t!” H is friend brings in p ic-
tu res of the p tient’s ho e show ing ound s
(E) Initi te sertr line
of ite s in every roo . The p tient h s been
u ne p loyed for 1 ye r, expl ining th t he
Questions 92 and 93
need s the ti e to t ke c re of his ho e. Which
A 38-ye r-old n w ith p st ed ic l history of of the follow ing is the ost likely d i gnosis?
chronic low er b ck p in nd hypertension presents (A) GAD
w ith friend to the e ergency roo co p l ining of
(B) H o rd ing d isord er
git tion nd he d che for 2 d ys. Upon fu rther
qu estioning, he d its to p st p sychi tric history (C) OCD
of MDD w ith psychotic fe tures. Soon fter p resen- (D) Obsessive-co pu lsive person lity
t tion he beco es confu sed nd nonresponsive. H is d isord er (OCPD)
ed ic tion list inclu d es p henelzine, risp erid one, (E) P nic d isord er
Que s tions : 90–100 183

95. A 31-ye r-old fe le is fou nd in D ll s, Tex s 98. A 35-ye r-old le is u rged to go to psy-
sitting in coffee shop for n entire d y. When chi trist by cow orker bec u se of frequent
sked to le ve the coffee shop u p on closing, nger ou tbu rsts. H e rep orts th t s long s
the p tient c nnot rec ll ny id entifying infor- he c n re e ber he h s h d “hot te p ter.”
tion or w here she is fro . The p olice re L tely, he h s been getting into to trou ble t
c lled , w ho escort her to n e ergency d ep rt- w ork d u e to u ltip le verb l confront tions
ent. When se rched , her ID c rd is fou nd w ith cow orkers, w hich occ sion lly esc l te
in her p u rse, d ocu enting th t the p tient is to the p tient throw ing v rious office objects
fro Michig n. Fu rther investig tion reve ls cross the roo . H e feels very gu ilty fter, but
her f ily rep orted her issing 2 w eeks go. he is u n ble to p revent his beh vior. H e h s no
Of note the p tient w s d op ted t you ng signific nt leg l proble s otherw ise. Which of
ge second ry to child bu se. Wh ich of the the follow ing is the ost likely d i gnosis?
follow ing is the ost likely d i gnosis?
(A) Antisoci l p erson lity d isord er
(A) Bord erline p erson lity d isord er (B) Bip ol r II d isord er
(B) Dep erson liz tion d isord er (C) Bord erline p erson lity d isord er
(C) Dissoci tive nesi (D) Cond u ct d isord er
(D) Dissoci tive id entity d isord er (E) Inter ittent exp losive d isord er
(E) M jor neu rocognitive d isord er d u e to
Alzhei er d ise se D IRECTION S (Questions 99 through 108): For
each of the follow ing patients, choose the most
Questions 96 and 97 appropriate diagnosis. Each lettered option may
be used once, multiple times, or not at all.
A 26-ye r-old fe le p resents to her pri ry c re
physici n w ith d ifficu lty sleep ing. She co pl ins (A) Antisoci l p erson lity d isord er
of const ntly fid geting in bed w ith const nt urge
(B) Avoid nt p erson lity d isord er
to ove her legs. She gets u p nd w lks rou nd in
ord er to help relieve it. This occu rs ost nights of (C) Bord erline p erson lity d isord er
the w eek nd h s gre tly interfered w ith her bil- (D) Dep end ent p erson lity d isord er
ity to sleep . Physic l ex in tion is neg tive for ny (E) H istrionic p erson lity d isord er
bnor lities. H er l b w ork show s low MCV, low (F) N rcissistic p erson lity d isord er
ferritin, nd he oglobin of 9.0 g/ d L. (G) Obsessive-co p u lsive p erson lity
d isord er
96. Which of the follow ing is the ost likely
(H ) P r noid p erson lity d isord er
d i gnosis for this p tient?
(I) Schizoid person lity d isord er
(A) GAD (J) Schizotyp l person lity d isord er
(B) Inso ni d isord er
(C) N on-REM sleep rou s l d isord er 99. A 45-ye r-old w o n recently is rele sed fro
(D) Obstru ctive sleep p ne p rison for r ed robbery. She h s nu erous
(E) Restless legs synd ro e j il sentences for ste ling, bribery, nd rson.
She end orses th t she s w p sychi trist s
97. Which of the follow ing is the ost p p rop ri- child for “beh vior l proble s,” getting into
te step in the tre t ent of this p tient? trou ble t school nd t ho e. Upon further
questioning, she rec lls p eriod of ti e
(A) Initi te hyd rocod one. torturing her f ily’s p et c t s child .
(B) Initi te iron rep l ce ent.
(C) Initi te lor zep . 100. A 57-ye r-old n presents to his pri ry
(D) Initi te el tonin. p hysici n w ith co p l ints of bd o in l p in.
(E) Initi te risp erid one. H e is cco p nied by his other, w ith w ho
184 6: Diffe re ntia l Diag nos is a nd Ma na g e me nt

he lives. The p tient ppe rs to d efer to his see the Chief of the d ep rt ent, nd how he
other to nsw er ny of the qu estions d u r- c nnot be “bothered ” by low -level e ployees.
ing the interview. The p tient t lks bou t how
he c lls his other throughou t the w orkd y 105. A 46-ye r-old n is reco end ed to see
to sk her trivi l qu estions nd so she c n help e p loyee he lth s he continu ou sly believes
hi ke ny, everyd y d ecisions. H e h s his colle gu es t w ork re ou t to s bot ge
recently been ore nxiou s s he fe rs th t she his e p loy ent. As resu lt, he h s w ritten
is “old er” nd y d ie soon, “w hich w ou ld thre tening e ils to his peers nd su pervi-
le ve e lone, help less, nd u n ble to cop e.” sors. When confronted w ith the f ct th t he
h s been invited to t ke p rt in soci l ctivi-
101. A 30-ye r-old n w orks in lighthou se d u r- ties w ith others, he st tes th t he is being
ing the overnight shift by hi self. H e h s “tricked into co p l cency” nd pl ns to sue
no friend s, prefers to be lone in his sp re the org niz tion.
ti e, nd d oes not d esire to ke person l
or inti te rel tionships. H e presents d ue to 106. A 21-ye r-old w o n w ho is in her third ye r
incre sed stress now th t he h s cow orker t of college p resents to ther p ist u p set th t
his job. she is u n ble to ke friend s. She exp resses
th t she w ou ld like to join school clu b to
102. A 34-ye r-old w o n h s just gone throu gh eet new p eop le nd ke friend s bu t she is
her second d ivorce nd is cu rrently in too shy. She d oesn’t go ou t to p rties d esp ite
tu u ltu ous rel tionship w ith n she et being sked , bu t “they w ou ld n’t like e
1 onth go. H er ex in tion reve ls sc rs nyw y.”
on her w rist, w hich she expl ins re fro cut-
ting herself “w hen I get ngry or up set.” She 107. A 34-ye r-old w o n is new ly d i gnosed w ith
often feels e pty insid e bu t d its to intense hyp ertension. Upon interview, she d its
ch nges of ood , qu ickly beco ing d espon- th t she st ys l te t her job on regul r b sis
d ent or ngry. She reports it is ll her f u lt she bec use she spend s n excessive ount of
feels this w y. ti e on e ch project, ensuring everything is
in ex ct ord er nd properly for tted . She
103. A 45-ye r-old w o n presents to her p ri ry triple-checks not only her ow n but ll of her
c re ppoint ent p rovoc tively d ressed for colle gues’ w ork. She d oesn’t believe th t oth-
rou tine ex in tion. She exu ber ntly exp l ins ers re “c rrying their lo d ” but believes she is
her cu rrent rel tionship w ith so eone she h s the only one w ho excels t her job. She is d is-
et only online nd not in person. She goes ppointed , how ever, bec use her fellow col-
into gre t d et il reg rd ing his erits, cl i ing le gues re ll ble to le ve t the ppropri te
th t “he is the one.” H ow ever, t the end of the ti e.
interview she sks the p hysici n if he is single.
108. A 32-ye r-old w o n p resents to her OB/
104. A 32-ye r-old n p resents to the e ergency GYN for routine c re. She is noted to be
d ep rt ent fter suffering l cer tion to his d ressed w ith ulticolored sc rves, b ngles,
left ind ex finger. When ed ic l stu d ent nd l rge nu ber of cryst l jew elry. She
tte p ts to ev lu te hi he rebu kes hi , st tes she is p l re d er s profession,
excl i ing “I only t lk to ttend ing p hysi- believing she h s the tools to u nlock p er-
ci ns.” H e is noted to be t lking lou d ly on his son’s fu tu re. She c e to her p point ent
telephone bout how su ccessful he h s been fter re d ing her strologic l forec st, bu t w s
in the fin nci l rket, how he is su re he w ill concerned bec u se of the “color of y ur .”
CHAPTER 7

Prac tic e Te s t 1
Que s tions

D IRECTION S (Questions 1 through 107): For each Questions 3 and 4


of the multiple-choice questions in this section,
select the lettered answ er that is the one best A 44-year-old w om an p resents to her p rim ary care
response in each case. d octor w ith m u ltip le com p laints, inclu d ing w eak-
ness in her low er extrem ities, bloating, head aches,
interm ittent loss of ap p etite, and back p ain. A care-
Questions 1 and 2 fu l review of sym p tom s reveals m any other vagu e
sym p tom s. H er com p laints d ate back to ad oles-
A 29-year-old m an w ith a history of bip olar d isor-
cence and she has seen m any d octors. Thorou gh
d er presents to the psychiatric em ergency d ep art-
w orku p s, inclu d ing an exp loratory lap arotom y,
m ent saying that he is the king of “Pum bar” and
have failed to u ncover any clear, anatom ic, or
need s everyone’s allegiance for the u p com ing w ar
p hysiologic cau se.
w ith the Martians. In the p ast few d ays, he has slep t
a total of 3 hou rs bu t says that he is not tired . H e has
3. Which of the follow ing is the best ap proach to
spent all of his m oney soliciting phone sex. N ow , he
this patient?
is agitated , d em and ing, and threatening.
(A) Tell her any physical w orku p is
1. Which of the follow ing is the best treatm ent u nnecessary.
for this p atient in the acu te setting? (B) Tell her to com e back in 1 m onth and , if
(A) Carbam azep ine (Tegretol) the sym ptom s are still present, you w ill
initiate a p hysical w orku p .
(B) Divalp roex sod iu m (Dep akote)
(C) Tactfu lly ask her w hy she is inventing
(C) H alop erid ol (H ald ol)
sym ptom s.
(D) H yd roxyzine (Atarax)
(D) Assess her for other p sychiatric
(E) Lithiu m illnesses.
(E) Initiate a p hysical w orku p and arrange
2. After treating the p atient acu tely, a m ed ica-
for follow -u p in a year’s tim e.
tion is requ ired for ongoing treatm ent of his
bip olar d isord er. You find that he has a history
4. Which of the follow ing p ersonality d isord ers
of agranulocytosis. Which of the follow ing
w ou ld m ost likely be com orbid in the above
m ed ications w ou ld be the best choice for his
p atient?
treatm ent?
(A) Avoid ant
(A) Antip sychotic m ed ication
(B) Bord erline
(B) Carbam azep ine
(C) Obsessive-com p u lsive
(C) Divalp roex sod iu m
(D) Schizoid
(D) Lithiu m
(E) Schizotyp al
(E) Lorazep am (Ativan)

205
206 7: Pra c tic e Te s t 1

Questions 5 and 6 7. Which of the follow ing sym p tom s is consi-


d ered a “negative sym ptom ” in this patient’s
A 50-year-old sin gle m an h as in creasin gly becom e p resentation?
a con cern to his n eigh bors. H e w orks at a com ic-
book store, d resses in od d , ou td ated cloth es, an d (A) Au d itory hallu cinations
d isp lays p oor eye con tact. Althou gh h e tend s (B) Delu sions
to keep to him self, h e h as told neighborhood chil- (C) Flat affect
d ren th at th ere are w itch es w h o live d ow n th e (D) Loose associations
street.
(E) Paranoia
5. Which of the follow ing is the m ost likely 8. The p atient is started on m ed ication and
d iagnosis? m any of her sym p tom s im p rove. She begins
(A) Bipolar d isord er a new job and d oes w ell. One year later, she
(B) Bord erline p ersonality d isord er is brou ght to her d octor florid ly p sychotic,
actively hearing voices, and extrem ely p ara-
(C) Schizoid p ersonality d isord er
noid . She believes that her boss is trying to kill
(D) Schizop hrenia her. She has an u p p er respiratory viral illness
(E) Schizotyp al p ersonality d isord er that she believes to be the w ork of a foreign
governm ent. She d iscontinu ed her m ed ica-
6. The patient’s brother brings him to a d octor tion 4 w eeks ago becau se she felt too sed ated .
because, since the d eath of their mother, the In the p ast year, her cigarette sm oking habit
patient’s paranoia has cau sed him to qu estion has d ecreased to one p ack p er d ay. What is the
his neighbors’ activities. H e has since m oved m ost likely cau se of her exacerbation?
into a hotel that he could not afford in ord er
to get aw ay from the “spies” living next d oor. (A) A reaction to the viral illness
Which of the follow ing is the most appropriate (B) Decreased cigarette sm oking
intervention? (C) Med ication noncom p liance
(A) Antip sychotic (D) Stress from w ork
(B) Benzod iazep ine (E) Treatm ent refractory illness
(C) N o treatm ent
Questions 9 and 10
(D) Psychod ynam ic p sychotherapy
(E) Selective serotonin reu p take inhibitor A 44-year-old man complains to his doctor that he is
(SSRI) alw ays tired and is having d ifficulty getting out of
bed in the morning. Upon questioning, he reveals he
Questions 7 and 8 has three or four d rinks each night and perhaps more
on the w eekend s, but d enies he has any problem w ith
A 25-year-old fem ale college grad u ate is brou ght alcohol. A d iagnosis of alcohol use d isord er is mad e
to her d octor by her m other. Described as “od d ” and the patient comes to your office in acute alcohol
since she lost her job a year ago, the p atient has w ithd raw al. He subsequently has a w ithd raw al sei-
com p lained of hearing voices and believes that zure and is ad mitted to the intensive care unit.
her bod y is receiving Wi-Fi com m u nications for a
cou nterterrorist op eration. H er m other notes she 9. Which of the follow ing laboratory find ings
has been isolating herself in her room . She is alert w ou ld be m ost likely fou nd in this p atient?
and oriented bu t su sp iciou s and gu ard ed on exam -
ination. H er affect is flat and her sp eech reveals (A) Decreased p rothrom bin tim e
loose associations. A com p lete m ed ical w orku p is (B) Elevated or d ep ressed liver enzym es
negative. (C) H igh blood -alcohol level
(D) H yp erm agnesem ia
(E) Throm bocytosis
Que s tions : 5–14 207

10. Which of the follow ing m ed ications w ou ld be to w orsening d epressive sym ptom s and acute sui-
m ost im portant to ad m inister? cid ality. She has had m u ltiple trials of m ed ications
w ithou t significant im p rovem ent. A cou rse of elec-
(A) Diazepam
troconvulsive therap y (ECT) is d eterm ined to be the
(B) H alop erid ol next ap p ropriate step.
(C) Lorazep am
(D) Phenytoin 12. While u nd ergoing the ECT, the treatm ent
(E) Valp roic acid team w ishes to m onitor im provem ent in her
d epression. Which of the follow ing tests has
11. A 28-year-old woman is brought to the emer- the greatest reliability and valid ity for this
gency department for active suicidal ideation p u rp ose?
with a plan to overdose on acetaminophen. (A) Beck Dep ression Inventory
She has no history of a psychiatric illness but
(B) Draw -A-Person Test
endorses many criteria for major depressive
disorder (MDD), including poor sleep for the (C) H alstead -Reitan N eu rop sychological
past 2 weeks. She recently lost her job and Battery
fears that she may not be able to pay her rent. (D) Rorschach Test
Attempts to obtain collateral information have (E) Them atic Ap p ercep tion Test (TAT)
been unsuccessful. You believe the patient
requires inpatient evaluation but her insurance 13. After one of her treatm ents, the p atient com -
company denies authorization for inpatient p laints of m em ory im p airm ent. Which of the
care, alternatively authorizing eight outpatient follow ing tests w ou ld be the m ost ap p rop riate
visits with a psychiatrist. You speak to the to assess her com plaint?
weekend on-call physician-reviewer and report
(A) Beck Dep ression Inventory
that the patient remains unsafe and wishes to
be discharged. Upon learning that the patient (B) Brow n-Peterson Task
does not have a history of psychiatric illness, the (C) Bu lim ia Test—Revised
reviewer fails to authorize inpatient care, despite (D) Eating Disord er Inventory 2 (EDI-2)
your assessment. Which of the following is the (E) State-Trait Anxiety Inventory
most appropriate intervention?
(A) Ad m inister an antipsychotic m ed ication 14. A 67-year-old w oman w ith a history of
and reevalu ate the p atient in 1 hou r. d epression presents to your office for evaluation.
Her symptoms of poor appetite, insomnia,
(B) Ad m it the p atient on an em ergency
low energy, and feelings of hopelessness have
certificate to an inpatient facility.
w orsened recently. She has been on several
(C) Begin antid ep ressant therap y and d ifferent serotonin-specific reuptake inhibitors
arrange for outp atient follow -u p the (SSRIs), w hich you learn have not resulted in
next d ay. complete remission of her symptoms. Which of
(D) Explain to the p atient that her the follow ing med ications w ould be the most
insu rance com p any d id not au thorize appropriate to prescribe?
hosp italization and d ischarge her w ith
follow -u p care. (A) Citalopram
(E) Prescribe a m ed ication to help her (B) Flu voxam ine
sleep, arrange for follow -up care, and (C) Paroxetine
d ischarge her from the hospital. (D) Sertraline
(E) Venlafaxine
Questions 12 and 13

A 34-year-old w om an suffering from severe m ajor


d epressive d isord er is ad m itted to the hospital d u e
208 7: Pra c tic e Te s t 1

Questions 15 and 16 (D) H yp ochond riasis


(E) Panic d isord er
A 29-year-old w om an p resents to the em ergency
d ep artm ent w ith her 3-year-old child reporting that
18. Which of the follow ing m ed ications is the m ost
the child suffered a seizu re w hile at hom e. H ospi-
efficaciou s in the treatm ent of this illness?
tal record s verify that this is the third em ergency
d ep artm ent visit in as m any w eeks for the sam e (A) Clonid ine (Catap res)
p resentation. N eu rologic w orku p for seizu re d is- (B) H alop erid ol
ord er w as negative. Initiation of an anticonvu lsant (C) Lithiu m
has been ineffective. The m other becom es very fru s-
(D) Prop ranolol
trated , d em and ing that her son be ad m itted to the
hosp ital for further testing. (E) Sertraline (Zoloft)

15. Based on the above p resentation, w hich of 19. A w om an being treated for m ajor d ep ression
the follow ing d iagnoses is m ost likely in the is brou ght to the em ergency d ep artm ent after
child ? being fou nd u nconsciou s by a neighbor. The
neighbor states that over the p ast few d ays
(A) Conversion d isord er the w om an had been com plaining of severe
(B) Factitiou s d isord er head aches. She also m entions that the w om an
(C) N o d iagnosis often enjoys red w ine. The w om an’s blood
(D) Seizu re d isord er pressu re is record ed as 220/ 110 m m H g.
Which of the follow ing shou ld be ad m inistered
(E) Sep aration anxiety d isord er
im m ed iately?
16. The m other is ad d itionally m ost likely to su f- (A) Alp ha-blocker
fer from w hich of the follow ing? (B) Beta-blocker
(A) Bipolar d isord er (C) Brom ocrip tine (Parlod el)
(B) Ep ilep sy (D) Calciu m channel blocker
(C) Major d ep ressive d isord er (E) Dantrolene sod iu m (Dantriu m )
(D) Posttrau m atic stress d isord er (PTSD)
Questions 20 and 21
(E) Schizop hrenia
A 38-year-old m an w ith schizop hrenia has had
Questions 17 and 18 nu m erou s hosp italizations and m any trials w ith
variou s antip sychotic m ed ications. H e continues to
A 26-year-old m an is being evalu ated in the em er-
be sym p tom atic, w ith d erogatory au d itory hallu ci-
gency d ep artm ent for su d d en onset of chest p res-
nations, p aranoia regard ing his neighbors, and poor
sure and d yspnea. This is his third em ergency
self-care. H e d enies su icid al or hom icid al id eation.
d ep artm ent visit for sim ilar sym p tom s for w hich he
H e has recently been started on clozap ine (Clozaril),
rep orts “I feel like I’m going to d ie.” An electrocar-
w ith significant im p rovem ent in his cond ition. H e
d iogram (ECG) and stress test w ere norm al. Urine
d oes not d rink alcohol or u se illicit d rugs, and he
toxicology w as negative. The p atient d enies risk
d enies ad d itional m ed ical history.
factors for heart d isease and d oes not have a fam ily
history of heart d isease.
20. Which of the follow ing ad verse effects is
associated w ith this m ed ication?
17. Which of the follow ing is the m ost likely
d iagnosis? (A) Brad ycard ia
(A) Acu te m yocard ial infarction (B) Galactorrhea
(B) Acu te stress d isord er (C) H yp ertension
(C) Deliriu m (D) Seizu res
(E) Weight loss
Que s tions : 15–27 209

21. Which of the follow ing blood tests w ill requ ire (C) Paranoid p ersonality d isord er
frequ ent, regu lar m onitoring for this p atient? (D) Schizoid p ersonality d isord er
(A) Calciu m level (E) Schizotyp al p ersonality d isord er
(B) Com p lete blood cou nt w ith d ifferential
25. A 28-year-old sep arated w om an is referred
(C) Electrolytes
from her p rim ary care d octor for evalu ation
(D) Thyroid fu nction tests for d ep ression. While she ad m its to occasional
(E) Urinalysis p eriod s of d ysp horia, she claim s to “alw ays
feel em p ty insid e.” Up on fu rther qu estion-
Questions 22 and 23 ing she d em onstrates a p ervasive p attern
of u nstable relationship s, p oor self-im age,
You ad m it an 83-year-old w id ow ed fem ale for
im p u lsiveness, and irritability. Which of the
fu rther evalu ation as she is no longer able to care
follow ing d iagnoses is the m ost ap p rop riate
for herself at hom e. She has lost 30 lb in the p ast
to consid er?
year, has p oor hygiene, and ad m its to increasing
forgetfulness. (A) Antisocial p ersonality d isord er
(B) Bord erline p ersonality d isord er
22. Which of the follow ing tests w ou ld best help (C) Dep end ent p ersonality d isord er
to m ake the correct d iagnosis?
(D) H istrionic p ersonality d isord er
(A) Blessed Rating Scale (E) Schizoid p ersonality d isord er
(B) Folstein Mini-Mental State Exam ination
(MMSE) Questions 26 and 27
(C) Geriatric Rating Scale
A 28-year-old single m an w ith a 10-year history of
(D) Glasgow Com a Scale schizophrenia has been taking his m ed ications reg-
(E) Mental Statu s Exam ination (MSE) u larly. H e now p resents w ith w orsening hallucina-
tions and prom inent thought d isorganization.
23. Which of the follow ing d isord ers w ou ld be
m ost im p ortant to ru le ou t as a cau se of her 26. Which of the follow ing neu rop sychological
clinical p resentation? tests w ou ld best d eterm ine his ability to
(A) Generalized anxiety d isord er organize and correctly p rocess inform ation?
(B) Major d ep ressive d isord er (A) Bend er Gestalt Test
(C) Obsessive-com p u lsive d isord er (B) Draw -A-Person Test
(D) Panic d isord er (C) Lu ria-N ebraska N eu rop sychological
(E) Posttrau m atic stress d isord er Battery
(D) Mini-Mental State Exam ination
24. A 35-year-old patient is encou raged to seek (E) Wisconsin Card Sorting Test (WCST)
“p rofessional help” by his cow orkers. H e
d enies pervasive d epression or anxiety, bu t 27. The above test m easu res fu nctioning at w hich
u pon interview he is od d ly d ressed , expresses of the follow ing lobes of the brain?
u nu sual beliefs and thinking, som e paranoia
regard ing his cow orkers’ m otivations, and (A) Cerebellar
has few close friend s. H e d enies d elu sions (B) Frontal
or hallu cinations, and there is no su icid al or (C) Occip ital
hom icid al id eation. Which of the follow ing (D) Parietal
d iagnoses is the m ost appropriate to consid er? (E) Tem p oral
(A) Avoid ant p ersonality d isord er
(B) N arcissistic p ersonality d isord er
210 7: Pra c tic e Te s t 1

Questions 28 and 29 31. The p atient retu rns 1 m onth later for a follow -
u p exam ination and rep orts th at he exp e-
A 30-year-old w om an w ithout prior p sychiatric his- rienced a gen eralized seizu re. Laboratory
tory is brought to the em ergency d epartm ent by the investigation reveals that the p henytoin level
p olice after being arrested for “breach of the p eace.” is 6.5 m g/ d L (norm al, 10–20 m g/ d L). Which
The w om an w as observed acting irrationally at a of the follow ing is the m ost ap p rop riate
local bu siness w here she d em and ed to sp eak w ith intervention at this tim e?
the presid ent of the com p any claim ing that she had
new id eas for prod u ct d evelopm ent. The patient (A) Ad d a benzod iazep ine.
rep orts that she has not slep t for d ays and that (B) Ad d p henobarbital.
her m ood is “fabulou s.” Urine hum an chorionic (C) Assess com p liance.
gonad otrop in is p ositive. Illicit su bstances w ere not (D) Discontinu e p henytoin and begin
d etected . d ivalproex sod ium .
(E) Increase the p henytoin d ose.
28. Which of the follow ing ad d itional find ings
w ou ld m ost likely be p resent in her history or
32. A 24-year-old w om an w ith a history of
m ental status exam ination (MSE)?
schizophrenia tells you that she w ou ld like
(A) Daytim e sleepiness to becom e p regnant. H er husband has no
(B) Dep ressed affect history of m ental illness. What d o you tell
(C) Racing thou ghts her is the chance of her offsp ring d evelop ing
schizophrenia?
(D) Visu al hallu cinations
(E) Weight loss (A) 1%
(B) 2%
29. Which of the follow ing m ed ical d isord ers can (C) 5%
also present w ith sim ilar sym ptom s? (D) 8%
(A) Cirrhosis (E) 12%
(B) Diabetes m ellitu s
(C) H yp erglycem ia 33. A 28-year-old m arried fem ale p atient is ad m it-
ted to the hospital w ith bizarre behavior and
(D) Rheu m atoid arthritis
d isorganized thinking. Consid eration is given
(E) Thyroid d isord er to a d iagnosis of schizophreniform d isor-
d er. To help w ith the d iagnosis, the patient is
Questions 30 and 31 ad m inistered a test that consists of view ing a
A 24-year-old m an w ith a history of seizu re d is- set of 10 inkblots sequ entially. The exam iner
ord er, alcohol, and cocaine u se d isord er has been scores the patient’s responses to the blots by
incarcerated for assau ltive behavior. The p atient is noting the content of the p ercep tion, the area
evaluated by a neurologist, who prescribes phenytoin of the blot that form s the basis of the response,
(Dilantin). and the asp ects of the area that are u sed to
form the resp onse.
30. Which of the follow ing sid e effects is m ost In w hich of the follow ing classes d oes this
likely associated w ith this m ed ication? projective test belong?

(A) Ebstein anom aly (A) Associations


(B) Gingival hyp erp lasia (B) Choice of ord ering
(C) H ep atic failu re (C) Com p letions
(D) H yp ertension (D) Constru ctions
(E) Leu kocytosis (E) Self-exp ression
Que s tions : 28–39 211

34. A m other and her 17-year-old son p resent to 36. An 89-year-old m arried w om an w ith no p rior
you r p sychiatric clinic seeking fam ily therap y. p sychiatric history bu t a history of m u ltiple
The m other rep orts that she and her son have m ed ical p roblem s is ad m itted for failu re to
a tum ultu ou s relationship and are constantly thrive. Upon history, the patient d oes not seem
argu ing w ith each other. The son ad m its that acclim ated to her su rrou nd ings. Which of the
abou t a year ago he d isclosed to his m other follow ing tests w ou ld best help to d eterm ine
that he is gay. H e feels that because of his her level of confu sion?
m other ’s strong religiou s beliefs, she has not
(A) Fargo Map Test
accep ted his sexu ality and this has been the
sou rce of tension in their relationship. The (B) Sp atial Orientation Mem ory Test
m other firm ly believes that her son is “ju st (C) Stroop Test
confu sed ” and need s cou nseling. She has (D) Tem p oral Orientation Test
heard of a particular therapy that can change (E) Wisconsin Card Sorting Test
the sexu al orientation of an ind ivid u al and
asks that you cond u ct this on her son. Which 37. A Malaysian m an is brou ght into cu stod y by
of the follow ing is the appropriate next step? the police after m u rd ering his friend . While
(A) Ask the son if he is w illing to p u rsu e he d oes not rem em ber com m itting the act, he
this treatm ent. d oes recall being insu lted by him at an earlier
tim e. Which of the follow ing cond itions is
(B) Engage the son in this therap y based off
m ost associated w ith this presentation?
on w hat you know .
(C) Inform the m other that su ch therap y is (A) Am ok
u nethical. (B) Dhat
(D) Learn m ore abou t the treatm ent first (C) Ganser synd rom e
and ask the fam ily to retu rn w hen you (D) Koro
are know led ge able enou gh to cond u ct (E) Latah
this.
(E) Refer the fam ily to a therap ist w ho Questions 38 and 39
specializes in such treatm ents.
A 26-year-old w oman is d iagnosed w ith schizophre-
35. An 18-year-old m an is referred to his college nia. The psychiatrist d ecid es to treat her symptom s
counseling center d ue to failing grad es. It is w ith a high-potency antipsychotic med ication. While
d eterm ined that he has stop ped attend ing her symptom s appear to improve on the med ication,
classes, preferring to rem ain in his d orm room . she d evelops acu te, painfu l muscle spasm s of her
H is room m ates claim that he stays u p “all jaw .
night,” w ith little ap parent sleep. The p atient
claim s that he has d iscovered som ething that 38. Which of the follow ing m ed ications shou ld be
w ill m ake him a m illionaire. On m ental sta- ad d ed ?
tu s exam ination, he app ears d iap horetic w ith (A) Benztrop ine (Cogentin)
p sychom otor agitation. H is speech is rap id (B) Cholinergic agonist
and his affect is eu phoric. H e d enies su icid al
(C) Clozap ine
or hom icid al id eation but d isp lays significant
grand iosity. H is insight is p oor. Which of the (D) Methylp henid ate (Ritalin)
follow ing is the m ost likely d iagnosis? (E) Risp erid one (Risp erd al)

(A) Cannabis intoxication 39. Despite compliance with the above regimen, the
(B) Cocaine (stim u lant) intoxication patient continues to suffer with extrapyrami-
(C) MDD w ith p sychotic featu res dal symptoms. As a result of this, she abruptly
(D) Schizoaffective d isord er stops taking her antipsychotic medication. It is
(E) Schizop hrenia decided to switch to another medication. Which
Que s tions : 40–49 213

45. A 28-year-old m ale m ed ical stu d ent is fou nd are no signs of trau m a or overd ose. A review
to have an enlarged testicle d u ring a rou tine of her chart reveals that the p atient w as w ell
p hysical exam ination. The stu d ent reports the night before and w ent to sleep w ithou t
that it has been grad ually enlarging for several incid ent. You d eterm ine that the p atient’s
m onths. The physician asks w hy he d id not u nresponsiveness is psychogenic. Which of
report these find ings earlier. “I’m su re it’s the follow ing find ings is m ost likely to be
nothing,” the stud ent replies. Which of the ap p arent on her exam ination?
follow ing typ es of resp onses d oes this m ost
(A) Abnorm al electroencep halogram (EEG)
likely rep resent?
(B) Cold -caloric–ind u ced nystagm u s
(A) Confid ence (C) Decreased resp irations
(B) Denial (D) Elevated tem p eratu re
(C) Rationalization (E) N onsaccad ic eye m ovem ents
(D) Repression
(E) Su p p ression 48. A 24-year-old w om an is referred by her fam -
ily p ractice d octor after the Christm as holi-
46. A 45-year-old Asian-Am erican w om an is d ay. She gives a significant history of fatigu e,
brou ght to the em ergency d ep artm ent by her w eight gain, and hyp erp hagia d u ring the w in-
hu sband , w ho rep orts that for the past 3 d ays ter m onths. She d escribes a “sad m ood ,” poor
his w ife has not been sleeping w ell, has been sleep, problem s concentrating, and “can’t
experiencing bad d ream s, and ap pears “in a w ait” u ntil her vacation to Florid a. Which of
d aze” w ith a sense of feeling “nu m b.” The the follow ing is the m ost likely d iagnosis?
patient end orses feeling anxiou s bu t d oes not
(A) Acu te stress d isord er
know w hy. She has been u nable to p erform
her u su al activities of d aily living, poorly con- (B) Generalized anxiety d isord er
centrating on tasks and frequently “ju m p ing” (C) Major d ep ressive d isord er w ith seasonal
w hen startled . One w eek ago, the p atient w as p attern
d ischarged from the hosp ital after exp erienc- (D) Persistent d ep ressive d isord er
ing an anap hylactic reaction to IV contrast d ye (d ysthym ia)
w hile u nd ergoing an im aging p roced u re for (E) Su nd ow ning synd rom e
sinusitis. Althou gh she cannot recall specifics,
her husband verifies the history, ad d ing that 49. A 27-year-old w om an is brou ght to the em er-
the d octors “thought she w as going to d ie.” gency d ep artm ent by her p arents, w ho rep ort
Upon retu rning to the hosp ital, she exp eri- that their d au ghter is u nable to recall her
ences intense fear abou t revisiting the sam e nam e. The em ergency d ep artm ent p hysician
hospital from w hich she w as recently d is- rep orts that a com p lete neu rologic w orku p is
charged . Which of the follow ing is the m ost w ithin norm al lim its. Collateral inform ation
app rop riate d iagnosis? reveals that the p atient had ep isod es in w hich
(A) Acu te stress d isord er she w ould take u nplanned trips, som etim es
for d ays or w eeks, w ithou t notice, and w ou ld
(B) Ad ju stm ent d isord er
retu rn u nable to recall the ep isod e. A review of
(C) Generalized anxiety d isord er (GAD) her m ed ical chart notes a p ast history of pos-
(D) Major d ep ressive d isord er (MDD) sible sexual abu se as a child . Urine toxicology
(E) Posttrau m atic stress d isord er (PTSD) is negative and she d oes not take any m ed ica-
tions. Which of the follow ing is the m ost likely
47. A 40-year-old w om an w ithou t p ast p sychiatric d iagnosis?
history is ad m itted to the hospital for the treat-
(A) Deliriu m
m ent of d epression. Du ring m orning rou nd s,
the patient appears unresp onsive and d oes (B) Dissociative am nesia w ith d issociative
not, ind eed , respond to verbal stim u li. There fu gu e
212 7: Pra c tic e Te s t 1

of the following medications would be the most patient’s mother calls you and reports that her
appropriate choice? son is more agitated and confused. Physical
examination reveals tachycardia, dilated pupils,
(A) H alop erid ol
and flushed skin. Which of the following would
(B) Loxapine (Loxitane) be the most appropriate next step?
(C) Perp henazine (Trilafon)
(A) Discontinu e benztropine and p rescribe
(D) Pim ozid e (Orap )
am antad ine (Sym m etrel).
(E) Qu etiapine
(B) Increase haloperid ol to 5 m g tid .
Questions 40 and 41 (C) Increase benztropine to 2 m g bid .
(D) Discontinu e halop erid ol and p rescribe
A 30-year-old w om an presents w ith a history of risp erid one.
chronic but rapid and intense m ood sw ings, feelings (E) Prescribe lorazep am 1 m g bid .
of em ptiness, low self-esteem , chronic suicid al id e-
ation, as w ell as frequ ent but brief sexu al relation- Questions 43 and 44
ships, anger outbu rsts, and self-m u tilation.
A 25-year-old single w om an w ho carries the p rovi-
40. Which of the follow ing d isord ers is this p atient sional d iagnosis of d epend ent personality d isord er
m ost likely to be su ffering from ? is referred for p sychological testing. You d ecid e to
(A) Avoid ant p ersonality d isord er ad m inister the Rorschach.
(B) Bipolar d isord er
43. Which of the follow ing asp ects of this typ e
(C) Bord erline p ersonality d isord er
of psychological testing w ou ld be the m ost
(D) Depend ent personality d isord er
im p ortant in this case?
(E) N arcissistic personality d isord er
(A) Ask sp ecific qu estions w ith item ized
41. The above p atient is referred for d ialectical resp onses.
behavioral therapy (DBT). Which of the fol- (B) Determ ine how a p atient feels abou t
low ing techniqu es is the m ost im portant in you .
her therapy? (C) Lack of stru ctu re allow s for a variety of
resp onses.
(A) Aversion
(D) Provid e nu m erical scores.
(B) Eye-m ovem ent d esensitization and
rep rocessing (EMDR) (E) Resu lts allow for easy statistical
analysis.
(C) Flood ing
(D) H om ew ork
44. You ad d itionally d ecid e to ad m inister the
(E) System atic d esensitization Minnesota Mu ltiphasic Personality Inventory
2 (MMPI-2). Which of the follow ing aspects of
42. A 21-year-old man presents w ith a 7-month this typ e of p sychological testing w ould be the
history of bizarre behavior, paranoid delusions, m ost im portant in this case?
and auditory hallucinations that comment on
his appearance. After a thorou gh evaluation, (A) Assesses test-taking attitud es.
you d iagnose him w ith schizophrenia and (B) Instru ctions are u nam bigu ou s in natu re.
prescribe haloperid ol 5 m g bid . One w eek later, (C) Item s d esigned to sep arate norm al
the patient retu rns for a follow -up exam ination subjects from those w ith psychiatric
and reports that, althou gh his symptom s illness.
have imp roved , he now experiences mu scle (D) More sensitive in p icking u p gend er-
stiffness in his arms and neck. You p rescribe specific issu es.
benztropine 1 mg bid and sched ule a follow -up (E) Most researched and w ith norm ative
ap pointment in 2 w eeks. One w eek later, the d ata.
214 7: Pra c tic e Te s t 1

(C) Major d ep ressive d isord er (A) Bip olar I d isord er


(D) Partial com p lex seizu res (B) Bip olar II d isord er
(E) Posttrau m atic stress d isord er (C) Cyclothym ic d isord er
(D) Major d ep ressive d isord er
Questions 50 and 51 (E) Persistent d ep ressive d isord er
A 20-year-old college stu d ent is referred for testing (d ysthym ia)
to evaluate poor acad em ic perform ance. H e rep orts
that he has alw ays “stru ggled ” to pass his classes 53. You are asked to evalu ate a 30-year-old m ale
d espite stu d ying for m any hours. H e attend s all of p risoner w ho reports a d ep ressed m ood and
his lectures and is able to pay attention, yet he d oes suicid al id eation. Du ring your interview and
not seem to be able to ad equ ately learn the m aterial. m ental statu s exam ination, you note that
You susp ect he has a learning d isord er. the prisoner d oesn’t give correct or sp ecific
answ ers, respond ing instead to you r qu es-
50. Which of the follow ing tests w ou ld be the m ost tions w ith approxim ate answ ers. Which of the
ap prop riate to help d eterm ine this p atient’s follow ing is the m ost ap p rop riate d iagnosis?
p roblem ? (A) Cap gras synd rom e
(A) Draw -A-Person Test (B) Dissociative am nesia
(B) Minnesota Mu ltip hasic Personality (C) Ganser synd rom e
Inventory 2 (MMPI-2) (D) Major neu rocognitive d isord er
(C) Wechsler Ad u lt Intelligence Scale— (d em entia)
Revised (WAIS-R) (E) Mu nchau sen synd rom e
(D) Wechsler Intelligence Scale for Child ren
(WISC) 54. A 75-year-old m an is ad m itted to the hospi-
(E) Wechsler Mem ory Test (WMT) tal follow ing a serious su icid e attem pt. The
p atient exhibits clinical featu res of d ep res-
51. Which of the follow ing form u las is used to cal- sion w ith severe neu rovegetative sym ptom s
cu late the intelligence quotient (IQ)? and has a p ast history of su icid e attem p ts.
The m ed ical chart reveals that he is p rescribed
(A) Actu al IQ/ theoretical IQ × 100 levod op a/ carbid op a, atorvastatin, asp irin,
(B) Chronological age/ p erform ance IQ × and nifed ip ine for a card iac arrhythm ia. H e
100 rem ains extrem ely help less and hop eless,
(C) Mental age/ chronological age × 100 w ith little ap p etite, and ongoing su icid al
(D) Mental age/ full-scale IQ × 100 id eation w ith several lethal p lans. Which of
the follow ing treatm ents w ou ld be the m ost
(E) Perform ance IQ/ verbal IQ × 100
ap p rop riate?
52. A 26-year-old female graduate student reports to (A) Diazep am (Valiu m )
you a 4-week history of a depressed mood that (B) Electroconvu lsive therap y (ECT)
has caused her significant difficulty in attending (C) N ortrip tyline (Pam elor)
her classes. The patient reports difficulty falling
(D) Risp erid one
asleep at night, red uced appetite and weight
loss, poor energy, and passive suicidal ideation. (E) Su p p ortive p sychotherap y
A careful review of her history reveals that for
the past 2 years she also experienced brief and 55. A 38-year-old w om an presents to your office
distinct periods of an elevated and expansive w ith an 8-w eek history of sym ptom s of
mood, a decreased need for sleep, and an d ep ressed m ood w ith an increased appetite
increase in activities, although she was still able and 10-lb w eight gain, hyp ersom nia, a “heavy
to function adequately. Which of the following feeling” in her bod y, and rejection sensitivity.
is the most likely diagnosis? Up on fu rther qu estioning she ad m its to being
Que s tions : 50–60 215

able to brighten w hen sp end ing tim e in activi- d ie” bu t d enies any plan or intent. Upon d is-
ties she u sually enjoys. She rep orts to you that cussion w ith the w ife present, they both con-
she recently enrolled in grad uate school and firm a p ast episod e w here for several w eeks
is having trou ble w ith m any of her classes. last year he w as “u p ,” w ith increased energy,
Which of the follow ing d iagnoses is the m ost little sleep , w anting to have sex several tim es
likely? p er d ay, talking fast, gam bling a large su m of
m oney, and attem pting to rem od el the hom e
(A) Ad ju stm ent d isord er
d esp ite having no experience. H e d enies
(B) Major d epression w ith atypical features hallu cinations or p aranoia. H e occasionally
(C) Major d ep ression w ith m elancholic d rinks one to tw o glasses of w ine on w eek-
featu res end s and d enies any illicit d ru g u se. H e has
(D) Persistent d epressive d isord er not been taking any m ed ications recently as
(d ysthym ia) he thou ght he cou ld “tou gh it ou t.” Which of
(E) Sleep d isord er the follow ing m ed ications w ou ld likely be the
m ost ap p rop riate to begin?
56. A 33-year-old m an changes his first nam e (A) H alop erid ol
to honor a m u sician w hom he id olizes. H e
(B) Lam otrigine
recently bou ght the sam e gu itar as the m u si-
cian and form ed a rock band to play his m usic. (C) Lithiu m
Du ring p ractice, the m an d resses like his id ol. (D) Sertraline
Which of the follow ing d efense m echanism s (E) Valproic acid
d oes this behavior best rep resent?
Questions 59 and 60
(A) Fixation
(B) Id ealization A 42-year-old healthy m an is u nd ergoing a lot of
(C) Id entification stress at his w ork. As a result, he is having a d if-
(D) Projection ficu lt tim e falling and staying asleep . H is p rim ary
care d octor has since p rescribed him a m ed ication
(E) Regression
to help w ith his insom nia. While he is sleep ing bet-
ter, he calls one m orning several w eeks later w ith
57. A 60-year-old m an w ith schizop hrenia sits
com plaints of a very painfu l penile erection w hich
m otionless in his chair. The patient is m u te and
has lasted throu ghout the evening.
reacts very little to his environm ent. H is eyes
ap pear fixated on a d istant object. At tim es,
59. Which of the follow ing m ed ications has m ost
he assum es bizarre postu res and im itates the
likely been p rescribed ?
m ovem ents of others. Which of the follow ing
is the best d escrip tion of his behavior? (A) Flu oxetine (Prozac)
(A) Absence seizu re (B) N ortrip tyline (Pam elor)
(B) Catalep sy (C) Paroxetine (Paxil)
(C) Catap lexy (D) Sertraline (Zoloft)
(D) Catatonia (E) Trazod one (Desyrel)
(E) Partial com plex seizu re
60. Which of the follow ing is the m ost ap propri-
ate next step ?
58. A 37-year-old m arried m ale presents w ith a
history of m ultiple d epressive ep isod es. For (A) Tell him to d ecrease the d ose.
the past 5 w eeks he has felt sad , w ith crying (B) Tell him to stop the m ed ication and
spells, term inal insom nia, low energy, but restart in 3 d ays.
increased ap p etite and w eight gain. H e has (C) Tell him to stop the m ed ication and
had passive su icid al id eation of “w anting to m onitor his cond ition.
216 7: Pra c tic e Te s t 1

(D) Tell him to stop the m ed ication and go d ischarge, she is still exp eriencing significant p ain.
im m ed iately to the em ergency room . She has no other m ed ical problem s or allergies. She
(E) Tell him to stop the m ed ication and is cu rrently taking oral contracep tives and a m ono-
have sexual intercourse or m asturbate. am ine oxid ase inhibitor (MAOI) for her d epression.

Questions 61 and 62 63. Which of the follow ing analgesics shou ld be


m ost avoid ed in this p atient?
A 66-year-old man reports a history of excessive
w orry about his d aughter since she moved aw ay (A) Acetam inophen
from the area 1 year ago. H is w ife of 43 years verifies (B) Cod eine
his complaint, ad d ing “he w orries about everything.” (C) Ibu p rofen
Recently, his w ife mad e plans to travel abroad to (D) Mep erid ine
visit friend s. The patient is u nable to accomp any his (E) Oxycod one
w ife because of chronic obstructive pulmonary d is-
ease cau sed by years of heavy smoking and is very 64. Tw o m onths later the patient retu rns for a fol-
anxious about her leaving. H e reports su bsequent low -u p visit and rep orts that she is p regnant.
d ifficulty falling asleep, excessive d aytim e fatigue, Which of the follow ing interventions is the
and trou ble concentrating at w ork. m ost app rop riate?

61. Which of the follow ing m ed ications w ou ld be (A) Continu e the MAOI.
the m ost ap propriate to treat his sym p tom s? (B) Discontinu e the MAOI.
(A) Alprazolam (Xanax) (C) Discontinu e the MAOI and initiate
treatm ent w ith flu oxetine after 2 w eeks.
(B) Busp irone (Bu Spar)
(D) Discontinu e the MAOI d u ring the first
(C) Diazep am (Valiu m )
trim ester only.
(D) Lorazep am (Ativan)
(E) Discontinu e the MAOI and begin
(E) Propranolol (Ind eral) m aintenance ECT.

62. Several w eeks later, the patient rep orts that 65. A 62-year-old hom eless m an p resents to
the m ed ication you prescribed has helped the em ergency d epartm ent w ith confu sion,
everything bu t his d ifficu lty falling asleep . H e agitation, im p aired gait, and nystagm u s. H is
rep orts that d u ring the p ast w eek he has slep t vital signs are stable and an ophthalm ologic
only 4 hours p er night. After carefu l consid er- exam ination is w ithin norm al lim its. The
ation, you d ecid e to begin a trial of a 1-w eek p atient is u nable to recall the d ate and has
su pply of m ed ication to help his insom nia. d ifficu lty su staining attention. Which of
Which of the follow ing m ed ications w ou ld be the follow ing is the m ost appropriate initial
the m ost app rop riate to prescribe? intervention?
(A) Diphenhyd ram ine (Benad ryl) (A) Ad m inister IV d extrose.
(B) Lorazepam (Ativan) (B) Ad m inister p arenteral thiam ine.
(C) Oxazep am (Serax) (C) Obtain a com p u ted tom ograp hic scan of
(D) Tem azep am (Restoril) the brain.
(E) Zaleplon (Sonata) (D) Perform a breathalyzer.
(E) Perform a lu m bar p u nctu re.
Questions 63 and 64

A 25-year-old w om an w ith a history of m ajor 66. You are a community psychiatrist who is seeing
d epressive d isord er, single episod e, in rem ission is a patient for the first time. The patient informs
ad m itted to the hosp ital for rem oval of a fibroid . H er that he has ju st m oved into the area and
p roced u re is com pleted w ithout d ifficu lty, but u p on need s to establish him self w ith a p sychiatrist
Que s tions : 61–71 217

to continue his treatm ent for d ep ression and ap p ears confu sed and is d iap horetic. A u rine toxi-
anxiety. On fu rther evalu ation, you learn that cology screen is negative.
he w as previou sly arrested for child p ornog-
rap hy and served several years in p rison. H e 68. Which of the follow ing is the m ost im portant
is cu rrently in therap y to ad d ress this issu e. intervention?
You are su rp rised to find that after the d isclo-
(A) Ad m inister brom ocrip tine.
sure, you feel angry tow ard the patient as you
have tw o you ng d au ghters of you r ow n. You (B) Ad m inister IV d antrolene sod iu m .
are concerned abou t w hether it m ay be d iffi- (C) Ad m inister naloxone (N arcan).
cult for you to treat the patient in an u nbiased , (D) Ap p ly cooling blankets.
nonjud gm ental m anner becau se of his crim i- (E) Discontinu e m ed ications.
nal history. Which of the follow ing is the next
m ost ap prop riate step ? 69. Which of the follow ing laboratory abnorm ali-
(A) Continu e to see the patient bu t lim it the ties is m ost likely to be p resent in this patient?
tim e spent w ith him . (A) Anem ia
(B) Im m ed iately refer the patient to another (B) Decreased transam inases
p hysician in you r clinic. (C) Elevated blood u rea nitrogen (BUN )
(C) Inform the p atient that you are u nable (D) Elevated creatine p hosphokinase (CPK)
to see him and refer him back to the
(E) Leu kop enia
com m unity.
(D) Obtain the patient’s therap y record s so 70. An 18-year-old m an is brou ght to th e
that you are aw are of his progress. emergency department by the paramedics after
(E) Seek a consu ltation w ith an exp erienced being involved in a motor vehicle accident. His
colleague regard ing you r feelings medical chart reports a history of substance use.
tow ard the p atient. Which of the following tests can most likely
confirm the diagnosis of chronic substance use
67. A 39-year-old execu tive w ithout p sychiatric with physical dependence?
history bu t a history of hyp ertension rep orts
d rinking up to six cu ps of caffeinated coffee (A) Breath analysis
p er d ay. H e board s a p lane sched u led for an (B) Elevated heart rate
18-hou r flight on w hich only d ecaffeinated (C) N aloxone injection
beverages are served . Which of the follow ing (D) Seru m liver function stu d ies
sym ptom s is he m ost likely to experience? (E) Urine toxicology screen
(A) Depressed m ood
71. A 72-year-old man without prior psychiatric
(B) H ead ache
history presents to the outpatient clinic with a
(C) Irritability
recent history of memory difficulty. The patient
(D) Mu scle cram p ing has had a stable level of consciousness and
(E) N au sea denies any current or past substance use. He has
not been prescribed any new medications. The
Questions 68 and 69 medical chart reveals a history of megaloblastic
anemia and a subtotal gastrectomy for severe
A 25-year-old m an w ith a history of schizophrenia
peptic ulcer disease. Which of the following ini-
p resents to the em ergency d ep artm ent w ith severe
tial interventions is the most appropriate?
m u scle stiffness and an elevated tem p eratu re. H is
casew orker claim s that he is com pliant w ith his (A) Discu ss long-term extend ed -care facility
m ed ications and d enies that he u ses alcohol or ad m ission.
d rugs. H is vital signs d em onstrate elevated tem - (B) Obtain a forensic exam ination to
p erature, blood p ressu re, and p u lse. The p atient evalu ate com p etency.
218 7: Pra c tic e Te s t 1

(C) Obtain a neu rologic consu lt. (A) BUN and creatinine
(D) Ord er a m agnetic resonance im aging (B) CBC
scan of the brain. (C) ECG
(E) Ord er a thorou gh laboratory w orku p . (D) Liver fu nction stu d ies
(E) Thyroid -stim u lating horm one
72. A 37-year-old w om an presents to the em er-
gency d ep artm ent com p laining of severe d iar- 76. A 27-year-old w om an has been prescribed
rhea, nau sea, and a coarse trem or. The m ed ical flu oxetine for d ep ression for the p ast year. She
chart notes a history of bip olar I d isord er. com es to you com plaining of m ed ication sid e
Which of the follow ing w ou ld be the m ost effects and w ishes to d iscontinu e her antid e-
ap prop riate initial intervention? p ressant. As her p hysician, you d iscu ss w ith
(A) Obtain renal fu nction stu d ies. her the ap p rop riate risks and benefits, and
(B) Obtain a seru m m ed ication level. you d iscontinu e her treatm ent. Tw o w eeks
later, she retu rns com p laining of d ep ressive
(C) Obtain a u rine p regnancy test.
sym p tom s characterized by sleep and appetite
(D) Obtain a u rine toxicology screen. changes, poor concentration, and a d epressed
(E) Prescribe antid iarrheal m ed ication. m ood for m ost of the d ay. You d eterm ine that
she is experiencing a recurrent m ajor d ep res-
Questions 73 and 74 sive episod e and d ecid e to prescribe her a
d ifferent class of m ed ication to treat her sym p -
A 72-year-old m an is brou ght into the em ergency
tom s. After d iscussing the appropriate precau -
room via am bu lance after su ffering w hat is su s-
tions w ith her, you prescribe an MAOI. Within
pected to be a stroke in his right p arietal region.
1 w eek, she begins exp eriencing irritability,
abd om inal pain and d iarrhea, and au tonom ic
73. Which of the follow ing areas w ould be m ost
instability characterized by hyp ertension and
likely to have d eficits d u e to the stroke?
tachycard ia. H er tem p eratu re is 103.5°F. The
(A) Long-term im plicit m em ory p atient also rep orts exp eriencing jerking of
(B) Long-term p roced u ral m em ory her m uscles and vivid visu al hallucinations
(C) Tactile m em ory of colorfu l flow ers spinning tow ard the sky.
(D) Verbal m em ory She states that she has follow ed your d irec-
tions carefu lly w hile taking this m ed ication.
(E) Visu al nonverbal m em ory
Which of the follow ing cond itions m ost likely
accou nt for this p resentation?
74. Which of the follow ing tests w ould best id en-
tify the abnorm ality in that region? (A) H allu cinogen intoxication
(A) Minnesota Mu ltip hasic Personality (B) Malignant hyp ertherm ia
Inventory 2 (C) N eu rolep tic m alignant synd rom e (N MS)
(B) Rey–Osterrieth Test (D) Serotonin synd rom e
(C) Rorschach Test (E) Tyram ine-ind u ced hyp ertensive crisis
(D) Wechsler Intelligence Scales
Questions 77 and 78
(E) Wisconsin Card Sorting Test
A 17-year-old girl is referred to the school nu rse for
75. A 32-year-old m arried m an is d iagnosed w ith
frequ ent ep isod es of vom iting in the bathroom d u r-
bip olar I d isord er. Variou s treatm ent op tions
ing lu nch breaks. H er friend s rep ort that, d esp ite
are d iscussed , and he agrees to begin lithiu m
alw ays talking abou t w anting to lose w eight, she
carbonate to treat his sym ptom s of m ania.
eats “tw ice as m u ch as anybod y else.” The p arents
Which of the follow ing tests is the m ost
are called to the school and recall that a recent bill
im p ortant to ord er p rior to initiating treatm ent
from their charge accou nt at the local p harm acy
w ith lithiu m ?
Que s tions : 72–82 219

ind icated a large nu m ber of laxative p u rchases. The 80. Which of the follow ing com p lications w ou ld
girl rep orts that her m ood is fine, and her record s be the m ost likely in this p atient?
at school ind icate that she is an above-average
(A) Diarrhea
stu d ent.
(B) H eat intolerance
77. Which of the follow ing m ed ical com p lications (C) Leu kocytosis
w ou ld be m ost likely in this p atient? (D) Menorrhagia
(A) Acid osis (E) Osteop orosis
(B) Dental carries
81. Which of the follow ing w ou ld be the m ost
(C) Diarrhea likely p red iction of a good ou tcom e in this
(D) H yp erkalem ia patient?
(E) H yp erchlorem ia
(A) 10%
78. Which of the follow ing m ed ications w ou ld be (B) 20%
the m ost ap propriate initial treatm ent for her (C) 50%
sym p tom s? (D) 75%
(A) Bu propion (E) 90%
(B) Citalop ram
Questions 82 and 83
(C) Lithiu m
(D) Risp erid one A 70-year-old w om an rep orts a d ep ressed m ood
(E) Valp roic acid an d insom n ia 1 m on th follow in g th e d eath of
her h u sban d . Im m ed iately after h is d eath , sh e
79. A 70-year-old m an w ith m u ltip le m ed ical began to feel th at sh e w ou ld be “better off d ead ,”
problem s is su sp ected to have had a stroke, alth ou gh d en ies an y active su icid al id eation . For
affecting his ability to speak. Which of the th e first several w eeks sh e h ad a red u ced ap p e-
follow ing tests w ou ld best assess the natu re of tite w ith a several p ou nd w eigh t loss, bu t this h as
his speech d ifficu lty? im p roved . At tim es, sh e believes th at sh e can h ear
his voice calling to her. Sh e d en ies any feelin gs
(A) Bend er Gestalt Test of w orthlessness bu t feels gu ilty abou t n ot bein g
(B) Boston Diagnostic Ap hasia Exam ination able to d o th e right th in gs for h im before h e d ied .
(C) Folstein MMSE Sh e occasionally gets d istracted and forgets that
(D) Sentence Com p letion Test he h as d ied . Sh e is able to brigh ten w hen sp eak-
in g abou t sp en d in g tim e w ith h er gran d ch ild ren .
(E) Stroop Test
Wh en talkin g w ith oth ers, sh e believes th at h er
feelin gs are n orm al.
Questions 80 and 81

A 16-year-old fem ale high school stu d ent is referred 82. Which of the follow ing is the m ost likely
by her parents for an evalu ation. One year prior to d iagnosis?
the evalu ation, the girl began restricting her food (A) Ad ju stm ent d isord er w ith d ep ressed
intake and started a rigorou s exercise program to m ood
im prove her appearance. Aspiring to be a m od el,
(B) Bereavem ent
the girl lost 25 lb bu t rem ained preoccu pied w ith her
app earance d espite w eighing only 85 lb. H er friend s (C) Major d ep ressive ep isod e
rep orted that she constantly referred to herself as (D) Persistent d ep ressive d isord er
being “fat” and d id not seem interested in d ating. (d ysthym ia)
The girl continu ed to lose w eight and w as relu ctant (E) Schizoaffective d isord er
to d iscu ss her cond ition.
220 7: Pra c tic e Te s t 1

83. Which of the follow ing is the m ost app rop ri- 86. Which of the follow ing d iagnoses w ou ld be
ate course of action? the m ost likely cause of his sym ptom s?
(A) Ad m ission to a hosp ital. (A) Circad ian rhythm sleep –w ake d isord er
(B) Evalu ate for a sleep d isord er. (B) Major d ep ressive d isord er
(C) N o treatm ent. (C) Obstru ctive sleep ap nea hyp op nea
(D) Prescribe antid ep ressant m ed ication. (D) Rap id eye m ovem ent (REM) sleep
(E) Prescribe sed ative-hyp notic m ed ication. behavior d isord er
(E) Restless legs synd rom e
84. A 25-year-old m an rep orts a 5-year history of
excessive hand w ashing and a p reoccu p ation 87. Which of the follow ing is the treatm ent of
w ith feeling clean. The thou ght of contracting choice for his sym ptom s?
an infectiou s d isease p ersists throu ghout the
(A) Antid ep ressant m ed ication
d ay even thou gh he m akes attem p ts to ignore
it. H is cond ition has p rogressively w orsened (B) Benzod iazep ine m ed ication
and has cau sed significant im p airm ent w hile (C) Breathing air u nd er p ositive p ressu re
at w ork and at hom e. Which of the follow ing (D) N asal su rgery
m ed ications w ou ld be the best initial choice to (E) Uvu lop alatop lasty
treat his sym ptom s?
(A) Antid epressant Questions 88 and 89
(B) Antiep ilep tic A 60-year-old m ale p atient w ith schizop hrenia w ho
(C) Antip sychotic has been stable for years on a low -p otency antip sy-
(D) Benzod iazep ine chotic agent begins exp eriencing parkinsonian-like
(E) Lithiu m sid e effects. H is p hysician p rescribes a d ru g to alle-
viate som e of these sid e effects. One w eek later, the
85. A 28-year-old w om an w ith no p sychiatric his- p atient is seen in the em ergency d ep artm ent w ith
tory is arrested for shop lifting. She claim s she d ilated pup ils, d ry m ou th, w arm skin, and tachycar-
d oes not rem em ber d etails of the crim e, and d ia. H e is also exp eriencing the new onset of visu al
that for “a w hile” she has had trou ble recalling hallu cinations.
d etails from things she’s recently read . You are
called to assess her, bu t w ant to ensure she’ll 88. Which of the follow ing w ou ld be the m ost
give good effort on testing. Which of the fol- ap p rop riate m ed ication to ad m inister?
low ing tests w ou ld be the m ost ap p rop riate to (A) Anticholinesterase
ad m inister? (B) Atrop ine
(A) Benton Visu al Retention Test (C) Benzod iazep ine
(B) Clock Draw ing (D) Dantrolene sod iu m
(C) Test of Mem ory Malingering (E) H alop erid ol
(D) Wechsler Intelligence Scale
(E) Wisconsin Card Sorting Test 89. After the ap p rop riate intervention, the p atient
exp eriences nau sea and vom iting and su bse-
Questions 86 and 87 qu ently has a seizu re. Which of the follow ing
m ed ications shou ld be ad m inistered next?
A 52-year-old obese m an exp eriences excessive d ay-
(A) Atrop ine
tim e sleepiness and a d epressed m ood . H is w ife
rep orts that he snores lou d ly and is restless w hile (B) Ep inep hrine
sleeping. There is no evid ence of regu lar alcohol or (C) Lorazep am
illicit su bstance u se, and the p atient d oes not have a (D) Physostigm ine
p rior p sychiatric history. (E) Prochlorp erazine (Com p azine)
Que s tions : 83–95 221

90. A 40-year-old w om an com p lain s of 3 to 93. A 48-year-old w oman successfu lly completes
4 m onths of feeling sad , w ith trou ble falling an inpatient program for alcohol d etoxification
asleep , d ecreased ap petite and som e w eight for w hich she w as prescribed chlord iazepoxid e.
loss, d ifficu lty concentrating at w ork, and Upon d ischarge, the patient is prescribed
fatigu e. She ad m its to su icid al id eation w ithou t d isulfiram. Soon after d ischarge, the patient
sp ecific plan or intent. There is no history of attend s an office party w here she ad m its to
m ania. She also m entions longstand ing and having a few d rinks. She has been comp liant
repetitive thoughts w here she w orries abou t w ith her prescribed med ication and d oes not
“germ s.” As a resu lt, she w ashes her hand s have any active med ical problem s. Which of
very frequ ently, som etim es end ing u p w ith the follow ing symptom s is she most likely to
cracked and chapp ed skin. She realizes that experience?
her w orries are unrealistic, but she cannot
(A) Blu rred vision
keep from d oing it. Which of the follow ing
m ed ications w ou ld be the m ost app rop riate to (B) Eu p horia
p rescribe? (C) H igh blood p ressu re
(D) Urinary retention
(A) Busp irone
(E) Vom iting
(B) Clom ip ram ine (Anafranil)
(C) Doxep in (Sinequ an) 94. A 22-year-old m an w ith a history of bipolar d is-
(D) Flu voxam ine (Lu vox) ord er is prescribed lithium carbonate to treat
(E) Phenelzine (N ard il) his sym p tom s. Du ring a w eekend ru gby tou r-
nam ent, he hu rts his knee and an orthoped ic
Questions 91 and 92 physician p rescribes a m ed ication to red u ce
his sym p tom s of p ain and sw elling. Although
A 56-year-old m an w ith a long history of alcohol u se the patient reports relief from this m ed ication,
d isord er and elevated liver function tests is ord ered he begins to exp erience abd om inal pain, d iar-
by the court to enroll in an inp atient d etoxification rhea, and d row siness. Which of the follow ing
program for his alcohol u se d isord er after nu m erou s m ed ications w ou ld m ost likely contribu te to
DWIs. After 2 d ays in treatm ent, he begins to exp eri- the prod u ction of these sym p tom s?
ence trem ors, sw eating, flu shing, and anxiety.
(A) Acetam inophen
91. Which of the follow ing m ed ications w ou ld be (B) Asp irin
the m ost approp riate to prescribe? (C) Cod eine
(A) Alp razolam (D) Ibup rofen
(B) Chlord iazep oxid e (Libriu m ) (E) Mep erid ine
(C) Disu lfiram (Antabu se)
95. A 32-year-old new ly m arried m an w ith a p ast
(D) Lorazepam
history of m ajor d ep ressive d isord er p resents
(E) Phenobarbital w ith a 4-w eek history of d ep ression, insom -
nia, anergia, p oor concentration, and anhe-
92. Shortly after this tim e, the m an begins to d onia. H e has had p assive su icid al id eation
rep ort visu al hallu cinations and becom es agi- w ithou t a p lan, as w ell. H e has not taken
tated . Which of the follow ing m ed ications citalop ram for several years, and , d esp ite
shou ld be prescribed to treat these sym ptom s? rem ission of his sym p tom s, he had signifi-
(A) Fluoxetine cant sexu al d ysfu nction w hen taking it. H e
(B) H alop erid ol w ishes to restart a m ed ication bu t is greatly
concerned abou t the im p act on his m arriage.
(C) Lithiu m
Which of the follow ing m ed ications w ou ld be
(D) Lorazep am
the m ost ap p rop riate?
(E) Sertraline
222 7: Pra c tic e Te s t 1

(A) Citalop ram 99. Which of the follow ing is the m ost effective
(B) Flu voxam ine treatm ent for this type of d rug abuse?
(C) Paroxetine (A) Abstinence
(D) Mirtazapine (B) Antid ep ressant agents
(E) Venlafaxine (C) Antip sychotic agents
(D) Dialectic behavioral therapy
96. A patient w ho is hu m an im m u nod eficiency
(E) Exp osu re and resp onse p revention
viru s (H IV)-p ositive rep orts 4 w eeks of a
d epressed m ood , low energy, p oor sleep, and
100. A 30-year-old man w ith schizophrenia is
hopelessness. H is app etite is negligible, and
brought in w ith his mother, w ith w hom he
he has been refu sing to eat or d rink anything
lives. His med ical chart d ocuments numerous
for 2 d ays. Which of the follow ing m ed ications
but adequate medication trials including
w ou ld be the m ost appropriate to p rescribe?
risperidone, olanzapine, quetiapine, and halo-
(A) Bupropion perid ol. Despite his compliance he continues
(B) Buspirone to have significant paranoia, d elusions, and
(C) Flu oxetine aud itory hallucinations, telling him that he is
(D) Methylphenid ate going to die. H e has no other medical problems,
and both he and his mother d eny any alcohol or
(E) Sertraline
d rug use. Which of the follow ing medications
w ould be the most appropriate to prescribe for
97. A 43-year-old w om an p resents w ith a new
this patient?
history of d ep ressive sym p tom s, inclu d ing
insom nia, poor ap petite and w eight loss, low (A) Arip ip razole
energy, and d istractibility. On m ental statu s (B) Clozap ine
exam ination she is asked to cou nt backw ard (C) Flu p henazine
by 7’s, beginning at 100. Which of the follow ing
(D) Qu etiap ine
asp ects is being assessed by this test?
(E) Zip rasid one
(A) Attention
(B) Fu nd of know led ge 101. You are treating a 32-year-old w om an for bip o-
(C) Mathem atics skills lar I d isord er w ith a com bination of m ed ica-
(D) Rem ote m em ory tions. She d evelops hair loss w hich eventually
resolves on its ow n. Which of the follow ing
(E) Verbal m em ory
m ed ications is m ost likely resp onsible for this
sid e effect?
Questions 98 and 99
(A) Carbam azepine
A 19-year-old college stu d ent rep orts that he and
(B) Clozap ine
his friend s have been experim enting w ith “hu ffing”
on cam pu s. H is room m ate rep orts that he has been (C) Divalp roex sod iu m
accu mu lating typew riter correction fluid , nail polish (D) Olanzap ine (Zyp rexa)
rem over, and m od el airp lane glu e. (E) Zip rasid one (Geod on)

98. Which of the follow ing w ou ld he m ost likely 102. A 34-year-old man is referred by his internist
exp erience d u ring intoxication? for depression. For the past 2 months he has
(A) Conju nctival injection been suffering from anhedonia, crying spells,
frequent aw akenings, poor appetite, and low
(B) Depressed reflexes
energy. He is subsequently diagnosed w ith a
(C) Dim inished response to pain major d epressive episod e and recommend ed to
(D) Increased ap petite begin citalopram 20 mg daily. After discussion
(E) Staring into space of the risks and benefits of the med ication, he
Que s tions : 96–107 223

expresses concern about possible sexual d ys- (C) Ad d an anxiolytic m ed ication.


function as he is currently in a new relationship (D) Discontinu e m ethylp henid ate and
w ith a cow orker. Which of the follow ing symp- p rescribe a m ixed am phetam ine
toms are the most likely w ith this med ication? com pou nd .
(A) Decreased libid o (E) Discontinu e m ethylp henid ate and
(B) Prem ature ejaculation p rescribe an antid ep ressant.
(C) Priap ism
105. A 35-year-old w om an presents w ith ep isod ic
(D) Retrograd e ejaculation anxiety and com p laints that occasionally
(E) Sexually transm itted d isease w hen hearing som ething for the first tim e, that
she has actually heard it before. She expresses
Questions 103 and 104 her concern that she is “going crazy.” Which
of the follow ing term s best d escribes this
A 21-year-old m ale college stud ent w ithou t p rior
p henom enon?
m ed ical or psychiatric history is evalu ated for p oor
w ork p erform ance. The stu d ent rep orts to the d ean (A) Déjà entend u
that he find s it hard to follow through w ith assign- (B) Déjà vu
m ents and is easily d istracted by environm ental (C) Folie à d eu x
stim uli. The d ean com m ents to the stu d ent abou t
(D) Jam ais vu
how frequ ently he interru p ts others d u ring conver-
sations and observes that it seem s that the stud ent (E) La belle ind ifférence
is not p aying attention to w hat he is saying. Over-
106. A 19-year-old m an is brou ght to the em er-
all, his grad es are p oor excep t in the class he con-
gency d ep artm ent by the p olice for an evalu -
sid ers “the m ost interesting class I have ever had .”
ation. The w ritten p olice rep ort states that
H e often forgets requ ired m aterials for classes and
the p atient has been calling 911 for the past
his assignm ents are frequ ently late. The stu d ent’s
5 w eeks, reporting that he is being spied on
p arents report that he w as evaluated for sim ilar d if-
by aliens from a d istant p lanet. The p atient
ficu lties w hile in p u blic school, bu t he seem ed to
rep orts that he is receiving m essages from the
im p rove w hen the fam ily m oved ou t of the area and
aliens throu gh his com pu ter and that he hears
he attend ed p rivate school.
voices in his head com m enting on his appear-
103. Which of the follow ing is the m ost likely ance. H e has not been sleep ing w ell at night
d iagnosis? becau se he has been “gu ard ing his bed room .”
You note that his affect is flat, and he ap p ears
(A) Attention-d eficit/ hyperactivity d isord er tired d uring your examination. A urine toxicol-
(ADH D) ogy screen is negative. Which of the follow ing
(B) Bipolar I d isord er is the m ost likely d iagnosis?
(C) Bip olar II d isord er (A) Bip olar I d isord er w ith p sychotic
(D) Cond u ct d isord er featu res
(E) Learning d isord er (B) Brief p sychotic d isord er
(C) Major d ep ressive d isord er (MDD) w ith
104. The p atient is p rescribed m ethylp henid ate for p sychotic featu res
the above sym ptom s. After an ad equate trial
(D) Schizoaffective d isord er
and d ose, he continu es to d isp lay som e sym p -
tom s w hich interfere w ith his school fu nction- (E) Schizop hreniform d isord er
ing, althou gh he d enies significant sid e effects.
Which of the follow ing actions w ou ld be the 107. An 18-year-old w oman w ith recently d iag-
m ost ap prop riate next step? nosed schizophrenia is acutely psychotic and
in labor w ith her first child . The obstetrics ser-
(A) Ad d a low -d ose am phetam ine. vice requests a psychiatric consultation for an
(B) Ad d an antid ep ressant. appropriate and safe med ication to use in ord er
224 7: Pra c tic e Te s t 1

to control the patient’s psychotic symptoms. 110. A 7-year-old boy w ith d ifficu lty in school need s
Which of the follow ing w ould be the best fu rther evalu ation of acad em ic p otential.
choice?
(A) Chlorp rom azine (Thorazine) 111. You w ou ld like to ad d ress the d eficits of a
(B) H aloperid ol (H ald ol) 69-year-old m an w ith nonflu ent sp eech and
form u late a sp ecific treatm ent p lan.
(C) Loxap ine (Loxitane)
(D) Perphenazine (Trilafon)
Questions 112 through 116
(E) Thiorid azine (Mellaril)
Select the d ru g m ost likely to cause the associated
D IRECTION S (Questions 108 through 116): The
sym ptom s.
follow ing group of questions are preceded by a
list of lettered options. For each questions, select (A) Cocaine
the one lettered option that is most closely asso- (B) Lysergic acid d iethylam ine (LSD)
ciated w ith it. Each lettered option may be used (C) Mariju ana
once, multiple times, or not at all.
(D) N icotine
Questions 108 through 111 (E) Op iates
(F) Phencyclid ine (PCP)
Match the clinical presentation w ith the ap propriate
neu ropsychological test. 112. An 18-year-old high school student with con-
junctival redness, increased appetite, dry mouth,
(A) Beck Depression Inventory tachycardia, and a sensation of slowed time.
(B) Bend er Gestalt Test
(C) Blessed Rating Scale 113. A 31-year-old m an w ith m iosis, brad ycard ia,
(D) Boston Diagnostic Aphasia Exam ination hyp otension, hypotherm ia, and constipation.
(E) Mini-Mental state Exam ination (MMSE)
(F) Minnesota Mu ltiphasic Personality 114. An assau ltive 26-year-old m an w ith vertical
Inventory 2 (MMPI-2) nystagm u s, echolalia, paranoid id eation, and
(G) Rey–Osterrieth Test hallucinations.
(H ) Rorschach Test
(I) Wad a Test 115. A 16-year-old girl w ith abd om inal cram ps,
(J) Wechsler Ad u lt Intelligence Scale— confu sion, palpitations, and m u scle tw itching.
Revised (WAIS-R)
(K) Wechsler Intelligence Scale for Child ren 116. A 21-year-old m an w ith tachycard ia, d ilated
(WISC) p upils, hallucinations, and com p laints of chest
p ain.
108. A 65-year-old fem ale p hysician w ho is now
concerned about m em ory d ifficulties.

109. A 35-year-old w om an w ith a p rotein S d efi-


ciency show s evid ence of left sid e hem i-neglect.
CHAPTER 8

Prac tic e Te s t 2
Que s tions

D IRECTION S (Questions 1 through 4): For each the lettered answ er that is the one best response in
of the follow ing vignettes, select the one lettered each case
option that is most closely associated w ith it. Each
lettered option may be used once, multiple times, 5. An 18-ye r-ol w o n recently i gnose
or not at all. w ith first e iso e of schizo hreni grees
to t ke n nti sychotic e ic tion to
(A) Cit lo r hel ecre se her h llu cin tions, bu t she is
(B) Flu oxetine ntly o ose to t king ny e ic tion
(C) Mirt z ine th t y c u se her to g in excessive w eight.
(D) P roxetine Which of the follow ing e ic tions w ou l be
(E) Sertr line the ost ro ri te to rescribe?
(F) Venl f xine (A) Cloz ine (Cloz ril)
(B) Ol nz ine (Zy rex )
1. A 29-ye r-ol new ly rrie le w ith (C) Qu eti ine (Seroqu el)
signific nt e ressive sy to s, inclu ing
(D) Ris eri one (Ris er l)
inso ni n w eight loss, w s nonco li nt
w ith e ic tions in the st u e to sexu l (E) Zi r si one (Geo on)
ysfunction.
6. A 40-ye r-ol rrie fe le w ithout st
sychi tric history is referre by her inter-
2. A 45-ye r-ol n w ith severe jor e res- nist. She h s been feeling “ ow n” for sever l
sion on h igh ose of n n ti e ress nt w eeks, w ith little bility to enjoy herself. Since
evelo s incre se bloo ressu re. th t ti e, her slee h s been isru te , her
etite re uce , n she’s felt “tire .” She
3. A 50-ye r-ol ivorce fe le being tre te for h s lso been king ist kes t w ork n is
jor e ression oesn’t refill her rescri tion w orrie bou t being fire . She its to h v-
n 2 ys l ter h s he che, u scle ches, ing thou ghts of killing herself, bu t she enies
n n use . ny l n or intent to o so. She enies h llu -
cin tions or elu sions. Which of the follow ing
4. A 33-ye r-ol w o n w ith ersistent e re- e ic tions w ou l be the ost ro ri te
ssive isor er ( ysthy i ) h s history of oor for this tient?
co li n ce w ith e ic tion s, occ sion lly (A) Ari i r zole
issing oses. (B) Cit lo r
(C) Methyl heni te
D IRECTION S (Questions 5 through 41): For each of (D) N ortri tyline
the multiple-choice questions in this section select
(E) Tr nylcy ro ine

247
248 8: Pra c tic e Te s t 2

7. A 25-ye r-ol w o n resents to her ri ry oes not feel like her con ition w ill i rove,
c re octor co l ining of su en onset of she enies ny su ici l i e tion. She rinks
intense fe r n feeling th t she w s going one to tw o rinks once or tw ice er onth,
to ie, w hich occurre w hile she w s sto e bu t enies illicit ru gs. She is su bsequ ently
in tr ffic e rlier in the w eek. At th t ti e she begu n on flu oxetine, w hich is gr u lly
bec e short of bre th, i horetic, n tre - incre se to 60 g. She enies si e effects,
u lous, n she cou l feel her he rt oun ing. lthou gh she h s not ex erience signific nt
H er initi l i u lse w s to rive to n e er- lessening of her e ressive sy to s. H ow
gency e rt ent, bu t her istress su bsi e long shou l her cu rrent os ge be continu e
on its ow n in bou t 20 inu tes. Which of the before sw itching to nother e ic tion?
follow ing shou l be the next ost ro ri te
(A) 3 ys
cou rse of ction?
(B) 1 w eek
(A) Con u ct thorou gh e ic l screening (C) 2 w eeks
for he rt n lu ng ise se.
(D) 4 w eeks
(B) Prescribe selective-serotonin reu t ke
(E) 6 w eeks
inhibitor (SSRI) n sk to see the
tient b ck in 1 w eek.
10. A 56-ye r-ol w o n w ith schizo hreni h s
(C) Prescribe short- cting benzo i ze ine been st ble on h lo eri ol for ny ye rs.
s nee e in c se of nother e iso e. She enies ny current sychotic sy to s
(D) Re ssu re the tient th t her sy to s n h s been co li nt w ith her e ic tions.
re ost likely benign n not c u se She resents for rou tine follow -u oint-
for concern. ent. Which of the follow ing tests shou l be
(E) Refer the tient to sychi trist. inistere ?
(A) Abnor l Involu nt ry Move ent Sc le
8. You re seeing 45-ye r-ol n w ith his-
(AIMS)
tory of schizo hreni . H e is cu rrently co -
li nt w ith his nti sychotic e ic tions (B) Beck De ression Inventory (BDI)
bu t re ins sy to tic, w ith u itory h l- (C) Brief Psychi tric R ting Sc le (BPRS)
lu cin tions n r noi . On ent l st tu s (D) P tient H e lth Qu estionn ire (PH Q)
ex in tion (MSE), he s e ks coherently n (E) Positive n N eg tive Sy to Sc le
rticul tely, bu t kes no sense bec u se ny (PAN SS)
of the w or s he uses re of his ow n invention.
Which of the follow ing ter s best escribes 11. A n brings his w ife to the octor bec u se
this MSE sign? she h s beco e ore sy to tic over the
(A) Cl ng ssoci tions st sever l w eeks. She is i gnose w ith
schizo hreni n h s recently sto e her
(B) Echol li
e ic tions. She h s e thre ts tow r
(C) Flight of i e s her hu sb n bec u se she believes th t he is n
(D) N eologis s i ostor w ho looks ex ctly like her hu sb n .
(E) Wor s l Which of the follow ing ter s best escribes
this sy to ?
9. A 34-ye r-ol w o n resents to sychi -
(A) A ok
trist w ith 6-ye r history of e tiness n
e ression for “ ost of the ti e,” w ith low (B) C gr s syn ro e
self-estee , chronic inso ni , n ifficulty (C) Cot r syn ro e
king ecisions. She enies ny roble s (D) Cou v e syn ro e
w ith her etite or energy, n , w hile she (E) Koro
Que s tions : 7–17 249

12. A 69-ye r-ol w o n w ithout rior sychi tric (C) Echol li


history is seen by her ri ry c re hysici n (D) P lil li
ue to “crying s ells.” She st tes th t she h s (E) P r r xis
felt s since the su en e th of her husb n
5 w eeks go. Since th t ti e, she h s h iffi- Questions 15 and 16
culty slee ing n f tigue. She h s lso lost sev-
er l oun s ue to not enjoying her foo . She A w e lthy, ivorce 48-ye r-ol w o n resents
is ble to cheer u w hen s en ing ti e w ith to you fter being rreste for sho lifting. The
her chil ren, but feels es eci lly lonely t night. tient its th t she h s been ste ling for ye rs,
She is so ew h t nxious bec use she occ - lthou gh she is e sily ble to ffor the objects sto-
sion lly he rs her ece se husb n ’s voice, len. She st tes th t she ste ls “on the s u r of the
es eci lly e rly in the orning or l te in the o ent” n th t these i u lses see foreign n
evening. While she feels her future is “ble k,” istressing.
she enies ny suici l i e tion or l n. Which
of the follow ing i gnoses is ost likely? 15. Which of the follow ing i gnoses is the ost
likely?
(A) Bere ve ent
(B) Bi ol r isor er (A) Antisoci l erson lity isor er
(C) M jor e ressive isor er (MDD) (B) Inter ittent ex losive isor er
(D) Persistent e ressive isor er (C) Kle to ni
( ysthy i ) (D) Pyro ni
(E) Schizo ffective isor er (E) Trichotillo ni

Questions 13 and 14 16. Which of the follow ing ter s lies to the
f ct th t these i u lses re istressing to her?
You re ske to ev lu te 12-ye r-ol boy with his-
tory of Tourette isor er n obsessive-co ulsive (A) Delu sion l
isor er. He is brought in to the oint ent with his (B) Ego- ystonic
rents, who escribe the severity of his illness n (C) Ego-syntonic
how he is often te se in school. During the interview (D) Moo congruent
the boy inter ittently kes obscene gestures.
(E) Moo incongru ent
13. Which of the follow ing ter s best escribes
17. A 49-ye r-ol n co es to you co l in-
this sy to ?
ing of he ches, e ory loss, isorient -
(A) Ble h ros s tion , n occ sion l r lysis th t ffects
(B) Bru xis his r s n l sts sever l hou rs. Du ring
(C) Co ro r xi th e MSE, you notice th t th e tient is giv-
ing v gu e nsw ers to ny qu estions (e.g.,
(D) Echo r xi
th ere re six toes on th e foot n 2 + 2 = 5).
(E) Torticollis Which of th e follow in g ter s best escribes
th is resen t tion?
14. While obt ining the history, the tient is lso
note to re e t hr ses you u se i e i tely (A) A ok
fter you s y the . Which of the follow ing (B) Dere liz tion
ter s best escribes this sy to ? (C) G nser syn ro e
(A) Co rol li (D) N eg tivis
(B) Dys rthri (E) Piblokto
250 8: Pra c tic e Te s t 2

18. A sychi trist iscovers th t she is fru str te believe her thoughts coul be he r through the
n e sily ngere w ith one of her tients w lls. She feels th t this is n inv sion of her riv cy.
for no obvious re son. While t lking to col-
le gue, she its th t the tient re in s her 21. Which of the follow ing ter s best escribes
of her busive f ther. Which of the follow ing the belief th t the tient’s thoughts tow r
best escribes the clinici n’s re ction? the neighbor w ere res onsible for the lost
regn ncy?
(A) Countertr nsference
(B) Dis l ce ent (A) Dis l ce ent
(C) Projection (B) I e s of reference
(D) Re ction for tion (C) M gic l thinking
(E) Tr nsference (D) Projection
(E) Re ction for tion
Questions 19 and 20
22. Which of the follow ing ter s best escribes
A 19-ye r-ol w o n resents w ith co l ints the tient’s fe r th t her thoughts cou l be
of fe r, rehension, n tre bling w ithou t ny overhe r ?
know n re is osing situ tions. On ex in tion,
you note brow n skin, s oky brow n rings on the (A) Echol li
ou ter corne , n occ sion l r i , jerky, u r ose- (B) Thou ght bro c sting
less sw inging of the r s w hich e rs to w orsen (C) Thou ght control
w ith volu nt ry ove ent. (D) Thou ght insertion
(E) Tr nsference
19. Which of the follow ing ter s best escribes
the tient’s ocul r fin ings?
Questions 23 and 24
(A) Arcus senilis
A 25-ye r-ol n resents to the e ergency roo
(B) Bru shfiel s ots
fter being brou ght in by his girlfrien . She h s
(C) K yser–Fleischer rings been concerne s “he h s not been cting right.”
(D) Su bconju nctiv l he orrh ge She st tes he h s been st ying u ll night over
(E) X nthel s the l st sever l ys but still “very ctive” uring
the yti e. H e h s re rr nge the fu rniture n
20. Which of the follow ing ter s best escribes tte te to re o el their b throo , es ite his
the bnor l ove ents seen in this tient? l ck of ex erience w ith constru ction. H e h sn’t
show n u for w ork recently, n she is fr i he
(A) Athetoi
w ill be fire . While he its th t he h sn’t sle t,
(B) Choreifor he cl i s to feel “gre t!” H e is so ew h t ifficu lt
(C) H e ib llis u s to interview s he w lks roun the roo , h n-
(D) Myoclonu s ling ll the equ i ent. When confronte w ith his
(E) Myotoni not going to his w ork, he st tes “Why shou l I . . . I
on the verge of iscovering cu re for AIDS,
Questions 21 and 22 w hich lso h ens to be for u l for econo ics,
w hich w ill lso get ri of overty.” H e enies lco-
A 21-ye r-ol w o n is brought to the sychi tric hol or ru g u se, bu t u on qu estioning, its to
e ergency e rt ent fter c lling the olice to erio in his l te teens w hen he felt e resse , w ith
tu rn herself in. She cl i s th t she w s res onsible ifficu lty slee ing, w eight loss, low energy, gu ilt,
for the loss of her neighbor’s regn ncy. She believes n hel lessness/ ho elessness. H e w s su bse-
her neg tive thoughts tow r the w o n c u se her qu ently tre te w ith “so e sort of nti e ress nt”
isc rri ge. On further qu estioning, she tells you for roxi tely 18 onths, bu t he never follow e
th t she felt thre tene by her neighbor bec u se she u fter th t ti e.
Que s tions : 18–29 251

23. Which of the follow ing is the ost ro ri te (A) Conversion isor er
i gnosis for this tient? (B) F ctitiou s isor er
(A) Bi ol r I isor er (C) Illness nxiety isor er
(B) Bi ol r II isor er (D) M lingering
(C) Cyclothy ic isor er (E) So tic sy to isor er
(D) M jor e ressive isor er w ith
sychotic fe tu res 27. Which of the follow ing ter s best escribes
this tient’s ini iz tion of the severity of
(E) Schizo hreni
his sy to s?
24. Which of the follow ing e ic tions w ou l be (A) Déjà enten u
the ost ro ri te to rescribe? (B) Déjà vu
(A) A itri tyline (C) Folie à eu x
(B) Cloz ine (D) J is vu
(C) Flu oxetine (E) L belle in ifférence
(D) H lo eri ol
28. A 67-ye r-ol w o n w ith u l on ry c rci-
(E) V l roic ci
no n secon ry br in et st ses recently
r fte w ill in the resence of her f ily
25. A 36-ye r-ol w o n w s begu n on l r -
ttorney. She h s history of jor e ressive
zol (X n x) 3 ye rs go for nic isor er.
isor er (MDD) th t is now in re ission. She
After w tching new s re ort on television,
eci es th t her chil ren w ho re w ell est b-
she bec e frightene bou t iction. She
lishe in their c reers o not nee ny inheri-
bru tly sto e the e ic tion 2 ys go
t nce n th t her est te w oul best serve
n h s since been ex eriencing cu te, intense
ch rity. To secure the v li ity of the w ill, the
nxiety, tre ors, sw e ting, n l it tions.
tient sks her sychi trist to sub it let-
Which of the follow ing e ic tions w ou l be
ter to her ttorney reg r ing her co etency.
the ost ro ri te to rescribe?
Which of the follow ing w ou l be the ost
(A) Di ze i ort nt f ctor in eter ining this w o n’s
(B) Flu zenil test ent ry c city?
(C) Pro r nolol (A) Actu s reu s
(D) Sertr line (B) A history of MDD
(E) V l roic ci (C) Know le ge of her n tu r l heirs
(D) The resence of conserv tor of erson
Questions 26 and 27
(E) The resence of ju ge to w itness the
A evou t hu sb n fin s th t his w ife is h ving n signing of the w ill
ff ir w ith his best frien . One w eek l ter, he fin s
th t he c nnot w lk. A thorou gh neurologic w orku 29. You re consu lte to see 77-ye r-ol H is-
f ils to reve l c u se to his su en r legi . H is nic le w ith schizo ffective isor er,
neu rologic ex in tion is not consistent w ith u er e ressive ty e, itte to the hos it l for
or low er otor neuron fin ings. Des ite this r - chest in. H is w orku h s reve le u l o-
tic is bility, he see s qu ite un ffecte by it n ry e bolis , n the e ic l te w ishes
e otion lly. to l ce chest tube in the tient, bu t the
tient refu ses. The ri ry te h s ske
26. Which of the follow ing i gnoses w ou l be you to eter ine his c city to refu se the
the ost ro ri te in this tient? roce u re. Which of the follow ing fe tures is
252 8: Pra c tic e Te s t 2

the ost i ort nt in eter ining ecision- Questions 32 and 33


king c city in this tient?
You re ske to erfor “co etency to st n
(A) Ability to co u nic te tri l” ev lu tion of n 18-ye r-ol n w ho w s
(B) Age of tient rreste for violently ss u lting his girlfrien . The
(C) Agree ent w ith tre t ent te tient h s no st sychi tric history, n frien s
reco en tions re ort th t he h s never e onstr te ny violent
(D) Presence of sychosis ten encies. The tient re orts th t his rel tion-
shi w ith his girlfrien “w ent ow nhill” shortly
(E) S nish s e king
fter they gr u te fro high school n his girl-
frien sto e c lling hi . On MSE, his ffect is
30. A e uty sheriff serves you su b oen for
constricte n his oo is re orte s “cr zy.”
the recor s of one of your tients w ho is the
Thou ght rocesses re go l irecte , n he enies
efen nt in civil li bility l w su it. Which of
ny h llu cin tions or elu sions. The tient is
the follow ing is the ost ro ri te cou rse
u n ble to correctly erfor si le c lcu l tions.
of ction?
E ch ti e he is ske to u lti ly, su btr ct, or
(A) Rele se the tient’s recor s to the ir of nu bers, his nsw ers re w rong by one
l intiff bec u se the su b oen overri es or tw o igits. For ex le, he res on s “22” w hen
tient consent. ske to u lti ly 7 by 3.
(B) H n over only the infor tion th t is
relev nt to the c se. 32. Which of the follow ing i gnoses is the ost
(C) Cont ct the tient n sk if she or he likely in this c se?
w ou l like the infor tion rele se . (A) Conversion isor er
(D) Rele se the tient’s recor s irectly to (B) F ctitiou s isor er
the cou rt. (C) M jor neu rocognitive isor er
(E) Refu se to s e k to the sheriff in or er to (D) M lingering
int in confi enti lity.
(E) Schizo hreni
31. A 23-ye r-ol gr u te stu ent you h ve
33. Which of the follow ing st te ents best
been seeing in long-ter sychother y h s
escribes the bove tient’s ro u ction of
sto e ying his bills, es ite being rei -
sy to s n otiv tion?
bu rse by his insu r nce co ny. Which of
the follow ing is the next ost ro ri te (A) Consciou s ro u ction of sy to s to
cou rse of ction? ssu e the sick role.
(A) Cont ct the tient’s insu r nce co ny (B) Consciou s ro u ction of sy to s to
n requ est th t they issue you nother obt in secon ry g in.
y ent. (C) Unconsciou s ro u ction of sy to s
(B) Directly ress this issu e w ith u e to u nconsciou s conflicts.
the tient t the next sche u le (D) Unconsciou s ro u ction of sy to s to
oint ent. ssu e the sick role.
(C) Inqu ire w ith f ily e bers w hether (E) Unconsciou s ro u ction of sy to s to
the tient h s fin nci l roble s. obt in secon ry g in.
(D) N otify collection gency to obt in
rei bu rse ent. 34. You re ske to ev lu te 68-ye r-ol n
on the in tient e icine service for incre s-
(E) Sen the tient letter of ter in tion.
ing confu sion. The tient w s itte
2 ys e rlier for neu oni . After erfor -
ing ent l st tu s ev lu tion, you su s ect
eliriu . Fu rther history fro the tient’s
Que s tions : 30–39 253

w ife in ic tes th t the tient ro e ou t 36. Which of the follow ing w ou l be the ost
of school in the seventh gr e. Which of the ro ri te next ste in the n ge ent of
follow ing tests w ou l best ssess his bility to this tient?
int in n focu s ttention?
(A) A inister thi ine or lly (PO) before
(A) Cou nting by 2’s to 20 IV flu i s n glu cose.
(B) R n o letter test (B) A inister thi ine intr venou sly (IV)
(C) Seri l sevens before IV flu i s n glu cose.
(D) Seri l threes (C) A inister thi ine IV fter IV flu i s
(E) Si le c lcu l tions n glu cose.
(D) A inister n loxone IV before IV flu i s
35. A 7-ye r-ol girl is brou ght to the e ergency n glu cose.
e rt ent for ev lu tion of sore thro t (E) A inister n loxone IV fter IV flu i s
n fever. H er rents took her to the e i - n glu cose.
trici n’s office bou t 1 w eek go n he rec-
o en e flu i s n be rest. Within the 37. The bove tient is st bilize n itte
l st 2 ys, the tient evelo e ys h gi to the ICU. After 3 ys, he begins to beco e
n severe b o in l in. H istory fro the confu se g in, w ith visu l h llu cin tions,
rents in ic tes the tient h s been nxiou s tre ors, i horesis, n elev te bloo res-
n i u lsive over the l st onth. There h s su re n u lse. Which of the follow ing is the
lso been rke ecline in school erfor- ost ro ri te tre t ent for this tient?
nce, n she h s not been intereste in
l ying w ith her frien s. Physic l ex in - (A) A inister b rbitu r te.
tion is re rk ble for signific nt erythe (B) A inister benzo i ze ine.
over the osterior h rynx w ith gr y exu - (C) A inister ition l thi ine.
te. There re br sions in the region of the (D) A inister n nti sychotic.
tient’s l bi . Co lete bloo cou nt show s (E) A inister hy r l zine.
w hite bloo cell (WBC) cou nt of 14,000/ µL
w ith left shift. Which of the follow ing is the 38. A you ng w hite le, ge u nknow n, is brou ght
next ost ro ri te ste ? into the e ergency roo u nres onsive to
(A) Arr nge for f ily eeting to qu estioning. H is vit ls e onstr te nor l
eter ine s fe is osition. te er tu re, low u lse n bloo ressure,
(B) Cont ct the st te’s Chil Protective n ecre se res ir tions. H e e rs le,
Services w hile kee ing the tient s fe. w ith u ils th t re constricte n ini lly
res onsive. A inistr tion of w hich of the
(C) Det in the rents w hile you notify the
follow ing w ou l be ost likely to i rove his
olice.
con ition?
(D) N otify the rob te court to h ve the
tient leg lly re ove fro the (A) Disu lfir (Ant bu se)
llege er etr tor. (B) Benzo i ze ines
(E) Refer the tient to the f ily’s (C) Flu zenil
e i trici n. (D) N loxone
(E) Thi ine
Questions 36 and 37
A 48-ye r-ol le is brou ght into the e ergency 39. A 77-ye r-ol w o n w ithout rior sychi tric
roo (ER) vi n bu l nce. H e s ells of lcohol, is history is brought into her f ily hysici n’s
covere in vo it, n not res onsive to qu estions. office w ith her husb n . H e is concerne th t
H e is unste y n u ncoo er tive w ith the hysic l she is “ e resse ” n nee s tre t ent. H e
ex in tion, bu t he is note to h ve isconju g te escribes her w ith r w ing e otion lly over
eye ove ents. the st ye r or tw o, w ith gr u l ecline in
254 8: Pra c tic e Te s t 2

her bility to c re for herself. They h ve been (D) R ce


un ble to t ke rt in their u su l soci l ctivi- (E) Religion
ties, n the tient “ju st w n ers roun the
hou se uring the y.” H e h s lso notice 41. The tient w its u ntil her f ily n ri ry
th t she is forgetful, often is l cing ite s, c re te le ve the roo . She then sks you to
n ixing u n es of cqu int nces. When hel her co it su ici e, s her rognosis is
ske , the tient enies ny ifficulties, st t- very oor. Wh t is the ost ro ri te next
ing she feels “fine,” n th t her husb n w or- course of ction?
ries too uch. She ntly refutes roble s
w ith her e ory, inste bl ing hi for (A) Discu ss her re sons for su ici e in ore
oving things roun in their house. She h s et il.
no signific nt e ic l roble s. U on MSE, (B) Ex l in how she c n obt in leth l oses
she is le s nt n coo er tive w ith questions, of e ic tion.
lthough efensive t ti es. H er ffect is neu- (C) Infor the f ily i e i tely.
tr l but full. There is no su ici l or ho ici l (D) Pl ce the tient in restr ints.
i e tion, n she enies ny sychotic sy - (E) Refer her to hysici n w ho erfor s
to s. H er Mini-Ment l St te Ex in tion is eu th n si .
20/ 30. H er hysic l is essenti lly u nre rk-
ble. Which of the follow ing w oul be the D IRECTION S (Questions 42 through 49): Match
ost ro ri te ri ry tre t ent for this the clinical presentation w ith the appropriate neu-
tient? ropsychological test. Each lettered heading may be
(A) Cit lo r selected once, more than once, or not at all.
(B) G l nt ine (A) Beck De ression Inventory (BDI)
(C) Ginkgo bilob (B) Ben er Gest lt Test
(D) Me ntine (C) Blesse R ting Sc le
(E) Zi r si one (D) Boston Di gnostic A h si Ex in tion
(E) Folstein Mini-Ment l St te Ex in tion
Questions 40 and 41 (MMSE)
You re ske to see 37-ye r-ol w hite C tholic (F) Minnesot Mu lti h sic Person lity
fe le w ith en -st ge ov ri n c ncer bec u se she Inventory 2 (MMPI-2)
h s tol her oncologist th t she w nts to ie. When (G) Rey–Osterrieth Test
you ro ch the tient’s be , you see c chectic (H ) Rorsch ch Test
bu t s iling w o n su rrou n e by her hu sb n n (I) Stroo Test
tw o you ng u ghters. She h s history of e res-
(J) W Test
sion bu t is not un er ny tre t ent for it now . “I
re y to go, Doctor,” she s ys to you . H er oncologist (K) Wechsler A u lt Intelligence Sc le—
h s tol you th t he w oul like to try new che o- Revise (WAIS-R)
ther y for w hich the tient is goo c n i te, (L) Wisconsin C r Sorting Test (WCST)
bu t the tient h s refu se ; he is concerne bou t
her suici l otenti l. 42. A 40-ye r-ol w o n w ho scores 26/ 30 on
the Folstein MMSE g ve ny nsw ers of, “I
40. Which of the follow ing ch r cteristics incre se on’t know, I’ too tire to nsw er.” You w nt
this rticu l r tient’s risk of co itting to ssess for the ossibility of e ression.
su ici e?
(A) Age 43. A 65-ye r-ol n h s ifficu lty org nizing,
sequ encing, n l nning ctivities but no
(B) Gen er
i ir ent in e ory.
(C) M rit l st tu s
Que s tions : 40–51 255

44. The f ily of n 80-ye r-ol n w ith neu - chorionic gon otro in is ositive. The tient’s
rocognitive isor er ( e enti ) h s ske rents e n to know the resu lts. Which of
you to ev lu te his bility to continu e to live the follow ing is the ost ro ri te cou rse
in his cu rrent environ ent. You w ish to sk of ction?
his f ily n frien s their o inion of how he
(A) Disclose the resu lts in eeting w ith
h s been oing.
the tient n her rents.
(B) Encou r ge the tient to iscu ss the
45. A 16-ye r-ol boy w ith f ily history of resu lts w ith her rents.
intellectu l is bility resents w ith longst n -
(C) Invoke testi oni l rivilege.
ing oor school erfor nce n ggressive
beh vior tow r eers. (D) N otify the rents of the resu lts bec u se
they y her e ic l bills.
(E) Re ort the resu lts to Chil Protective
46. A 37-ye r-ol n h s history of voi ing
Services.
soci l situ tions, no close frien s, n ref-
erence for being lone. H e h s been escribe
Questions 51 and 52
by others s u ne otion l n et che . You
woul like to ev lu te this tient’s erson lity
The tient is 31-ye r-ol , ivorce le, w ithou t
style w ith rojective test.
rior sychi tric history, referre by his w ork for
ev lu tion. H e st tes he h s felt “ ow n” since his
47. You w oul like to quickly ssess for oten- se r tion n ivorce sever l onths go. H e h s
ti l neurocognitive isor er in 75-ye r-ol been slee ing oorly, frequ ently w king u u ring
wo n itte to the e ergency e rt ent the night, then feeling exh u ste u ring the y.
for f ilure to thrive. He h s h little interest in foo , losing 10 lb, n
he h s been e sily istr cte t w ork, resu lting in
48. A 45-ye r-ol w o n h s long history of ist kes n re ri n . While he feels hel less
u nst ble rel tionshi s, self-injuriou s beh vior, n th t “I’ll never feel better,” he enies ny su i-
n ffective inst bility. She oes not eet cri- ci l i e tion or ho ici l i e tion. H e rinks 1 to
teri for jor e ressive, bi ol r, or nxi- 2 beers on the w eeken s n enies illicit ru g u se.
ety isor er. You w ish to e loy n objective H e s okes 1/ 2 ck of cig rettes er y n h s
test to give fu rther evi ence for erson lity no jor e ic l roble s exce t for bor erline
isor er. hy ertension. H e t kes u ltivit in bu t no other
e ic tions regul rly. After fu rther iscu ssion, he
grees to begin tri l of e ic tion. The tient
49. You re ske to reo er tively ev lu te is begu n on flu oxetine 20 g ily. H e is co li nt
he is heric o in nce in n 18-ye r-ol w ith the e ic tion n follow s u regu l rly. After
left-h n e w o n w ith history of seizu re 4 w eeks he re orts slight i rove ent in his sy -
isor er w ho is bou t to u n ergo su rgery to to s. Of note, he initi lly h nxiety n loose
re ove seizu re focu s in her left he is here. stools, bu t he enies current si e effects.

DIRECTIONS (Questions 50 through 83): For each 51. Which of the follow ing w ou l be the ost
of the multiple-choice questions in this section select ro ri te next ste ?
the lettered answer that is the one best response in
(A) A ri i r zole.
each case.
(B) A lithiu .
50. A 16-ye r-ol girl is brou ght to you r office (C) Discontinu e the flu oxetine.
by her rents to get regn ncy test. She (D) Incre se flu oxetine to 40 g.
consents for the ev lu tion bu t requ ests th t (E) Sw itch to sertr line.
you kee the resu lts confi enti l. Bet –hu n
256 8: Pra c tic e Te s t 2

52. H e eventu lly chieves fu ll re ission of his (D) Refer her for electroconvu lsive ther y
e ressive sy to s n continu es on the (ECT).
e ic tion. H ow ever, he is concerne bou t (E) Tell her to sto her e ic tion
his rognosis n otenti lly sto ing the i e i tely.
nti e ress nt in the fu tu re. Wh t o you tell
hi is his roxi te risk of evelo ing 55. A 45-ye r-ol n is brought to the sychi tric
fu rther e iso e in the fu tu re? e ergency roo by olice fter being foun
(A) 10% scre ing w hile cutting ow n tree in the loc l
rk. H e tells you th t the ghost of his w ife,
(B) 20%
w ho ie 3 onths go, is living in the tree.
(C) 30% The n h s no revious sychi tric history,
(D) 40% but he re orts being very e resse since the
(E) 50% e th of his w ife n h s h suici l i e tion.
H e is ishevele n e rs to h ve ignore
Questions 53 and 54 his erson l hygiene for so e ti e. H is vit l
signs re nor l. Which of the follow ing is the
A 35-ye r-ol w o n resents to you r office w ith
ost ro ri te next course of ction?
referr l fro sychologist for sychi tric tre t-
ent. She h s been su ffering “the blues” for sever l (A) Obt in u rine toxicology screen.
onths, inclu ing crying s ells, low energy, n (B) Prescribe n nti e ress nt.
hy erso ni . She is convince th t her fi ncé is (C) Prescribe n nti sychotic.
bout to bre k u w ith her bec use she h s been e t- (D) Prescribe n nti sychotic n n
ing ore th n usu l. So eti es, how ever, she feels nti e ress nt.
“ju st fine.”
(E) Prescribe oo st bilizer.
53. Wh ich of th e follow in g w ou l be th e ost
Questions 56 and 57
ro ri te tre t en t of choice for th is
tient?
You re seeing 56-ye r-ol n w ith history of
(A) A itri tyline (El vil) lcohol u se isor er. H e co l ins bou t not being
(B) Div l roex so iu (De kote) ble to slee n requ ests slee ing i . H e enies
(C) Flu oxetine (Proz c) erv sive e ression or ny ch nge in his etite,
energy, or concentr tion. H e cl i s th t otherw ise
(D) Phenelzine (N r il)
“everything is going w ell.”
(E) Zi r si one (Geo on)
56. Which of the follow ing e ic tions w ou l
54. The tient res on s w ell to tre t ent, n
be the ost ro ri te to rescribe for this
fter sever l onths retu rns to you r office for
tient?
follow -u visit. She tells you she h s just
been rrie n w ou l like to beco e reg- (A) Al r zol (X n x)
n nt. She n her hu sb n h ve been h ving (B) Di ze (V liu )
u n rotecte sex for onth. Which of the fol- (C) Tr zo one (Desyrel)
low ing w ou l be the ost ro ri te next (D) Z le lon (Son t )
cou rse of ction?
(E) Zol i e (A bien)
(A) A nother nti e ress nt to revent
ost rtu e ression. 57. H e grees to begin the ro ri te bove e -
(B) Ask how i ort nt regn ncy is to her. ic tion. Which of the follow ing si e effects
(C) Discu ss the risks n benefits of w oul be ost i ort nt to w rn hi bout?
continu ing her tre t ent u ring (A) Anorg s i
regn ncy. (B) I otence
Que s tions : 52–61 257

(C) Incontinence Questions 60 and 61


(D) Pri is
A 38-ye r-ol tient w ith history of bi ol r isor-
(E) Retrogr e ej cul tion er h his first ( n only) nic e iso e 18 onths
go. H e w s tre te for 1 ye r w ith oo st bi-
Questions 58 and 59 lizer, n h s not t ken ny sychotro ics since th t
58. You re seeing 9-ye r-ol boy w ith history ti e. H e h s co e in for n ev lu tion t the urging
of e ression n suici l i e tion. H e h s of his w ife, w ho is concerne bout his frequent cry-
been t king flu oxetine (Proz c) for 6 onths ing s ells. The tient its th t he h s felt ore
n h s h goo res onse. H is other e resse over the l st sever l w eeks, w ith ssoci-
sks to see you fter his oint ent n tells te inso ni , low energy, oor etite n 5-lb
you th t she is concerne bou t new s re orts w eight loss, ifficu lty concentr ting, n nhe o-
s ying th t serotonin-s ecific reu t ke inhi- ni . H e enies ny su ici l i e tion. Fortu n tely,
bitors (SSRIs) incre se the risk of su ici e in these sy to s h ve not interfere w ith his w ork.
chil ren. She sks you to consi er sto ing H e h s no jor e ic l roble s n is on no
the e ic tion. Which of the follow ing is the e ic tions. H e rinks one to tw o gl sses of w ine
best cou rse of ction? sever l ti es er ye r n enies illicit ru g u se.

(A) Ask the other, “Wh t’s re lly 60. Which of the follow ing e ic tions w ou l be
bothering you bou t you r son being on the ost ro ri te to rescribe this tient?
nti e ress nts?”
(A) Bu ro ion
(B) Discu ss the risks n benefits of
flu oxetine n other SSRIs in the (B) Flu oxetine
e i tric o u l tion. (C) H lo eri ol
(C) Sto the flu oxetine i e i tely s the (D) L otrigine
tient is inor n it is his other’s (E) V l roic ci
w ish.
(D) Sw itch to itri tyline (El vil). 61. Which of the follow ing si e effects w ou l be
(E) Sw itch to roxetine (P xil). the ost i ort nt to iscu ss w ith the tient
rior to st rting?
59. A onth l ter, the boy n his other return to (A) At xi
your office. She infor s you th t fter her son’s (B) N u se
l st oint ent, she h s been giving hi
(C) N eu tro eni
St. John wort bec use she believes he nee s
“n tur l re e y.” Which of the following (D) R sh
woul be the ost ro ri te course of ction? (E) Se tion

(A) Incre se the ose of his nti e ress nt Questions 62 and 63


s St. John w ort w ill low er bloo
concentr tions of the nti e ress nt. A 60-ye r-ol w hite fe le w i ow is brought to you r
(B) Refer the c se to soci l services for chil clinic by her ughter, w ho re orts th t her other
bu se. s i to her this fternoon th t she “w nte to en it
(C) Tell the other there is no evi ence th t ll.” She re orts thou ghts of over osing on her ills,
St. John w ort is n effective tre t ent w hich she h s stock ile . This tient h s no rior
for e ression. history of su ici e tte ts bu t h s h thou ghts of
su ici e on ulti le rior occ sions over the cou rse
(D) Tell the other to sto the St. John w ort
of her ne r-lifelong history of e ression. She lost
i e i tely.
her hu sb n to su en he rt tt ck 1 ye r go, but
(E) Tell the other to w tch ou t for si e her u ghter s ys th t she still h s ny very su -
effects inclu ing hotosensitivity n ortive frien s w ith w ho she s en s signific nt
izziness.
258 8: Pra c tic e Te s t 2

ou nts of ti e. She still c rries on her vi hobby (C) 25%


of g r ening, lthough t slow er ce bec u se of (D) 50%
her w orsening rthritis. She ex resses w orries over (E) 75%
recent “big ro ” in the v lu e of retire ent stocks
she h s. 66. A 24-ye r-ol gr u te stu ent resents to her
University He lth C re Syste w ith the chief
62. Which of the follow ing is the ost signific nt co l int of “ e ression.” U on et ile ques-
f ctor itig ting her risk for su ici e? tioning she reve ls history of intense oo
(A) G r ening hobby sw ings since e rly olescence, w ith short
(B) N o rior su ici e tte ts (hours to ys) erio s of ys hori , ltern t-
ing w ith ti es w here she feels “nor l.” She
(C) R ce
its th t she is e sily ngere , occ sion lly
(D) Slow er ove ents secon ry to rthritis bre king ite s n su erfici lly cutting on her-
(E) Soci l su orts self w hen or bore . She h s been un ble to
kee boyfrien for ore th n sever l onths,
63. Which of the follow ing e ic l con itions but st tes th t she w ill frequently rtici te
w ou l be the ost i ort nt to ru le ou t s in “one-night st n s” bec use she often feels
otenti l c u se of e ression in this tient? lonely n e ty. She enies ersistent inso -
(A) H y ercholesterole i ni , nergi , or nhe oni , lthough gets e s-
ily istr cte w hile in lectures. While w hen
(B) Phos holi i et bolis isor ers
cutely u set she y feel ssively suici l,
(C) Thin b se ent e br ne ise ses she enies ny st or current l n. Which of
(D) Thyroi illness the follow ing tre t ents w oul be the ost
(E) Try to h n eficiency ro ri te to begin for this tient?
(A) Ari i r zole
Questions 64 and 65
(B) Cit lo r
A 20-ye r-ol college so ho ore is brou ght to the (C) Di lectic l beh vior l ther y (DBT)
e ergency e rt ent fter being fou n sse ou t
(D) Lithiu
in her b throo . She h been vo iting, w hich she
(E) V l roic ci
its now w s self-in u ce . H er history is not ble
for bre ku 4 onths go w ith her boyfrien of
67. A 24-ye r-ol n w ith history of schizo-
2 ye rs. H er w eight is in the 82n ercentile for her
hreni resents to the e ergency e rt-
height. She is enorrheic. There is no history of
ent. H e h reviou sly t ken h lo eri ol
binge e ting.
(H l ol), but now refuses to t ke it. Bec use of
this, you eci e to try flu hen zine (Prolixin),
64. Which of the follow ing o you ost ex ect to
w hich see s to itig te his sy to s. One
fin on her hysic l ex in tion?
w eek l ter, he le ves the hos it l. H e contin-
(A) C fé u l it s ots ues the e ic tions you rescribe n on
(B) Dent l ec y follow -u visit you notice he oves slow ly
(C) P l ble s leen n h s festin ting g it. Block e of w hich
(D) Pectus exc v tu of the follow ing rece tors ost likely ccounts
for the bove si e effect?
(E) Pitting fingern ils
(A) B s l g ngli cetylcholine rece tors
65. Wh t is the likelihoo th t with tre t ent she (B) D 4 rece tors
will ke full recovery over the next 10 ye rs? (C) 5-hy roxytry t ine-2 (5-H T2)
(A) 5% (D) Mesoli bic D 2 rece tors
(B) 10% (E) N igrostri t l D 2 rece tors
Que s tions : 62–74 259

68. A 16-ye r-ol boy beg n h ving re etitive eye (D) Psycho yn ic sychother y
blinking t the ge of 6. By ge 12, he beg n (E) Pu nish ent
blu rting stereoty e hr ses su ch s “B lls!”
n “Shitty!” s w ell s king kissing noises. 71. The chil un ergoes lengthy tri l of the rec-
Conco it nt w ith this, he h incre sing if- o en e ther y s bove, but his con ition
ficu lty w ith ying ttention in school n continues without signific nt i rove ent.
occ sion lly is l ye w il , isinhibite Which of the following woul be the ost
beh vior. Which of the follow ing w ou l be ro ri te h r cologic tre t ent of choice?
the ost effective tre t ent for the ri ry
(initi l) con ition? (A) C rb ze ine (Tegretol)
(B) Cloni ine (C t res)
(A) Cloni ine (C t res)
(C) Des o ressin (D v )
(B) Flu oxetine (Proz c)
(D) Di henhy r ine (Ben ryl)
(C) Methyl heni te (Rit lin)
(E) I i r ine (Tofr nil)
(D) Ris eri one (Ris er l)
(E) To ir te (To x) Questions 72 and 73

69. A 12-ye r-ol boy h s recently been rreste An 8-ye r-ol boy is referre by his e i trici n for
for setting fire to neighbor ’s b rn. H e h s fu rther ev lu tion. H is history is not ble for being
been in trou ble before for l ying w ith fire- o te t ge 6 onths. H is f ily history is sig-
w orks in the neighborhoo , n w hen he w s nific nt for biologic l other w ith severe lcohol
10 ye rs ol he w s su s en e for lighting use isor er. H e h s stru ggle w ith ll of his su b-
tches t his school. Which of the follow ing jects n h s been in s eci l e u c tion cl sses s
i gnoses w ou l ost likely be co orbi in resu lt. H e cts ore concrete th n his eers, n h s
this tient? few frien s. Of note, his intelligence qu otient (IQ)
h s recently been e su re t 60.
(A) Au tis s ectru isor er
(B) Con u ct isor er 72. Which of the follow ing w ou l be this chil ’s
(C) Intellectu l is bility ost ro ri te i gnosis?
(D) M jor e ressive isor er (A) Au tistic s ectru isor er
(E) Tic isor er (B) Mil intellectu l is bility
(C) Mo er te intellectu l is bility
Questions 70 and 71
(D) Severe intellectu l is bility
A 9-ye r-ol boy w ith history of n or l evel- (E) Profou n intellectu l is bility
o ent l ilestones h s been w etting his be for
the l st 6 on ths. H is ren ts re ort th t th e be 73. Given the bove tient’s history, w hich of the
w etting initi lly occu rre s or ic lly, bu t for the follow ing hysic l fin ings w ou l ost likely
l st 2 onths it h s been h enin g bou t every be ex ecte ?
oth er n ight. This h s c u se hi sign ific nt
ou nt of sh e n h s been li iting his soci l (A) Cleft l te
inter ctions s he is u n ble to go to slee overs or (B) Congenit l blin ness
w y to c . (C) H y erextensible joints
(D) Microce h ly
70. Which of the follow ing interventions w ou l (E) Pro inent j w
be the ost ro ri te?
(A) Beh vior l ther y 74. A 5-ye r-ol girl h s co e un er the c re of
(B) Cognitive ther y st te Chil Protection Services. H er other w s
know n to be using cr ck coc ine n IV heroin,
(C) Di lectic l beh vior l ther y
n su orting her h bit w ith rostitution.
260 8: Pra c tic e Te s t 2

The girl’s te or ry foster rents re orte been ski ing cl sses n not turning in ssign ents
to st te w orkers th t she w s in iscri in tely s he believes he w ill u to tic lly gr u te n
soci l w ith u lts bu t often oesn’t e r to ss the b r bec use of his bilities n intelligence.
seek or res on to co forting. In her soci l
inter ctions w ith other 5-ye r-ol chil ren t 76. Which of the follow ing w ou l be the ost
her reschool, she is note to often st y by her- likely i gnosis for this tient?
self but is escribe s being in ro ri tely
(A) Bi ol r isor er, cu rrent e iso e
ngry w ith her eers. Which of the follow ing
e resse
w ou l be the ost ro ri te i gnosis in
(B) Bi ol r isor er, cu rrent e iso e nic
this c se?
(C) Cyclothy ic isor er
(A) Attention- eficit/ hy er ctivity isor er (D) Persistent e ressive isor er
(ADH D) ( ysthy i )
(B) Au tis s ectru isor er (E) Schizo ffective isor er, bi ol r ty e
(C) Con u ct isor er
(D) L ngu ge isor er 77. Which of the follow ing w ou l be the ost
(E) Re ctive tt ch ent isor er ro ri te h r cologic tre t ent for this
tient?
75. A 39-ye r-ol n is rreste fter being (A) Cit lo r (Celex )
fou n in e rt ent store, stu rb ting by
(B) Div l roex so iu (De kote)
ru bbing w o n’s shoe long his enis. H e
relu ct ntly its th t he h s co itte this (C) Flu oxetine
ct on nu erou s occ sions lthou gh h s not (D) I i r ine
gotten c u ght or rreste . H e feels gre t e l (E) Lor ze (Ativ n)
of sh e n guilt, bu t he is u n ble to sto the
beh vior. Which of the follow ing is the ost Questions 78 and 79
likely i gnosis?
A 36-ye r-ol w o n resents w ith 7 w eeks of
(A) Fetishistic isor er e resse oo ost of the ti e ( lthou gh w s
(B) Frotteu ristic isor er h y for short erio w hen she w on $150 in the
(C) Pe o hilic isor er lottery), sense of ho elessness, n thou ghts of
(D) Tr nsvestic isor er su ici e for the l st onth. She lso re orts incre se
etite n h s been slee ing u to 16 hou rs y.
(E) Voyeu ristic isor er
On ent l st tus ex in tion, her oo n ffect
re ys horic ost of the ti e, but she l u ghs in
Questions 76 and 77
res onse to hu orou s st te ents.
A 27-ye r-ol l w stu ent resents to you s
referr l for the tre t ent of first e iso e of jor 78. Which of the follow ing i gnoses w ou l be
e ression, initi lly i gnose by his ri ry c re the ost ro ri te in this tient?
hysici n 8 w eeks go. On his ent l st tu s ex i- (A) Bi ol r I isor er, cu rrent e iso e
n tion, the tient’s s eech is extre ely r i , if- e resse
ficult to interru t, n he quickly ju s fro one
(B) Bi ol r II isor er, cu rrent e iso e
to ic to nother. H e s ys th t his oo h s i rove
e resse
gre tly since he l st s w his ri ry c re octor: “In
f ct, I’ on to of the w orl .” The tient re orts 3 to (C) M jor e ressive isor er, w ith ty ic l
4 hou rs of slee er night for the l st w eek, “not th t fe tu res
I nee th t uch slee nyw y.” The tient h s (D) M jor e ressive isor er, w ith
been s en ing ll of his oney buying first e ition el ncholic fe tu res
textbooks n re rr nging the fu rniture in the l w (E) Persistent e ressive isor er
school inistr tion buil ing. H ow ever, he h s ( ysthy i )
Que s tions : 75–82 261

79. Which of the follow ing cl sses of e ic tions the hos it l, n she h s been telling her f -
w ou l be the ost effective in this tient? ily bout her concerns reg r ing the govern-
ent controlling her for the l st 2 w eeks. The
(A) Anticonvu ls nts
tient’s st sychi tric history is signific nt
(B) Lithiu for three rior hos it liz tions over the l st
(C) Mono ine oxi se inhibitors (MAOIs) 20 ye rs. At the ti e of her first ission,
(D) Tricyclic nti e ress nts (TCAs) she resente w ith 3-w eek history of nic
(E) Ty ic l (first-gener tion) nti sychotics sy to s n sychosis. During her secon
hos it liz tion, she w s suici l n note to
80. A 42-ye r-ol ccount nt resents for long- be e resse w ith sychotic sy to s. During
ter sychother y tre t ent. In the first ses- her thir ission, she w s g in e resse
sion, you fin th t he h s been ivorce three w ith ro inent isorg nize beh vior. Her
ti es. The oor qu lity of his ro ntic n ent l st tus ex in tion u on resent tion
other rel tionshi s re the focus of his concerns. to the hos it l reve ls looseness of ssoci tion
Over the course of onths, you fin out th t he n elusion l sy to s w ithout ro inent
is erfectionist n inflexible w ithin rel tion- oo co onent. Which of the follow ing is
shi s. He ty ic lly i e lizes his rtner, then the ost ro ri te i gnosis for this tient?
ev lues her, before en ing the rel tionshi (A) Bi ol r isor er, w ith sychotic fe tu res
su enly. At ti es, he loses “ sense of self” in
(B) M jor e ressive isor er (MDD) w ith
rel tionshi s, not know ing w ho he is or w h t
sychotic fe tu res
he w nts. During this ti e, he relies on others
to vise hi on w h t he shoul o. H ow ever, (C) Schizo ffective isor er
if he t kes this vice n his l ns f il, he uses (D) Schizo hreni
this to ev lue the one w ho vise hi . The (E) Uns ecifie sychotic isor er
tient kes it very cle r th t he sought your
tre t ent s sychother ist “bec use I’ll Questions 82 and 83
only cce t the best in the city.” The tient lso
The e ic l te on n in tient service c lls you
re orts to you th t he h s so e unique bilities
to hel the w ith 72-ye r-ol fe le tient w ho
of erce tion th t llow hi to etect w h t or
w s itte for congestive he rt f ilu re. They
w ho w ill rovi e the best ossible outco e for
re ort th t t night she beco es confu se , think-
hi . For ex le, he icke you s ther ist
ing th t she is in hotel. She kes u nre son ble
not only bec use of your re ut tion, but lso
e n s of the nu rsing st ff n beco es isru -
bec use of your office hone nu ber, w hich
tive on the w r , interfering in the nu rses’ w ork.
h s igits in scen ing v lue or er. Which of
She h s the ost ifficu lty fro 10 pm u ntil bou t
the following i gnoses is the ost ro ri te
2 a m, w hen she u su lly f lls slee . In the orning,
for this tient?
she is better oriente n u ch ore coo er tive. A
(A) Bor erline erson lity isor er review of her e ic tions reve ls th t she is t king
(B) N rcissistic erson lity isor er ci eti ine 400 g PO bi , fu rose i e 40 g PO q ,
(C) Obsessive erson lity isor er tenolol 50 g PO q , igoxin 0.125 g PO q , n
i henhy r ine 50 g PO qhs n 25 g PO qhs
(D) Other s ecifie erson lity isor er
(secon ose 3 hou rs ost first ose for inso ni ).
(E) Schizoty l erson lity isor er
82. Which of the follow ing e ic tions is ost
81. A 43-ye r-ol w o n is hos it lize uring likely to c u se this tient’s confu sion?
sychotic e iso e in w hich she h the elu-
sion th t she is being controlle w ith co - (A) Atenolol
uter rogr evelo e by the De rt ent (B) Ci eti ine
of Ho el n Security. Accor ing to her f ily, (C) Digoxin
she sto e t king her regi en of queti ine (D) Di henhy r ine
n lithiu 4 w eeks before this ission to (E) Fu rose i e
262 8: Pra c tic e Te s t 2

83. Which of the follow ing neu rotr ns itter sys- bu t she re ins sychotic, w ith u itory
te s is ost irectly i lic te in the etiol- h llu cin tions telling her to kill herself.
ogy of her confu sion?
(A) Acetylcholine 87. A 53-ye r-ol rrie le with history of
(B) Do ine jor e ressive isor er n oor co li nce
ue to signific nt sexu l ysfunction now res-
(C) G - inobu tyric ci (GABA)
ents with recurrence of e ressive sy to s
(D) Serotonin with rke hy erso ni n low energy.
(E) N ore ine hrine
88. A 22-ye r-ol fe le gr u te stu ent w ith
D IRECTION S (Questions 84 through 89): The fol-
bi ol r isor er beco es regn nt.
low ing question is preceded by a list of lettered
options. For each of the questions in this section
match the clinical vignette with the most appropriate 89. A 37-ye r-ol n w ith history of nic
medication to prescribe. e iso es now h s 3 w eeks of e ression, w ith
inso ni , oor etite, w eight loss, low
M tch the clinic l vignette w ith the ost ro ri te energy, n ecre se concentr tion.
e ic tion to rescribe.
DIRECTIONS (Questions 90 through 112): For each
(A) Ari i r zole
of the multiple-choice questions in this section select
(B) Bu ro ion
the lettered answer that is the one best response in
(C) Chlor ro zine each case.
(D) Cloz ine
(E) H lo eri ol 90. A clinic l rese rch sychi trist t u niversity
(F) L otrigine A h s been running ou ble-blin l cebo-
(G) Lithiu controlle tri l of new nti e ress nt.
The sychi trist conclu es fter st tistic l
(H ) N ortri tyline
n lysis of the t th t there is no better
(I) P roxetine effect of the ru g co re to l cebo. The
(J) Ris eri one sychi trist’s colle gu es t five se r te
(K) Ris eri one Const institu tions (u niversities B, C, D, E, n F) h ve
(L) Sertr line ru n i entic l stu ies n eter ine th t there
(M) V l roic ci is tre t ent effect of the ru g co re
to l cebo. Given this infor tion, one y
(N ) Venl f xine
conclu e th t the rese rch tri l t u niversity A
resu lte in w hich of the follow ing?
84. A 32-ye r-ol n with schizo ffective isor er
h s goo benefit with nti sychotic e ic tions (A) H igh v ri nce
but is oorly co li nt ue to l ck of insight (B) Low - re ictive v lu e
n re uce cognition. (C) St n r error
(D) Ty e I error
85. A 48-ye r-ol ivorce w o n h s history (E) Ty e II error
of schizo hreni n is co li nt w ith her
e ic tion, bu t she resents to the e er- Questions 91 and 92
gency roo fter evelo ing severe su nbu rn
es ite only being ou tsi e for sever l hou rs. A 56-ye r-ol n w ith sever l ec e history
of schizo hreni h s been t king nu erou s if-
86. A 41-ye r-ol w o n w ith chronic schizo hre- ferent nti sychotics, lbeit regu l rly, for 27 ye rs.
ni h s been trie on sever l first- n secon - H is sy to s re re son bly w ell controlle , w ith
gener tion nti sychotics t ther eu tic oses, occ sion l, nonco n h llu cin tions n so e
Que s tions : 83–96 263

r noi . H e h s been ble to w ork s volu nteer 94. A 29-ye r-ol rrie n resents to you r
n lives in grou ho e. Abou t 5 ye rs go, he out tient sychi tric clinic co l ining of
evelo e w rithing ove ents of his w rists n nxiety n e ression. H is soci l history
fingers th t is e r w hen he goes to slee . reve ls th t he is ju nior f cu lty e ber of
loc l u niversity, w orking bout 60 hours er
91. Which of the follow ing si e effects is ost w eek. H e enjoys his job but re orts stress ris-
consistent w ith this c se? ing fro his rit l rel tionshi bec use his
w ife ccuses hi of not s en ing “qu lity
(A) Ak thisi
ti e” w ith her n his being “ rrie to the
(B) Dystoni job.” Accor ing to Erik Erikson, in w hich st ge
(C) N eu role tic lign nt syn ro e (N MS) of evelo ent oes this tient currently
(D) P rkinsonis h ve conflict?
(E) T r ive yskinesi (A) Au tono y versu s sh e n ou bt
(B) I entity versu s role iffu sion
92. Which of the follow ing n to ic stru ctu res
is ost likely i lic te in the etiology of his (C) In u stry versu s inferiority
ove ents? (D) Inti cy versu s isol tion
(E) Tru st versu s istru st
(A) B s l g ngli
(B) Cerebellu Questions 95 and 96
(C) Front l cortex
(D) Mi br in A 46-ye r-ol n resents w ith e iso es of “bl ck-
ing out,” w here he w ill not re e ber erio s l sting
(E) Motor cortex
for sever l inutes. During those ti es, he is tol by
others th t he h s n o ex ression on his f ce n
93. You re seeing 66-ye r-ol Vietn veter n
oesn’t res on to his n e. In ition, he co -
t Veter n’s A inistr tion hos it l. H e
l ins of ifficulty in rel tionshi s u e to ecre se
h s been hos it lize nu erous ti es w ith
sex rive n irrit bility. H e s en s gre t e l of
si il r resent tions. H e co l ins of ongo-
ti e w riting ow n his thoughts in his e oirs.
ing night res reg r ing fighting in Vietn ,
w ith frequ ent fl shb cks, intru sive thou ghts,
95. Which of the follow ing ition l sy to s
oor slee , n incre se st rtle. Other th n
w ou l be ost likely resent in this tient?
co ing in for his oint ents, he h s been
u n ble to go out in ublic bec u se he often (A) De ression
feels th t he is u n er tt ck, fe ring th t so e- (B) H y erreligiosity
one is going to ju ou t of the bu shes n (C) Obsessive-co u lsive beh viors
“ bush” hi . H e h s been tre te w ith the
(D) Slee isor ers
follow ing e ic tions, ll w ith only o er-
te i rove ent in his sy to s: flu oxetine, (E) Urin ry incontinence
sertr line, roxetine, cit lo r , irt z -
ine, i i r ine, n itri tyline. H e h s 96. A slee - e rive electroence h logr is
been co li nt w ith venl f xine 375 g ily. erfor e on the tient s rt of his co -
H e h s been sober fro lcohol n heroin for lete w orku . Which of the follow ing lobes
20 ye rs. Which of the follow ing e ic tions of the br in w oul ost likely e onstr te
w ou l be the ost benefici l for this tient? bnor lities in this tient?

(A) Bu ro ion (A) Front l


(B) H lo eri ol (B) Cerebell r
(C) Lor ze (C) Occi it l
(D) Pr zosin (D) P riet l
(E) V l roic ci (E) Te or l
264 8: Pra c tic e Te s t 2

97. After being c lle to the e ergency e rt- (A) Beets


ent to ev lu te you ng w o n w ho over- (B) Li be ns
ose on b rbitu r tes, you fin her in be w ith (C) Liver
her eyes close . She o ens her eyes briefly in
(D) Pe nu ts
res onse to in n e onstr tes flexion
fro in bu t kes no sou n s. Which of the (E) White w ine
follow ing best re resents her Gl sgow Co
Sc le score? 100. A 66-ye r-ol n w ith history of schizo hre-
ni h been int ine on thiori zine (Mel-
(A) 7 l ril) 700 g q for ny ye rs, fter nu erous
(B) 6 unsuccessful tri ls w ith v rious nti sychotics.
(C) 5 Over the l st 8 onths, he h s been co l in-
(D) 4 ing of he ring erog tory voices, n his ose
h s been gr u lly incre se to 1,000 g q .
(E) 3
H e h s no signs of t r ive yskinesi . Which
of the follow ing si e effects w oul be the ost
Questions 98 and 99
concerning t this ose?
You h ve been tre ting 34-ye r-ol n for jor (A) Consti tion
e ression w ith serotonin-s ecific reu t ke inhibi-
(B) Dry eyes
tor (SSRI) for the l st 2 onths. H e w s initi lly
st rte on flu oxetine 20 g/ for the first onth, (C) N e hrogenic i betes insi i u s
n then you r ise the ose to 40 g/ for the l st (D) Pig ent ry retino thy
onth. While he h s toler te the e ic tion w ith- (E) Urin ry retention
out signific nt si e effects, his e ressive sy to s
h ve re ine refr ctory es ite the incre se in os- 101. A 24-ye r-ol n w ithou t rior sychi tric
ge. You h ve eci e to sw itch his e ic tion to history is itte to the sychi tric w r
henelzine (N r il), ono ine oxi se inhibitor w ith isorg niz tion, r noi , git tion,
(MAOI), in ho es of eliciting better nti e ress nt n co n u itory h llu cin tions tell-
res onse. ing hi to kill hi self. H e is given tw o oses
of h lo eri ol 5 g or lly. A roxi tely
98. Which of the follow ing w ou l be the best 8 hou rs l ter, he evelo s torticollis. Which of
str tegy in king the tr nsition fro the the follow ing w ou l be the ost ro ri te
SSRI to the MAOI? tre t ent?
(A) Begin t ering the ose of the SSRI (A) Acet ino hen
w hile incre sing the ose of the MAOI (B) Benztro ine
si ult neou sly. (C) Cyclobenz rine
(B) St rt the MAOI u ntil ther eu tic levels (D) Ibu rofen
h ve been re che , then t er the SSRI.
(E) Pro r nolol
(C) Sto the SSRI bru tly n begin
the MAOI t n equi otent ose the 102. A 28-ye r-ol w o n w ith history of bi o-
follow ing y. l r isor er is itte to the e ic l service
(D) Sto the SSRI bru tly n i e i tely bec u se of w e kness, ent l st tu s ch nges,
begin the MAOI. n seru so iu of 154 ol/ L. Accor -
(E) T er the SSRI n 5 w eeks fter the ing to her boyfrien , for the l st 2 w eeks she
l st ose begin the MAOI. h s been rinking “lo s n lo s of w ter”
bu t co l ining of being thirsty often, n
99. After he is sw itche to the henelzine, w hich lso frequ ently u rin ting. Which of the follow -
of the follow ing foo s/ rink shoul the ing e ic tions w ou l be ost likely res on-
tient ost voi ? sible for this tient’s resent tion?
Que s tions : 97–107 265

(A) C rb ze ine enies ny suici l i e tion. H is st e ic l his-


(B) H lo eri ol tory is signific nt for i betes ellitu s, tw o yo-
(C) Lithiu c rbon te c r i l inf rctions, tri l fibrill tion, n coron ry
rtery by ss gr ft. The tient’s w ife w ishes hi to
(D) Qu eti ine
st rt e ic tion.
(E) V l roic ci
105. Which of the follow ing nti e ress nts shou l
Questions 103 and 104 be ost voi e in this rticu l r tient?
A 58-ye r-ol n w ith long history of o i te u se (A) Bu ro ion
isor er h s been u sing intr venou s heroin ily,
(B) I i r ine
roxi tely $50–60 er y. H e h s not been
(C) Mirt z ine
ble to int in ny signific nt sobriety es ite
nu erous in tient n resi enti l reh bilit tion (D) Sertr line
rogr s. H e l st u se heroin 12 hou rs go n h s (E) Venl f xine
su bsequ ently been itte to the in tient sychi-
tric w r for etoxific tion. 106. Which of the follow ing nti e ress nts w ou l
ost likely resu lt in cognitive ecline in this
103. Which of the follow ing e ic tions w ou l be tient?
the ost ro ri te to tre t his w ith r w l (A) Bu ro ion
sy to s?
(B) Cit lo r
(A) A nt ine (Sy etrel) (C) Flu oxetine
(B) Cloni ine (D) P roxetine
(C) H lo eri ol (E) Venl f xine
(D) Lor ze
(E) N loxone 107. A 35-ye r-ol w o n w ith bi ol r isor er
n frequ ent hos it liz tions resents to the
104. After the tient co letes his etoxific tion e ergency e rt ent fter t king n over-
on the u nit, he continu es to be otiv te to ose of lithiu in su ici e tte t. Which
bst in fro heroin u se. H e is referre to n of the follow ing signs w ou l be ost likely to
out tient ictions rogr , bu t w ishes occur?
to t ke e ic tion to ecre se his heroin (A) Ab o in l in
u se fter isch rge. Which of the follow ing
(B) Acu te ystoni
e ic tions w ou l be the ost ro ri te to
rescribe? (C) Leg in
(D) P r noi elu sions
(A) D ntrolene
(E) P resthesi s
(B) Disulfir
(C) Flu zenil Questions 108 and 109
(D) Meth one
(E) Sertr line A 67-ye r-ol w o n resents to you r office w ith
long history of jor e ression. She h s trie
Questions 105 and 106 nu ber of ifferent nti e ress nt e ic tions
in the st bu t h s not h re ission w ith ny
A 75-ye r-ol n w ith jor neu rocognitive is- of the . Currently, she is on lithiu , venl f xine
or er u e to Alzhei er ise se resents w ith his (Effexor), nortri tyline (P elor), lor ze , ris-
w ife to you r clinic for ev lu tion. She is concerne eri one, n benztro ine. She is st rting to believe
bec u se he h s been “ ow n” recently, co l ining th t her next- oor neighbors re ou t to h r her.
of e ression. H is slee h s been w orse, n his You eci e to begin tri l of electroconvulsive ther-
etite n energy h ve been oor. The tient y (ECT).
266 8: Pra c tic e Te s t 2

108. Which of the follow ing ini u ti es shou l 112. A 45-ye r-ol w o n w ith rior history of
the seizure l st for o ti l effectiveness in this jor e ression is itte to the sychi-
tient? tric in tient w r . She h s been severely
e resse , w ith oor slee , ecre se e-
(A) 10 secon s
tite, low energy, n su ici l i e tion. She
(B) 15 secon s now believes th t her neighbors re cons ir-
(C) 60 secon s ing to u r er her, n occ sion lly c n he r
(D) 120 secon s n’s voice th t tells her she shou l ie.
(E) 180 secon s After s e king w ith the neighbors, you fin
ou t th t they brou ght her to the hos it l. They
109. Which of the follow ing verse effects w ou l h ve been goo frien s for 20 ye rs n h ve
be the ost likely u ring the cou rse of ECT? notice the tient h s not been le ving the
hou se uch, h s t ken to looking out of her
(A) Br in ge w in ow s w ith su s iciou s gl nces, n h s not
(B) C t toni cle ne her hou se in the l st 2 onths. Which
(C) Fr ctu res of the follow ing e ic tion co bin tions
(D) Me ory i ir ent w ou l be the ost ro ri te tre t ent for
(E) Vo iting this tient?
(A) Flu oxetine n ris eri one
Questions 110 and 111
(B) Lor ze n cloz ine
(C) P roxetine n thyroi u g ent tion
A 26-ye r-ol n w ithou t rior sychi tric history
resents to sychi tric e ergency e rt ent (D) Sertr line n lithiu
w ith r noi , visu l h llu cin tions, feelings of (E) Sertr line n lor ze
u nre lity, e erson liz tion, n extre e git tion.
A u rine toxicology screen is ositive for hencycli- Questions 113 through 116
ine (PCP).
M tch the clinic l vignette w ith the ost likely
su bst nce-in uce isor er. E ch lettere o tion
110. Which of the follow ing w ou l be the best
y be u se once, u lti le ti es, or not t ll.
i e i te tre t ent for the git tion in this
tient? (A) Alcohol intoxic tion
(A) Chlor ro zine (B) Alcohol w ith r w l
(B) Flu oxetine (C) C nn bis intoxic tion
(C) Lor ze (D) Coc ine intoxic tion
(D) Tr zo one (E) Coc ine w ith r w l
(E) Trihexy heni yl (Art ne) (F) H eroin intoxic tion
(G) H eroin w ith r w l
111. Which of the follow ing tre t ents w ou l be (H ) Inh l nt intoxic tion
the ost ro ri te if this tient w ere not (I) Meth het ine intoxic tion
git te ? (J) N icotine w ith r w l
(A) Aci ific tion of the urine (K) N itrou s oxi e intoxic tion
(B) Cheese (L) PCP intoxic tion
(C) Su ortive c re (M) Psilocybin w ith r w l
(D) Vit in B12 (N ) 3,4-Methylene ioxy eth het ine
(E) Vit in E (MDMA) (Ecst sy) intoxic tion
Que s tions : 108–118 267

113. A 23-ye r-ol n is brou ght to the e ergency lettere o tion y be u se once, u lti le ti es,
e rt ent by the olice. H e show s git tion, or not t ll.
vertic l nyst g us, n n lgesi .
(A) Benztro ine
(B) Cloz ine
114. A 62-ye r-ol ho eless n itte to sy-
(C) Desi r ine (N or r in)
chi tric unit begins h ving elev te vit l signs,
visu l h llucin tions, i horesis, tre or, n (D) Flu oxetine
seizu res. (E) H lo eri ol
(F) Lithiu
115. A 43-ye r-ol w o n resents to the sychi- (G) Lor ze
tric e ergency e rt ent co l ining of (H ) Ol nz ine
e ression n suici lity. She is observe to be (I) Phenelzine
f tigue , irrit ble, n with ys horic ffect. (J) V l roic ci
(K) Venl f xine
116. A 30-ye r-ol u sici n resents in the
ou t tient clinic. H e co l ins of fever n 117. A 34-ye r-ol n w ith schizo hreni
chills, runny nose, n use , bo y ches, i rrhe , resents w ith fever n chills. H e is foun to
n b o in l cr s for the l st 24 hours. be b ctere ic n h s WBC cou nt of 900/ µL.

Questions 117 and 118 118. A 41-ye r-ol w o n on e ic tions for her
bi ol r isor er resents w ith f tigu e, w eight
For the follow ing scen rios, choose the e ic tion g in, col intoler nce, consti tion, n
ost likely ssoci te w ith the si e effects. E ch ecre se concentr tion.

You might also like